301

Surprises and Counterexamples in Real Function Theory

  • Upload
    others

  • View
    34

  • Download
    1

Embed Size (px)

Citation preview

Page 1: Surprises and Counterexamples in Real Function Theory
Page 2: Surprises and Counterexamples in Real Function Theory

TEXTS AND READINGS IN MATHEMATICS 42

Surprises and Counterexamples in Real Function Theory

Page 3: Surprises and Counterexamples in Real Function Theory

Texts and Readings in Mathematics

Advisory Editor C. S. Seshadri, Chennai Mathematical Institute, Chennai.

Managing Editor Rajendra Bhatia, Indian Statistical Institute, New Delhi.

Editors R. B. Bapat, Indian Statistical Institute, New Delhi. V. S. Borkar, Tata Inst. of Fundamental Research, Mumbai. Probal Chaudhuri, Indian Statistical Institute, Kolkata. V. S. Sunder, Inst. of Mathematical Sciences, Chennai. M. Vanninathan, TIFR Centre, Bangalore.

Page 4: Surprises and Counterexamples in Real Function Theory

Surprises and Counterexamples in Real Function Theory

A. R. Rajwade A. K. Bhandari Panjab University

Chandigarh

~HINDUSTAN U ULJ U BOOK AGENCY

Page 5: Surprises and Counterexamples in Real Function Theory

Published by Hindustan Book Agency (India) P 19 Green Park Extension New Delhi 110 016 India

email: [email protected] www.hindbook.com

ISBN 978-81-85931-71-5 ISBN 978-93-86279-35-4 (eBook) DOI 10. 1007/978-93-86279-35-4

Copyright © 2007, Hindustan Book Agency (India)

Digitally reprinted paper cover edition 2011

No part of the material protected by this copyright notice may be repro­duced or utilized in any form or by any means, electronic or mechanical, including photocopying, recording or by any information storage and retrieval system, without written permission from the copyright owner, who has also the sole right to grant licences for translation into other languages and publication thereof.

All export rights for this edition vest exclusively with Hindustan Book Agency (India). Unauthorized export is a violation of Copyright Law and is subject to legal action.

ISBN 978-93-80250-16-8

ISBN 978-81-85931-71-5 ISBN 978-93-86279-35-4 (eBook)DOI 10.1007/978-93-86279-35-4

Page 6: Surprises and Counterexamples in Real Function Theory

v

Contents

Chapter 1: Introduction to the real line lR and some of its subsets 1

§ 1.1. The real number system ........ . ........ . .. .... . .. .. .. .... . . . .... ... 1 § 1.2. Irrational and transcendental numbers ...... .. ............. .. ........ 3 § 1.3. The numbers e and 7r •••..••..••••••.•••••••.•••••••••••••••••••••• 10 § 1.4. The Cantor ternary set ...... .. ..................................... 18 Exercises ................................................................. 24

Chapter 2: Functions: Pathological, peculiar and extraordinary 25

§ 2.1. Some pathological functions . . ............. . ............... ... ...... 25 § 2.2. The Cantor function .......... . .. : ................................. 29 § 2.3. The length of the Cantor function .................................. 33 § 2.4. Some more curious functions .... .. .... . .... ..... ...... .. .. . ........ 37 § 2.5. Algebraic functions ................................................. 50 Exercises . ........... .. ................................................... 51

Chapter 3: Famous everywhere continuous, nowhere differentiable functions: van der Waerden's and others 56

§ 3.1. van der Waerden 's function ............... . .. .. .................... 56 § 3.2. Some more properties of van der Waerden's function .... . ........... 61 § 3.3. A geometric example ........... ... ........ .. ............. .. . ....... 71 § 3.4. An analytic example ...................................... .. ....... 76

Chapter 4: Functions: Continuous, periodic, locally recurrent and others 78

§ 4.1. The intermediate value property ................................... 78 § 4.2. Boundedness and attainment of bounds ..... . ........... ...... . .... 81 § 4.3. Locally recurrent functions ...... .... .. ....... .. ......... . .......... 84 § 4.4. Periodic functions ............. ..... ................................ 86 § 4.5. Horizontal Chords ......................... ..... .................... 96 § 4.6. Continuous functions and integration .. ... ...... .. ................. 100 § 4.7. Continuous functions and their graphs .............. . ...... .. ..... 104 § 4.8. Convex functions ..... . ... . ... . ................................... 107 Exercises ................................................................ 109

Page 7: Surprises and Counterexamples in Real Function Theory

vi

Chapter 5: The derivative and higher derivatives 111

§ 5.1. Continuity of the first derivative .................................. 111 § 5.2. The tangent and the first derivative ............................... 113 § 5.3. Implications of the first derivative being equal to zero ............. 116 § 5.4. Rolle's theorem and the first mean value theorem ................. 121 § 5.5. Points of inflexion ................................................. 126 § 5.6. Geometric interpretation of the second and the third derivatives ... 133 § 5.7. Taylor's theorem and L'Hospital's rule ............................ 138 Exercises ................................................................ 151

Chapter 6: Sequences, Harmonic Series, Alternating Series and related results 155

§ 6.1. Sequences and series of real numbers .............................. 155 § 6.2. The series 2: ~ and Euler's constant I ............................ 159 § 6.3. Some number theoretic aspects of the harmonic series ............. 172 § 6.4. The restricted harmonic series ..................................... 179 § 6.5. Rearrangements of the alternating series 2: (_~)n ................. 181 Exercises ................................................................ 193

x· Chapter 7: The infinite exponential XX

results and related

195

§ 7.1. The equation x Y = yX and the parameterization ................... 195 § 7.2. The infinite exponential ........................................... 204 § 7.3. Applications and examples ........................................ 220

Appendix I 226

§ A.1. Stirling's formula and the trapezoidal rule ........................ 226 § A.2. Schwarz differentiability .......................................... 234 § A.3. Cauchy's functional equation f(x + y) = f(x) + f(y) ............. 244

Appendix II 248

Hints and solutions to exercises .......................................... 248

Bibliography

Index

281

289

Page 8: Surprises and Counterexamples in Real Function Theory

vii

Preface

Our aim in this book is to consider a variety of intriguing, surprising and appealing topics, and nonroutine proofs of the usual results of real function theory. The reader is expected to have done a first course in real analysis (or advanced calculus), since the book assumes a knowledge of continuity and differentiability of functions, Rolle's theorem, the mean value theorem, Taylor expansion and Riemann integration. However, no sophisticated knowledge of analysis is required and a student at the masters or advanced undergraduate level should have no difficulty in going through the book.

Though this book has some part of the title common with the book "Coun­terexamples in Analysis" by Gelbaum and Olmstead, it is totally different in nature and contents. Some examples and counterexamples (fifteen or twenty) in our book are essentially the same as given in the book by Gelbaum and Olmstead, but otherwise the intersection is small.

This book contains a number of surprising and unexpected results. It is meant to be a reference book and is expected to be a book to which one turns for finding answers to curiosities which one comes across while studying or teaching elementary analysis. For example: We know that continuous functions defined on an interval satisfy the intermediate value property. Are there functions which are not continuous but have this property in every interval?(Example 4.1.2) If a one to one onto function is continuous at a point, is its inverse also continuous at the image of that point?(Example 2.4.17) Most would believe that "if Y = f(x) is a function defined on [a, b] and c E (a, b) is any point, then the tangent to its graph r f exists at the point (c, f (c)) if and only if f is differentiable at c" - see § 5. 2 for a negative answer. Where does one find easily accessible details of everywhere continuous, nowhere differentiable functions?(Chapter 3)

Chapter 1 of the book gives an introduction to algebraic, irrational and transcendental numbers. It has several results about the numbers e and 11' and contains a detailed account of the construction of the curious Cantor ternary set.

In Chapter 2, we consider functions with extraordinary properties. For example, an increasing function f : [0,1] -+ [0,1]' 1(0) = 0, 1(1) = 1, the length of whose graph is equal to 2. Another example studied is a function defined on the entire real line that is differentiable at each point but is monotone in no interval.

Chapter 3 discusses, in detail, functions that are continuous at each point but differentiable at no point.

Chapters 4 and 5 include the intermediate value property, periodic func-

Page 9: Surprises and Counterexamples in Real Function Theory

viii

tions, properties of derivatives, Rolle's theorem, Taylor's theorem, L'Hospital's rule, points of inflexion, tangents to curves etc. The geometric interpretation of the second and the third derivatives and some intricate aspects of Riemann integration are also included in this chapter.

Chapter 6 discusses sequences, series and Euler's constant ,. The restricted harmonic series is a beautiful topic included here. The surprising rearrange­ments of alternating harmonic series, leading to Riemann's theorem are also discussed in this Chapter. Some number theoretic: aspects are also treated.

x· In Chapter 7, the infinite exponential XX with its peculiar range of

convergence is studied in detail. We have included an analytic proof of its convergence as well as a very revealing graphical proof.

Appendix I deals with Stirling's formula, a specialized topic of Schwartz's differentiability and some curious properties of Cauchy's functional equation.

Some exercises, conforming to the spirit and style of the book, are included at the end of each chapter. Hints and/or full solutions for the exercises are provided in Appendix II.

References for most of the material in the book are given at the end. Cer­tainly, many other interesting topics could have easily found a place in the book, but there are limitations of time and space.

We hope that the book will be useful to students and teachers alike.

Department of Mathematics Panjab University, Chandigarh India October 2006

A.R. Rajwade A.K.Bhandari

Page 10: Surprises and Counterexamples in Real Function Theory

Chapter 1

Introduction to the real line JR;. and some of its subsets

§ 1.1. The real number system

1

The system of real numbers has evolved as a result of a process of successive extensions ofthe system of natural numbers (i.e., the positive whole numbers). We shall denote by N, Z, IQ, IF. and C, respectively, the sets of natural numbers, integers, rational numbers, real numbers and complex numbers. The sets IQ, IF. and C form a field with respect to the usual operations of addition and multiplication. The fields IQ and IF. are ordered fields:

Definition 1.1.1 An ordered field is a field F that contains a subset P such that (i) P is closed with respect to addition; that is,

x E P , yEP ===} x + YEP.

(ii) P is closed with respect to multiplication, that is,

x E P , yEP ===} x.y E P.

(iii) For all x E F, exactly one of the following three statements is true:

x E P; x = 0; -x E P,

where 0 is the additive identity of the field F. A member x of F is called positive if and only if x E P and is called negative

if and only if -x E P. Inequalities in an ordered field are defined by: x < y if and only if y - x E P and x :=:; y if and only if y - x E P or x = y. If F is an ordered field and if x E F, then 1 x I, called the absolute value of x, is defined to be x in case x 2: 0 and to be -x in case x < O.

Suppose that F is an ordered field. Let u E F and A ~ F. If x :=:; u for every x E A, then u is called an upper bound of A. A non empty subset A of F is called bounded above in F if and only if there exits an element of F which is an upper bound of A. If s is an upper bound of A and if s is less than or equal to every other upper bound of A, then s is called the least upper bound or supremum of A, denoted by sup A. Similarly one defines the notion

Page 11: Surprises and Counterexamples in Real Function Theory

2 Chapter 1

of being bounded below, lower bound and the greatest lower bound or infimum of a non-empty set A. The infimum of A is denoted by inf A.

Definition 1.1.2. A complete ordered field is an ordered field F in which a least upper bound exists for every non-empty subset of F which is bounded above in F.

From any of the well-known constructions of the real number system, it follows that the set IE. of real numbers is a complete ordered field.

Let F be an ordered field. A sequence in F is a function with values in F whose domain is the set of natural numbers N. Its values are denoted by an and the sequence itself by {an}. A sequence {an} is said to converge and to have a limit a if and only if for every E in the set P of positive numbers, there exists N E N such that 1 an - a 1< E for all n > N. A sequence that is not convergent is said to be a divergent sequence. A sequence {an}, where the terms an are members of an ordered field F, is called a Cauchy sequence if for every E E P, there exists N E N such that 1 am - an 1 < E for all m, n > N. It follows that every convergent sequence is a Cauchy sequence. The completeness property of IE. is equivalent to the fact that every Cauchy sequence in IE. is convergent. Not all fields can be ordered in the sense of Definition 1.1.1. We have:

Example 1.1.3 The field C can not be ordered, i.e., it possesses no subset P satisfying the properties of Definition 1.1.1. Indeed, assume that there does exist such a subset P of C. Consider the complex number L Since L f. 0, there are two possibilities. The first is that L E P, in which case L 2 = -1 E P, whence [4 = 1 E P. Since [2 and [4 are additive inverses of each other, it is impossible for both of them to be in P. We thus obtain a contradiction. The other alternative is that -[ E P, in which case (_L)2 = -1 E P, whence (_L)4 = 1 E P, and we arrive at the same contradiction.

It can be shown that the fields Q as well as IE. admit only one ordering (see, for example [61]). The set P of positive elements of IE. is the set of squares of the elements of IE.. The set of positive elements of Q is Q n P.

However, there are fields which can be ordered in more than one ways.

Example 1.1.4. Let m be any positive integer which is not a perfect square. Let F = {a + bfo 1 a, b E Q}. It is easy to see that F is a field under the usual operations of addition and multiplication of real numbers. Let P be the set of usual positive elements of IE. (i.e., squares) and let pi = P n F. Then pi serves as a subset of positive elements of F according to Definition 1.1.1. A second way in which F is an ordered field is provided by the subset p lI defined by

a + bylffi E P" {::=::} a - bylffi E P;

that p lI satisfies the three requirements of Definition 1.1.1 can be easily verified.

Example 1.1.5. The ordered field Q of rational numbers is not complete (i.e., does not satisfy the requirements of Definition 1.1.2). Let

A. = {r E Q 1 l' > 0 , 1'2 < 2}.

Page 12: Surprises and Counterexamples in Real Function Theory

1.1 The real number system 3

The set A is non empty ( 1 E A) and is bounded above by 2. Let us assume that Q is complete. Then there must be a positive rational number c that is the supremum of A. Since there is no rational number whose square is equal to 2, either c2 < 2 or c2 > 2. Assume first that c2 < 2 and let d be the positive

2 number d = ~ min{ (~+~)2 ,I}. Then c+d is a positive rational number greater

than c whose square is less than 2, i.e., (c + d)2 < c2 + d(c + 1)2 < 2; but then c + dE A, whereas c is an upper bound of A. Assuming c2 > 2, let d be the

2

positive number d = 2(C;1)2' Then c - d is a positive rational number less than

c whose square is greater than 2, i.e., (c - d)2 > c2 - d(c + 1)2 > 2. Since c - d is an upper bound of A which is less than the least upper bound c, we arrive at a contradiction.

Recall that the set Q of rational numbers is dense in lR (in the usual dis­tance topology). We close this section by recording a somewhat surprising result, which will be needed later.

Theorem 1.1.6. Let () be an irrational number. Then the set of n11mbers of the form m + n() , m, nEZ, is dense in R

Proof. Let E > 0 be an arbitrary real number. We shall first find m, n such that 0 < m + n() < E. Choose N such that 1:t < E and then for each n = 0,1,2, ... ,N, choose m (= m(n), i.e., m depends on n) such that 0< m + n() < 1 (for example, m = - [n ()] will do, where for a real number x, [x] denotes the largest integer less than or equal to x). Then we have N + 1 distinct numbers m(n) +n(), 0:::; n:::; N, in [0,1] and therefore there exist two such numbers which are at a distance less than 1:t apart (box principle), say, 1 (ml + nl ()) - (m2 + n2 ()) 1< 1:t < E, i.e., 1 (ml - m2) - (nl - n2) () 1< E. Thus, we find a number of the form m + n () such that 1 m + n () 1< E. By changing signs we get a number 0 < m + n () < E. Now, given any real number r, find an integer k such that k(m + n ()) < r < (k + l)(m + n ()). We thus get a number of the required form which is as close to r as we want.

§1.2. Irrational and transcendental numbers

A real number ex is called an algebraic number, if ex is a root of a nonconstant polynomial with rational coefficients. Let A denote the set of all such real numbers. In the usual notations, the inclusions

give rise to the following disjoint unions:

lR = Q U (lR "Q) = Q U II ; lR = A U (lR "A) = A U 1I' ,

where, II = lR " Q and 1I' = lR "A As Q ~ A, it follows that 1I' ~ II.

Definition 1.2.1. Elements of II are called irrational numbers while those of 1I' are called transcendental numbers.

Page 13: Surprises and Counterexamples in Real Function Theory

4 Chapter 1

Perhaps it is very surprising that while on one hand it is not immediately clear that'll' is non empty, on the other hand, it turns out that (in a well defined sense) almost all real numbers are transcendental. Not only that, it is hard to identify individual numbers as being transcendental and such identifications, as have been made are mathematical epics. According to a Cambridge story, G.H. Hardy was prepared to resign his chair in favour of anyone who proved that the Euler's constant

'Y = lim (1 + ~ + ... + 2. -logn) n-+oo 2 n

was irrational, let alone transcendental! It was in 1873 that Hermite proved that

1 1 e=I+-+ + ...

I! 2!

is transcendental and in 1882, Lindemann did the same for 7T; the status of 'Y is still undecided.

It will give some idea of the difficulty of transcendence questions if we note that the transcendence of 2V2 was the seventh in the list of the famous 23 unsolved problems that David Hilbert presented, at the International Congress of 1900 in Paris, as signposts for twentieth century Mathematics. Not only that but speaking informally at a seminar in Gottingen, twenty years later, Hilbert declared that none of the audience would live to see a solution of this problem. As it happened, Hilbert was wrong: the problem was settled some twelve years later by Gelfond and Schneider, while several questions that Hilbert believed to be easier are still unanswered.

A beautiful result that settles the transcendence of 2V2 is the following:

Theorem 1.2.2.(Gelfond-Schneider, 1934) Ifa -::f- 0 or 1, a E A, (3 E lInA, then a(3 E 'll'.

For a self-contained proof of this result the reader is referred to [44]. Taking a = 2, (3 = v'2 shows that 2V2 is transcendental.

In 1851, Liouville was the first to exhibit a class of transcendental numbers, viz. numbers of the form adl0 + a2/102! + a3/103! + . .. ,0 :S ai :S 9, with infinitely many ai's nonzero. This class is uncountable whereas the set A can be shown to be countable. This already shows that most numbers are transcendental.

Theorem 1.2.3. (Liouville, 1851) The real number

0011111 L = L 10i! = 10 + 102 + 106 + 1024 + . .. = 0.1100010 ...

i=l

is transcendental.

Proof. Let a be an algebraic number. Then it satisfies a polynomial P(x) with integral coeffcients. Suppose that the degree of P(x) is n. We first claim

Page 14: Surprises and Counterexamples in Real Function Theory

1.2 Irrational and transcendental numbers 5

that then a can not be the limit of a sequence of distinct rational numbers Pi/ qi satisfying

1 pdqi - a 1< k/q~+l , (*)

where k is a fixed number. For otherwise, the mean value theorem gives P(pdqi) - P(a) = (pdqi - a)P'(~i) for some ~i between Pi/qi and a, i.e.,

1 P(pdqi) 1 = 1 pdqi - a II P'(~i) I < (k/q~+1) I P'(~i) I

< (k/q~+1)(1 P'(a) I +1) ,

(since P(a) = 0)

(by(*) )

if i is large enough, since ~i ---+ a, (as i ---+ oo,pdqi ---+ a) and P' is a polynomial, so continuous. This gives I qi P(pdqi) 1< (k/qi)(1 P'(a) + 1 I), if i is large. Here the left hand side qi P(pd qi) E Z, since P(Pi/ qd is a polynomial in pd qi of degreen with integer coefficients, while the right hand side tends to 0 as i tends to infinity, i.e., the absolute value of the right hand side is less than 1 (note that Pi/qi being distinct implies that qi ---+ 00 as i ---+ 00). It follows that the left hand side is equal to 0 for all i 2: J, say, i.e., P(pdqi) = 0 for i 2: J, which is a contradiction since P is a polynomial of degree n and so has no more than n zeros. This proves the claim.

Now going back to the number L, we have L = 1/10+1/102+0/103+0/104 + 0/105 + 1/106 + 0/107 + ... , so that its decimal expansion equals 0.1100010 .... as stated in the theorem, with the ith nonzero decimal digit equal to 1 in the i! th place. If we truncate this decimal expansion after the ith nonzero digit, we obtain a rational approximation pi/lOi!, which differs from L by less than 2/lO i !+1. Indeed, we have

L = (1/10+ 1/102 +0/103 +0/104 +0/105 + 1/106 + .. -+ l/lO i !)+O/lOi !+1 + ... , v "

truncated at this digit

and we call the quantity in brackets pdqi, which is equal to Pi/lOi! (common denominator equal to 1Oi!), and then

i!+l (HI)! L - pdqi = 0/10 + ... + 1/10 + ... (i+I)! = 0 + ... + 1/10 + ...

= (1/10(i+1)!)(1 + 1/10i+2 + 1/1O(i+3)(i+2) + ... )

< (1/10(i+1)!) ·2 (the sum being a sub-geometric progression)

= 2/(10i!)i+l

:::; 2/(10i!)n+l ,

for any n 2: i; and so, since the qi are all distinct ( qi = lO i !), the claim above yields that L can not satisfy a polynomial equation of degree n with integer coefficients and hence it follows that L is not algebraic.

Page 15: Surprises and Counterexamples in Real Function Theory

6 Chapter 1

For the general Liouville number L = L a;/10i! , 0 ::; ai ::; 9, a slight modi­fication ofthe above argument shows L to be transcendental. Alternatively, one could use binary expansion and write L in the form L = L a;/2i! (ai = 0,1) and then, exactly as in Theorem 1.2.3, L may be shown to be transcendental. This was the first encounter of a class oftranscendental numbers (in 1851). The Gelfond-Schneider theorem mentioned above guarantees another such class of transcendental numbers. To check specific numbers for transcendence is an extremely difficult job. Results that help in this direction are the following:

Theorem 1.2.4. If (3 is a positive number such that 2{3, 3{3, 5{3, 7{3, l1{3, ... are all integers, i.e., if p{3 is an integer for every prime p then (3 itself is an integer.

For a proof see [41J.

The hypothesis of the above theorem can be weakened and indeed a deeper argument given in [41J to prove Theorem 1.2.4 yields the following result.

Theorem 1.2.5.(Siegel) If (3 is a positive real number such that 2{3, 3{3, 5{3 are integers, then (3 is an integer.

It is an open question whether the hypothesis of Theorem 1.2.5 can be reduced to requiring only that 2{3 and 3{3 be integers. If that were possible, as is conjectured, the result would be the best possible, since 2log 3/ log 2 = 3 is an integer but log3/log2 is irrational (for, if it is equal to p/q, then qlog3 = p log 2, or 3q = 2P , which is impossible). Theorem 1.2.5 tells us that if (3 is irrational, at least one of 2{3, 3{3, 5{3 is not an integer.

The following far-reaching extension of Theorem 1.2.2 was proved by Baker (see [6]) in 1966.

Theorem 1.2.6. If a1, ... ,an are algebraic numbers, ai ::j:. 0,1, and if (31, ... ,(3n are algebraic numbers which are different from 0 and 1 and are linearly independent over Q, then af' ag2 ••• a~n is transcendental.

The case n = 1 is Theorem 1.2.2. As an example, it follows that the num­ber 2v'2.3v3.5V5 is transcendental.

Example 1.2.7. Can a rational or an irrational number raised to a ratio­nal or an irrational power be rational or irrational? All the eight possibil­ities are as follows: (i) (irrational)irrational = rational : Observe that

(J2v'2)v'2 = (J2)v'2.v'2 = 2 and that J2v'2 is irrational, being the square

root of Hilbert's number 2v'2 (Theorem 1.2.2). It was actually proved to be transcendental by Kuzmin in 1930. (ii) (irrational)irrational = irrational: Yes; (v'"2)v'2, as above.

(iii) (rational)irrational = irrational : Yes; 2v'2 is irrational, by Theorem 1.2.2. (iv) (rational)irrational = rational: Yes; 2log23 = 3. However, a rational number raised to an algebraic irrational power is irrational, by Theorem 1.2.2.

Page 16: Surprises and Counterexamples in Real Function Theory

1.2 Irrational and transcendental numbers

(v) (irrational)rational = rational: Yes; (y'2)2 = 2.

(vi) (irrationaltational = irrational :Yes; trivially, (y'2)1 = y'2. (vii) (rational)rational = rational: Yes; 22 = 4.

(viii) (rational)rational = irrational :Yes; 2~ = y'2 is irrational. The reader is referred to [52] for more such results.

Example 1.2.8. The number ~ = 2::~1 t is transcendental, where Co

0, Ci+l = 2C;.

7

To see this, observe that Ci are rapidly increasing powers of 2. Write out a few and then check (by induction) that if k ~ 1, then k ::; Ck-l and that C%_l ::; 2Ck - 1 • It follows that kCk-l ::; C%_1 ::; 2Ck - 1 = Ck, i.e., kCk-l ::; Ck and so 2kck- 1 ::; 2Ck = Ck+l, or (2 Ck - 1 )k ::; Ck+l, or c~ ::; Ck+l (note that x Y - yX ::; 0 for (x, y) in regions IV and II of the plane (see Chapter 7); hence c% - 2Ck ::; 0, since (Ck' 2) is in the region IV).

Now, let E,k = 2::7=1 t· Then ~ = Pk/Ck, where (Pk, Ck) = 1 (for ~k = (a sum of even integers+l)/ck)' Now suppose ~ is algebraic of degree n (n > 1), then

because, {_1_ + ... } < _1_ + terms of a geometric progression, all of the Ck+2 - Ck+2

form 1/2T , from Ck+2 onward::; 2/Ck+2 < I/Ck+l and c~ ::; Ck+l, by above. It follows that

and so 1 c~+l(~-Pk/Ck) 1< 1, if k is large, which gives that 1 Pk/Ck-~ 1< l/cZ+l and since Pk / Ck are distinct, this contradicts the claim in the beginning of the proof of Liouville's theorem.

Finally, there remains the case n = 1, i.e., ~ is rational, say, ~ = P / q (p, q integers). Choose k such that Ck > q. Then 0 < (~- ~k)qck = (P/q -Pk/Ck)qCk = PCk - Pkq, which is an integer.

However, (~ - ~k)qck = qCk(2::i>k+l1/ci) < qCk.(2/ck+d ::; 2q/C~-1 < 2 / c~ - 2 , since c~ ::; Ck+l and k is so-chosen that Ck > q; and it follows that for k ~ 3, (~- ~k)qck < 1, contradicting the above observation that it is an integer.

There are many outstanding problems regarding the irrationality of num­bers. In particular, many irrational numbers can be explicitly exhibited, e.g., Vn is irrational for all positive integers n which are not perfect squares. Also y'2 + y'3, y'2 + y'3 + y'6 etc. are irrational. In what follows, we give a couple of such interesting results, which yield irrationality of some classes of numbers. The proofs of these results require familiarity with the basic concepts of field extensions.

Theorem 1.2.9. Let aI, ... ,an be positive integers, none perfect squares and

Page 17: Surprises and Counterexamples in Real Function Theory

s Chapter 1

. . . Th h 2n lb· b (<0 1 02 C ) 1 0 1 eoprzme zn pazrs. en tea ge raze num ers a l a2 ... ann 2, Ci = , , are linearly independent over Q.

Proof. We use induction on n. For n = 1, the result is trivial. So suppose the result is true for n - 1 and we prove it for n.. Define a tower of fields: Ko = Q , KI = Q(Jal) , K2 = K I (y'a2) , ... ,Kn = Kn-I(Fn). We first prove that each Ki is of degree 2 over K i- l . To see this we again use induction. For n = 1, clearly [KI : Kol = 2. So, suppose the result is true for n - 1 and we are to show that [Kn : Kn-ll = 2. If not, then Kn = K n- l , i.e., Fn E K n- I = K n- 2(Jan-l) and so Fn = /3 + ,Jan-I, /3" E K n- 2 , or an = /32 + ,2an_ l + 2/3'Jan-l. Here Jan-I ~ K n- 2 , by induction hypothesis, so /3, = 0. If, = 0, then Fn = /3 E K n - 2 , which contradicts the induction hy­pothesis for n -1 numbers aI, a2,··· ,an-2, an· If /3 = 0, then Fn = ,Jan-l

and so vi (an-l an) = an-I, E K n- 2, which again contradicts the induction hypothesis for n - 1 numbers aI, ... ,an-2, an-I an. This completes the proof of the observation and hence [Kn : Ql = 2n.

By induction hypothesis, the set {( a~1 ... a~'::...-11) 1/2 I Ci = 0,1} is linearly independent over Q and therefore constitutes a basis of K n - l over Q, as [Kn - l :

Ql = 2n - l , by above. Let E ~ K ~ L be a tower of fields. Let {al, ... ,as} be a basis of K

over E and {/3I, ... ,/3d be a basis of Lover K. Then it is easy to see that {ai/3j 11 ::; i ::; s, 1 ::; j ::; t} is a basis of Lover E.

Now, {(a~1 ... a~'::...-11 )1/2 lei = 0, I} is a basis of K n - l over Q and {I, Fn} is a basis of Kn over K n- l , the result follows.

Example 1.2.10. As an application of the above result it follows that an expression like 7JI9/4 - 3V7 + SV6/5 is irrational.

While we are at it, we prove the following very useful:

Theorem 1.2.11. Let m be a square-free integer greater than 1 and let a, /3" E Q( J m) = {a + brm I a, b E Q}. A necessary and sufficient condition that there exist u, v, w E Q such that

uVa + vJ1j + wJ1 = 0

is that there exist ).., JL, v E Q( J m) such that

a : /3 : , = )..2 : JL2 : v2.

(1)

(2)

Proof. Write a = al +a2Vm, /3 = bl +b2Vm, , = Cl +C2Vm, al,a2,bl ,b2, Cl, C2 E Q. Then (1) => (ufo + vV,B)2 = (-wy'r)2 => u2a + v2/3 + 2uv~ = w 2 , => ~ E Q(Jm), i.e., that a/3 is a square in Q(Jm), say, a/3 = l/v2

and similarly /3, = 1/)..2, ,a = I/JL2, with )..,JL, v E Q(Jm). On dividing, it follows that a//3 =)..2 /JL2 and ah =)..2 /v2 , which together give (2).

Conversely, let (2) hold. Then fo : V,B : y'r = ).. : JL : v, i.e., say, fo = c).., V,B = CJL, y'r = cv, where C E Q(Jm). Then Va = c(al + a2Vm) , V,B = c(b i + b2 Vm) , y'r = C(CI + C2Vm) and we want to solve for u, v, w (not all zero) in Q, with ufo + vV,B + w"fY = 0, i.e., u(al + a2Vm) + v(b i + b2Vm) +

Page 18: Surprises and Counterexamples in Real Function Theory

1.2 Irrational and transcendental numbers 9

W(CI + C2Vm) = 0, i.e., we want to solve the equations ual + vb l + WCI

o , ua2 + vb2 + WC2 = 0, simultaneously for u, v, W, nontrivially, in Q. As these are two equations in three variables, there exist a nontrivial solution u, v, W in Q, as required.

Observe that when (2) is satisfied, (1) becomes u).. + Vf-l + wv = o.

As an application, we have the following:

Example 1.2.12. 59V(90 - 14v7)+4V(4555 + 1721v7) = 145V(26 + 2v7).

Indeed, let m = 7 and let a = 90-14v7, (3 = 4555+ 1721v7, 'Y = 26+2v7. We need to verify that there exist u, v, W (given respectively as 59, 4, -145) such that uya + vVfJ + w,;=y = 0 (i.e., (1)). It is enough to verify (2), i.e.,

that a : (3 : 'Y = )..2 : f-l2 : v 2. We have ah = (90 - 14v7)(26 + 2v7) = (317-68v7)/81 = ((17-2v7)/9)2, and (3h = (4555+ 1721v7)/(26+2v7) = (47168 + 17818v7) /324 = ((151 + 59v7)/18)2, so a = «17 - 2v7)/9)2.'Y , (3 = «151 + 59v7)/18)2 . 'Y and of course 'Y = 12 . 'Y. These give us the ratios )..2 : f-l2 : v 2 and proceeding as in the proof of Theorem 1.2.11, one checks easily that u = 59, v = 4, W = -145 is a solution of the system of equations

17 151 -u+ -V+W = 0 9 18

-2 59 gU+ 18v = 0

Remark 1.2.13. From elementary field theory it follows that if K/ F and L/ K are algebraic extensions, then so is L/ F. From this, it is easy to deduce that the set of all algebraic real numbers is a subfield of JR., containing Q, i.e., the sum, difference, product and quotient of two algebraic numbers is algebraic.

If 8 is a rational number, it follows that cos Jr8 and sin Jr8 and hence cot Jr8 = ~f; ;~ are algebraic numbers (indeed, for 8 = p/ q, (cos 7- + L sin 7-)q =

(ePrp/q)q = eL7rp = ±1, yields polynomial equations with rational coefficients satisfied by cos Jr8 and sin Jr8). As an application, we have:

Example 1.2.14. The number g(8) = 2:~=1 8j(n(n + 8)) is transcendental (and hence irrational) for infinitely many rational numbers 8 E (0,1).

Proof (sketch). Assuming the partial fraction expansion of Jr cot Jr8, i.e.,

00 28 Jr cot Jr8 = L 82 _ n2 '

n=2

Page 19: Surprises and Counterexamples in Real Function Theory

10 Chapter 1

the proof proceeds as follows:

00

g(()) = 1 - 1/(1 + ()) + I)I/n - 1/(n + ())) , (1) n=2

00

g(l- ()) = :l)I/n - 1/(n + 1 - ())) n=l

= (1- 1/(2 - ())) + (1/2 - 1/(3 - ())) + ... 00

= 1 + 2)I/n - 1/(n - ())) . (2) n=2

(1) and (2) imply that

1 00 ')

g(l- ()) - g(()) = 18 + 2)2 ()/(()2 - n~)) + n=2

= 1 ~ () + £=(2 ()/(()2 - n 2 ))

n=l

1 = 1 _ () + 1T cot 1TB

Since for () rational, cot 1T() is algebraic, whence 1T cot 1T() is transcendental (as 1T

is transcendental, see next section), therefore, at least one of g(1 - ()) and g(()) is transcendental.

§ 1.3. The numbers e and 1T

In this section we investigate two special real numbers e and 1T, their irrationality and transcendence. We have tried to collect interesting facts about them at one place. We begin with the following:

Theorem 1.3.1.{i) The series 1 + 1/1! + 1/2! + ... is convergent. Its limit is called e. Moreover 2 < e < 3. (ii) The number e is irrational. (iii) Let an = (1 + l/n)n+l, bn = (1 - l/n)n, Cn = (1 + l/n)n. Then en increases, an > Cn for all nand limn--+oo an, limn--+oo Cn both exist and are equal to e. Further, limn--+oo bn also exists and equals l/e.

Proof. (i) The partial sums Sn = 1 + 1/1! + 1/2! + ... + l/n! < 1 + 1 + 1/2 + 1/22 + ... + 1/2n- 1 = 3, are bounded above by 3 and below by 2 and are clearly increasing as n increases and so tend to a limit e say, as required.

Page 20: Surprises and Counterexamples in Real Function Theory

1.3 The numbers e and 7r

(ii) We have 0 < e - Sn and that

e - Sn = l/(n + I)! + l/(n + 2)! + ... = (l/(n + 1)!)(1 + l/(n + 2) + l/(n + 3)(n + 2) + ... ) < (l/(n + 1)!)(1 + l/(n + 1) + l/(n + 1)2 + ... ) = (l/(n + 1)!)(1/(1 - l/(n + 1)))

= l/n!· n .

11

Now if e = p/q, say, with 1 ::; p, q ,(p, q) = 1, then 0 < p/q - Sn < l/n!· n. In this taking n = q we get 0 < p/q - Sq < l/q!. q, and multiplying by q!, this becomes 0 < (q - I)! . p - q! . Sq < l/q ::; 1. Here (q - I)! . p and q! . Sq are both integers and so their difference p say, is an integer, i.e., 0 < p < 1, which is not possible. (iii) We first prove that Cn = (1 + l/n)n --+ e as n --+ 00. Using the binomial theorem

(l+l/n)n=l+n(l/n)+(n(n -1)/2!)(1/n2)+(n(n -l)(n - 2)/3!)(1/n3 )+ ...

= 1 + 1 + (1/2!)(1 - l/n) + (1/3!)(1 - l/n)(l - 2/n) + ... + + (l/n!)(l - l/n)(l - 2/n)··· (1- (n - l)/n) (*)

< 1 + 1 + 1/2! + 1/3! + ... + l/n!

<e

Further, each term on the right hand side of (*) increases as n increases and the number of terms increases too as n increases, i.e., (1 + l/n)n increases as n increases, and it is bounded above by e and so tends to a limit TJ ::; e. However, (l+l/n)n 2 1+1+(1/2!)(1-1/n)+(1/m!)(1-1/n)(1-2/n) ... (1-(m-1)/n), (by (*) again), if n 2 m. Now keeping m fixed and letting n --+ 00, we get that TJ 2 1 + 1 + 1/2! + ... + l/m! and this is true for all m. Letting m --+ 00, we get TJ 2 e.

Short proofs of the other parts are as follows. The arithmetic mean of the n + 2 numbers 1, 71/(71 + 1), n/(n + 1), ... , n/(n + 1) is greater than their geometric mean. This gives (1 + n)/(n + 2) > ((n/(n + l))n+1 )1/(n+2), i.e., on taking reciprocals 1 + 1/ (71+ 1) < (1 + l/n )(n+1)/(n+2), i.e., (1 + 1/ (n+ 1) )(n+2) < (1 + 1/ n) (n+1), showing that an decreases as n increases.

Next, bn+1 = (1 - l/(n + l))(n+1) = (n/(n + l))(n+1) = l/an; hence bn increases, as required.

Finally, the arithmetic mean of the n+1 numbers 1, l+l/n, l+l/n, ... , 1+ l/n is greater than their geometric mean. Hence (1 + n(l + l/n))/(71 + 1) > ((1 + l/n )n)l/(n+1), i.e., ((1 + 1/ (n + 1) )(n+1) > (1 + l/n)n, showing that Cn in­creases as 71 increases. Since Cn --+ e (already proved), so an = cn(1 + l/n) --+ e too. Finally, l/bn = 1/an-1 --+ l/e as required.

Remark 1.3.2. The value of e has been calculated with great accuracy (see [63], page 101):

2.71828182 ... < e < 2.71828184 ...

Page 21: Surprises and Counterexamples in Real Function Theory

12 Chapter 1

We next consider the number 7r. A reference to an excellent account of 7r, its computation and all other developments through the ages, is the exhaustive article " The Ubiquitous 7r ", by Dario Castellanos [22].

With the help of computers, 7r has now been calculated to thousands of decimal places:

7r = 3.14159265358979323846 ... .

Many mnemonics have been composed to remember and write down the value of 7r, the number of letters in each word representing successive digits of 7r. A well known mnemonic is: How I want a drink, alcoholic of course, after the heavy lectures involving quantum mechanics.(15 digits).

One of the most basic result in which 7r appears is the sum of the following infinite series:

Theorem 1.3.3. 1 + 1/22 + 1/32 + 1/42 + ... 7r2 /6.

We shall present here a simple proof given m [78]. First we prove the following simple

Lemma 1.3.4. 2::;;'=1 cot2 (k7r/(2m + 1)) = m(2m -1)/3 .

Proof. By equating the imaginary parts in the formula

cos n 0 + [sin n 0 = (cos 0 + [sin 0) n = sinn 0 (cot 0 + [) n

= sinn 0 t (~) . [k cotn - k 0, k=O

we obtain the identity

sinnO = sinn 0 [(~) cotn - 1 0 - (~) cotn - 3 0 + G) cotn - 5 0 _ ... ] ,

where 0 < 0 < 7r /2. Take n = 2m + 1 and write this in the form sin (2m + 1)0 = sin2m+1 0·Pm(cot2 0), where Pm (x) = (2n~+1)xm - (2n~t )xm-1 + em5+1)xm-2-... + (;:tD is a polynomial of degree m. Since sin 0 =f. 0, if 0 < 0 < 7r /2, it follows from above that Pm (cot2 0) = 0 if and only if (2m + 1)0 = k7r , k E Z. Therefore, Pm(x) vanishes at the m distinct points Xk = cot2 (7rk/(2m + 1)), for k = 1,2, ... ,m. Since the degree of Pm(x) is m, these are all the zeros of Pm(x) and their sum is _(_(2~+1)/e~+1)), which proves the lemma.

Proof of Theorem 1.3.3. Start with the inequality sin x < x < tan x for 0< x <7r/2. Take reciprocals and square to obtain cot2 x < 1/x2 < 1 + cot2 x. Now put x = k7r /(2m + 1), where k, m are integers, 1 :::; k :::; m, and sum from k = 1 to k = m to get

m m m

L cot2(k7r/(2m+ 1)) < ((2m+ 1)2 /7r2 ) L 1/k2 < m+ L cot2(k7r/(2m + 1)). k=l k=l k=l

Page 22: Surprises and Counterexamples in Real Function Theory

1.3 The numbers e and 7f 13

Putting in the value of the sums, using the lemma, we get

m

m(2m - 1)/3 < ((2m + 1)2/7f2) L 1/k2 < m + m(2m - 1)/3, k=l

which, on dividing throughout by (2m + 1)2/7f2 and then letting m -+ 00,

proves the theorem. One of the most beautiful expressions for 'if, given by John Wallis in 1655,

called Wallis' product, is the following:

Theorem 1.3.5 .

'if

2 . ( 2·2·4·4·6···(2m)·(2m) )

J~oo 1·3·3·5·5· .. (2m - 1) . (2m + 1) .

Proof. Let In = Ja1T/

2 sinn e dx. Integration by parts easily gives In = ((n -1)/n)In - 2 (n> 1). Since Ia = 'if/2, h = 1, it follows that

hm = ((2m - 1)/2m) . ((2m - 3)/(2m - 2))··· (1/2).(7f/2) (1)

hm+l = (2m/(2m + 1)) . ((2m - 2)/(2m - 1))··· (2/3).1 (2)

Now in the range 0 ::; e ::; 'if /2, we have 0 ::; sin e ::; 1 (in fact < 1 if e < 'if /2) and so sinne > sine. sinn e = Sinn+le for all e E (O,7f/2). It follows that (see Figure 1.1) 0 < hmH < hm < hm-l.

y = sin2m-1e

y = sin2me y = sin2m+le

Figure 1.1

Dividing by hmH gives

(*)

Here the extreme right hand term is equal to (2m + 1)/2m (see (1) and (2)), which tends to 1 as m -+ 00. So letting m -+ 00 in (*), we get 1 < limm-+oo hm/ hm+l ::; 1. However using (1) and (2), we get

~ = (~) (~) (~) (~) ... (2m - 1) (2m - 1) (2m + 1) , I2mH 2 2 242m - 2 2m 2m

Page 23: Surprises and Counterexamples in Real Function Theory

14 Chapter 1

and letting m --+ 00, we get the result.

Corollary 1.3.6.

1. (2m m!)4 71" 1m

m-Hx) ((2m)!)2 . (2m + 1) 2

Proof. The right side of Wallis' product is

2 . 2 . 4 . 4 ... 2m . 2m (2 ·4···2m)2 1·3·5··· (2m - 1)(2m + 1) (1· 3·5··· (2m - 1))2(2m + 1)

(2m . m!)2 (2 . 4· . ·2m)2 (1 . 3 . 5 ... (2m - 1))2 (2m + 1) . (2 . 4 ... 2m)2

(2m. m!)2 . (2m. m!)2

(1·2·3·4··· (2m - 1) . 2m)2(2m + 1) (2m . m!)4

((2m)!)2(2m + 1)·

We shall now describe some methods to compute the value of 71".

1. By inscribed and circumscribed polygons. Euclid, in the fourth century B.C., had proved that 3 < 71" < 4, but it was not until the third century B.C. that Archimedes attacked the problem of the determination of 71" systematically. Using polygons inscribed in and circumscribed to the circle, whose number of sides are successively doubled, he obtained for 71" the bounds 3~~ < 71" < 3t. The bound 3t = ;2 is often referred to, erroneously, as the Archimedian value. Archimedes meant this as an upper bound on the value of 71".

Archimedes' method remained essentially unchanged except for better ap­proximations to 71" obtained by taking larger and larger doublings, until the advent of Calculus.

2. An analytic expression for 7r . Consider the identity

(sin 0) /0 = cos(O /2) . sin(O /2) / (0/2)

= cos(0/2) . cos(0/4). sin(0/4)/(0/4)

= cos(0/2)· cos(0/4)· .... cos(0/2n). sin(0/2n)/(0/2n).

As n --+ 00, sinO/2n/(0/2n) --+ 1 and we obtain Euler's formula

(sin 0)/0 = lim (cos(0/2)··· cos(0/2n)) = cos(0/2)· cos(0/4)· cos(0/8) ... n-+oo

Putting 0 = 71"/2, this gives, on use of the formula cos(0/2) = )(1 + cosO)/2, the following result, giving 2/71" as a limit of a sequence, which was first given by Fancois Viet a in 1593:

2/71" = )(1/2 + 1/2)(1/2)). (1/2 + 1/2)(1/2 + 1/2)(1/2))) .....

Page 24: Surprises and Counterexamples in Real Function Theory

1.3 The numbers e and 1f 15

The convergence of this expression was proved by F.Rudio in lS91 (see [9S]). Vieta's formula is the first analytical expression ever obtained for 1f.

Now, taking logarithms in the Euler's formula above (noting that the loga­rithm is a continuous function):

log sin B -10gB = log cos B/2 + log cos B/4 + 10gcosB/S + ...

On differentiating with respect to B, this gives

l/B = cot B + (1/2) tan(B /2) + (1/4) tan(B /4) + (l/S) tan(B /S) + ...

Putting B = 1f / 4, we obtain

4/1f = 1 + (1/2)tan(1f/S) + (1/4)tan(1f/16) + (1/S)tan(1f/32) + ...

The calculation of the number 1f by this formula is equivalent to a geometrical calculation. It was undertaken by Rene Descartes in the seventeenth century.

There have since been many formulae giving 1f as a rapidly converging series. These make use of various methods, as for example Euler's summation formula, f-function and the Taylor's series for tan-1 x. We give here an arctan series formula for the calculation of 1f.

3. The arctan method. The formula

(i)

can be derived as follows: Let a be the angle given by tana = 1/5; then tan2a = 2tana/(1 -

tan2 a) = 5/12 and tan4a = 2tan2a/(1 - tan2 2a) = 120/119 = tan(1f/4) + 1/119. Thus 40: > 1f/4, say 40: - 1f/4 = (3, so that tan(3 = (tan40:­tan1f/4)/(1 + tan 4a tan 1f/4) = 1/239, which finally gives 4a - 1f/4 = (3 = tan-l (1/239), i.e., 1f/4 = 4tan- 1 (1/5) - tan- 1 (1/239), as required. This for­mula was discovered by John Machin in 1706.

We also have the expression 1/(1 + x 2) = 1 - x 2 + x4 - x 6 + ... ,Ix 1< l. As the above series is uniformly convergent, we may integrate term by term to get

00

tan- 1 x = 2) _1)nx2n+l /(2n + 1) (lxl<l) (ii) n==O

This is the series discovered by James Gregory in 1671, which we shall obtain in Chapter 7. For x = 1, it gives the Leibnitz's celebrated series 1f/4 = 1 -1/3 + 1/5 - ... which requires 2000 terms to give three decimal figures of 1f. Using (i) and (ii), we get

1f = (16/5)(1 - 1/(3·25) + 1/(5.252 ) - 1/(7.253 ) + ... ) -(4/(239))(1-1/(3·57121) + 1/(5.571212) - 1/(7.571213) + ... )

Page 25: Surprises and Counterexamples in Real Function Theory

16 Chapter 1

which is well suited for the calculation of Jr. In 1706, using this series, Machin did the computation to 100 decimal digits. Similar series for arctan have been used by many to compute Jr accurately. The reader is referred to pages 90~96 of [22].

In 1882, Lindemann proved that Jr is transcendental (see [74]). Here we give Niven's ingenious proof (see [75]) that Jr is irrational.

Theorem 1.3.7. The real number Jr is irrational.

Proof. Suppose Jr = alb (a, bEN). Define polynomials f(x) = xn(a-bx)n/n! and g(x) = f(x) - f"(x) + f(4)(x) _ ... + (-1)nf(2n)(x), where the positive integer n will be chosen later. Now,

n! f(x) = xn (an + (~) an- 1( -bx) + (~) an~2( -bx)2 + ... + (-bX)n)

= anxn _ (~) an~lbxn+l + (~) an~2b2xn+2 _ ... + (_1)nbnx2n.

Differentiating successively we get

n !1'(x) =nanxn- 1 - (~) (n + l)an~lbxn + G) (n + 2)an- 2b2xn+l _ ... +

(_1)nbn2nx2n- 1 ,

n !f"(x) =n(n - 1)anxn-2 -(~}n + l)nan~lbxn-l+

+ (~}n + 2)(n + 1)an- 2b2 xn - ... + (-1)nbn2n(2n - 1)x2n~2 ,

The last equation is the only one which is free of x.It follows that f(O)=1'(O)= ... f(2n~1) (0) = 0, while n !f(2n) (0) = (2n)! (_l)n bn, so that f(2n) (0) is an integer. In any case:

f(O) , l' (0) , ... , f(2n) (0) are all integers. (1)

Further, it can be easily checked that f(x) = f(a/b - x) for all x and so differentiating successively, we get 1'(x) = - 1'(a/b - x), f"(x) = + f"(a/b -x), .... Hence f(Jr) = f(a/b - Jr) = f(O) = 0, 1'(Jr) = 1'(0) = 0, ... , f(2n-l)(Jr) = f(2n)(0) = ((2n)!/n!)(-1)nbn E Z. In any case

(2)

Page 26: Surprises and Counterexamples in Real Function Theory

1.3 The numbers e and 7r 17

Note that in (1) and (2), only j(2n)(0) and j(2n)(7r) are nonzero, the rest are in fact zero. Now,

d(g'(X) sinx - g(x) cos x) '() . I/() (). '( ) dx =g x cosx+smxg x +g x smx-cosxg x

= (g(x) +gl/(x))sinx.

But 9 = j - j(2) + j(4) - ... + (_1)n j(2n), and so gl/ = j(2) - j(4) + j(6) _ ... + (_1)nj(2n+2). Adding, we get 9 + gl/ = j + (_l)n j(2n+2) = j, as j(2n) = (-1)nbn(2n)!/n!, and so j(2n+l) = j(2n+2) = O. Thus d(9'(x)sin~;;g(x)cosx)

j(x) sinx. Hence

fa" j ( x) sin x d x = [9' ( x) sin x - 9 ( x) cos x] :

= g(7r) + g(O)

= j(7r) - j(2)(7r) + ... + (_1)nj(2n)(7r) + j(O) - j(2)(0) + ... + (_1)nj(2n)(0)

= (_1)n(j(2n)(7r) + j(2n)(0)) (the rest being all zero) ,

= 2(-1)n(2n)!bn /n!,

which is an integer, since (2n)! In! is an integer. Thus

fa7r j (x) sin x dx is an integer. (3)

However, for 0 < x < 7r, we have 0 < j(x) sin x < 7rnan/n!2 2n , because, j(x) = xn(a - bx)n/n!; hence f'(x) = xn.n(a - bx)n-I(-b) + (a - bx)n.nxn-I/n! = O,i.e.,xn-I(a - bx)n-I(-bx + (a - bx)) = 0, giving x = 0, or alb = 7r, or a/2b = 7r /2. But as 0 < x < 7r, the only relevant solution is x = a/2b = 7r /2. The value at 0 and 7r is 0, so that 7r /2 is a maximum and this maximum value is j(a/2b) sin 7r/2 = (a/2b)n(a - b.a/2b)n In! = (a/2b)n(an)/22nn! = 7rnan /22nn!. Now, 7rnan /2 2nn! -+ 0 as n -+ 00, so if we choose n large enough, we can ensure that 0 < j(x) sin x < 1/5, and so

0< fa" j(x) sinxdx < 7r/5 < 1,

which is a contradiction to (3). This proves that 7r is irrational. We now turn to the transcendence of e and 7r, which is considerably more

difficult to establish than their irrationality. A complex number is said to be algebraic ij, as in the case oj real numbers,

it is a mot oj some non-constant polynomial with rational coefficients. The transcendence of e and 7r can be deduced from:

Theorem 1.3.8 (Lindemann). Ij AI, ... ,An; aI, ... , an are algebraic num­bers (possibly complex), ai :j: aj jor i :j: j, Ai :j: 0 jor all i, then L~I Ai eai :j: O.

Page 27: Surprises and Counterexamples in Real Function Theory

18 Chapter 1

For a proof of this result, the reader is referred to [101].

Corollary 1.3.9. The number e is transcendental.

Proof. Take arbitrary AI, ... ,An E Ql, then Ale + A2e2 + ... + Anen -::j:. O.

Corollary 1.3.10. The number 71 is transcendental.

Proof. If 71 is algebraic, then, as observed in Remark 1.2.13, 7f~ is algebraic (as ~ obviously is) and hence by Theorem 1.3.8, e'" + eO -::j:. 0, but e'rL = -1 and eO=l; which gives a contradiction.

It is not known whether the numbers 7fe and 71 + e are transcendental or algebraic. However, we have the following interesting:

Theorem 1.3.11 ([17]). At least one of the numbers 7fe and 71 + e is tran­scendental.

Proof. Suppose to the contrary, that both 7fe and 71 + e are algebraic. By Remark 1.2.13, the number A = (71 + e)2 - 47fe = (71 - e)2 is algebraic. It is easy to see that if a 2 is an algebraic number, then so is a. Thus 71 - e and hence (71 + e) + (71 - e) = 271 would be algebraic, which is not true.

§1.4. The Cantor ternary set

In this section we describe and establish some properties of the Cantor ternary set, which is a source of several counter examples in real function the­ory. As a preparation, we first discuss some results in point set topology.

Theorem 1.4.1. Let 0 be an open and bounded subset of lE.. Then 0 can be uniquely expressed as a disjoint union of (at most) countably many open intervals.

Proof. Let a EO. Since 0 is open, there exist an open interval (a - E, a + E) ~ O. Now consider the closed set S = (- 00, a] nO c (here 0 C denotes the comple­ment of 0), i.e., the points not in 0, but lying to the left of a. Let a = sup S, which we know belongs to S, since S is closed and bounded above. It follows that (a, a] ~ O.

Similarly, let (3 = inf([a, (0) nO C), so that (3 is the smallest element of [a, (0) n Oc. Then again [a,(3) ~ O. Hence (a,(3) ~ O,a,(3 ~ 0, i.e., (a,(3) is the' largest' open interval contained in 0 which contains the point a. It is called a component interval of 0 and is clearly uniquely determined by a (or indeed by any other point of (a, (3) as the starting point instead of the point a).

Thus, 0 is a disjoint union of open intervals, the component intervals. To see that they are at most countable, we select a rational number in each of them. Being disjoint intervals, these rationals are distinct so that there is a 1-1 correspondence between these component intervals of 0 and a subset of the rationals.

Now let F be a closed, bounded nonempty set and let a = inf F, (3 = sup F, so that a, (3 E F, since F is closed. The interval [a, (3] is called the smallest

Page 28: Surprises and Counterexamples in Real Function Theory

1.4 The Cantor ternary set 19

closed interval containing F. Let 0 = [a,;3] "F. It is easy to see that 0 is open. Indeed, 0 = FC n [a,;3] = FC n (a,;3) (since a,;3 1- FC), which is an open set, since FC and (a,;3) are both open. It follows, by Theorem 1.4.1 that 0 is a disjoint union of at most countably many open intervals. These are called the complementary intervals of F (five of them are shown in Figure 1.2).

F 0 F 0 ~ 0 F 0 F 0 ~ ( ) ( ) e( ) ( ) ( ) e]

a F Ff3 Figure 1.2

Since F is closed, F', the set of all limit points of F, is contained in F. We ask: When is F ~ F', i.e., when is each point of F a limit point of F? Such closed subsets of IE. are called perfect sets. Now a point of F (indeed of any set) is either an isolated point (i. e., a point such that there exists some open interval centred at the point which contains no point of F other than itself) of F or a limit point of F. Thus, if F has no isolated point, then all points of F are limit points of F, i.e., F ~ F' ( i.e., F is dense in itself). Since F' ~ F (F being closed), we get F = F', i.e., F is perfect. We characterize all the isolated points of F in the following:

Theorem 1.4.2 Let F be a closed, bounded, nonempty subset @f IE.. A point I of F is an isolated point of F if and only if (i) I is a common end point of two complementary intervals of F, or (ii) I = a, where (a, a + c5) is a complementary interval of F, or (iii) 1=;3, where (;3 - c5',;3) is a complementary interval of F.

Remark 1.4.3. The theorem says that either points of the type y shown in Figure 1.3, i.e.,

( )e( x y

Figure 1.3

z

the common end points of the two complementary intervals (x, y) and (y, z) of F are isolated points of F or a is an isolated point of F if a complementary interval of F begins at a, i.e., is (a, a + c5)

a a+8 )

Figure 1.4

or ;3 is an isolated point of F if a complementary interval of F ends at ;3, i.e., is (;3 - c5',;3) and that there are no other isolated points of F.

Page 29: Surprises and Counterexamples in Real Function Theory

20 Chapter 1

8'

Figure 1.5

Proof of Theorem 1.4.2. The points mentioned in the theorem are clearly isolated points of F. Conversely, let, be an isolated point of F. Then there exists an E > 0 such that (r - E" + E) has no point of F except " i.e., (r - E, ,) ~ 0, where 0 is [a, jJ] " F, as defined above (proof to be modified for, = a) and (r" + E) ~ 0 (proof to be modified for, = /3), i.e., (r - E, ,) and (r" + E) are both subsets of two complementary intervals of F meeting at " as required.

For, = a or , = /3, a similar argument yields the result.

Theorem 1.4.4. The set S of all sequences of the type (al' a2, a3, ... ) where ai = 0 or 1, is uncountable.

Proof. Indeed there is a one-to-one correspondence between S and all the real numbers in [0,1], except for at most countably many exceptions. In fact, writing each real number r E [0,1] in its binary expansion, i.e., r = .ala2a3 ... (a; = 0,1), we see that the mapping r B (al,a2,a3, ... ) is a one to one, onto mapping between Sand [0,1]' the exceptions mentioned being those reals which have two expansions, viz. certain rational numbers as for example 1/2 = .011111 ... = .100000 ... , and being a subset of rational numbers, there are at most count ably many of these.

We are now in a position to define and establish some properties of the curious Cantor ternary set C. Let F = [0,1]. Now remove the middle third open interval (1/3,2/3). From the two remaining intervals, again remove the middle third open intervals, i.e., remove (1/9,2/9) and (7/9,8/9), and so on. What remains, is the Cantor ternary set C. Thus, for example, the points

0, 1; 1/3, 2/3; 1/9, 2/9, 7/9, 8/9; ...

will never be removed and therefore belong to C (see Figure 1.6):

[ ( ) ( ) ( ) ( ) ( ) ( ) ( ) ] ~ o 119 2/9 113 2/3 . 7/9 8/9 1

Figure 1.6

As C is a countable intersection of closed sets Fl = [0,1], F2 = [0, ~] U [~, 1], F3 = [0, i] u [~,~] U [~,~] U [~, 1], ... , it follows that C is a closed set. Since no two removed intervals are of the type (x, y), (y, z), we see that C has no isolated point (see Theorem 1.4.2); in fact, we get:

Theorem 1.4.5. The subset C of IE. is perfect, i.e., C' = C.

To understand C more clearly, it is essential to look at the ternary expansion

Page 30: Surprises and Counterexamples in Real Function Theory

1.4 The Cantor ternary set 21

of real numbers. Let us therefore start first with a digression on the decimal expansion of real numbers T E [0,1].

Each T has a unique decimal expansion: T = .TIT2T3 ... (Ti = 0,1 ,2, ... ,9), except those T for which Ti = 0 for all i 2: io, for some io or for which Ti = 9 for all i 2: i~ for some i~. For example:

1/2 = .500000 ... = .499999 ... i.e. Ti = 0 or 9 for i 2: 2 ;

1/4 = .250000 ... = .249999 ... i.e. T i = 0 or 9 for i 2: 3 ;

1/125 = .008000 ... = .007999 ... i.e. Ti = 0 or 9 for i 2: 4 ;

etc. Such numbers are a subset of the rational numbers and therefore countable. Note further that

• the numbers in the interval [0,1/10] will have an expansion of the type T = .OT2T3 ... , beginning with a O. For example, 1/10 = .09999 ... (= .10000 ... ) , 1/11 = .09090 ... , 1/12 = .08333 ... , etc. ,

• the numbers in the interval [1/10,2/10] have an expansion of the type T = .lT2T3 . . . , beginning with a 1. For example 1/10 = .1000 ... (this is in [0,1/10] as well as in [1/10,2/10]),1/5 = .1999 ... (= .2000 ... ) etc. ,

and so on. Finally,

• the numbers in the interval [9/10, 1] have an expansion of the type T = .9T2T3 .•. , beginning with a 9.

Next we divide each of [0,1/10], [1/10,2/10], ... , [9/10,1] into ten equal parts again. Take the interval [0,1/10]' which gets divided into [0,1/100]' [1/100,2/100]' ... , [9/100,10/100]. Here the numbers in the first part have a o in the second place of their decimal expansion, those in the second part have a 1 in the second place and so on; and similarly for the third place, ... .

Keeping this in mind, we now work in ternary expansions, so that we have just three digits 0,1,2. Here each number in [0,1/3] will begin its ternary expansion with a 0, while numbers in [1/3,2/3] begin with a 1, and numbers in [2/3,1] with a 2. Note that 1/3 and 2/3 both fall into two categories .

.00 .. . 01... . 02 ... . 10 ... . 11. . . . 12 ... . 20 ... . 21.. . .22 ...

• ( ) ( • • ) ( ) • 0 1/9 2/9 113 4/9 5/9 2/3 7/9 8/9

Figure 1.7

Next when we divide [0,1/3] into three equal parts, the numbers in [0,1/9] will have a 0 in the second place, those in [1/9,2/9] a 1 in the second place and those in [2/9, 1/3] a 2 in the second place and so on; and so on for the third , fourth, ... , place. The full chart, up to nine parts, is as in Figure 1.7. Looking at the above chart, we easily get the following:

Page 31: Surprises and Counterexamples in Real Function Theory

22 Chapter 1

Theorem 1.4.6. In the construction of the Cantor set C, the removed intervals consist of those numbers of [0, IJ that have a 1 somewhere in their ternary expansion, i.e., C is the set of those real numbers in [0,1] that can be written with only 0 's and 2 's in their ternary expansion.

We now have the following bijective mappings:

C B {r E [0,1]1 r = .r1r2 ... ri = 0 or 2}

B {s E [0,1]1 s = .SlS2 ... Si = 0 or I} (replacing 2's by l's)

B {( Sl, S2, ... ) I Si = 0 or I} = S (in the notation of Theorem 1.4.4)

B all real numbers in [0,1] except countably many. (by Theorem 1.4.4)

Hence we get the following:

Theorem 1.4.7. The Cantor ternary set C is uncountable.

This would seem even more surprising when we note that the total of the lengths of the intervals removed in the construction of C is equal to 1. Indeed, the removed lengths equal 1/3 + 2 . 1/9 + 4 . 1/27 + ... = (1/3) . (1 + (2/3) + (2/3)2 + ... ) = (1/3)(1/(1- 2/3)) = 1/(3 - 2) = 1. So it would seem that the set of leftover points ( i.e., the set C) is rather small (as far as set inclusions go or in the language of measure theory, not discussed here, C is of "measure zero") compared to the set of all real numbers in [0,1]' but is not as small as it appears to be, at first sight, as Theorem 1.4.7 shows.

Remark 1.4.8. Since the end points of the removed open intervals form a countable set, it follows that C contains points other than these end points. In ternary expansion, these end points of the removed open intervals are .0222 ... , .2000 ... , .00222 ... , .02000 ... , .2022 ... , .22000 ... , ... , i.e., with either all O's or all 2's after a stage. An example of a point of C, which is not amongst the count ably many listed above is .20202020 ....

Although C is very thinly scattered over [0,1], the following even more surprising result holds:

Theorem 1.4.9. The set of distances between points of C fills the unit interval, i.e., given c E [0,1], there exist x,y E C such that y - x = c .

This theorem was first proved using geometric methods and published in the Polish journal Wektor in 1917, by H.Steinhauss. In 1940, Randolph gave a proof that appeared in [90]. There is plenty of literature on this subject. Here we give a very neat proof of a slightly more general result, which is due to W.R.Utz [112].

Theorem 1.4.10. Let c E [0,1] and let 1/3 ~I m I~ 3. Then there exist x, y E C such that y - mx = c .

Taking m = 1, we get Theorem 1.4.9.

Proof. Consider the Cantor set C on the x-axis as well as the y-axis. Let 2: consists of all points (x, y) of the unit square with corners (0,0), (0,1), (1, 1),

Page 32: Surprises and Counterexamples in Real Function Theory

1.4 The Cantor ternary set 23

(1, 0) such that x belongs to the Cantor subset C of [0,1] on the x-axis and y belongs to the Cantor subset C of [0,1] on the y-axis. The result will follow if we can show that for c, m, as above, the line y - mx = c meets ~ in at least one point (x, y).

We divide the unit square [0, 1] x [0, 1] = S into three equal vertical strips, by inserting two vertical line segments in S. Delete from S the interior of the middle strip as well as its top and bottom open segments (see Figure 1.8).

y

2 ......,......,..~-...., __ "" (l , I)

(0.0) (I , 0) x

Figure 1.8

Next delete the interior of a similar horizontal middle third strip so that there remains four closed corner squares, each of area 1/9 and separated by a region shaped like a cross. It is easily observed that:

• In case the general line A : y = mx + c meets S and if 1/3 ::::;1 m I::::; 3, then A meets at least one of the four smaller squares .

•• Indeed, the above observation is true of any square besides S, whose sides are parallel to the two axes.

Now start with the unit square S and let 1/3 ::::;1 m I::::; 3. If A : y = mx + c meets S, then by the first observation above, A meets at least one of the four corner squares that remain when the cross is removed from S. Call that square Sl. Since A meets Sl, we may repeat the above process (using the second observation) to get S2 (see Figure 1.8) such that A meets S2, and so on. This leads to a shrinking sequence of compact squares: S = So :J Sl :J S2 :J ... ; their diameters tending to zero, such that AnSi ::J 0. By Cantor intersection theorem (see [103] ), the intersection of all S;'s is nonempty. But the intersection of all S;'s is contained in ~, which completes the proof of the theorem.

Taking m = -1 in the above result yields:

Page 33: Surprises and Counterexamples in Real Function Theory

24 Chapter 1

Corollary 1.4.11. For c E [0,1], there exist x, y E C such that x + y = c.

Taking m = -1 and replacing c by 2c (so that the line A becomes y -x + 2c, and it still meets S for all c E (0,1)), we get:

Corollary 1.4.12. If c E (0,1), then c lies midway between two points x, y of c. Example 1.4.13. (A perfect nowhere dense set). A nowhere dense set is a set whose closure has no interior point. The above discussion shows that the Cantor ternary set C, constructed above is an example of a perfect nowhere dense subset of R

Exercises

1.1. Let E ~ IE. and 0 > O. For x E E, define

E(x,o) = En (x - 0, x + 0) .

(i) If for some 0, E(x,o) = {x}, then x is said to be an isolated point of E. (ii) If for some 0, E(x, 0) is an infinite set, but countable, then x is said to be a sparse point of E. (iii) If for all 0, E(x, 0) is uncountable, then x is said to be a dense point of E. By considering intervals with rational end points, prove that the set of isolated points of E and the set of sparse points of E are both countable.

1.2. Let E be any subset of R Prove that the following statements are equivalent: (i) Any bounded monotone sequence of points in E converge to a point of E. (ii) For any A ~ E (A i:- 0, A bounded), sup A and inf A both belongs to E. (iii) Every sequence {xn } of points of E satisfying

lim {sup{1 Xn - Xm II n,m 2': N}} = 0 N-+=

converge to a point in E.

1.3. Find a sequence {an} of real numbers such that an -+ 0 as n -+ 00, but nan converge to a transcendental number.

Page 34: Surprises and Counterexamples in Real Function Theory

Chapter 2

Functions: pathological, peculiar and extraordinary

§2.1. Some pathological functions

25

One of the first standard examples of a real-valued function that is dis­continuous everywhere is

f (x) = { 1, if x E Q -1, if x ~ Q

Example 2.1.1. The function defined above is discontinuous everywhere but its absolute value is continuous everywhere.

A slightly more difficult job is to construct an example of a function that is continuous at only one point (e.g. at x = 0) but discontinuous everywhere else. The following function is one such example:

f(x) = { x, ~f x E Q -x, if x ~ Q.

Now if one is asked to construct a function, defined in say [0,1], that is continuous at each irrational but discontinuous at each rational, it would require quite some thought. The function

{l/q , ifx=p/qEQ,x:f:O, (p,q)=l,

f(x)= 1, ifx=O

0, ifx~Q,

where (p, q) denotes the g.c.d. of p and q, serves the purpose. The proof goes as follows:

It is immediately clear that f is discontinuous at each rational, for whatever a rational Xo = p/q, (p, q) = 1, may be, there exist irrational numbers a as close to xoas we like and then given E > 0, I f(xo) - f(a) 1=11/q - ° 1= l/q, which is not less than any given E for I Xo - a I less than any J if E ~ l/q. Now to prove that f is continuous at each irrational a E [0,1]' we proceed as

Page 35: Surprises and Counterexamples in Real Function Theory

26 Chapter 2

follows: Let E > 0 be given. Define a set S of rational numbers by putting the rational p/q in S if and only if q :::; I/E and p/q E (0: - 1,0: + 1), i.e.,

S = {p/q E (0: - 1,0: + 1) 1 q = 1,2, ... ,[I/E]}.

Then S is obviously a finite set, as q takes finitely many values and so does p. Consequently, we may choose a o-neighbourhood of 0: which is free of elements of S (see Figure 2.1), i.e., there exists a 0 such that (0: - 0,0: + 8) n S = 0. Thus, given E > 0 we arrived at a 0 and we now show that 1 f(x) - f(o:) 1 < E if Ix-o:l< o.

• • • ( 8 • 8 • • S2 Sj a Sj+l

Figure 2.1

So let x E (0: - 0,0: + 0). If x is irrational, then 1 f(x) - f(o:) 1 = 0 < Eo If x is rational, then x, not being in S, has denominator larger than I/E, i.e., x =p/q, (p,q) = 1, q> I/E, so If(x) - f(o:) 1 = 11/q - 01 = l/q < Eo Thus, for all x E (0: - 0,0:+ 0), rational or irrational, we have 1 f(x) - f(o:) 1 < E, which means that f(x) is continuous at 0:.

Remark 2.1.2. The above proof, as well as a proof using sequences (rather than neighbourhoods, as in the above) are both given by J.H. Staib, in [105].

Remark 2.1.3. It is instructive to draw a rough graph of this function. First note that the function can be defined on whole of IE. by the same def­inition and that the function is periodic with period 1 in (0,00) and also in (-00,0), for if x is irrational, so is x + 1 and so f(x) = f(x + 1) = 0, while if x = p/q, gcd (p, q) = 1, then x + 1 = (p + q)/q, gcd (p + q, q) = 1, and again f(x + 1) = l/q = f(x) (here p may have any sign) and of course f(n) = 1 if n E N. Further, f(x) = f( -x), i.e., f is symmetrical about the y-axis, so it is enough to draw the graph of fin [0,1]. A rough sketch is given in Figure 2.2.

f(1I2)=1I2

t f (0) = 1

I i

o 116 115 114 113 2/5 3/5 2/3 3/4 4/5 5/6

Figure 2.2

Page 36: Surprises and Counterexamples in Real Function Theory

2.1 Some pathological functions 27

ReIllark 2.1.4. Classically, the function f given above has been defined to be o at x = O. This was because if we take a sequence Pn/qn of rationals tending to 0, then qn must tend to infinity and as such f(Pn/qn) = l/qn -t 0 hence it is natural to define f(O) = 0 so that f becomes continuous at x = O. However, o is a rational number and we want f to be discontinuous at x = 0, so it is even more natural to put f(O) = 1, say.

ExaIllple 2.1.5 The function defined above also serves as an example of a function which is continuous at uncountably many points (i.e., all irrational numbers) but is nowhere differentiable. To this end, it is clearly enough to look for points of differentiability in 0 ::; x ::; 1. Since f is not even continuous at nonzero rationals, it is enough to look at the irrationals in [0, 1] and the point O. We show that f is not differentiable at any of these points. First let x = O. Let {h;} be a sequence of irrationals with limit equal to 0, (e.g.J2 -1.4, J2 - 1.41, J2 - 1.414, ... ). Then lim(f(O + hi) - f(O)/h i) = lim( -l/hi), which does not exist, and so f(x) is not differentiable at x = O.

Let now x be an irrational in [0,1], so that 0 < x < 1. First choose a sequence {hi} of reals having limit 0 and such that x + hi is irrational for each i (e.g. hi = l/i). Then lim(f(x + hi) - f(x))/h i = lim(O/hi) = O.

Next suppose that x has decimal representation .a1a2 ... an .. ; and choose hi = .a1a2 ... ai - x (which is an irrational). Since x -:j:. 0 we have ai -:j:. 0 for some i and indeed for i as large as we like, for otherwise, x would be a rational. Let N be the least (first) integer such that aN -:j:. 0, i.e.,

N -1 zeros ~

X =.000 . .. 00aNaN+1 ...

Then f(x + hi) = f(.a1a2 ... ai) 2':: 1/10i if i 2':: N , for .a1a2··· ai = al/lO + a2/102 + ... + ad10i = (a110i-1 + ... + ai)/10i and so f(.a1a2 ... ai) 2':: l/lO i

(note that any cancellation in this fraction (only 2 or 5 can be cancelled) will reduce the denominator and so increase the value of I), while 1 hi I::; l/lOi for 1 hi 1= .00 ... OaH1··. = 0/1O+0/102+···+0/lOi+ai+1/10H1+ ... ::; l/lOi. It follows that (f (x + hi) - f (x)) / hi 2':: 1 if i 2':: N and so f is not differentiable at x, by the observation in the last paragraph.

This example qualifies as pathological, at least to a beginner and clearly reveals the power of careful analytic reasoning.

ExaIllple 2.1.6. The above example thus gives us a function f defined on the entire real line IR which is continuous at each irrational point of IR but discontinuous at each rational point. Note that the set of discontinuities for f is a dense set and that each of the points of discontinuities is removable. Because if the function is redefined at a to have the value 0, (also for a = 0) then since limx-+a f(x) = 0 = f(a), as redefined, f becomes continuous at a. Another observation about the function f is that it gives the example of a function which is everywhere finite but everywhere locally unbounded. For, if f were bounded in an E-neighbourhood N of a, then for all min in N, the denominator n would be bounded, and hence the numerator m would be too.

Page 37: Surprises and Counterexamples in Real Function Theory

28 Chapter 2

But this would permit only finitely many rational numbers in the interval N, which is not true.

Since a function, continuous at all irrational numbers but discontinuous at all rational numbers exist, it is tempting to believe that a function that is continuous at each rational but discontinuous at each irrational also exists. However the surprise here is that no such function can exist! A beautiful elementary proof of this curious result was given in 1881 by Vito Volterra. We have:

Theorem 2.1.7 Let (a, b) be any open interval (possibly (-00,00)). There exists no function g: ( a, b) ~ lR that is continuous at each rational number but discontinuous at each irrational number.

Proof (Volterra). Suppose to the contrary that such a function 9 exists, defined on an open interval (a, b). Let f be the function constructed above, that is continuous at the irrational numbers but discontinuous at the rational numbers. Let Xo be a rational in ( a, b) and take a = 1. By the continuity of 9 at xo, there exists a 8 > 0 such that

(xo - 8,xo + 8) c (a, b) , 1 g(x) - g(xo) 1< 1/2 for all x E (xo - 8,xo +8) .

Choose c < d so that [c, d] is a closed subinterval of (xo - 8, Xo + 8). Then for any x,y E [c,d] ,

1 g(x) - g(y) 1 :::; 1 g(x) - g(xo) 1 + 1 g(xo) - g(y) 1< 1/2 + 1/2 = 1 = a.

Now, take any irrational ~ in (c, d), so that f is continuous at ~ and by the preceding argument, there exist al < bl , with [aI, btl C (c, d) such that 1 f(x) - f(y) 1< 1 for all x, y E [aI, bl ]. To summarize then, for all x, y E [aI, bl ],

(note the inclusions [aI, bl ] C (c, d) C (xo - 8, Xo + 8) C (a, b)), we have 1 g(x) - g(y) 1< 1 and 1 f(x) - f(y) 1< 1 (remember a = 1).

We next repeat this argument, starting with the open interval (aI, br) and taking a = 1/2, then again a = 1/4, ... ,and generally a = 1/2n. This generates a strictly decreasing sequence of closed intervals (except for (a, b)):

(a,b):::J [al,bl]:::J [a2,b2] :::J ... :::J [an,bn] :::J ...

such that for all x, y E [an, bn], we have 1 g(x) - g(y) 1< 1/2n and 1 f(x) -f(y) 1< 1/2n. The denseness property of the rationals and irrationals plays a central role at each step, providing at least one point of continuity for 9 and f alternatively in any open subinterval. By the nested interval property, there exists at least one point p contained in all the [ai, bi ] and thus both 9 and fare continuous at p, giving a contradiction, since 9 is continuous where f is not or vice versa.

An immediate corollary to this is the following:

Theorem 2.1.8. There can exist no continuous function 'f! : lR ~ lR, mapping the rational numbers to irrationals and the irrational numbers to rationals.

Page 38: Surprises and Counterexamples in Real Function Theory

2.2 The Cantor function 29

Proof. Consider the function f which is continuous at each irrational but discontinuous at each rational. Suppose, to the contrary, that a continuous function cp: lR ---+ lR exists that maps rationals to the irrationals and the irra­tionals to the rationals. Let h(x) = f(cp(x)). Then for x E Q, cp(x) is irrational and so f is continuous at cp(x) and cp is continuous everywhere by hypothesis; hence we observe that h is continuous at each x E Q.

For an irrational number y, let {xn } be a sequence of rationals that tends to y. Then limn--+oo h(xn ) = limn--+oo f(cp(x n )) = 0, since Xn E Q implies that cp(xn ) is irrational and hence f(cp(x n )) = 0. However, h(y) = f(cp(y)) "1O, since y is irrational and so cp(y) is ra,tional. Hence h is discontinuous at each irrational number. These observations contradict Theorem 2.1.7, thus completing the proof.

For some historical remarks on the difficulties Volterra encountered see [31].

Remark 2.1.9. A direct proof of Theorem 2.1.8, using cardinality arguments goes as follows: Suppose to the contrary that an everywhere continuous func­tion cp: lR ---+ lR exists with cp(Q) <;;:; 1I and cp(lI) <;;:; Q. It follows that cp takes at least two values, one rational and one irrational, say p and a. By the in­termediate value theorem, since cp is continuous, cp assumes all values in [p, a], where without loss of generality, p < a, and since p "I a, we get that the set [p, a] is uncountable. On the other hand, the set {cp(Q)} is countable, since Q is countable. Further, since cp(lI) C Q , cp(lI) is countable and consequently cp(lR) is countable. This contradiction shows that cp can not exist.

Remark 2.1.10. This paragraph requires some knowledge of measure theory. We have constructed a function f that is continuous at the irrationals and discontinuous at the rationals, i.e., points of continuity of f constitute a set of measure 1 while points of discontinuity of f form a set of measure 0. Af­ter Volterra's theorem, it may be rather surprising to note that there do exist functions h(x) with these characteristics reversed, i.e., points of continuity of h constitute a set of measure ° while points of discontinuity of h form a set of measure 1. The interested reader may look up [110].

§2.2. The Cantor function

Recall that in §1.4 we constructed the Cantor ternary set C. We now give certain labels to the intervals left out after each stage, as follows:

First step: On deleting the open interval (1/3,2/3) from I = [0,1], we are left with the two intervals h = [0,1/3] and h = [2/3,1]' each of length 1/3.

Second step : On deleting the open middle thirds (1/9,2/9) from 11 and (7/9,8/9) from h, we are left with 22 = 4 intervals III = [0,1/9]' h2 = [2/9,3/9]' h1 = [6/9,7/9],122 = [8/9,9/9]' each oflength (1/3)2,

and so on, i.e., by induction, we complete the construction as follows:

Page 39: Surprises and Counterexamples in Real Function Theory

30 Chapter 2

Induction step: Suppose the kth step is complete (k EN), i.e., on removal of the open middle thirds of 2k- 1 intervals, there remain 2k closed intervals, each of length (1/3)k. Then the (k + l)th step consists in removing the open middle thirds of these 2k intervals to be left with 2k+l closed intervals, each of length (1/3)k+l.

We continue this process indefinitely and define C to be the points of I that do not get removed. Example of such points are the end points of each removed interval viz:

{1/3, 2/3; 1/9,2/9,7/9,8/9; 1/27,2/27, ... ; ... } (*)

which form only a countable subset of C. More precisely, if Fl = [0, 1], F2 = [ 1] [2 ] _ [ 1] [2 3 3 n -1_1 0'3 U 3,1, ... , Fn - 0'3n-1 U 3n-I'3n-I]U ... U[~,lJ, then C = n~IFn'

3/4 / ...................................................................................................................................................................................................................................... ~----,

1/2 ......... .

114········ ········-1 --. U2 = 2/9, (32 = 1/3

x

+ o ]/9 2/9 ]/3 2/3 7/9 8/9

Figure 2.3

We are now in a position to define the Cantor function <f> : [0,1] --+ [0,1]. Define <f>(0) = 0, <f>(1) = 1, and <f>(x) = 1/2 if 1/3 ::::: x ::::: 2/3. This defines <f>

at the end points of h,h and on the deleted open interval (1/3,2/3). Next, define <f>(x) = 1/4 if 1/9 ::::: x ::::: 2/9 and <f>(x) = 3/4 if 7/9 ::::: x ::::: 8/9 (see Figure 2.3). We repeat the process indefinitely, thereby defining <f> on each deleted interval and its end points and at 0,1, i.e., <f> is defined on each deleted open interval and at countable set (*) defined above and at 0 and l.

Let now x be a point at which <f> is not defined (i.e., let x E C). Then for each kEN, x belongs to the interior of exactly one of the 2k left over intervals of length (1/3)k after the kth step of our construction of C. Call this interval [Cl:k,;Jk]' Being of length (1/3)k, we have;Jk = Cl:k + (1/3)k. Further, <f> has been defined at the end points of these left over intervals, viz. , <f>(;JJ.:) =

Page 40: Surprises and Counterexamples in Real Function Theory

2.2 The Cantor function 31

<I>(ak) + (1/2k). This should best become clear by an example (see Figure 2.3): <I>(a2) = 1/4, <I>(/32) = 1/2, and <I>(/32) = <I> (a2) + (1/2)2, or 1/2 = 1/4 + 1/4, as required. Now when the (k + l)th deletion takes place, [ak' /3k] is subdivided into three equal parts and the middle third is removed. Then, x belongs to one of the two leftovers [ak+l, /3k+1], say .

• • • • x ~k+l

Figure 2.4

Hence:

We now define <I> at x by :

(:j:)

which is natural. Indeed, by (t), <I>(ak) is increasing and is bounded above by 1, <I>(/3k) is decreasing and is bounded below by 0, so both sequences {<I>(ak)} and {<I>(/3d} tend to limits and since <I>(/3k) = <I>(ad + (1/2k), these limits are equal and this common value is defined to be <I>(x). This defines <I> on [0,1] as a continuous, nondecreasing function with values in [0,1]. Moreover <I> is con­tinuous in [0,1], as it is clearly continuous at each point of the deleted intervals and at there end points too, as a little thought will show. For continuity at points of C, the defining equation (:j:) is equivalent to the continuity of <I> at xl We have thus proved (i) and (ii) of the following

Theorem 2.2.1 The Cantor function <I>: [0,1] -+ [0,1] with <I>(O) = 0, <I>(1) = 1, has the following properties:

(i) <I> is continuous, (ii) <I> is monotone increasing (not strictly increasing), (iii) <I> is not differentiable at any point x of C, (iv) <I> is differentiable at each point x which is not in C and for such x,

<I>'(x) = 0.

Proof. (i) and (ii) have been proved, while (iv) is trivial since x ~ C implies that x is in a removed interval and hence is constant in a neighbourhood of x. Thus <I>'(x) exists and is 0. We now proceed to prove (iii).

At each step of deletion of open intervals, a part of the Cantor function gets defined. Let us join the end points of this partially defined function <I>, at each step, by straight lines. Thus, for k = 1, we have the graph as in Figure 2.5 and for k = 2, the graph is shown in Figure 2.6.

Page 41: Surprises and Counterexamples in Real Function Theory

32 Chapter 2

3/41·················································· ............................................. __

112 1/21············································ /'--../

X, x, = 1/3 Xl = 2/3 x, = 1 X,

Figure 2.5 Figure 2.6

Here note that the horizontal graph (i.e., the actual if» continues undisturbed, once it appears, whereas the other parts consist of 2k congruent portions with positive slope. Indeed, for k = 1, the slope equals (1/2)/(1/3) = 3/2 while for k = 2, the slope is (1/4)/(1/9) = 9/4 = (3/2)2, and so on.

Now to prove (iii). Since x E C, so x is in a closed left over interval [X2i-i, X2i] at each step of deletion. It follows that if>(X2i) - if>(X2i-d = 1/2k; but X2i - X2i-i = 1/3k. These give: if>(X2i) - if>(X2i-d = (3/2)k(X2i - X2i-d. Now recall the inequality (easy to check): max(a//3,,/J) 2': (a + ,)/(/3 + J), for a" 2': 0; /3, J > O. Using this, we get

max[(<I>(x2i) - <I>(X))/(X2i - x), (<I> (x) - <I>(X2i-d)/(x - x2i-dl 2: (3/2)k ,

where x E (X2i-i, X2i)' So either the left or the right derivative of if> at x can not exist, i.e., if> is not differentiable at x.

Example 2.2.2. The above discussion also yields an example of a nowhere dense set A of real numbers and a continuous mapping of A onto the closed unit interval [0,1]. Let A be the Cantor set C defined previously. Let x E C, and let 0,C1 C2C3 ..• be its ternary expansion, where Cn = 0 or 2 for n = 1,2, ... and let

Ci C2 C3 ¢(x)=0'222'" ,

where the expansion on the right is to be regarded as a binary expansion in terms of the digits 0 and 1. Clearly ¢(C) is a subset of [0,1]. To see that [0,1] <;;; ¢(C), choose an arbitrary y E [0,1] and a binary expansion of y:

y = O.b i b2 b3 ..•

Then x = 0.(2bi )(2b2 )(2b3 ) ... ,

evaluatecl in the ternary system, is a point of C such that ¢(x) = y. Continuity of ¢ is not difficult to establish, using geometric considerations given in the above description of the Cantor function.

Page 42: Surprises and Counterexamples in Real Function Theory

2.3 The length of the Cantor function 33

§2.3. The length of the Cantor function

The Cantor function <I> has another very astonishing property, viz., the length (from ° to 1) of its graph is 2. Before we prove this, we need to define what we mean by the length of a curve that is continuous and nondecreasing (i.e., increasing but not strictly increasing). Denote by G f the graph of the function f: [a, b] -t JR.. The range of f is then [j(a), f(b)]. We define the length >..( G f) of G f by the equation

>"(Gf) = sup [L: ((Xi - xi_d2 + (f(Xi) - f(Xi_1)2)1/2] , !"

where p is any partition: a = Xl < X2 < ... < Xn-l < Xn = b of [a, b]. Since for a, (3 E JR., (a2 + (32)1/2 :::; I a I + I (3 I (with strict inequality when a, (3 are both non-zero), we have ((x;-xi-d 2+(f(Xi)- f(x;_d)2)1/2 < (Xi-Xi-d+(f(Xi)­f(Xi-d) and so L:((Xi - Xi_1)2 + (f(Xi) - f(Xi_d?)1/2 :::; b - a + f(b) - f(a).

This proves the following

Lemma 2.3.1. If >"(Gf) denotes the length of the graph Gf, then >"(Gf ) :::; b - a + f(b) - f(a). Moreover, if f is a continuous nondecreasing function defined from [0,1] onto itself, with f(O) = 0, f(l) = 1, then >"(Gf) :::; 2 .

Suppose now that f is continuously differentiable (i.e., that the derivative f' is continuous in [0,1]). Using the Mean Value Theorem, we get:

Lemma 2.3.2. Let f be a continuous nondecreasing function defined from [0, 1] onto itself, with f(O) = 0, f(l) = 1. If f'(x) exists in [0,1] and is continuous throughout, then >.. (G f) < 2.

Proof. Since f is nondecreasing, l' :::: 0. If l' were equal to ° throughout [0,1], then f(x) = k and >'(Gf ) = 1. Next, f'(x) can not be larger than 1 throughout [0,1], nor it can be less than 1 throughout [0,1] (see Figures 2.7 and 2.8).

o a i3 Figure 2.7 Figure 2.8 Figure 2.9

So, there exists /, J > ° and an interval [a, (3] C (0,1) such that (see Figure 2.9)

° < /:::; f'(t) :::; J < 1 (for all t E [a,(3]) (1)

Page 43: Surprises and Counterexamples in Real Function Theory

34 Chapter 2

Denote by )<1, A2, A3, the lengths of the parts of G f' lying respectively above [0, a], [a,,8], [,8,lJ. Then A(G f) = Al + A2 + A3 and the obvious estimates (Lemma 2.3.1) yields:

Al :::; a - 0 + f(a) - f(,8) = a + f(a) ,

A2 :::; ,8 - a + f (,8) - f (a) ,

A3 :::; 1 - ,3 + f(l) - f(,8) = 2 - ,8 - f(,8) ,

which only imply that A(Gf) :::; 2. However, we shall show that A2 :::; A((,8 -a) + (j(,8) - f(a))), where 0 < A < 1, and that would prove the Lemma.

Now, ~ E b,b] gives 1 + ~ = (1 + ~)1/2(1 + ~)1/2 ~ (1 + ,)1/2(1 + e)1/2, since ~ ~" ~ ~ e, i.e.,

(2)

with A = (1 + ,)-1/2 < 1. Hence, when a :::; u < v :::; ,8, there exists a point T) E (u, v), such that

f(v) - f(u) = 1'(T))(v - u)

and of course l' (T)) E b, bJ (see (1)). Hence

((v - U)2+(j(V) - f(U))2)1/2 =((v - U)2 + 1'2 (T))(v _ u)2)1/2

= (v - u){l + 1'2 (T))}1/2

:::; (v - u)A(l + 1'(T)))

= A(v - u)(l + (j(v) - f(u))/(v - u))

= A.(v - u + f(v) - f(u)) , (4)

(3)

(by (3))

(by (2))

(by (3))

with A < 1. Now let a = to < t1 < ... < tn-l < tn = ,8 be any partition of [a,,8]. Then

n " ( 2 . 2) ]/2 ~ (ti - ti-d + (j(ti) - }(ti-r)) ;=1

n

:::; LA (ti - ti-l + f(ti) - f(t.i-l)) ;=1

= A((,8 - a) + (j(,8) - f(a)))

:::; A(,8 - a) + (j(,8) - f(a))

(with A < 1, by (4))

(with A < 1)

(asA<l).

Taking supremum, it follows that A2 ::; A(,8 - a) + (j(,8) - f(a)) and hence A2 < (,8-a)+(j(,8)- f(a)), as 0 < A < 1. It now follows that Al +A2+A3 < 2, as required.

Remark 2.3.3. It is not straightforward to produce an example of an in­creasing or even a nondecreasing continuous function f on [0, 1J (or indeed any

Page 44: Surprises and Counterexamples in Real Function Theory

2.3 The length of the Cantor function 35

increasing function f) with >.( G I) = 2. By Lemma 2.3.2, such a function, if constructed, can not be continuously differentiable. The following example, of a continuously differentiable function, goes part of the way towards this. Let in(x) = xn, 0::::: x::::: 1. The graphs of these in's are shown in Figure 2.10. As n increases, the graph flattens out to nearing a horizontal line, almost up to 1.

B

o A

Figure 2.10

o p

Figure 2.11

Q

It is clear from the figure that >'(G(fn)) increases with n and gradually ap­proaches the length 0 A + 0 B = 2, but it really never makes it to 0 A + 0 B as in --+ i, where i(x) = ° if ° ::::: x < 1 and i(l) = 1, and so the best we can say is that given E > 0, there exists a natural number N such that >.( G In) > 2 - E

if n :::: N. As E --+ 0, N --+ 00 and it follows that

(6)

However, lim>. (G In) :j:. >. (lim G I J, for the left hand side is greater than or equal to 2 (by (6)), whereas, the limit of Gin is the graph of j, which is discontinuous at 1 and so >'(limGln) = 1. To improve on (6), we proceed as follows (see Figure 2.11).

Let ° < P < 1. Then clearly >'(G In) = Arc OP + Arc PQ > p + 1 - in(P), and letting n --+ 00, this gives that lim>'(Gln) :::: 1 + p, provided P < 1. Thus

L = lim >.( G In) :::: P + 1 for all ° < P < 1 n-HX)

(7)

We may now give the following:

Corollary 2.3.4. Let in(x) = xn in [0,1]. Then limn-+oo >'(G/J = 2.

Proof. Let L = lim >'(GIN)' If L < 2, say L = 2 - 6 (6) 0), it would follow, by (7) above that 2 - 6 :::: P + 1 for all ° < P < 1, i.e., that P ::::: 1 - 6 for all ° < P < 1, which is false if p is chosen to be larger than 1 - 6.

Example 2.3.5. A more artificial example of a continuous, nondecreasing function if(X) on [0,1] with >'(G/J > 2 - E is the one shown in Figure 2.12; or indeed, a continuous strictly increasing function, with this property, is shown in Figure 2.13 below.

Page 45: Surprises and Counterexamples in Real Function Theory

36 Chapter 2

o

Graph of feCx)

A

Figure 2.12

B

p o

B B

p o A

Figure 2.13 Figure 2.14

In Figure 2.12, >..(GfJ = OA + AB = 1 - E + AB > 1 - E + BP = 2 - E, as required.

However, as E -+ 0, fE(X) -+ f(x) = 0 in [0,1); 1 at x = 1 (see Figure 2.14), and the length of this function is 1.

Similar conditions hold in Figure 2.13, and indeed, if we want a continuous and differentiable, strictly increasing function fE(X), we may round off the cor­ners in Figure 2.13 and get what is required.

We are now in a position to prove the following:

Theorem 2.3.6. For the Cantor function <I>(x) defined on the closed interval [0,1]' the length >"(Gq,) of its graph Gq, is equal to 2.

Proof. In view of Lemma 2.3.1, it is enough to prove that >"(G.p) ;:::: 2. To this end, choose a positive integer k and compute the approximation of the length for the partition

Pk : 0 = Xl < X2 < ... < X2k+1 = 1 ,

consisting of end points Xi of the 2k intervals that remain after the kth step of our construction of C. Thus, for example, for k = 1, Pl is given by

1 2 Pl : 0 = Xl < X2 = - < X3 = - < X4 = 1

3 3

and for k = 2, P2 is given by (see Figure 2.6)

1 2 1 2 7 8 P2 . 0 = Xl < X2 = - < X3 = - < X4 = - < Xs = - < X6 = - < X7 = - < Xs = l.

. 9 9 3 3 9 9

Call the corresponding sum approximating >"(Gq,) to be L k . Then

2 k +1

Lk = L ((Xi - Xi_d 2 + (<I>(Xi) - <I>(xi_d)2) 1/2 .

i=2

Here Lk is the length of the approximating polygon. We now have to calculate the value of Xi - Xi-l and of <I>(Xi) - <I>(xi-d for all i and each k. Looking at

Page 46: Surprises and Counterexamples in Real Function Theory

2.4 Some more curious functions 37

the cases k = 1,2 (Figures 2.5 and 2.6), we see that the contribution of the horizontal portions in Lk equals

((2/3 - 1/3)2 + 0)1/2 + ((2/9 - 1/9)2 + 0?/2 + ((8/9 - 7/9)2 + 0)1/2 + ...

Observe that the first term is equal to L 1 , sum of the first three terms is equal to L 2 , and so on. Thus, the contribution is

1/3 + 1/9 + 1/9 + ... = 1/3 + 2/9 + 4/27 + ... + 2k - 1 /3 k

= (1/3)(1 + (2/3) + (2/3)2 + ... + (2/3)k-1)

= 1 - (2/3)k.

The other portion of the graph consists of 2k congruent portions with slope 3/2 for k = 1, (3/2)2 for k = 2, (3/2)3 for k = 3 and so on. Then the contri­bution to Lk from this portion equals 2k((1/3k)2 + (1/2k)2)1/2 = (2 2k (1/3 2k + 1/22k ))1/2 = ((2/3)2k + 1)1/2. So now Lk = 1- (2/3)k + (1 + (2/3)2k)1/2, and since A(Gq,) = supLf,), we see that this contribution is less than or equal to A(Gq,). Letting k -+ 00, we get limLk = 1 + 1 = 2::; A(Gq,).

§2.4. Some more curious functions

It is not surprising that the Cantor Function <l>(x) has its derivative equal to 0 almost everywhere, since on all the removed intervals, it is, by definition, horizontal. As such, it fails to be a strictly increasing function. For a strictly increasing function, one would think it to be impossible that the derivative is 0 almost everywhere and the existence of such a function therefore, would definitely come as a surprise to most. \Ve now describe such a function (see [108]), i.e., we construct:

Example 2.4.1. A function 8(x) on the closed interval [0,1] which is (i) continuous therein, (ii) strictly increasing, with 8(0) = 0,8(1) = 1 , (iii) 8'(x) = 0 wherever 8'(x) exists and is finite.

Let x E (0,1] and write x in binary form as:

00

x = 1/2ao + 1/2a1 + 1/2a2 + ... = L 1/2ar T=O

Now define 8(0) = 0, and 8(x) = L~o pT /(1 + p)ar , where p is a positive real number. For x = 1, we have 1 = 1/2 + 1/22 + 1/23 +"', so that aT = r + 1 for each r 2 o. It follows that 8(1) = 1/(1 + p) + p/(1 + p)2 + p2 /(1 + p)3 + ... = 1, on summing a geometric series. Thus 8(0) = 0, 8(1) = 1, as required in (ii) above.

Now let 0 < x ::; 1. Then by the definition of e, 0 = e(O) < e(x). Let

Page 47: Surprises and Counterexamples in Real Function Theory

38 Chapter 2

o < x < Y :S 1 and write

:r = 1/2ao + 1/2a1 + 1/2a2 + ... , (ao < al < a2 ... ),

Y = 1/2bo + 1/2b1 + 1/2b2 + ... , (bo < b1 < b2 ... ).

Let s be the smallest integer such that as i=- bs (i.e., ao = bo, a1 = b1 ,

... ,as-1 = bs- 1, as i=- bs). Since x < y, we see that as > bs and we have

8(x) = 1/(1 + p)ao + p/(l + p)a 1 + ... + ps-l /(1 + p)a S - 1 + pS /(1 + p)as + .. . 8(y) = 1/(1 + p)bo + p/(l + p)b1 + ... + ps-l /(1 + p)b S - 1 + pS /(1 + p/s + .. .

Here the first s terms are equal and since bs < as, we have 1/(1 + p)bs > 1/(1 + p)as. It follows that 8(y) > 8(x) (even if p = 1), i.e., we have thus proved (ii) above, i.e., that 8(x) is strictly increasing in [0,1].

Next, for x = 1/2ao + 1/2a1 + ... and n 2': 0, let Xn = L:ar < n 1/2ar and let Yn = Xn + 1/2n. Then Xn :S x :S Yn, because Xn :S x is trivial, while x = Xn + L: 1/2ar (summation over aT > n) :S Xn + 1/2n +1 + 1/2,,+2 + ... = Xn + 1/2" = Yn' Let k" be the number of those suffixes for which ai :S n. Then

pkn

8(y,,) - 8(x,,) = (1 + p)" (t)

where clearly 0 :S k" :S n (note that even if we allow ar's to be 0, 0 :S k" :S n + 1). This is best seen by the following:

(i) Take, for example,

x = 1/22 + 1/23 + 1/25 + 1/26 + ... = 0/21 + 1/22 + 1/23 + 0/24 + 1/25 + 1/26 + ... = .011011 ... ,

and so ao = 2, al = 3, a2 = 5, a3 = 6,.... Take n = 5, say; then X5 L:ar < 5 1/2ar = 1/22 + 1/23 + 1/25 , and Y5 = 1/22 + 1/23 + 1/25 + 1/25 = 1/22+ 1/23 + 2/25 = 1/22 + 1/23 + 1/24. So now 8(Y5) = 1/(1 + p)2 + p/(l + p)3 + p2/(1 + p)4 and 8(X5) = 1/(1 + p)2 + p/(l + p)3 + p2/(1 + p)5. Hence 8(Y5) - 8(X5) = p2 /(1 + p)4 _ p2 /(1 + p)5 = p3 /(1 + p)5 = pks /(1 + p)5, where k5 = 3 is equal to the number of 7", 0 :S r :S 5, with aT :S 5.

(ii) If all the terms in x are present, say, x = 1/2 + 1/22 + 1/23 + 1/24 + ... = .1111111 ... , then

X5 = 1/2 + 1/22 + 1/23 + 1/24 + 1/25 ,

Y5 = 1/2 + 1/22 + 1/23 + 1/24 + 1/25 + 1/25

= 1/2 + 1/22 + 1/23 + 1/24 + 2/25

= 1/2 + 1/22 + 1/23 + (1/24 + 1/24)

= 1/2 + 1/22 + 1/23 + 1/23

= 1.

Page 48: Surprises and Counterexamples in Real Function Theory

2.4 Some more curious functions 39

Then 0(Y5) = 1, 0(X5) = 1/(1 + p) + p/(1 + p)2 + ... + p4/(1 + p)5, and so, after simplifications, 0(Y5) - 0(X5) = p5/(1 + p)5 = pkn /(1 + p)5, where kn is the number of r's such that ar ::; 5, which is 5 in this case. It now follows that

pkn

lim (0(Yn) - 0(xn)) = lim ( ) = 0, n--+oo n--+oo 1 + P n

since for p < 1, we have 1/(1 + p)n ~ 0 and pkn is bounded (even if kn does not tend to 0), while if p > 1, we have pkn /(1 + p)n ::; pn /(1 + p)n = 1/(1 + 1/ p)n ~ O. So 0 is continuous in (0,1]. As for continuity at x = 0, note that lim;r--+o 0(x) = ° (since as x ~ 0, more and more of the ar's in the representation x = L: 1/2ar are zero) and as 0(0) = 0,0 is continuous at x = ° too. Finally if p = 1, we have 0(x) = L:~o F /(1 + 1)ar = L:~o 1/2ar = x and hence 0 is continuous at 1. We have thus proved (i) above, i.e., that 0 is continuous in [0,1].

Let now P"l1. We prove the curious result that if, for a point a E [0,1], 0'(a) exists, then 0'(a) = O.

Using (t) above, we get (0(Yn) - 0(xn))/(Yn - xn) = (pkn /(1 + p)n).2n, where Xn < a < Yn and where {xn}, {Yn} are sequences constructed above for a point a. Since 0'(a) = limn--+oo(0(Yn) - 0(xn))/(Yn ~ xn), we see that limn--+oo pkn .2n /(1 + p)n exists and is finite and we have to show that this limit is 0 (under the assumption that p "I 1). Suppose that the limit in question is not O. We get a contradiction as follows: We know that if lim an is finite and nonzero, then lim(an/an-d = 1. It follows, then, that lim(pkn ((2/(1 + p))n)/pkn-l((2/(1 + p))n-1)) = 1, i.e., that

Since p ¥c 1, this means lim(kn - kn-d exists and equals k, say. But now an -+ A implies that (a1 + a2 + ... + an)/n -+ A. This gives that (k2 - k1 + k3 - k2 + ... + kn - kn-d/n -+ k, i.e., (kn - k1)/n -+ k, i.e. kn/n -+ k. But o ::; kn ::; n, so kn/n -+ 0 or 1, depending on how kn behaves as n ~ 00, i.e., k = 0 or 1. Now use (:j:) to give pO = (1 + p)/2 or 1 = (1 + p)/2, i.e., p = 1, giving a contradiction.

Remark 2.4.3. This remark requires some knowledge of measure theory. The property (iii) above says that either 0 ' (x) does not exist or if it does, it is 0. By Lebesgue's theorem [97], 0' (x) does exist and is finite almost everywhere, since o is an increasing function. It follows that 0 '(x) = 0, almost everywhere.

It is worthwhile to compare this function with the Cantor function. In the latter, <l>(x) is a horizontal line in (1/3,2/3); (1/9,2/9), (7/9,8/9); ... ; i.e., almost on the entire line and as such, the fact that <l>'(x) = ° almost everywhere, is obvious. Further, <l> is increasing, but not strictly increasing. In the present case, the function 0 is strictly increasing and still 0' (x) = ° almost everywhere. The reader is advised to make a rough sketch of the function 0. He should plot points with x = 1/2,1/4,3/4, ... and take, say, p = 1/2. For

Page 49: Surprises and Counterexamples in Real Function Theory

40 Chapter 2

x = 1/2, aD = 1, al = a2 = a3 = ... = 0, but this does not satisfy the requirements given in the beginning, i.e., aD < al < a2 < .... The way to do this is to write 1/2 = 1/4 + 1/8 + 1/16 + ... = 1/22 + 1/23 + 1/24 + ... , giving aD = 2, al = 3, a2 = 4, .... Hence 0(1/2) = L p')(1 + p)ar = 1/(1 + p) = 2/3 (on summing the geometric progression, since we are taking p = 1/2). Similarly for points x = 3/4 or x = 1/8.

A much shorter, but slightly more difficult example, that does all the above, is given by Freilich in [36].

There is no dearth of functions that are peculiar in one way or the other. There is one more example that we would like to take up in detail, which we think, is quite curious. The function n in question has the following astounding properties:

• n (x) is differentiable (and therefore continuous) for all real x .

• n(x) is monotone in no interval (however small).

We now give a series of lemmas that gradually build up to our required function n (also see [54]).

Lemma 2.4.4. Let r, s E lit (i) r > s > O::::} (r - s)/(r2 - S2) < 2/r , (ii) r,s > I::::} (r + s - 2)/(r2 + s2 - 2) < 2/s.

Proof. Easy.

Lemma 2.4.5. Let tp(x) = 1/ v(1+ 1 x I), x E R Then, for two real numbers a, b, a i- b,

(b ~ a) lb tp(x) dx < 4min{tp(a), tp(b)}.

Proof. Without loss of generality we may suppose a < b (otherwise consider the interval [b, a] and the integral from b to a equals the negative of the integral from a to b).

Case 1: 0::; a < b. Then by Lemma 2.4.4

(b ~ a) lb tp(x) dx = (b ~ a) lb v(~: x) (since 1 x 1= x as O::;a < b)

= 2 (J(T+b) -V(1 + a))) (putting x = tan2 B) (b - a)

= 2 (J(T+b) -V(1 +a))) (l+b)-(I+a)

4 < (by Lemma 2.4.4) J(T+b) = 4min{tp(a),tp(b)} ,

Page 50: Surprises and Counterexamples in Real Function Theory

2.4 Some more curious functions 41

since '{J is decreasing, so the minimum is at b. Case 2: a < b ::::; 0. Now '{J( -x) = '{J(x), so changing x to -x, we land up in Case 1, with '{J unchanged and so Case 2 follows.

Case 3: a < ° < b. In this case J: dx/J(l+ I x I) = J~ dx/J(l - x) +

J~ dx/J(l + x). Evaluating by the substitution x = sin2 (), x = tan2 () re­

spectively, this equals 2( ~ - 1) + 2( J(f+b) - 1). Hence

I-_1_1b dx -2 (/f+b+VI=a-2) - b-a a J1+lxl- (1+b)+(1-a)-2

4 <---

VI=a'

again by Lemma 2.4.4. But I is also equal to 2( VI=a+ /f+b- 2)/((1- a) + (1+b)-2). It follows that 1< 4min{1/VI=a , 1//f+b} = 4min{'{J(a),'{J(b)}. This completes the proof of the Lemma.

The graph r of'{J is drawn in Figure 2.15. At the point (0,1), it is easy to check that the right hand derivative equals -1, while the left hand derivative equals +1. Thus '{J is not differentiable in (0,1) and the slope ofr (on the right and the left) is shown in the Figure 2.15. As x ---+ ±oo, y ---+ 0.

y (0, 1)

%n: (0, %)

-3 -1 o 2 3

Figure 2.15

Lelllllla 2.4.6. Let'{J be as in Lemma 2.4.5 and let

n

1jJ(x) = :~.::>i'{J(Ai(X - (};i)) i=l

where C1, C2, ... ,cn ; AI, A2, ... ,An are positive real numbers; (};1, (};2, ... ,(};n are any real numbers. Then

1 lb (b _ a) a 1jJ(x) d x < 4 min{1/J(a) , 1jJ(b)} ,

where a, b are distinct real numbers.

Page 51: Surprises and Counterexamples in Real Function Theory

42 Chapter 2

Proof. On putting t = ).(x - a), we see that

1 lb 1 l.A(b-a) tp(t) dt - tp().(x-a))dx=- --b - a a b - a .A(a-a) ).

1 l.ACb-a) = tp(t)dt.

)'(b - a) - ).(a - a) .A(a-a)

Now, Lemma 2.4.5 immediately gives the result.

Figure 2.16

Remark 2.4.7. To understand the function 1/J, it is helpful to build it up in stages:

1. The function tp(x - a) is just the translate of tp(x) by an amount of a.

2. For the function tp()'x) , taking). = 2,3,4, ... , the graph looks like the following: The right hand gradient of tp().x) , at x = 0, is equal to -)., which tends to -00 as ). --+ 00, as is clear from Figure 2.16. Similarly, the left hand gradient is equal to +00.

3. The function ctp(x) has graph with each ordinate multiplied by c.

Then 1/J is just a finite sum of such functions ctp().(x - a)).

Lemma 2.4.8. Let 1/Jl, 1/J2,'" be any sequence of functions as in Lemma 2.4.6. For any real number x and any positive integer n, define

(*)

Suppose that L::l1/Jn(a) is convergent to, say, s < 00, for some real number a. Then (i) the series L~=l '¥n(x) = L~=l u; 1/Jn(t)dt) is uniformly convergent on every bounded subset ofJR, with limit function F(x), say; (ii) F(x) is differentiable at x = a and FI(a) = s(i.e., FI(a) = L~=l '¥ I(a) = L~=l1/Jn(a) = s, by hypothesis (*)). In particular, ifL~=l 1/Jn(t) = f(t) < 00,

for all t E JR, then F is differentiable everywhere on JR and FI = f.

Page 52: Surprises and Counterexamples in Real Function Theory

2.4 Some more curious functions 43

Proof. Let b be a real number with b 21 a I. For -b ::; x ::; b, we have

which, by Lemma 2.4.6, is less than or equal to 41al min{ 1Pn(a), 1Pn(O)} + 41x­al. min{'l/'n(x) , 1Pn(a)} = 4Ial1Pn(a) + 41x - al·1Pn(a) ::; 12b.1Pn(a), by looking at the graph of 1Pn and since min{ 1Pn(x), 1Pn(a)} ::; 1Pn(a). By Weierstrass' M-test, uniform convergence on [-b, b] now follows.

To prove F'(a) = s, let E> 0 be given. Choose N such that

00

10 L 1Pn(a) < E/2 . (1) n=N+l

Since each 1Pn(x) is continuous at x = a, there exists a 6> 0, such that for all 0< Ihl < 6, we have

1

11a+

h I

h a 1Pn(t)dt - 1Pn(a) < E/2N, (2)

for all n = 1,2, ... ,N. Using Lemma 2.4.6 again, we have, for 0 < Ihl < 6,

I F(a + hl- F(a) - sl = I~ { (~la+h 1Pn(t)dt) - 1Pn(a)} I ::; t, I (~ la+h 1Pn(t)dt) - 1Pn(a) I

:J;:+l { (~ la+h 1Pn(t)dt) + 1Pn(a)}

CX)

< E/2 + L 51Pn(a), (by (2) and as by Lemma 2.4.6, n=N+l

< E/2 + E/2 = E

This completes the proof of Lemma 2.4.8.

k Jaa+h 1Pn(t)dt ::; 41Pn(a)).

(by (1)).

Lemma 2.4.9. Let h,I2, ... ,In be disjoint open intervals and let (Xi be the mid points of the Ii. Let E; d1 , d2, ... , dn be positive real numbers. Then there exists a function 1P(x), as in Lemma 2.4.6, such that foreach i, we have

(i) 1P((Xi) > di ,

(ii) 1P(x) < di + E , if x E Ii ,

(iii) 1P(x) <E, if x~hUIzU ... UIn.

Page 53: Surprises and Counterexamples in Real Function Theory

44 Chapter 2

Proof. Choose Ci = di + f/2 and write <pi(X) = Ci<P(>'i(X - ai)), where the function <P is as defined in Lemma 2.4.5 and Ai are chosen so large that

(3)

(one only needs to check this inequality at an end point of h since here, the quantity x - ai is least). Now take 'lj;(x) = <PI (x) + <P2(X) + ... + <Pn(x). Since the Ii are disjoint and since <Pi(X) takes its maximum value Ci at ai, properties (i), (ii), (iii) are easy to verify. Indeed, take i = 1, say. Then

'lj;(ad = <PI (ad + <p2(ad + .. . = Cl<P(O) + <p2(ad + .. . = (d1 + f/2)<p(O) + <P2(al) + ... > d1 ,

(4)

since <p(O) = 1 and since each <P is greater than 0 everywhere. This yields (i).

y

y=dn

A rough sketch of \If

----~~r-----~--~+---------r_--------~+_----~~~x

o U2

Figure 2.17

Also,

'lj;(ad ::::; (d1 + f/2).1 + f/2n + f/2n + ... + f/2n < d1 + f (by (3)), , , .. n-l terms

which proves (ii).

Page 54: Surprises and Counterexamples in Real Function Theory

2.4 Some more curious functions 45

For (iii), observe that if x ~ Ui~ll;, then as 'PI takes its maximum value Cl at al and the rest of 'P's are, in any case, less than E/2n, we have

'l/J(X) ~ E/2n + E/2n + ... + E/2n < Eo

Theorem 2.4.10. Let {ad and {,8i}, i = 1,2,3, ... , be disjoint countable subsets of lit Then there exists a real valued, everywhere differentiable function F( x) on JR., satisfying the following properties: (i) F'(ai) = 1 , (ii) F' (,8i) < 1 for all i , (iii) 0 < F'(x) ~ 1, for all x.

Proof. We construct the partial sums fn(x) = L~=l 'l/Jk(X) with the following three properties, by induction on n:

An : fn(ai) > I-I/n for all i = 1,2, ... ,n;

En : fn(x) < 1 - I/(n + 1) for all x E JR. ;

Cn : 'l/Jn(,8i) < I/2n.2n ,for all i = 1,2, ... ,n .

First, let n = 1. Choose an open interval h, with midpoint al such that ,81 ~ h. By Lemma 2.4.9, with E = dl = 1/4 so that Cl = dl + E/2 = 3/8, we obtain JI(x) = 'l/Jl(X), which satisfies: (i) 'l/Jl(ad > 1/4, (ii) 'l/Jl(X) < 1/4+ 1/4 = 1/2 if x E h, (iii) 'l/Jl(X) < 1/4 if x ~ h. Here (i) implies that 'l/Jl(ad > 0, which gives AI, while (ii) and (iii) imply that 'l/Jl(X) < 1/2 for all x, which is E l . Finally (iii) is Cl and now our induction starts.

We next have the induction step. Let n > 1 and suppose fn-l(X) = L~:i 'l/Jk(X) has been constructed to satisfy An-I, En-I, Cn-l. Select dis­joint open intervals h, h, ... ,In such that for each i E {I, 2, ... ,n}, ai is the mid-point of Ii and Ii n {,8l, ,82, ... ,,8n} = I/) and

(5)

for all x E Ii and where Ii = I/(n(n + 1)) - I/2n.2n > O. Now apply Lemma 2.4.9, with

E = I/2n.2n ,di = I-I/n - fn-l(ai) ,i = 1,2, ... ,n (6)

to obtain 'l/Jn(x), and take fn(x) = L~=l 'l/Jk(X), We now verify An, En, Cn for these functions 'l/Jn(x) and fn(x). Cn follows trivially; for, according to (iii) of Lemma 2.4.9, 'l/Jn(x) < E if

x ~ h U ... U In and since ,8i ~ h U ... U In for all i = 1,2,3, ... ,n, we get 'l/Jn (,8i) < E = I/2n.2n for all i = 1,2, ... n, giving Cn, as required.

Next, to check An, observe that for i = 1,2, ... ,n,

fn(ad = fn-l(ai) + 'l/Jn(ai)

> fn-l(ai) + di (by (i) of Lemma 2.4.9)

= 1 - I/n (by (6)).

Page 55: Surprises and Counterexamples in Real Function Theory

46 Chapter 2

Finally, to check Property En, if X E Ii (for some i), then we have

in (X) =in-l(X) + '!f;n(X)

< (fn-l(ai) + b) + (d i + E)

= (fn-l(ai) + di ) + b + E

= 1 - l/n + l/(n(n + 1))

= l-l/(n+ 1);

(by (5) and by (ii) of Lemma 2.4.9)

(on rewriting)

. 1 (by (6) and smce b = ( )

n n+ 1 1 1

- 2n.2n ' E = 2n.2n)

while if x 1:. hUh U ... In, then

in(x) = in-l (X) + '!f;n(X)

< l-l/n + E

< 1-1/(n+1)

(by E n - l and by (ii) of Lemma 2.4.9)

(since E = 1/2n.2n).

This verifies En.

Construction of the function F(x) on the real line, as required in the theorem, can now be achieved. By Lemma 2.4.8, limn -4oo 2:~=1 '!f;k(X) = limn -4oo in(x) exists (call it FI(x)) and is the term-by-term derivative of F(:r) = 2:~=1 \[!n(x), i.e., FI(x) = limn -4oo in(x). Hence:

(i) FI(ai) = limn -4oo in (ai) 2 1, by An, and also::; 1 by En; giving FI(a;) = 1, as required.

(ii) 0 < FI(x) for all x, since FI(x) = 2:~1 \[!~(x) = 2:~=1 '!f;n(x) > 0, as each '!f;n(x) > 0; also FI(x) = limn -4oo in(x) ::; 1, by En.

(iii) Choose n > i. We have

00

k=l

n-l 00

= 2: '!f;dfJi) + 2: '!f;kC,6i ) k=l k=n

00

k=n 00

< 1 - l/n + 2: 1/2k.2k (by En and by en) k=n

< 1 - l/n + (1/2n).1

< 1-1/2n < 1,

(estimating the geometric progression)

Page 56: Surprises and Counterexamples in Real Function Theory

2.4 Some more curious functions 47

which finally completes the proof of Theorem 2.4.10.

Theorem 2.4.10 gives an elegant construction of a function n(x), satisfying the most unexpected properties mentioned earlier.

Example 2.4.11. There exists a real valued function 12 such that

(i) 12 is differentiable for all real x and 12' is bounded for all real x,

(ii) 12 is not monotone in any subinterval, however small, of the real line.

Indeed, let {ad and {,8i}, i = 1,2,3, ... , be disjoint dense subsets of the real line (e.g., the {ad could be the set of all rationals and the {,8i} the set of rationals + a fixed irrational such as V2, say). Using Theorem 2.4.10 construct a function F on JR such that

(i) F'(ai) = 1 for all i ,

(i i) F' (,8i) < 1 for all i ,

(iii) 0 < F'(x) ~ 1 for all real x ,

and a function G on JR such that

(i)' G'(ai) < 1 for all i ,

(ii)' G' (,8i) = 1 for all i ,

(iii)' 0 < G'(x) :::; 1 for all real x ,

and write n(x) = F(x) - G(x). Then

n'(ai) = F'(ai) - G'(ai) > 0,

12' (,6i ) = F' (,8i) - G' (,8i) < 0,

-1 < n'(x) = F'(x) - G'(x) < 1,

(using (i) and (i)')

(using (ii) and (ii)')

(using (iii) and (iii)')

(7)

(8)

(9)

In particilar, (9) shows that n'(x) is bounded. Finally, since the sets {ai} and {,8d are both dense, 12 can not be monotone in any interval.

This function 12 has many more remarkable properties; we give a few here:

Corollary 2.4.12. n(x) has a local maximum and a local minimum in every interval of JR.

Proof. Let [a, b] be any interval, a < b. Choose a,,8 E [a, b] from our dense subsets, a < ,8; then n'(a) > 0, 12'(,8) < O. It follows that n(x) takes an absolute maximum value in [a,,8] at some I E (a, ,8). Hence n(r) is a local maximum value for O(x).

Page 57: Surprises and Counterexamples in Real Function Theory

48 Chapter 2

/ ~_=_n_(x_) __ ~ ____ __

a a b

Figure 2.18

Similarly there exists a local minimum.

Corollary 2.4.13. (For readers familiar with Lebesgue integration). The func­tion n' (x) is Lebesgue integrable but not Riemann integrable.

Proof. (i) We first note that the function n(x) is absolutely continuous in [a, b], for, let a = Xo < Xl < ... < J~n-1 < Xn be any set of points in [a, b] such that

n

~)Xi - Xi-r) < 6. i=l

By the first mean value theorem, n(Xi) - n(Xi-r) = (Xi - Xi-r)n'(~i)' Xi-1 < ~i < Xi· It follows that

n n n

L In(Xi) - n(xi-dl :::: L(Xi - xi-dln'(~i)1 :::: L(Xi - Xi-r) < 6, i=l i=l i=l

since In'(x)1 < 1. Thus n(x) is absolutely continuous on [a,b], as claimed. Hence by results from the theory of Lebesgue integration (see [97]), it follows that n' (x) is Lebesgue integrable, as required; and indeed,

n(X) = n(a) + l x n'(t)dt (10)

Next, we claim that n'(x) is not Riemann integrable in [a, b]. Suppose, if possible, that n'(x) is Riemann integrable in [a, b]. Then by Lebesgue's well-known theorem (see [97]), n'ex) is continuous almost everywhere in [a, b]. However, if n'(t) is continuous at t, then it is easy to see that n'(t) = 0 (for, say n'(t) > 0; then there exists a neighbourhood (t - 6, t + 6) of t, throughout which, n'(x) > O. However, there exists a (3 of our dense set in (t - 6, t + 6) at which n' ((3) < 0, a contradiction).

Thus n'(t) = 0 almost everywhere on [a, b]. Hence by (10), n(x) = n(a) for all X E [a, b], i.e., n(x) is a constant, which gives a contradiction, since n is not monotone anywhere.

Next, we would like to mention an example of a function f: IE. --+ IE. with a proper local maximum at each point of a set S that is everywhere dense in IE. (for example S could be the set of rational numbers Q).

Page 58: Surprises and Counterexamples in Real Function Theory

2.4 Some more curious functions 49

Definition 2.4.14. We say that a function f : IR ---+ IR has a proper local maximum at a point x = a, if there exists an open neighbourhood N(a) of a, such that f(x) < f(a) for all x E N(a) ,,{a}.

It is easy to see (A.Schoenfties, Die Entwickelung der Lehre von den punk­tmannigfaltigkeiten, Bericht, Erstattet der Dentschen Mathematiker - Vereini­gung, 1900) that for any function f, the set S of points at which f has a proper local maximum, is at most countable. Indeed, let B be the set of all open in­tervals in IR with rational end points. For each a E S, pick a neighbourhood N(a) E B such that f(x) < f(a) for all x E N(a) " {a}. As B is countable, S is at most countable.

Example 2.4.15. It is trivial to sketch the graph of a continuous function f: IR ---+ IR which has a proper local maximum at infinitely many points in [0,1], say at each of the points a = 1,1/2,1/3, ... ; see Figure 2.19.

o 114 113 1/2 3/2 2

Figure 2.19

This brings up the question:

• Does there exist a continuous function f : IR ---+ IR, which has a proper local maximum at each point of a countable dense set?

It is not easy to visualize the graph of such a continuous function, but the answer to the question is known to be yes; see [81], page 63, Theorem 3. For a straightforward and simple example, also see [82].

Finally, we record some more examples/ counterexamples in the spirit of the above discussion.

Example 2.4.16. Let f be a function defined on the compact set [0,1] by

{ (_l)nn

f(x) = --n+1' 0,

if x is a rational, x = ~ in lowest terms, n > ° if x is irrational.

Then in any neighbourhood of every point of [0,1], the values of f come ar­bitrary close to the number 1 and -1 while always lying between them. This

Page 59: Surprises and Counterexamples in Real Function Theory

50 Chapter 2

gives an example of a bounded function having no relative extreme value on a compact domain.

Example 2.4.17 [37]. We now give an example of a monotonic function whose points of discontinuity form an arbitrary countable set.

Let A be any arbitrary nonempty countable set of real numbers aI, a2, ...

and let LPn be a finite or convergent infinite series of positive numbers with sum p. If A is bounded below and :1: is less than every point of A, let f(x) = O. Otherwise, define f(x) to be the sum of all terms Pm of LPn such that am ::; x. The function f is increasing on lEt continuous at every point not in A, and discontinuous with a jump equal to Pn at each point an, i.e.,

lim f(x) - lim f(x) = Pn. x---+a n + x---+a n -

Example 2.4.18 (R.E.Megginson; Math. Mag., 57(1984), 112). This is an example of a one-to-one onto function f : lE. -+ lE. which is continuous at some a E lE., but f- 1 is not continuous at f(a). Let

{

I ·f 1 N 2n' 1 X = n' n E

f(x)= ~, ~fx:3,5,7, .. . 2' 1f x - 2,4,6, .. .

x , otherwise

i.e., f(x) = x, except when x = 1,2,3, ... ; 1/2,1/3,1/4, ... ; but for 1,1/2, 1/3,1/4, ... , the image under f is 1/2,1/4,1/6,1/8, ... , respectively; and for 3,5,7, ... , the image under f is 1/3,1/5,1/7, ... respectively; and for 2,4,6, ... , the image is 1,2,3, ... respectively. So the set of values 1,2,3, ... ; 1/2,1/3,1/4, ... are just reshuffled by f and so, since everywhere else f(x) = x, we see that f is onto. So f- 1 exists and f is continuous at 0 (check !). However, f- 1 is not continuous at f(O) = 0, since f-1(1/n) = n for n = 3,5, 7, ....

§2.5. Algebraic functions

Definition 2.5.1. A function f defined on a subset E oflE. is called an algebraic function if for some positive integer n, there exists a sequence Po(x), P1 (x), ... , Pn(x) of polynomials such that for each x E E, the following relation holds:

For such a function f, there exists a least positive integer Tn such that the degrees of each of the polynomials Po(x), P1 (x), ... ,Pn(x) is less than or equal to Tn. We call (*), with this minimal pair n,Tn the minimum (*). Our object is to prove the following

Theorem 2.5.2. There is no algebraic function f whose derivative exists and is 1/ x f07' each x in the domain of definition of f.

Page 60: Surprises and Counterexamples in Real Function Theory

Exercises 51

Proof (H.R.Rollse [96]). Suppose that such a function f exists. Since l' (x) = 1/ x, 0 does not belongs to the domain of definition of f. Now differ­entiate minimum (*) with respect to x:

{P6(x) + P{(x)f(x) + p~(X)f2(X) + ... + p~(x)r(x)} + {PI (x)f' (x) + 2P2(x)f(x)f'(x) + ... + nPn(x)r- I(x)f'(x)} = O.

Since 1'(x) = l/x, this gives

{P6(x) + P{(x)f(x) + p~(X)f2(X) + ... + p~(x)r(x)} +{PI(x) + 2P2 (x)f(x) + ... + nPn(x)r- I(x)}.(l/x) = O.

Multiply this by x and regroup to get

(xp6(x) + Pdx)) + (xP{(x) + 2P2 (x))f(x)

+ (xP~(x) + 3P3 (x))f2(X) + ... + (XP~_I(X) +nPn(x))r- I (x) +xp~(x)r(x) = 0 m

Now (rt) shows that there exists an equation (*) with the same n, m but such that the last polynomial has a factor x. So without loss of generality, let (*) be such that x I Pn(x). But then (rt) implies that the last but one term, viz. xP~_l (x) + nPn(x) also has a factor x, since Pn(x) has factor x, and so on. Repeating this argument n times, we see that we may assume that each of Po (x), ... ,Pn (x) in (*) has a factor x. But 0 does not belong to the domain of f, so (*) can be divided by x, which reduces the degree of each Pi(x), i.e., m can be reduced, contradicting the assumption that m is minimal.

Definition 2.5.3. Ifn = 1, this minimum (*) becomes Po(x)+Pdx)f(x) = O. Such an f is called a rational function .

By an argument, similar to the one above, it can be proved that there is no rational function ¢ such that ¢'(x) = l/x.

An immediate application of Theorem 2.5.2 is the following

Proposition 2.5.4. The function log x is not algebraic.

Proof. The function log x has derivative equal to l/x in its domain of defini­tion, namely the set x > O.

Remark 2.5.4. It can also be shown that there is no nonzero algebraic func­tion f for which l' (x) = f (x) for all x belonging to the domain of definition of f.

Exercises

2.1. Let f be a continuous function on [a, b]. Prove that if s, t are distinct values taken by f in [a, b], then

6(S, t) = inf{lx - yl I f(x) = s, f(y) = t}

Page 61: Surprises and Counterexamples in Real Function Theory

52 Chapter 2

is positive. Prove that if r, s, t are values taken by f in [a, b], where r < s < t, then J(r,s):::; J(r,t). Prove further that J(r,s) < J(r,t).

2.2. Let f be defined on the entire real line and for J > 0, define

rp(x,J) = sup{lf(x)- f(y)1 I x-J < y < x+J} and 'ljJ(J) = sup{rp(x,J)lx E ~}.

Prove that

(i) 'ljJ is nondecreasing,

(ii) 'ljJ is bounded,

(iii) 'ljJ( J) ---+ 0 as J ---+ 0 if and only if f is uniformly continuous on lit

2.3. Let f be defined on~. Prove that the following statements are equivalent:

(i) For each sequence {xn }, convergent to a, the sequence {f(x n )} is conver­gent,

(ii) For some A, f(x) ---+ A as x ---+ a,

(iii) For each E > 0, there exists a J > 0 such that If(x) - f(y)1 < E whenever o < Ix - al < J and 0 < Iy - al < J.

2.4. Let rl,r2, ... , be an enumeration of the rationals in the interval [0,1]. For each x E [0,1], define the function ¢x on [0,1] by

¢x(Y) = {~ Let f be defined on [0,1] by

ex)

if y :::; x,

if y > x.

f(x) = L ¢x(rn)/2n . n=l

Prove that

(i) f is strictly increasing,

(ii) f is continuous at x E [0,1] if and only if x t/:- Q.

2.5. Let f be a continuous function on [0,1] with g(O) = 0 = g(I). Suppose, for every 0 < x < 1, there exists a k > 0 such that (i) 0:::; x-k < x < x+k:::; 1 (ii) g(x) = ~(g(x + k) + g(x - k)). Show that g is identically zero in [0,1].

2.6. Let f be a function such that for all real numbers x, y,

If(x) - f(y)1 :::; k Ix - yl

where 0 :::; k :::; 1 is a constant.

Page 62: Surprises and Counterexamples in Real Function Theory

Exercises 53

By considering the sequence x, f(x), f(j(x)), ... , for a real x, prove that there exists a unique real number Xo such that f(xo) = Xo.

By considering the function (1 +X2)1/2, or otherwise, show that this result is false if the above condition is replaced by the corresponding condition without the k.

2.7. Show that a continuous, rational-valued function must be a constant.

2.8. Prove that if f is a continuous real-valued function on [0, (0) and f(x) tends to a finite limit as x -+ 00, then f is uniformly continuous on [0,(0). Is the converse true? Justify your answer.

Prove that if g(x) is uniformly continuous on (-00,00), then g( y'X) is uni­formly continuous on [0,(0).

2.9. Let iI, 12, ... , be continuous functions on [0,1] with values in [0,1]. Define functions gl, g2, ... , from [0, 1] to lE. by

if k = 0,

if gk(X) < 1- 2-k,

if gk(X) ;::: 1- 2-k .

For x E [0,1], set g(x) = limk---+oo gk(X). Show that for x E [0,1] and E > 0, there exists a J > ° such that

Ix - yl < J, y E [0,1] :::} g(y) < g(x) + Eo

Deduce that g(xo) = sup 0::; x::;l g(x) for some Xo E [0,1].

2.10. For n ;::: 0, let Pn(x) = 1 + x + x 2 /2! + ... + xn In!. Show that Pn has no real zero if n is even and exactly one real zero if n is odd (use induction on n).

Using uniform convergence, justify that exp(x) = I:~o xr /r! is a differen­tiable function on lE., and that (exp(x))' = exp(x). Deduce that exp(x) is a nondecreasing function of x.

2.11. Suppose that f: (a, b) -+ lE. be continuous. Prove that a necessary and sufficient condition that f is uniformly continuous in (a, b) is that limx---+b- f(x) and limx---+ a + f(x) both exist.

2.12. Which one of the following statements is sufficient to determine the value f(O) of f(x)? Give the values when determined:

1. f(x) is continuous at x = ° and takes both positive and negative values in any neighbourhood of x = 0.

2. Given E > 0, there exists a 6 > ° such that If(x)1 < E for ° < Ixl < 6.

3. (j(h) + f( -h) - 2f(0))/h -+ 1 and f(h) -+ a as h -+ 0.

2.13. f is defined in [-1,1] by: f(O) = 0, f(x) = sign(sin(7r/x)), if x ¥- 0, where sign ° = 0, sign y = y/lyl (y ¥- 0). Prove that f(x) is integrable in

Page 63: Surprises and Counterexamples in Real Function Theory

54 Chapter 2

[-1,1].

2.14. Determine whether the given conditions are sufficient to ensure that f(x) tends to a finite limit as x ---+ 00

1. f(x) > 0, g(x) < 0, f(x) - g(x) ---+ 0 as x ---+ 00.

2. (f(x))2 ---+ A as x ---+ 00 (A < 00).

Distinguish between the cases A = 0, A ::j::. O.

2.15. Let f(x + y) = f(x).f(y). Prove that if f is not identically equal to 0, then f(x) > O. Prove also that if f is bounded in some interval [-X, X], then inf xE [-X ,X] f(x) > O.

2.16. Let f be defined in [0,1]' f(O) = O. For x E [0,1]' there exists an hex) > 0 such that f(x) - f(x') E Q if x' E (x - hex), x + hex)), for all 0 ::; x' ::; 1. Prove that f(x) E Q for all x E [0,1].

2.17. The functions f(x) = 0 and f(x) = xn have the property that for any integer Tn, (f(x))m = f(xm). Prove that there exists a function with this property which is discontinuous at each x E (-00,00).

2.18. Does there exist a real nonconstant function f such that f(x+p) = f(x) for all p > O?

2.19. Let f be integrable in [a, b] and suppose that f(x) ~ 0 and J: f = O. Given E > 0, show that there exists a closed interval [c, d] C [a, b] such that f(x) < l/r for all x E [c, dJ. Deduce that there exists a nested sequence of intervals

[a, b] ::J [a,b1 ] ::J [a2' b2 ] ::J ...

such that f(x) < l/r for all x E [ar, br]. Hence show that f(~) = 0 for some point ~ E [a, b].

2.20. Give two examples to show that a function f can satisfy one of the following properties without satisfying the other:

• Given E > 0, there exists a 15 > 0, such that Ix - 11 < 15 implies that If(x) - 11 < E .

• Given 15 > 0, there exists a E > 0, such that Ix - 11 < 15 implies that If(x) - 11 < E.

2.21. A continuous function f is such that f(x + 7T) = f(x) for all x E lIt Find necessary and sufficient conditions on f under which one can find another function 9 satisfying g(x + 7T) = g(x) for all x E JR, such that *" = f(x).

2.22. Give an example of a real-valued continuous function f for which f(O) = 0, and for which the sequences

1.f(I), 3.f(I/3), 5·f(1/5), ... ,(2n + 1).f(1/(2n + 1)), ...

Page 64: Surprises and Counterexamples in Real Function Theory

Exercises 55

and 2.f(1/2), 4·f(1/4), 6.f(1/6), ... ,2n·f(1/2n), ...

converge respectively to 1 and -1. Can such an f be differentiable at O? Justify your answer.

2.23. For n = 2,3,4, ... , let fn : [0,1] -+ [0,1] be the function whose graph is shown below;

Figure 2.20

Calculate:

1. limn -4oo(1;;-l(x)), for all x E [0,1].

2. limn -4oo gog 0 .. ·0 g, where g is the function h above.

---------n times

2.24. Given f(x) = psin x+q cos x+x2 , f(2) = 3, is f( -2) uniquely determined or not?

2.25. Prove that the function f(x) = Si~x - ~, ° < x ~ 1f/2, is positive and increasing.

2.26. Prove that for ° < x < 1f/4, (sinx)SinX < (cosx)cosx.

2.27. Prove that:

1. cos (sin x) > sin( cos x) , for all real x.

2. cos(sin-l x) < sin-l (cos x), ° ~ x ~ 1.

3. I sinrxl < rl sinxl, for integer r 2': 2, sinx -::f- 0.

2.28. Suppose f : IE. -+ IE. is continuous and for every rational number q, n ~

there exists an integer n such that f(1( ... f(q)) ... ), i.e., the nth iterate of f evaluated at q is 0. Prove or disprove: for every real number t, there exists an

n ~

integer n, such that f(1( ... f(t)) . .. ) = 0.

Page 65: Surprises and Counterexamples in Real Function Theory

56

Chapter 3

The famous everywhere continuous, nowhere differentiable functions: van der Waerden's

and others

§3.1. van der Waerden's function.

One of the most weird functions is that which is defined for all real x, is continuous at each x but is differentiable at no x. Geometrically, it would appear to be some kind of limit of the saw tooth function,

AAAAAAAAAAAAAAAAAAAAAAAAAAAAAAAAAAAAAAAAA

Figure 3.1

provided the limit does not degenerate into a straight line. The first example of a continuous nowhere differentiable function was given by Weirstrass (1815-1897), namely f(x) = L:~=o bn cos(an1fx), where b is an odd integer and a is such that 0 < a < 1 and ab > 1 + ~1f. Now there are many elegant con­structions of such functions (sometimes called the Weierstrass functions or the blancmange functions, see [109]); some very geometric, others very analytical (i.e., pictures very difficult to visualize), yet others a compromise. We begin with a delightfully simple example given by van der Waerden. The function is simply

00 00 1 <I>(x) = L 1>k(X) = L 2k 1>0(2kx) ,

k=O k=O

where 1>0 (x) is the distance of x to the nearest integer .

To elucidate with diagrams and explanations: 1>o(x) is simply equal to x, if 0< x :::; 1/2, and to -x + 1 if 1/2 :::; x :::; 1, in [0,1] and extended periodically on the entire real line. Its graph is shown in Figure 3.2:

Page 66: Surprises and Counterexamples in Real Function Theory

3.1 van der Waerden '8 function 57

y y

---r----------~----+x x o o % Y2 % 1

Figure 3.2 Figure 3.3

The next function ¢l(X) is equal to (1/2).¢o(2x) and so has two peaks in [0,1], each half as high as in ¢o; thus

{

X if 0::::; x ::::; 1/4 ,

( ) _ -x + 1/2 if 1/4::::; x::::; 1/2 , ¢l x -

X - 1/2 if 1/2::::; x ::::; 3/4,

x + 1 if 3/4::::; x ::::; 1 ,

in [0, 1] and extended periodically on the entire real line. Its graph is shown in Figure 3.3; and so it goes, the number of peaks doubling at each stage but each peak half as high as the previous one. Then one adds them " all " up; " all " meaning add up the first n to get

and take the limit as n ---+ 00 to get <I>(x) = limn - tcXJ <I>n(x), which exists, since for each x, <I>n(x) increases and is bounded above by (1/2)+(1/2)2+ .. ·+(1/2)n, i.e., 0 ::::; <I>n-l ::::; <I>n ::::; 1/2 + (1/2)2 + ... + (1/2)n ::::; 1.

Figure 3.4

Page 67: Surprises and Counterexamples in Real Function Theory

58 Chapter 3

We note that as n increases, the contribution to <[>n from the nth term, viz. (1/2n)cPo(2nx), gets more and more insignificant; indeed, even for n = 5,6, ... , the visual difference between <[>5 and <[>6, is negligible. The graphs of <[>1, <[>2, .. . are drawn in Figure 3.4.

Since IcPo(x)1 :::; 1/2 for all x, we have IcPn(x)1 = 11/2n.cPo(2nx)1 :::; 1/2n+1 = Mn+l' say. Since 2:: Mn is convergent, it follows, by Weierstrass' M-test, that <[>(x) is continuous for all real x, since each cPn(x) is continuous for all real x. We shall now show that <[>' (x) does not exist at any point. The following proof is given in [12].

Lemma 3.1.1. Suppose that the function <[>(x) is different'iable at some ~ E lEt Let Un :::; ~ :::; Vn, Un :::; Vn, Vn - Un -7 0 as n -7 00. Then

as n -7 00, or in other words,

lim <[>(~ + h) - <[>(~ - k) = <[>I(~) (h,k)-+(O,O) h+k

Proof. From the following diagram (Figure 3.5) it is clear that if 81 :::; 8 :::; 82

(81 ,8,82 E (0,1[/2)), then tan81 :::; tan8:::; tanB2 , i.e.,

y y = <v(x)

------~-----L----------------~--------~----------~x o Un

Figure 3.5

<[>(~) - <[>(un) <[>(vn) - <[>(un) <[>(vn) - <[>(~) ~~--~~ :::; :::; ,

~ - Un Vn - Un Vn - ~

and letting n -7 00 in this, the lemma follows, since the extreme members tend to <[>I(~), which is given to exist.

Page 68: Surprises and Counterexamples in Real Function Theory

3.1 van der Waerden's function 59

We construct suitable Un :S t; :S Vn (in the notation of Lemma 3.1.1) as follows:

For n = 1,2, ... , find in E Z such that in/2n :S ~ < (in + 1)/2n (indeed, given n, 2n~ lies between some consecutive integers: in :S 2n~ < in + 1). Take Un = in/2n and Vn = (in + 1)/2n. We now calculate the value of <r>(un) and <r>(vn). We have

¢k (un) = (1/2k)¢0 (2kun)

= (1/2k)¢0 (2k .in/2)n

= 0;

if k ~ n, because if k ~ n, then z = 2k.in/2n is an integer. Hence <r>(un) =

2::%:0 ¢k(Un) = 2::Z~~ ¢k(Un) and similarly <r>(vn) = 2::Z~~ ¢k(Vn). So now

(*)

We next show that ¢k(X) is linear in [un,vnJ if O:S k < n, i.e., that neither a peak nor a trough of any of ¢0,¢1, ... ,¢n-1 can occur in [un,vnJ. Indeed, peaks and troughs of ¢o, ¢1,· .. ,¢n-1 occur at the following points:

1; 1/2; 1/4,3/4; 1/8,3/8,5/8,7/8; ... ; 1/2n- 1, 3/2n- 1, ... ,(2n- 1 - 1)/2n- 1 ;

and none of these can lie strictly between Un and Vn, for Un = in/2n < tI/2 j < (in + 1)/2n = Vn, for some odd integer hand 0 :S j :S n - 1, which in turn implies that in < t2.2n-j < in + 1, for some odd integer t2, which is impossible, as required.

It follows that the quotient (¢dvn) - ¢dun))/(vn - un) = ±1 (see the diagrams (Figure 3.6)).

Figure 3.6

So now the (*) above becomes (<r>( vn) -<r>(Un))/(Vn -un) = 2::~~~ (±1). Letting n -+ 00, this gives (using Lemma 3.1.1) that <r>'(~) = 2:: %:0 (±1), which is absurd, since the right hand series, being convergent to <»' (~), must have nth

term tending to zero, which is not the case. That completes the proof of the

Page 69: Surprises and Counterexamples in Real Function Theory

60 Chapter 3

following striking

Theorem 3.1.2. The van der Waerden function 00

<I>(x) = L ¢k(X) , k=O

where ¢k(X) = (1/2k).¢o(2kx), ¢o(x) being the distance of x from the nearest integer, is continuous for all real x but differentiable for no real x.

By making slight modifications in the proof of Theorem 3.1.2, we can prove the following

Theorem 3.1.3 ([21]). The function <I>(x) has no finite one-sided derivative at any point.

Proof. Assume, to the contrary, that <I>~ (~) exists at some point ~ and equals R, say (R for right). As before, for an integer n :::: 1, let in be the integer such that

(in - 1)/2n ::; ~ < in/2n .

Let Un = in/2n, Vn = (in + 1) /2n (we want both of Un, Vn to the right of 0, so that as n -t 00, both Un and Vn tend to ~ .

• •

Define rn, Sn by

Then we have

• . / n In 2 = Un

Figure 3.7

<I>(Un) - <I>(~) = (R + Tn)(Un -~)

<I>(vn) - <I>(~) = (R + sn)(vn -~).

o < Un - ~ < 1/2n

o < Vn - ~ < 2/2n .

( i)

( i)'

( ii)

( ii)'

Since (<I>(un) - <I>(~))/(un -~) -t Rand (<I>(vn) - <I>(~))/(vn -~) -t R as Vn,Un -t ~ (i.e., as n -t 00), it follows, by (i) and (i)', that Tn, Sn -t 0 as n -t 00. Now

<I>(vn) - <I>(un) = (<I>(vn) - <I>(~)) - (<I>(U n) - <I>(~))

= (R + Sn)(Vn -~) - (R + Tn)(Un -~) (by (i) and (i)')

= R(vn - un) + sn(Vn -~) - Tn(Un -~)

= R.(1/2n) + sn(vn -~) - Tn(Un -~).

Page 70: Surprises and Counterexamples in Real Function Theory

3.2 Some more properties of van der Waerden's function 61

Hence, 2n(<L>(vn)-<L>(un)) = R+2nsn(vn-~)-2nTn(Un-~)' orO::; 12n{<L>(vn)­<L>(un)}-RI ::; 2nlsnl.2/2n-2nITnl.1/2n = 2Isnl-ITnl-+ 0, by (ii) and (ii)' and because Tn, Sn -+ 0 as n -+ 00, i.e., (<L>(vn) - <L>(un))/(vn - un) -+ R as n -+ 00

(since 1/(v - n - un) = 2n). However, as before, (<L>(vn) - <L>(un))/(vn - un) = L:~::~ ¢>~+(O, where ¢>~+(~) is the right derivative of ¢>k(X) at x = ~, and

again, as before, ¢>dx) is linear for each k with 0 ::; k < n and so ¢>/ = ±1,

giving (<L>(vn) - <L>(un))/(vn - un) = L:~::~(±1). Letting n -+ 00, this gives R = L:%"'=o(±1), which is impossible.

Similarly, the left hand derivative L at ~ can not exist.

For yet another variant and proof of Theorem 3.1.3, the reader is referred to [67].

Another interesting variation of the function <L> above is the function

00

w(x) = L 1/Jk(X) , k=O

where 1/Jk(X) = (1/2k)¢>o(23kx) and ¢>o(x), as before, being the distance of x from the nearest integer. This function may be shown to be continuous everywhere, with an infinite left derivative and an infinite right derivative at each point.

§3.2. Some more properties of van der Waerden's function

There are further questions, answers to which would throw more light on this beautiful function <L> (x).

(k-l)/2n (2k-l)l2n+1 ~ = kl2n (2k+1)l2n+1 (k+1)l2n

Figure 3.8

A few pertinent properties of the functions <L>n and <L> are the following; the proofs are vividly clear from the graphs: (i) If ¢>n(ex) = 0, at a point x = ex, then all subsequent ¢>'s are 0 at ex, i.e., ¢>n+l (ex) = ¢>n+2(ex) = ... = o. (ii) <L>(k/2n) = ¢>o(k/2n)+¢>1(k/2n)+ .. +¢>n_l(k/2n), for ¢>n(k/2n) = ¢>n(un) =

Page 71: Surprises and Counterexamples in Real Function Theory

62 Chapter 3

o (the un's are same as in Theorem 3.1.2), and so (ii) follows from (i). Let ~ = kj2n (0 :s; k :s; 2n), k odd, n = 0,1,2, .... Then the graph of ¢n, in a neighbourhood of ~ = kj2n is drawn in Figure 3.8.

For x E (kj2n, kj2n + E), a right neighbourhood of ~ = kj2n,

the slope of ¢i = {~1 ±1

if i 2: n , ifi=n-1,

if 0:S;i:S;n-2;

while for x E ((kj2n) - E, kj2n), a left neighbourhood of~,

We thus have:

{-I

the slope of ¢i = 1

±1

(iii) At the point x = ~ = kj2n, ¢i has

the right derivative = {~1 ±1

{-I

the left derivative = 1

±1

if i 2: n ,

if i = n - 1 ,

if 0 :s; i :s; n - 2 .

ifi2:n,

ifi=n-1,

if 0 :s; i :s; n - 2 ;

if i 2: n , ifi=n-1,

if 0 :s; i :s; n - 2 .

All this is clear from the graphs of ¢n and of the other ¢i. Now let m > n (n fixed); then <I>m(x) = <I>n-l(X) + ¢n(x) + ... + ¢m(x).

Hence for x E (kj2n, kj2n + E), <I>~(x) = <I>~_1 (x) - (m - (n - 1)) ~ -(X) as m ~ 00, by (iii). Similarly for x E (kj2n - E, kj2n), <I>~(x) ~ (X) as m ~ 00. This gives (iv) It follows from (iii) that for ~ = kj2n (n fixed, k odd),

<I>~(x) ~ { ~oo if ~<X<~+E,

if ~-E<X<~,

i.e., the right derivative <I>~(~) of <I>(x) at x = ~ equals +00, while the left derivative <I>'-(~) equals -(X).

In particular, we get:

(i) <I> (x) is not differentiable at points x of the form k j 2n (n = 0, 1, 2, ... ).

(ii) The graphical considerations given below also provide another proof of

Page 72: Surprises and Counterexamples in Real Function Theory

3.2 Some more properties of van der Waerden's function 63

the non differentiability of the function <I> at points not of the form k/2n (see T.H.Hildebrandt, [43]).

Let ~ be a real number, not of the form k/2n. Suppose, to the contrary, that <I>' (0 exists and is finite. Let (~ - 15, ~ + 15) be any open neighbourhood of ~. We shall construct three suitable points P, Q, R on the graph of <I>, with abscissa in (~ - 15, ~ + 15), as shown in Figure 3.9, such that

e = slope PQ - slope PR (= tanf3 - tan,!) = 1.

tan y =slope PR tan 13= slope PO

~-8

Figure 3.9

k

~+8

R

In view of Lemma 3.1.1, slope P R ~ <I> I (~) as 15 ~ 0 and so PQ can not possibly make a nonzero angle with P R, since, as 15 ~ 0, PQ and P R will tend to coincide, making a zero angle with each other, giving the required contradiction.

To construct P, Q, R, find, for each n = 0,1,2, ... , an integer kn such that kn/2" ::::: ~ < (k" + 1)/2n; choose n so large that [kn/2n, (kn + 1)/2n] C

(~- 15, ~ + 15). Let

P == (kn/2n,<I>(kn/2")),

R == ((k" + 1)/2n, <I> ((kn + 1)/2n)),

Q == ((2kn + 1)/2n+1,<I>((2kn + 1)/2n+1)),

where Q is the mid-point of P R.

Remembering that some of the powers of 2 in the denominator of the coor­dinates of P,Q,R are superfluous (e.g. 2k/2n+1 = k/2n, where, if k is even n

Page 73: Surprises and Counterexamples in Real Function Theory

64

will further get reduced), we get

<I>((2k + 1)/2n+l) - <I>(2k/2n+1) slope of PQ = (2k + 1)/2n+1 _ 2k/2n+1

(cPo((2k + 1)/2n+l) + ... + cPn(2k/2n+1)) (l/2n +1 )

(cPo (2k/2n+1 ) + ... + cPn(2k/2n+1 )) (1/2n+1)

cPo((2k + 1)/2n+1) - cPo(2k/2n+1) = 1/2n+1 + ...

cPn((2k + 1)2n+1) - cPn(2k/2n+1) + 1/2n+1

= co + C1 + ... + Cn-1 + Cn , say ,

Chapter 3

where Ci = ±1, Cn = 1 (this will be clear by looking at the Figure 3.10).

In exactly the same way, cancelling the 2 in either of the coordinates, we find that

= 100 + C1 + ... + Cn-1 + Cn-1 ;

where the same Ci feature as in (t) above (this too may be seen immediately by looking at the Figure 3.10). It follows that

slope PQ - slope PR = Cn = 1 ,

as required.

Page 74: Surprises and Counterexamples in Real Function Theory

3.2 Some more properties of van der Waerden's function 65

%1···················································· ............................................................................ ,'"

% .................................................................. :;0(

1/8 I················ :;0(

1116

o 1116 118 3116 V. 5/16 3/8 7/16 Y, 9116 5/8 11116 % 13/16 7/8 15116 1

Figure 3.10

In addition to Figure 3.10, the following particular cases should clarify further:

(i) Take n = 3, k = 0; then P= (0, 0), R= (1/8,<I>(1/8)), Q = (1/16, <I>(1/16));

slope PQ = co + El + E2 + E3 = 1 + 1 + 1 + 1,

slope P R = EO + El + E2 = 1 + 1 + 1.

(ii) Take n = 3, k = 3; then P = (3/8, <I>(3/8)), R = (1/2, <I>(1/2)), Q = (7/16, <I> (7 /16));

slope PQ = EO + El + E2 + C3 = 1 - 1 - 1 + 1,

slope P R = Eo + El + C2 = 1 - 1 - 1.

At the end of Hildebrandt's paper, referred to above, the editors ask the following question: For what values of x (if any), does <I>(x) has a definite infinite derivative? i.e., for what x (if any) is

lim <I>(x + h) - <I>(x - k) = 00 or _ 00 ? (h,k)-+(O,O) h + k

This question is answered by Begle and Ayres in [9]. Their result is given below.

Let ~ E JR, ~ not of the form k /2 n , where, without loss of generality, ~ E (0,1). Let In and On be the number of l's and O's respectively, in the first n terms of the dyadic (binary) expansion for ~ and let Dn = On - In. Then we have:

Page 75: Surprises and Counterexamples in Real Function Theory

66 Chapter 3

Theorem 3.2.1. Let ~ E (0,1) be a real number which is not of the form k/2n. Then

<I>'(~) = { ~oo if Dn --+ 00 as n --+ 00 ,

if Dn --+ -00 as n --+ 00 ;

and if limn-too Dn does 'not exist (as ±oo), then <I>'(O does not exist (even as ±oo).

Remark 3.2.2. One of the following

(i) Dn --+ 00 ,

(ii) Dn --+ -00,

(iii) Dn does not tend to any limit,

must always hold since Dn = On - In is a sequence of integers with Dl = 1 or -1; D2 = Dl ± 1 = 2, -2,0; ... , Dn = Dn- 1 ± 1 = ±1 ± 1 ± 1··· does not tend to any finite limit, since the nth term of the series ±1 ± 1 ± 1··· equals ±1, which does not tend to 0.

For numbers ~ of the form k /211" the theorem is not valid, but for such numbers, we already know that <I> , (~) does not exist, even as ±oo.

The numbers ~ = 0, 1 are both of the form k /2 n , so without loss of gener­ality, we may take ~ to be in (0,1).

Proof of Theorem 3.2.1 For every power 2n of 2, there exists an integer kn

such that kn/2n < ~ < (kn + 1)/2n

(indeed, 2n~ lies strictly between two consecutive integers kn and kn+1 ; strictly, since ~ -::f. k/2n). Now refer to Lemma 3.1.1, which says that <I>'(~) exists if and only if

L = lim (h, k)-t(O, 0)

h, k2: 0

<I>(~ + h) - <I>(~ - k) h+k

exists and then <I>'(~) = L (the implication ~ is trivial). Let ~ + h = (kn + 1)/2n, ~ - k = kn/2n (n = 0,1,2, ... ). If the limit in

(t) exists, then with these special values of hand k also it will exist. For the converse, we shall need the following results:

Lemma 3.2.3. For the function <I>(x), if the limit in (:j:), using the special values ~ + h = (kn + 1)/211" ~ - k = kn/2n exists, then the limit in (+J, for general hand k also exists and equals <I> , (~).

Proof. For these special values of hand k, since ~ + h, ~ - k have the form m/2n, we have <I>(~ + h) = <I>n-l(~ + h), <I>(~ - k) = <I>n-d~ - k) (see (ii)). So in the above limit (t), the value of the numerator equals

<I>n-l((kn + 1)/211,) - <I>n_l(kn/2n) 1/211,

Page 76: Surprises and Counterexamples in Real Function Theory

3.2 Some. more properties of van dcr Waerden's function 67

Now, each of ¢o, ¢l,'" ,¢n-l is linear in the interval (kn/2n, (kn + 1)/2n); indeed ¢i(X) = ±x + Ci (Ci = ±di/2m (i = 0,1, ... ,n - 1)). The function ¢n(x) is the first to have a peak at the mid-point (2kn + 1)/2n+l of kn/2n and (kn + 1)/2n. It follows that (see Figure 3.11):

<I>(X) = <I>n-l(X) = ¢o(x) + ... + ¢n-l(X) = Anx + f..Ln,

with An = I:7:o1 ±1, f..Ln = I:7:01 Ci, i.e., a linear function of x and so the numerator in (t) is the slope of <I>n-l (x) in (kn/2n, (kn + 1)/2n), which is equal to An.

<Pn-I

Figure 3.11

Now

{An + 1

the slope of <I>n(x) = An -1

(i)

As n increases to N say, slope <I>N = An ± 1 ± 1 ... (N - (n - 1) ±l's) and so if the limit in (t) exists, it has to be +00 or -00. We consider these two cases separately and being similar, it is enough to treat one of them, say the +00 case.

Let the limit in (t) (for the special h, k) equal 00 and we have to show that the general limit in (t) is equal to 00, i.e. we must show that given Jl,1, however large, there exists a 6 > 0 such that ((<I>(~ + 7]) - <I>(~))/7]) > M whenever 17]1 < 6. Here again, we take 7] > 0, as the case 7] < 0 is similar.

Since, for the special set of values of ~ + h, ~ - k, limit in (t) exists and is 00, so there exists an integer N such that for n > N, the slope

(<I>n(~ + h) - <I>n(~ - k))/(h + k) > M + 10/3. (ii)

Page 77: Surprises and Counterexamples in Real Function Theory

68 Chapter 3

Choose s with s/2N < ~ < (s+1)/2 N, and let 15 = min(~-s/2N, (s+1)/2N -~). Find n such that

r/2" < ~ < (r+1)/2n < ~+7) < (r+2)/2n. (iii)

Then either ~ lies in the first half of the interval or ~ + 7) in the last half of the interval.

• • • (s+ 1 )/2k

Figure 3.12-

Since 7) < 15, we have n > N. For later use, note also (using (iii)) that

~ - r /2" < 1/2" , 7) > 1/2n +1

By (ii), we see (using (i)) that

(iv)

the slope of <I>,,(x), between r/2n and (r + 1)/2", is > M + 10/3 (v)

and

the slope of <I>,,(x) , between (r + 1)/2" and (r + 2)/2n ,is> M + 4/3 (vi)

In particular, with x = ~, (i) gives, since <I>n is linear, (<I>n(~) - 1!n(r/2n))/(~­r/2n) > M + 10/3. However, ~ - r/2n < 0, so this gives (<I>n(~) - <I>,,(r/2n)) < (M + 10/3)(~ - r/2"), i.e.,

(<I>nW < 1!n(r/2n))+(M +10/3)(~-r/2n) (vii)

Also, for every x, <I>(x) - <I>n(x) < 2/3.2", because the left hand side is equal to

1 ( n+1) 1 ( n+2 ) ¢n+dx) + ¢n+2(X) + ... = 2n+1 ¢o 2 x + 2n+2¢0 2 x + ... 1 1 1 1 < -(- + - + ... ) (since ¢o(x) <-) - 2 2,,+1 2n+2 - 2 1 1

22n 1 1 1 1

= --- < ---42n- 1 32n- 1 '

as claimed. Taking x = ~, this together with (vii) gives:

<I>(~) < <I>(r /2") + (M + 1O/3)(~ - r /2n) + 2/3.2n (viii)

In exactly the same way we get, (using (vi)):

<I>(~ + 7)) > <I>(r/2n) + (M + 10/3)(1/2n) + (M + 4/3)(~ + 7) - (r + 1)/2n) (ix)

Page 78: Surprises and Counterexamples in Real Function Theory

3.2 Some more properties of van der Waerden's function 69

Subtracting (viii) from (ix) and dividing by T) gives, after a straightforward simplification,

(if>(~ + T)) - if>(~))/TJ > (M + 4/3) - (2~ - 2r/2n - (4/3).(1/2n))/T) (x)

But 2~ - 2r/2n - (4/3).(1/2n) < 4/3, for, by (iv), the left hand side is less than (1/2nH - (4/3).(1/2n))/T) < 2n+ 1 ((1/2n- 1) - (4/3).(1/2n)) = 4 - 8/3 = 4/3. Hence, finally, (x) gives (if>(~ + T)) - if>(~))/T) > M. This completes the proof of the lemma.

Now let ~ = .CIC2C3 ... (ci = 0 or 1) and let us see what the inequalities

say about the Ci and the Dn = On - In.

For n = 0, kl = 0 and (*) becomes 0 < ~ < 1.

For n = 1,

For n = 2,

if 0 < ~ < 1/2 ,

if 1/2 < ~ < 1 ,

if 0 < ~ < 1/4 ,

if 1/4 < ~ < 1/2,

if 1/2 < ~ < 3/4 ,

if 3/4 < ~ < 1 ,

Lemma 3.2.4. The slope of if>n-l (0 = Dn (for our ~), n 2: 1.

(*)

Proof. Use induction. For n = 1, we have to show that slope q'>o(~) = D 1 .

Here, the left hand side is equal to 1 if 0 < ~ < 1/2 and is -1 if 1/2 < ~ < 1; while the right hand side is equal to Cl = 1 if ~ belongs to the left half of (0, 1) and is equal to -1 if ~ belongs to the right half of (0,1) (see how the binary expansions go in (0,1)). That starts our induction.

Suppose the result is true for n, i.e., suppose that slope if>n-l = Dn (for ~); then we have to show that slope if>n = Dn+1 (for ~). Here, the left hand side is equal to

slope if>n-l (~) + {I -1

if ~ E (kn/2n, (2kn + 1)/2nH) ,

if ~ E ((2kn + 1)/2n+1, (kn + 1)/2n) ,

which is equal to Dn ± 1 = DnH , using induction hypothesis.

Page 79: Surprises and Counterexamples in Real Function Theory

70 Chapter 3

Lemma 3.2.5.

<{>/(~) = {:,~ , nonexistent (always, by Theorem 3.1.2), even as ± 00,

according as

Proof.

{oo,

lim Dn = -00, n--+oo

nonexistent .

= lim (slope of <{>n-l in (kn/2n, (kn + 1)/2n)) n--+oo

= lim Dn (by Lemma 3.2.4) . n--+oo

The proof of Theorem 3.2.1 now follows easily from the above lemmas.

Corollary 3.2.6. Let ~ be a rational number in (0,1), which is not of the form k/2n. If, in a complete period of the dyadic (binary) expansion of~,

(i) the number of 1 's exceed the number of 0 's, then <{>' (~) = -00,

(ii) the number of 0 's exceed the number of 1 's, then <{>' (~) = 00,

(iii) the number of 1 's and 0 's are equal, then <{>' (~) does not exist, even as ±oo.

Proof. This is quite obvious once one works out one or two typical exam­ples.

The following examples will clarify the above statements.

Example 3.2.7. Let ~ = 1/3 = .010101 ... in the binary representation; the period is 01. We have the following table:

n 1 2 3 4 5 .. . ...

In 0 1 1 2 2 .. . ...

On 1 1 2 2 3 .. . ...

Dll 1 0 1 0 1 .. . ...

We see that Dn does not tend to any limit.

Example 3.2.8. Let ~ = .01100101100101100 ... , then we have the following chart:

Page 80: Surprises and Counterexamples in Real Function Theory

3.3 A geometric example. 71

n 1 2345 678910 1112 ... In o 1 222 23444 4 ... On 1 1 123 44456 7 ... Dn 1 0 -1 0 1 21012 3 ...

We see that Dn -+ 00

Example 3.2.9 Let ~ = 1011011001011001 ... , i.e., ~ starts off with say, three off period terms initially. A table giving the values of In, On, Dn will immedi­ately make it clear that the initial 1, 0, 1 do not affect lim Dn.

So far we have basically only considered van der Waerden's function <I>(x) or its variants. This function is, geometrically, quite interesting and analytically reasonably straight forward in the demonstration of continuity and nondiffer­entiability. There are lots of other ways such continuous, nowhere differentiable functions can be constructed; some of them give a clear geometric picture of the functions while others lend themselves to a simple analytic proof of continu­ity and nondifferentiability. We close this chapter with two extreme examples: one where the geometric picture is very revealing; the other, where the analytic verification of continuity and non differentiability is very simple.

§3.3. A geometric example

We first look at the purely geometric construction, where it will be seen that verification of its continuity and non differentiability will be found to be no more difficult than in any of the previous cases. We shall see that the idea behind this geometric construction is extremely simple and elegant (see A.A. Blank, [13]). Given a line joining the two points (xl,yd, (X2,Y2), Xl < X2, YI =I Y2 (Figure 3.13), we construct a zigzag of three lines from it (Figure 3.14) by the following procedure:

o o

Figure 3.13 Figure 3.14

Let h = (X2 - xd/3, k = >"(Y2 - yd, where>.. is a fixed positive number, to be chosen later. Then Figure 3.14 is got simply by joining the vertices (Xl, yd,

Page 81: Surprises and Counterexamples in Real Function Theory

72 Chapter 3

(Xl + h, Y2 - k), (X2 - h, Yl + k), (X2' Y2). The differences in the ordinates of successive vertices are, respectively (Yl - k) - Yl, (Yl + k) - (Y2 - k), Y2 - (Yl + k), i.e., (1- A)(Y2 - yd, (2A - 1)(Y2 - yd, (1 - A)(Y2 - yd·

First condition on A: Choose A (if possible) such that

1. the maximum of the absolute values of these three differences equals (1- A)IY2 - Yll = IlIY2 - Yll, say; and

2. Il (= 1 - A) < 1.

Next, let the slope of the line joining (xl,yd to (X2,Y2) be m, so that m = ((Y2-Y,)) (m may have any sign). The slope of the zigzag lines are then

X2- X l

(check)

3(1 - A)m, 3(2A - l)m, 3(1 - A)m.

Second condition on A: Choose A (if possible) such that

1. the minimum of the absolute value of these three slopes equals 3(1 -2A)lml = vlml, say; and

2. v (=3(1-2A)) > 1.

Both of these sets of conditions are satisfied if 0 < A < 1/3. Indeed, the first condition on A means that (i) max(11 - AI, 12A - 11) = 1 - A and (ii) 1- A < 1. Here (ii) implies that 0 < A, while (i) implies that 12A -11::; 11- AI and 11 - AI = 1 - A. The latter condition means A < 1, while the former says -11 - AI ::; 2A - 1 ::; 11 - AI or since A < 1, -1 + A ::; 2A - 1 ::; 1 - A, i.e., o ::; A ::; 2/3. The conditions above then yields

o < A ::; 2/3 (A)

The second condition on A means that (i) min(312A - 11, 311 - AI) = 3(1 - 2A) and (ii) 3(1 - 2A) > 1. Here (ii) implies that A < 1/3, while (i) implies that 12A - 11 ::; 11 - AI and that 12A - 11 = 1 - 2A. This latter condition means 2A - 1 ::; 0 or A ::; 1/2, while the former says -(1 - A) ::; 2A - 1 ::; 1 - A (remember that 11 - AI = 1 - A, since 1 - A > 0), i.e., 0 :::; A :::; 2/3. The conditions above then give

o ::; A < 1/3 (B)

Both (A) and (B) are satisfied if and only if 0 < A < 1/3, as required. These conditions on A imply that the entire zigzag of three lines on the

open interval (Xl, X2) remains between the lines Y = Yl and Y = Y2· We take A = 3/10 for drawing a graph. Any A satisfying 0 < A < 1/3, will,

of course, do.

Page 82: Surprises and Counterexamples in Real Function Theory

3.3 A geometric example. 73

P (I, 1)

o 1/9 2/9 113 4/9 519 2/3 7/9 8/9 1

Figure 3.15 Figure 3.16

For the construction of our function, we fix a oX (with 0 < oX < 1/3). Start with the line segment joining (0,0), (1,1). Then construct the first zigzag of the three lines as described above to get Figure 3.15. Next construct zigzags on each of OPI , PI P2 and P2 P by the same procedure to get Figure 3.16 comprising a zigzag of nine lines and so on. Figure 3.17 shows the fourth step of this zigzag procedure, i.e., a zigzag of eighty one lines, drawn with oX taken as .3.

Figure 3.17

Page 83: Surprises and Counterexamples in Real Function Theory

74 Chapter 3

The abscissa of successive vertices of the nth step zigzag (nth approximating polygon) are the ternary points

Xn, i = 3n ' i = 0,1, ... ,3n

and each of these vertices continue to be vertices of all succeeding zigzags. They will be points on the graph of Weierstrass' function Q(x).

This sequential construction yields a function f defined on all ternary points xn , i and Q (x) is defined as the continuous extension of f to each point of the interval [0,1). We give below the details:

Let x n , i, X n , i-I be successive ternary points of order n. Then by the first condition on .\, we have

If(xn,i) - f(Xn,i-l)1 ::; J-ln , i = 0,1, ... ,3n . (I)

where J-l = 1 -.\. Moreover, the slope of the the segment joining the corre­sponding vertices satisfies, by the second condition on .\,

X n , i - X n , i-I (II)

where l/ = 3(1 - 2,\). Let now r E (0,1) be any real number. Then there exists a unique integer in

(for a given n), such that in -1::; 3n.r < in, i.e., (in -1)/3n ::; r < i n/3n, i.e., Xn,i n -1 ::; r < Xn,i n , or sayan::; r ::; bn. It is easy to verify that an ::; an+! , bn+1 ::; bn. Indeed, we have to prove that (in - 1)/3n ::; (in+l - 1)/3n+1 or that 3 (in - 1) ::; in+! - 1. Now, by the definition of in, in+! , we have that (i) in - 1 ::; 3n.r < in (ii) in+l - 1 ::; 3n+!.r < in+1 . Multiply (i) by 3 to get 3.in - 3 ::; 3n+1 .r < 3.in, i.e., 3n+!.r can be anywhere in the interval [3 in - 3,3 in). In any case the integer on its left is greater than or equal to 3 in - 3, i.e., in+! - 1 2: 3 in - 3, as required.

3in - 3 3in - 1

• • • •

Figure 3.18

Also, f(an+d and f(bn+1 ) both lie in the interval [J(an), f(bn)) (or [J(bn), !(an))). This gives us the sequence of nested intervals

(in case !(ai) > !(bi), consider [!(bi), !(ai)) etc.). The length of the nth

interval is equal to I!(bn) - !(an)1 ::; J-ln (by (I)), which tends to ° as n -+ 00,

Page 84: Surprises and Counterexamples in Real Function Theory

3.3 A geometric example. 75

since fJ, < 1. Hence by the nested interval property, there exists a real number ~ such that ~ E [f(an ), f(bn )] (for all n) and we define

O(r) = ~ (III)

Proof of continuity of O(x): Let t > 0 be given and choose n so large that fJ,n < t/2 and take J = 1/4n. We have to show that IO(x) - O(r)1 < t if Ix - rl < J. First, let x < r, and of course, Ix - rl < J. Then (see Figure 3.19)

But

IO(an) - O(r)1 = IO(an) - ~I

::; IO(an) - O(bn)1 (since ~ E [O(an), o (bn)])

= O(an) - O(bn) (since an and bn are ternary points)

::; fJ,n < t/2 ,

r bn+!

[ [ • ]

Figure 3.19

while IO(x) - O(an)1 ::; fJ,n < t/2. That completes the proof of continuity.

Analytic proof of nondifferentiability. We have

IO(bn) - O(r)1 + IO(r) - O(an)1 > IO(bn) - O(r)1 + -=-I0......:..(r....:....)_-_O......:..(a_n-'-'.)I bn - r r - an bn - an bn - an

(in fact either term on the left side is more than the corresponding term on the right side). Hence the above expression is more than or equal to IO(bn) - O(an)l/ (bn - an) 2: vn. Thus, at least one of the slopes n(bbl=~(r), n(r;,=~~an) has absolute value more than vn /2, which means that we can not place a bound on the steepness of the chords with end point at (r, O(r)) in any neighbourhood of r (as any neighbourhood contains [an, bn], if n is large), i.e., o is not differentiable at r.

. ]

Figure 3.20

Page 85: Surprises and Counterexamples in Real Function Theory

76 Chapter 3

Geometric proof of nondifferentiability. We consider two cases.

Case 1: Suppose that r is a ternary point of order n, say r = i/3n, then r is a ternary point of every higher order: r = 3k .i/3n+k and it follows from (II) that no bound can be placed on the steepness of the chords of the graph of D with one point at (r, D(r)), so D is not differentiable at r.

Case 2: Suppose that r is not a ternary point. Then an < r < bn, with strict inequality for all n. Then the steepness of one of the two chords joining (r, D(r)) to (an, D(an)) and (bn, D(bn)) is at least as great as the steepness of the chord C joining (an, D(an)) to (bn, D(bn)). For example, if D(an ) < D(bn ),

then in case (r, D(r)) lies above C, the chord joining (r, D(r)) to (an, D(an)) is steeper (Figure 3.21 (left)), while if (r, D(r)) lies below C, the chord joining (r, D(r)) to (bn, D(bn)) is steeper (Figure 3.21 (right)).

(r, n (r))

c (r, Q( r))

Figure 3.21

A similar argument holds if D(an) > D(bn), and again by (II), nondifferen­tiability follows.

§3.4. An analytic example

We end this discussion on everywhere continuous, nowhere differentiable function with the following example, which is not only short in its definition but also in the proof of its being continuous and nondifferentiable (see William C. Swift, [107]), but it is difficult to visualize its graph.

It is, as usual, enough to define our function <I> on [0, 1]. So let x E [0, 1] and write x in its ternary expansion

x = .XIX2 ... Xn ... (base 3)

so that each Xn = 0,1 or 2. Define <I>(x), written out in the its binary expansion as

<I>(X) = .YIY2··· Yn'" (base 2)

so that each Yn = ° or 1, where

YI = 1 if and only if Xl = 1 and Yn+1 = Yn if and only if Xn+l = Xn- (*)

Page 86: Surprises and Counterexamples in Real Function Theory

3.4 An analytic example 77

Example 3.4.1. Let x = .2101120001122 ... = .X1X2X3X4X5X6 ... , then <I>(x) = .0101101110011 ... = .Y1Y2Y3Y4Y5Y6 ... .

It is easy to verify that if x is expressible in two distinct ternary expansions, the two binary expansions for <I>(x) define the same real number, for example: If x = .21122222 ... = .21200000 ... , then according to the first expression for x, we have <I>(x) = .01100000 ... , while according to the second expression for x, we have <I>(x) = .01011111 ... and these two are equal since .100000 ... = .011111 ....

Proof of nondifferentiability of <I>(x). Let x = .X1X2·.· xnXn+1Xn+2 ... (base 3), and <I>(x) = .Y1Y2 ... YnYn+1Yn+2 ... (base 2). We construct a se­quence of real numbers {~n} tending to x, such that the differential quotient (<I>(~n) - <I>(x))/(~n - x) does not tend to any finite limit as the sequence {~n} approaches x. For each given n, let ~n = .X1X2 ... XnX~+lX~+2'.' (base 3), where we select x~+ll X~+2'.'. such that <I>(~n) = ·Y1Y2··· YnY~+1Y~+2 (base 2), and where Y;,+l =I- Yn+1, but otherwise the corresponding digits of <I>(x) and <I>(~n) are identical. This can be done, using (*), as a moment's thought reveals. This gives our required sequence {~n}, for which we have

o < I~n - xl :::: 1/3" and 1<I>(~n) - <I> (x) I = 1/2n+1,

and therefore, we have I(<I>(~,,) - <I>(x))/(~n - x)1 ~ (1/2)(3/2)", which shows that <I>(~n) - <I>(x))/(~n - x) can not tend to a finite limit as ~n tends to x, i.e., that <I>' (x) does not exist.

Remark 3.4.2. The function <I>(x) can be extended periodically on the entire real line. The only point to observe, then, is that <I>(x) is continuous at x = 0 and x = 1. For this, it is enough to check that <I>(O) = <I>(1). Since 0 = .000 .. . (base 3), we have <I>(O) = .000 ... (base 2), (by (*)), and since 1 = .222 .. . (base 3), we have <I>(1) = .000 ... (base 2) (again by (*)).

There have been many new constructions of everywhere continuous, nowhere differentiable functions, treated by many authors. Since we have had a good many examples of such functions already, we end this chapter here.

Page 87: Surprises and Counterexamples in Real Function Theory

78

Chapter 4

Functions : continuous, periodic, locally recurrent and others

In this chapter we shall study some important properties of functions. The stress will be on various examples, counter examples and nonconventional proofs. We begin with:

§4.1. The intermediate value property

We say that a real-valued function f, defined on an interval I has interme­diate value property if for a, bE I and for any point c between f(a) and feb), there exists a point ~ between a and b such that f (0 = c. We call this property IVP for short.

One of the few results that one proves about continuous functions is that every continuous function has IVP. The continuity of the function is only a sufficient condition for it to have IVP.

Example 4.1.1. A function may satisfy IVP, and yet fail to be continuous. For example, the function f defined on [-1, 1] by

f(x) = {sin ~ , 0,

if x -I- 0

if x = 0

has IVP in [-1,1], but fails to be continuous at the point x = O. Indeed, it is possible to construct a function that has IVP in every interval,

however small, but fails to be continuous.

Example 4.1.2. Let us take our domain to be (0,1). For x E (0,1), write x in its decimal expansion: x = .ala2a3 ... and let z = .ala3a5 .... Define the function f as follows:

(1) If z is not periodic, put f(x) = 0 .

(2) If z is periodic with its first period beginning with a2n-l, put f(x) = .a2na2n+2 a2n+4··· .

(Note that we can always regard the period as beginning with an odd suffix, e.g., 1/6 = .lt 666 ... , with period starting at t, i.e., at a2 but also 1/6 = . 16t 666 ... , with period beginning at t, i.e., at a3).

Page 88: Surprises and Counterexamples in Real Function Theory

4.1 The intermediate value property 79

To see that f has the required property, let I C (0,1) be an open interval. Find n so large that I contains a terminating decimal .al a2 ... a2n - l 000 .. . and with it all the numbers .ala2 ... a2n-la2na2n+l ... that begin with the same first 2n - 1 digits (the latter number is very near the former and I is open).

Now, let y = .b1b2 .. . be any number in (0,1). Choose a2n+l , a2n+3, ... ,

so that .al a3 ... a2n-l a2n+l a2n+3 ... is periodic with its first period starting at a2n- l , and hence, by our construction (2) , the number

is such that f(x) = y . Using (1) , it is not difficult to see that f is not continuous.

We now make the formal :

Definition 4.1.3. We say that the function f defined on an interval I has intermediate value property (also called the Darboux property), if for each x , y E I , x :::; y , the closed interval [I(x) , f(y)](or[l(y), f(x)]) ~ {fez) I x :::; z :::; y}.

A striking result (proved in the next chapter) is the following:

Theorem 4.1.4 (Darboux). If f'(x) (the derivative of f at x) exists for every x in [a , b], then f' has IVP in [a, b].

Since f' may exist, but fail to be continuous (for example, the function f(x) = x2 sin~ , if x ~ 0, f(O) = 0) , we get a class of noncontinuous functions satisfying IVP.

If f and g are continuous in [a, b], then f + g has IVP, as it is continuous. However, we have: Example 2.1.5. If the functions f and g have the IVP, it does not follow that f + g has the IVP. Let

<pc t) = t2 sin ~, if t ~ 0, <p(0) = 0 and 1/;( t) = t2 cos ~, if t ~ 0,1/;(0) = O. t t

Then

'( ) . 1 1 <p t = 2t sm - - cos -t t 1 . 1

1/;' (t) = 2t cos - + sm -t t

if t ~ 0, <p' (0) = 0

if t ~ 0, 1/;'(0) = O.

Let f(t) = (ip'(t))2, get) = (1/;'(t)) 2. Then f , g have IVP, since <p' , 1/;' do have it, by Theorem 4.1.4 (Note that if F has IVP, then F2 has IVP, for: F2(X) :::; C :::; F 2 (y) implies that either F(x) :::; ,jC :::; F(y) or F(y) :::; -,jC :::; F(x). Thus, there exists z E [x, y] such that F(z) = ,jC (or F(z) = -,jC) , so that F 2 (z) = c, as required). But (f + g)(t) = 4t2 + 1 if t ~ 0, (f + g)(O) = 0, so f + g does not have the IVP on any interval containing O.

Remark 4.1.6. Discontinuous funct ions with IVP can be such that their sum

Page 89: Surprises and Counterexamples in Real Function Theory

80 Chapter 4

has IVP, for example, let f, 9 be such that 1', g' exists in [a, b], then 1', g' have IVP and so has l' + g' = (f + g)' (by Theorem 4.1.4). Now consider the question: Suppose f is continuous on [a, b], 9 has IVP on [a, b], must f + 9 have IVP in [a, bJ? It is surprising that the answer is no. A counterexample is given in [73].

There is a partial converse to the IVP of continuous functions, namely:

Theorem 4.1. 7. Let the function f, defined in [a, b], have the property that

(i) for any c, dE [a, b], f assumes every value between f(c) and f(d),

(ii) no rational value is assumed infinitely often by f,

then f is continuous in [a, b].

Proof. Suppose f is discontinuous at x = ~, say, on the left; then the statement

"given E > 0, :J a 6> 0 such that If(x) - f(~)1 < E for all x E (~- 6, ~)"

is false, i.e., for some E > 0, f(x) > f(O + E for some x in (~- 6,~) for each 6, however small. So, there exists an increasing sequence Xl < X2 < ... < Xn < ... , approaching~, and with f(xn) > f(~) + E (for each n).

f(S) + E

m

f(S)

a XI X2 YI ... xi, ... Xn·· ·Y2 .. ~ b

Figure 4.1

Let m be any rational in [J(~), f(~) + E] C [J(~), f(XI)] (by above). So by (i), there exists a YI E (XI'~) with f (yI) = m. Select an Xi, of the sequence {xn} to the right of YI: YI < Xi, .

O~----------~------------~L----

~

Figure 4.2

Page 90: Surprises and Counterexamples in Real Function Theory

4.2 Boundedness and attainment of bounds 81

Again, as above, m E (j(~), f(~) + £] C (j(~), f(Xil )]. So by (i), there exists a Y2 E (Xil'~) with f(Y2) = m, and so on with Y3, Y4,···· Thus f assumes the rational value m infinitely often in [XI'~], contradicting (ii).

The IVP also yields the following very simple fixed-point theorem (see Figure 4.2):

Theorem 4.1.8. Let f : [a, b] ~ [a, b] be a continuo7Ls f7Lnctions. Then there exists a ~ E [a, bl s7Lch that f(~) = ~.

Proof. The function g(x) = f(x) -x has the property that g(a) = f(a) -a 2 0 and g(b) = f(b) - b ::::: 0 (since f(a), f(b) both lie in [a, b]) and so by IVP, g(~) = 0 for some ~ E [a, b].

Another way of saying this is that if f : [0, 1] ~ [0,1] is continuous, then the graph of f must cross the line Y = x.

§4.2. Boundedness and attainment of bounds

We know that a continuous function defined on a closed interval is bounded and attains its bounds. We now give two proofs of this important result, both different from the ones given in the usual Calculus courses.

Theorem 4.2.1. Let the f7Lnction f be continuous on [a, b]. Then there exists a ~ E [a, b] such that f(x) ::::: f(~) for all x E [a, b].

First Proof (W.B.Pennington, [79]). Let ao = a, bo = b. Define anH, bnH inductively from an, bn as follows. Let en = (an + bn)j2. If there exists a number a E [an, en] such that f(x) ::::: f(a) for all x E [en, bn], we let an+l = an, bn+l = en. If not (i.e., iffor each a E [an, en], there exists a number x E [en, bn], with f(a) < f(x)), we let an+l = en, bn+l = bn·

a

In either case, we have

Figure 4.3

an+! = an

bn+! = en

(iii) if x E [an, bn], there exists ayE [an+l, bnHl such that f(x) < f(y) (easily seen).

Page 91: Surprises and Counterexamples in Real Function Theory

82 Chapter 4

This inductive process defines a sequence of intervals [an, bn], n = 0,1,2, ... , such that

(i)' am ::; an ::; bm ::; bo, if n 2': m 2': 0,

(ii)' bn - an -+ ° as n -+ 00,

(iii)' if x E [a, b] and N E N u {OJ, there exists ayE [aN, bN] such that f(x) ::; fey)·

By (i)', an -+ ~ as n -+ 00 and'; E [am, bm] C [a, b] (for each m = 0,1,2, ... ). We shall prove that f(x) ::; f(O for all x E [a, b]. So, let a E [a, b] be any

point. We shall show that f(a) ::; f(~). Choose t > f(';)· Since f is continuous in [a, b], t > fey) for all y in a neighbourhood of .;, i.e., there exists a 6 such that

x E (~- 6,~+ 6) n [a,b] ===?- f(x) < t. (1)

By (ii)', choose N large enough to ensure that bN - aN < 6 and then by (iii)', choose {3 E [aN, bN] such that

f(a) ::; f({3) . (2)

Since,; E [aN, bN], we have 1{3 - ~I ::; bN - aN < 6 (and also {3 E [a, b]). Hence, by (1)

f ({3) < t . (3)

Thus, (2) and (3) imply that

f(a) < t . (4)

~ • • • •

I: Figure 4.4 Figure 4.5

It follows that f(a) < f(~) (for otherwise f(a) > f(~) and taking t = f(a), (4) would give f(a) < f({3), which is a contradiction ).

Second proof (Gerald Jungck [53]). If one of f(a) or feb) is a maximum, we are through. So, suppose that there exists a point c E (a, b) such that f(c) > f(a) and feb). By the continuity of f, f(c) > f(x) throughout small neighbourhoods of a, b :

f(c) > f(x) for all x E [a, a + E) U (b - E, b] (1)

Thus c E [a + E, b - t].

Page 92: Surprises and Counterexamples in Real Function Theory

4.2 Boundedness and attainment of bounds 83

a a+£ c b-£ b

Figure 4.6

Now let

S = b E (a, b) I :3 a point dE [r, b] with f(d) > f(x) , 'V x E [a, In.

We see that S i= 0, since, by (1), a+E E S, with d = c. Moreover, b is an upper bound for S, i.e., S is bounded. Let ~ = sup S. Then as a + E E S, we see that

(2)

(b, f(b))

(a, f(a))

o a a+£ y d b-s b

Figure 4.7

We prove ~ :::; b - E. Otherwise, there exists a point u E (b - E,~], which is in S and this means that there exists dE [u, b], such that f(d) > f(x) for all x E [a, u), in particular for x = c, i.e.,

f(d) > f(c) . (3)

• • • • • b-s cr d b

Figure 4.8

But by (1), f(c) > f(x) for all x E [a, a + E) U (b - E, b], in particular, for x = d; i.e.,

f(c) > f(d) . (4)

Page 93: Surprises and Counterexamples in Real Function Theory

84 Chapter 4

As (3) and (4) contradict each other, ~ ::; b - E, as claimed, i.e., (see (2)), ~ E [a + E, b - E].

We now prove that f(~) is maximum, i.e., that f(~) ~ f(x) for all x E [a, b]. Suppose not; then f(() > f(~) for some ( E [a, b], and so by the continuity of f, there exists <5 « E) such that

f(() > f(x) for all x E (~ - <5, ~ + <5) . (5)

But by the definition of ~ as sup S, there exists a point dE [c, d] such that

f(d) > f(x) for all x E [a, c) and [a, c) ::> [a, ~ - <5] (6)

Let f( 7)) be the greater of f ((), f (d). Then

f(rJ) ~ f(() > f(x) (by(5)) for all x E (~- <5,~ + <5) (7)

and

f(7)) ~ f(d) > f(x) (by(6)) for all x E [a, ~ - <5] . (8)

• • [ . . • • . . ~

a a+E ~ - 8 c ~ ~+8 d S b-E b

Figure 4.9

Equations (7) and (8) means f(7)) > f(x) for all x E [a, ~ + <5) and so clearly that 7) > ~ + <5. Hence ~ + <5 E S, which contradicts the definition of ~ as sup S.

Remark 4.2.2. This point ~ is the (unique) infimum of all points in [a, b] that yields the maximum value of f(x), i.e., ~ = inf{zlf(z) ~ f(x) for all x E [a, b]}.

b

Figure 4.10

§4.3. Locally recurrent functions

This small class of functions, still in its developing stages, is defined as follows:

Definition 4.3.1. A function f, defined in the interval [a, b], is said to be

Page 94: Surprises and Counterexamples in Real Function Theory

4.3 Locally recurrent functions 85

locally recurrent at the point c E ( a, b) if every deleted neighbourhood N D ( c) = (c - E,C+ E)" {c} of c contains a point x such that f(x) = f(c).

A function f is said to be locally recurrent everywhere in ( a, b) if it is locally recurrent at each point c of (a, b).

Example 4.3.2. (i) The function f(x) = xsin~, if x =I- 0, f(O) = 0, is locally recurrent at x = 0. (ii) The function f(x) = 1 if x is rational, ° otherwise, is locally recurrent everywhere.

Definition 4.3.3. The function f defined in [a, b] is said to be locally recurrent over right neighbourhoods at the point c E (a, b), if every right neighbourhood (c, c + E) of c contains a point x such that f(x) = f(c).

Similarly we define locally recurrent over left neighbourhoods at c and locally recurrent over right (or left) neighbourhoods everywhere.

The following questions arise immediately:

Question 1. Does there exist an everywhere locally recurrent function which is nonconstant and continuous? Question 2. Does there exist a function which is locally recurrent over right neighbourhoods everywhere and which is nonconstant and continuous?

Remark 4.3.4 At first it may seem that if f is locally recurrent everywhere, then it is locally recurrent over right neighbourhoods everywhere. In fact this is not the case. Locally recurrent everywhere means that for each point c, there exists either a left or a right neighbourhood of c where the value f(c) occurs again. So for some point c, it may be that f(c) occurs in the left neighbourhood but not the right neighbourhood (or vice versa), so f need not be locally recurrent over right neighbourhoods (or left neighbourhoods). This remark may therefore make the answers to the two questions above less surprising, viz. yes to question 1 and no to question 2.

In [20], K.A.Bush gives an example of a continuous nonconstant function on [0,1] which is locally recurrent everywhere. Interested reader may look up this example.

Our object here is to show that the answer to question 2 is no, i.e., we have the following:

Theorem 4.3.5. There is no nonconstant continuous function, which is locally recurrent over right neighbourhoods everywhere.

Proof. Suppose, to the contrary, that f is a nonconstant, continuous function that is locally recurrent over right neighbourhoods everywhere in [0, 1], say. Fix c E (0,1). Let S = {x E (O,l)lf(x) = f(c)}. Then SC = (0,1) " S is an open set. To see this, let 0: ESc; then f(o:) =I- f(c), say f(o:) > f(c). By the continuity of f, f(x) > f(c) in some open neighbourhood of 0: and so this open neighbourhood of 0: is contained in se, showing that se is open.

Write this open set se as a countable disjoint union of open intervals (we

Page 95: Surprises and Counterexamples in Real Function Theory

86 Chapter 4

can suppose that these are contained in (0,1)), i.e.,

00

se = U (an,bn). n=1

Here an, bn ~ se and so they are in S, i.e., f(an) = f(c). But f is locally recur­rent over right neighbourhoods everywhere and so in particular at x = an, i.e., there exists a point 13 in a right neighbourhood of an at which f(j3) = f(an), and so f(j3) = f(c), showing that 13 E S. But 13 E (an, bn) eSe, which gives the required contradiction (see Figure 4.11).

f(f3) fCc)

a b

Figure 4.11

§4.4. Periodic functions

We begin by defining periodic functions.

Definition 4.4.1. Let f be a real-valued function on lIt We say that f is periodic if there exists a real number p ::j:. 0, such that f (x + p) = f (x) for all real numbers x.

Definition 4.4.2. The real number p in Definition 4.4.1 is called a period of

f· Remark 4.4.3. If p is a period of f, clearly so is -p and hence also Ipl.

Definition 4.4.4. If there is a positive period w such that w ::; p for all positive periods p of f, then w is called the fundamental period (abbreviated as FP) of f (w is clearly unique, since if Wi is another one, then by the definition of the FP, w ::; Wi, and Wi::; w).

Theorem 4.4.5. Any period p is an integral multiple of w.

Proof. Since w is the FP, ° < w ::; Ipl. If w = Ipl, then p = ±w, as required; so suppose ° < w < Ipl. Subtracting a suitable positive multiple nw of w from Ipl, we see that there exists a unique positive integer n and a unique real number r such that Ipl = nw + r (n > 0, ° ::; r < w) and then f(x) = f(x + Ipl) = f(x + nw + r) = f(x + r) for all real numbers x. Thus, if r ::j:. 0, f would have a positive period r smaller than w. Hence r = 0, giving Ipl = nw.

Page 96: Surprises and Counterexamples in Real Function Theory

4.4 Periodic functions 87

TheoreIll 4.4.6. Let f be periodic with a nonzero period and let

J* = inf{J > 0 I f(x + J) = f(x) for all x E IE.}.

Then either J* = 0 or every period of f is an integral multiple of J* .

Proof. Suppose that J* i- 0 and let p be any nonzero period of f. If p = 0, then p = O.J* is an integral multiple of J*. If J* is a period of f, then by Theorem 4.4.5, p is a multiple of J*. If J* is not a period of f, there exists arbitrarily small numbers E such that

J* + E is a period of f (*)

(this is one of the two characterizing properties of the infimum, in particular, of J*). Write (uniquely) Ipi = nJ* + r, n > 0 an integer and 0 :::; r < J*. We wish to show that r = O. Suppose, to the contrary, that r > O. Let E > 0 be a positive real number such that J* + E is a period of f and r - nE > 0 (possible by (*)). Then, for all real x, we have

f(x) = f(x + Ipl) = f (x + nJ* + r)

= f(x + n(J* + E) + r - nE)

= f(x + (r - nE)) (by (*)).

Thus, r - nE is a period of f, with 0 < r - nE < r < J*, which contradicts the definition of J*. It follows that r = 0 and so Ipi = nJ* is an integral multiple of J*.

It is still not clear that this number 8*, defined above, is actually a period of f. This is the content of the following:

TheoreIll 4.4.7. If the infimum 8* of the positive periods of the periodic function f is nonzero, then it is a period of f.

Proof. Suppose that J* > 0 and is not a period. By a property of the infimum, there exists a small positive number, say E, 0 < E < 8*, such that J* + E is a period of f. But by Theorem 4.4.6, 8* + E = n8* (for some positive integer n) and this is impossible as then J* < J* + E < 2J*.

The following matter, up to Theorem 4.4.15, calls for a stronger background on the part of the reader than what is required for the rest of the matter in the book.

Theorem 4.4.6 has a group theoretic counterpart which is worth noting (see [29]). Let f be a real valued function. Let

PI = {p E IE. I f(x + p) = f(x) for all x E IE.} = the set of all periods of f together with 0

Page 97: Surprises and Counterexamples in Real Function Theory

88

Theorelll 4.4.8. PI is an additive subgroup of JR.

Proof. Clearly, 0 E PI. Let p, q E PI. Then

f(x + (p - q)) = f(x - q)

= f((x - q) + q) = f(x) ,

shows that p - q E Pj. Hence, Pj is a subgroup of JR.

Chapter 4

Definition 4.4.9. PI is called the group of periods of f and f is said to be periodic if \PI I > 1 (note that 0 is always in Pj).

Also, if G is any additive subgroup of JR, then there exists a function g on JR whose periods are precisely the elements of G, namely, the characteristic function Xa of G (Xa =1 if x E G, and 0 otherwise).

With this relationship between periodic functions on JR and additive sub­groups of JR, it is not difficult to show that Theorem 4.4.6 is equivalent to:

Theorelll 4.4.10. Every proper closed additive subgroup of JR is a discrete subgroup of the form aZ, where a > O.

The above theorem can be seen to be equivalent to

Theorelll 4.4.11. Every nontrivial additive subgroup G of JR is one of the following two types: (a) If there exists a least positive element w E G, then G =< w >, the cyclic group generated by w, (b) G is dense in JR (in the usual topology).

Let f be a function with PI =< w >, where w is the least positive element of PI. We call w the fundamental period of f. We now have the following:

Theorelll 4.4.12. Let f be periodic and suppose that f is continuous at some point ~. Then either f has a fundamental period, or f is a constant function.

Proof. Suppose f has no fundamental period. Then Pj is dense in lR. For every E > 0 there exists a 6 > 0 such that If(x) - fWI < E if Ix - ~I < 6. Let x' be any point in JR. Then x' - ~ E JR and Pj is dense in JR, so there exists a I E Pj, at a distance less than 6 from x' - ~, i.e., x' - ~ - I = t, say, is such that It I < 6, i.e., x' = ~ + I + t, where I E PI, It I < 6. Hence

If(x/) - f(~)1 = If(~ + I + t) - f(~)1

= If(~ + t) - f(~)1

< E,

since It I < 6. As E is arbitrary, f(x' ) = f(~). Here ~ is a fixed point while x' is any point. It follows that f is a constant.

Exalllple 4.4.12. The converse of the above theorem is not true, for example,

Page 98: Surprises and Counterexamples in Real Function Theory

4.4 Periodic functions 89

let

f(x) = {O, if x is rational 2 + cos 2nx , otherwise.

This function has fundamental period equal to 1, but is discontinuous every­where.

Now consider the quotient group JR/Ql, whose elements are cosets Ql + r (r E JR). Let 1* : JR/Ql ---+ JR be a function on JR/Ql. This induces a periodic func­tion f : JR ---+ JR, with Pj :J Ql, by defining f(x) = 1* (Ql+x). For this function f, clearly Pj :J Ql, as let p E Ql, then f(x+p) = 1*(Ql+x+p) = 1* (Ql+x) = f(x) (since Ql + p = Ql), i.e., p is a period and so Ql C Pj, as required.

Further, if 1* is one to one, then Ql :J Pj, for let p E Pj be a period; then f(x + p) = f(x) for all x, i.e., 1*(Ql + x + p) = 1*(Ql + x) for all x. But 1* is one to one, so Ql + P + x = Ql + x, i.e., p E Ql, as required. Hence we have proved the following

Theorem 4.4.14. If 1*: JR/Ql ---+ JR is a one to one mapping, then the induced function f : JR ---+ JR, given by f(x) = 1*(Ql + x), is periodic and its group of periods P j equals Ql.

Since card(JR/Ql) = card JR, it is possible to choose a map 1*: JR/Ql ---+ JR that is one to one. The resulting function f : JR ---+ JR has Pj = Ql, which is dense in JR and the range of f is uncountable. This construction will easily yield a periodic function whose graph is dense in the plane. Note also that there exist uncountable proper subgroups of JR and hence we get:

Theorem 4.4.15. There exist a nonconstant periodic function with uncount­ably many distinct periods.

Let now f and g be two periodic functions, with fundamental periods a, /3 respectively. We ask the question: When is the sum f + 9 periodic ? If we admit only continuous functions, the following result holds :

Theorem 4.4.16. Suppose f, 9 are continuous periodic functions with funda­mental periods a, /3 respectively. Then f + 9 is periodic if and only if al/3 E Ql

Proof. Let f and 9 be continuous periodic functions. If a function f is periodic, then so is - f, therefore it is enough to prove that if f, 9 and h = f + 9 are peri­odic, continuous functions, with periods a,/3" respectively and al,,/3I, ~ Ql, then h is a constant function. Because this shows that at least one of a I" /31, is rational and since the relation f + 9 = h may be written as f + (-h) = -g or 9 + ( - h) = - f, this also shows that at least one of a 1/3, ,1/3 is rational and that at least one of /31 a, ,I a is rational. Thus at least two of a I /3, /3 Ii, ,I a are rational; but then so is the third, i.e., all three are rational. In particular, if f, g, f + 9 (= h) are continuous and periodic, then al/3 is rational as required. So now it remains to prove the above assertion.

Let x, y be any given real numbers. Since ai, is irrational, the set {pa+q, I

Page 99: Surprises and Counterexamples in Real Function Theory

90 Chapter 4

p, q E Z} is dense in m. (see Theorem 1.1.6) and so there exists sequences {Pn}, {qn} of integers such that {PnO: + qn,} ---t x and similarly there exists sequences {sn}, {tn} of integers such that {snO: + tn,} ---t y. Now we have

and

Adding, we get

h(pno:+qn,)+h( sn!3-Hn,) = h(O) +h(PnO:+ sn/3) = h(O)+h(pno:+qn,+Snf3 +in,).

Letting n ---t 00, we get (by the continuity of j,g,h) that h(x) + h(y) = h(O) + h(x + y), or (h(y) - h(O)) + (h(y) - h(O)) = h(x + y) - h(O), which means that the function H(x) = h(x) - h(O) is additive, and so, being continuous, H(x) = ex (the equation 'P(x + y) = 'P(x) + 'P(y) for all x, y E m. is known as Cauchy's equation. It will be proved in Appendix I, that if'P is also continuous, then 'P(x) = ex, for some constant e). But H is periodic too and so c = 0, i.e., h(x) = h(y) for all x,y E m., i.e., that h(x) is a constant, as required.

The converse is trivial: If 0:, 13 are periods of f, 9 and 0:/13 = min (m, n E Z), then, for example o:n+f3m is a period of j+g, for, we have U+g) (x+o:n+f3m) = f(x + o:n + 13m) + g(x + o:n + 13m) = f(x + 20:n) + g(x + 213m) = f(x) + g(x) (since o:n = 13m), as required.

For general f and 9 with fundamental periods 0: and 13 respectively, we need to consider the two cases, namely 0:/13 E Q and 0:/13 tJ- Q.

Definition 4.4.17. We say that two non-zero real numbers 0:,13 are commen­surable if their quotient is a rational number, i.e., if there exists integers i,j such that io: + j 13 = o. Also, 0:,13 are called incommensurable if 0:/13 is not rational, i.e., if io: + jf3 = 0, i,j E Z implies that i = j = O.

Definition 4.4.18. Suppose that 0:/13 is a rational number, so that there exist integers i, j such that io: + j 13 = 0; then, = io: = - j 13. Such a , is called a common multiple of 0: and 13. The smallest positive such, is called the least common multiple (lcm) of 0: and 13.

Going back to the question posed just before the statement of Theorem 4.4.16, we note that it is easy to deal with the case 0:/13 E Q. For, in this case io: = -jf3 (for some i,j E Z) =" say. Then the sum h = j + 9 satisfies

h(x + ,) = f(x + ,) + g(x + ,) = j(x + io:) + g(x - j(3)

= j(x) + g(x) (since 0:,13 are FP's of j,g)

= h(x).

We have thus proved the following:

Page 100: Surprises and Counterexamples in Real Function Theory

4.4 Periodic functions 91

Theorem 4.4.19. Let f, g be periodic, with FP's a,j3 respectively. Suppose that a/j3 is a rational number. Then h = f + g is periodic and any common multiple I = ia = - j 13 is a period of f + g.

Remark 4.4.20. In general, the lcm I of a, 13 need not be the FP of f + g. However, in the special case when f(x) = a sin ax and g(x) = b cos j3x (so that the FPs of f and g are 27f / a and 27f / 13 respectively), where a, b, a, 13 are real numbers and a/j3 is rational, the lcm of 27f / a, 27f / 13 (which is same as that of a and 13) is the FP of f + g, i.e., we have the following

Theorem 4.4.21. Let f(x) = asinax, g(x) = bcosj3x, where a,b,a,j3 are any real numbers and a,j3 are commensurable (i.e., a/j3 is rational). Then the function h = f + g is periodic and the lcm I of the FPs of f and g is the FP of f + g.

Proof. Without loss of generality, we may suppose that a > 0,13 > ° (for if, say a < 0, we consider the function -asin(-ax), which equals asinax, with -a > 0). Now as we have seen, the lcm I of the FPs of f and g is certainly a period of h = f + g (Theorem 4.4.19). It remains to show that I is the FP.

If a = 13, then h(x) = asinax + b cos ax = ../a2 + b2 sin(B + ax), where cos B = va;+b 2 ' sin B = v)+b 2 ' and hence h is periodic, as required. So assume

that a =I- 13· Let T be the FP of h = f + g. Then

h(T + x) = h(x) (for all real x)

Taking x = 0, (*) gives

a sin aT + b cos j3T = b

(*)

(1)

Differentiating (*) gives h'(T + x) = h'(x) and putting x = a in this gives

aa cos aT - bj3 sin j3T = aa

As above, differentiating again and putting x = 0, gives

-aa2 sin aT - bj32 cos j3T = _bj32

and differentiating yet again and putting x = 0, finally gives

-aa3 cos aT + bj33 sin j3T = -aa3

Solving (1)-(4) in pairs, viz. (1), (3) and (2), (4), we get

sin aT = 0, cos aT = 1

sinj3T = ° , cosj3T = 1

(2)

(3)

(4)

(5)

(6)

Now, (5) implies that aT = n7f, aT = 2n7f (n integer), i.e., aT = 2n7f (as this covers both), i.e., T = n.(27f)/a = a multiple of the FP of sin ax (viz. 27f/a). Similarly, (6) implies that T is a multiple of the FP of cos j3x (viz. 27f / 13).

Page 101: Surprises and Counterexamples in Real Function Theory

92 Chapter 4

Since T is the FP of h(x), it is the least common multiple of the two FPs viz. 27r / 0:, 27r / (3, as required. This completes the proof of the theorem.

For a purely trigonometric proof of the above theorem and for the related material, the reader is referred to [10].

Let now f and 9 have FPs 0:, (3 respectively, where 0:/(3 is not rational. Theorem 4.4.16 tells us that if f and 9 are continuous then f + 9 cannot be periodic. However, in general, f + 9 can be periodic. Here is a beautiful

Example 4.4.22. We now give an example of bounded functions f, 9 with FPs 0:, (3 respectively, 0:/(3 not rational, such that f + 9 is periodic.

Define a set s = {x E IE. I x = io: + j (3, i, j E Z}.

Select a real number, such that for no k E Z, we have k, E S, i.e., none of 0, ±" ±2" ... E Z. This is possible since the set {io: / k + j (3 / k Ii, j, k E Z, k "I­O} is countable, so we may select any, in IE., outside of this set. Let

T = {y E IE. I y = io: + j (3 + k, , i, j, k E Z}.

For the sets S, T the following properties are easily verified:

1. If XES, the integers i, j, for which x = io: + j (3, are unique,

3. SC + S c SC, where SC is the complement of S in the reals, i.e., SC = IE. " S,

and

(1)' if yET, the integers i, j, k, for which y = io: + j (3 + k, are unique,

(2)' T + T c T,

(3)' T C + T eTc.

Now, finally define

and

{2- U1 + 2- lkl , if x E T : x = io: + j(3 + k"

f(x) = 0, if x E T C ,

{2- lil - 2- lkl , if x E T : ,]; = io: + j(3 + k"

g(x) = 0, if x E TC.

Then

h(x) = f(x) + g(x) = {2- lil. + 2- U1c' if x E T: x = io: + j(3 + k"

o , If x E T .

Page 102: Surprises and Counterexamples in Real Function Theory

4.4 Periodic functions

We shall now show that (a) f is periodic with FP equal to a, (b) 9 is periodic with FP equal to /3, ( c) h is periodic with FP equal to "(-

93

Because of the similar form of f, g, h, we shall only verify (a). To see that a is a period of f, note that

(i) if x ETc, then f(x) = 0 and x + a E T C + T C T C and so f(x + a) = 0, giving f(x) = f(x + a),

(ii) if x E T, say x = ia + j/3 + k",(, then

f(x + a) = f((i + l)a + j/3 + k"'()

=Tljl +Tlkl

= f(ia + j/3 + k"'()

= f(x) ,

(since x + a E T)

as required. To prove that a is the FP of f, we need to show that every period of f is an integral multiple of a. So let p be any period of f. If p ETc, then for x E T, x+p ETc and so f(x+p) = 0, but f(x) =1= o because x E T. So p 1. T C ,

i.e., any period p must belong to T and so p = la + m/3 + n"'( (l, m, n E Z) and since p is a period, f(O + p) = f(O), i.e., 2- lml + 2- lnl = 2° + 2° = 2. This is possible if and only if m = n = 0 and then p = la, an integral multiple of a, as required.

In the above example, we chose "'( such that none of k"'( (k = 0, ±1, ±2, ... ) belongs to S. That this is essential follows from the following:

TheoreIll 4.4.23. Let f, 9 have FPs a, /3 respectively, where aj /3 1. Q and sup­pose that not both of f, 9 are unbounded (i. e., at least one of f, 9 is bounded). If"'( is a real number such that k"'( E S for some integer k =1= 0, then the sum f + 9 can not have "'( as a period .

Proof. Without loss of generality, let f be bounded and suppose to the con­trary, that "'( is a period of f + g. then k"'( too is a period and so for all real y and x = ia + j/3 E S, we have (since a is a period of j),

f(y + x + k"'() = f(y + j/3 + k"'() - f(y + j/3) (1)

Now, since k"'( is a period of f + g, we have (as j/3 is a period of g)

f(y + j/3 + k"'() + g(y + j/3 + k"'() = f(y + j/3) + g(y + j/3)

i.e.,

f(y + j/3 + k"'()- f(y + j/3)=g(y + j/3)-g(y + j/3 + k"'()=g(y) - g(y + k"'() (2)

Page 103: Surprises and Counterexamples in Real Function Theory

94 Chapter 4

So, using (1) and (2), we get f(y + x + k,) - f(y + x) = g(y) - g(y + k,). In this, take x = /l.(k"() E S, /l = 0,1,2, ... n - 1, and we get respectively, the following equations:

f(y + k,) - f(y) = g(y) - g(y + k,)

f(y + 2k,) - f(y + k,) = g(y) - g(y + k,)

f(y + nk,) - f(y + (n - l)k,) = g(y) - g(y + k,).

Adding, we get f(y + nk,) = f(y) + n(g(y) - g(y + k,)). Since f is bounded, this gives

g(y)-g(y+k,) = 0 , I::/y E JR. (3)

Again, f(y + k,) + g(y + k,) = f(y) + g(y), since k, is a period of f + g, so g(y) -g(y+k,) = f(y+k,) - f(y), and it follows, by (3), that f(y+k,) = f(y) for all real numbers y, and hence that k, is a period of both f and 9 and so k, is an integral multiple of both a and (3. Thus, k, = pa = q(3, i.e., a/ (3 = q/p is a rational number, which gives the required contradiction.

By allowing f and 9 both to be unbounded, it is possible to have a sum f + 9 with a period, in the set S. We have:

Example 4.4.24. Let

and let

then

{j when x = ia +j(3 E S,

f(x) = 0 when x Ese,

g(x) = {-i when x = ia + j(3 E S, o when x Ese,

f(x) + g(X) = {j - i when x = ia + j(3 E S, o when x Ese.

The unbounded functions f and 9 have FPs a and (3 respectively (as in example 4.2.22), while their sum f + 9 has FP equal to, = a + (3 E S (check).

Remark 4.4.25. Note that the set S = {x E JR I x = ia+j(3,i,j E Z} is dense in JR if a/ (3 ¢ Q. This fact can be generalized in the context of complex function theory and then used to show that the only triply periodic analytic functions are constants - a result due to Jacobi (see E.Hille, Analytic Function Theory, Vol. II, Ginn, 1962).

In Theorem 4.4.16, the continuity conditions may be replaced by others that guarantee a/ (3 E Q; for example, we have the following

Page 104: Surprises and Counterexamples in Real Function Theory

4.4 Periodic functions 95

Theorem 4.4.26. Let f, 9 be periodic and have (without loss of generality) positive periods a, (3 respectively and suppose that f, 9 both have strict absolute maxima at the origin, i.e., f(O) > f(x) for all x E (O,a) (note that f(a) = f(O)) and g(O) > g(x) for all x E (0, (3) (note again that g((3) = g(O)). If f + 9 is periodic, then a/ (3 E Q.

Proof. Let I be a positive period of f + g. Then (f + g)(O) = (f + g) (f) i.e., f(O) + g(O) = f(f) + g(f). But f(O) 2: f(f), g(O) 2: g(f). So this is impossible unless equality holds in both places: f(f) = f(O), g(f) = g(O). Hence I is a multiple of a and a multiple of (3: ,= ma = n(3, giving a/ (3 = n/m.

/{\ /\ \ / \

/ \ I \

-1

Graph oCy = cos 7t X Graph oCy = cos ..J27t x

Figure 4.12

Example 4.4.27. The above mentioned simple condition suffices to show, for example, that cos TTX + cos V27TX is not periodic, for if f (x) = cos TTX and g(x) = cos V27TX, then the FPs of f, 9 are given by a = 2, (3 = 2/V2; further, both f,g have absolute maxima at the origin (Figure 4.12). So, if f + 9 were periodic, a/(3 = 2/(2/V2) = V2, would be rational, which is not the case, hence f + 9 can not be periodic.

There are other similar conditions, which, together with the periodicity of f, 9 and f + 9 imply that a/ (3 E Q. the interested reader may look up [23].

Example 4.4.28. Let

and

f(x) = {o , if x = (2n +.1)/2 for some nEZ, tan TTX , otherwIse;

g(x) = {o , if x = (2n + 1)/2V2 for some nEZ, tan V27TX , otherwise .

Here, f is periodic with FP equal to 1, 9 is periodic with FP equal to 1/ V2, but f + 9 is not periodic, a/ (3 ~ Q.

Page 105: Surprises and Counterexamples in Real Function Theory

96 Chapter 4

§4.5. Horizontal chords

Let f : [a, b] ---+ lR be a continuous function.

Definition 4.5.1. We say that f has a horizontal chord of length), if there is a point x such that both of x, x +). E [a, b] and f(x) = f(x + ).).

Example 4.5.2. (1) The function in Figure 4.13 has two horizontal chords of length 2 and one of length 4, viz. GA, GB, AB of length 2, GB of length 4.

° Figure 4.13 Figure 4.14

(2) The function f(x) = 1 defined on the entire real line, has horizontal chords of all lengths. (3) The function f(x) = x3 - X + 1 has a horizontal chord of length 2, viz. (-1,1) to (1,1) and two horizontal chords of length 1, viz. (-1,1) to (0,1) and (0,1) to (1,1) (see Figure 4.14). (4) The function f(x) = x 3 has no horizontal chords.

Remark 4.5.3. Suppose that f is a continuous periodic function of period p.

Then it is easy to see that (see Figure 4.15) the integral J:+P f(x) dx has value independent of a.

OA=a

o A Figure 4.15

Using this, we can prove the following result:

Proposition 4.5.4. Let f be a continuous and periodic function. Then f has horizontal chords of all lengths.

Page 106: Surprises and Counterexamples in Real Function Theory

4.5 Horizontal chords 97

Proof. Let p be the period. We have to show that for any real number ..\, there exists a number ~ such that f(~ +..\) = f(~). To see this, consider the integral It(f(x + ..\) - f(x)) dx, which is zero, by Remark 4.5.3. Hence g(x) = f(x +..\) - f(x) can not have a fixed sign on the interval (O,p) and so by the IVP (which holds since the function 9 is continuous), g(~) = ° for some ~ E (O,p), as required.

Indeed, we have the following result:

Proposition 4.5.5. Let f be continuous and periodic with period p. Then f has two horizontal chords of any given length ..\, with their left hand end points at different points of [0, p).

Proof. Consider the graph of the function g(x) = f(x +..\) - f(x).

p

Figure 4.16 Figure 4.17

We have g(p) = g(O); but 9 changes sign between ° and p, by a reasoning similar to the one used above. So the graph of 9 must have two possibilities as shown in the Figures 4.16 and 4.17, and so 9 vanishes at least twice between [O,p); say at ~ and T); i.e., f(~ +..\) = f(~), f(T) +..\) = f(T)) (~ -=J T)). This completes the proof.

Figure 4.18

By considering the function g(x) = 'P(x) - 'P(x - ..\), where 'P(x) = f(x + ..\) - f(x), we see, as before, that It g(x) dx = ° and so 9 changes sign in [O,p). It follows that 9 vanishes at some ~ E [0, p) (by the IVP, since 9 is continuous),

Page 107: Surprises and Counterexamples in Real Function Theory

98 Chapter 4

i.e., cp(~) = cp(~ - A), giving f(~ + A) - f(~) = f(~) - f(~ - A). This last equation may be interpreted as the following interesting:

Proposition 4.5.6. Let f: [a, b] -+ ffi. be periodic and continuous. Then it has a chord of any prescribed span (length), not necessarily horizontal, with its mid-point on the graph (see Figure 4.18).

For functions that are continuous but not periodic, the situation can be quite different. A continuous function may have no horizontal chord, as for example, a strictly increasing function. However, the following striking result holds:

Theorem 4.5.7 (The Horizontal Chord Theorem). Suppose that f : [0,1] -+ ffi. has a horizontal chord of length A; say f(O) = f(I), i.e., A = l. Then there are horizontal chords of lengths 1/2,1/3,1/4, ... , but not necessarily a horizontal chord of any other length.

The positive half of this theorem may be proved as follows: Let k = 2,3,4, ... and let g(x) = f(x + 11k) - f(x), so that g has domain [0,1- 11k]. We shall show that ° belongs to the range of g, i.e., that there exists ~ such that g(~) = 0, giving f(~ + 11k) = f(O, showing that f has a horizontal chord of length 11k as required.

Suppose not; then 9 is either positive for all x E [0,1- 11k] or negative for all x E [0,1 - 11k], for otherwise, by IVP, it would vanish at some ~, as required. Say g(x) > ° throughout [0,1 - 11k], i.e., f(x) < f(x + 11k) for all x in [0,1 - 11k]. Taking x = 0, 11k, 2Ik, ... ,(k - 1)lk (all in the interval [0,1- 11k]), we get

f(O) < f(llk) < f(2Ik) < ... < f((k - l)lk) < f(klk) = f(l) = f(O) ,

giving f(O) < f(O), a contradiction. Now consider the function shown in the Figure 4.19. It has a horizontal

chord of length 1 but none of length A with 1/2 < A < l.

('i4 , 'i4)

(%, -%) Figure 4.19

However, we require to show that for each p -::j:. 1 I k (k EN), there exists a function, that is continuous, has a horizontal chord of length 1, but none of

Page 108: Surprises and Counterexamples in Real Function Theory

4.5 Horizontal chords 99

length p for all p E [0,1/2]; so other examples have to be furnished. The following example is due to P.Levy (see [62]).

Let a E (0,1) be fixed with a i- 1/2,1/3,1/4, .... Consider the function

f(x) = sin2 (7rx/a) - xSin2 (7r/a).

Since f(O) = 0, f(l) = 0, we see that f has a horizontal chord of length 1; see Figure 4.20.

o a a+a

Figure 4.20

Now, if f has a horizontal chord of length a, then, there is an 0: such that f (0:) = f(o: + a), i.e., sin2 (7r0:/a) - o:sin2 7r/a = sin2 (7r(0: + a)/a) - (0: + a) sin2 (7r/a), i.e., asin2 7r/a = O. i.e., 7r/a = n7r, with n E N, since a > 0 (being the length of a horizontal chord), i.e., a = l/n (n EN). This contradicts the choice of a.

An interesting complement to the Horizontal Chord Theorem is the follow­ing result due to P.Levy:

Theorem 4.5.8. Suppose f: [0,1) -+ lE. is continuous with f(O) = f(l) and suppose f has no horizontal chord of length a. Then f has two horizontal chords of length 1 - a (0 < a < 1).

Proof. Let 9 be defined on lE. as the periodic extension of f with period 1. As a periodic continuous function, 9 has two horizontal chords of length a, with left hand end points 1'1,1'2 in [0,1) (see the result above Theorem 4.5.6). A horizontal chord of length a for g, starting at l' E [0,1), is a horizontal chord for f, unless l' + a> 1, but then

Figure 4.21

Page 109: Surprises and Counterexamples in Real Function Theory

100 Chapter 4

o < I + a-I < 1 and so a horizontal chord of length 1 - a for g (and also for 1) starts at I + a-I and ends at r-

As an immediate corollary of this, we prove the following result due to Rosenbaum (see [94]):

Theorem 4.5.9. Let f : [0,1] -+ lR be continuous, with f(O) = f(l) and let 71 E N. Then the graph of f has at least 71 horizontal chords whose lengths are integral multiples of 1/71.

Proof. For each of k = 1,2, ... ,71 - 1, if f has no horizontal chord of length a = k/n, then it has two horizontal chords of length 1 - a = (n - k)/n and vice versa. Thus, there always are 2 horizontal chords, either of length k/n or of length (71 - k)/n or one each of length k/n, (71 - k)/n and this is the case for each of k = 1,2, ... ,71 - l.

For 71 odd, this gives 2.(71 - 1)/2 = n - 1 horizontal chords of the required type and for k = 71, there always is a horizontal chord of length 1 = 71.(1/71) (an integral multiple of n). This gives the full set of 71 horizontal chords, as required.

For n even, 71 - 1 is odd and the counting is different, viz. either there are 2 horizontal chords of length k / 71 or two of length (71 - k) / n or one for each of k = 1,2, ... ,(n - 2)/2; in all 2.(71 - 2)/2 = n - 2 plus one oflength 1 = n.(l/n), giving the full set of 71 again.

§4.6. Continuous functions and integration

The usual proof of the result that a continuous function f defined in [a, b] is Riemann integrable uses the more sophisticated result that a continuous function in [a, b] is uniformly continuous. We here give a proof that blends well with the level of sophistication of the statement of the theorem.

With the usual notation, let f be a bounded function in [a, b] and let

P : a = Xo < Xl < ... < X" = b

be a partition of [a, b]. Let mi = infxE [Xi-l,Xi] f(x) and Mi = SUPxE [Xi-l,X;] f(x) and let Sgo and Sgo denote respectively the lower and upper (Darboux) sums, i.e.,

" n

Sgo = L mi(xi - Xi-I) and Sgo = L Mi(Xi - xi-d· i=l i=l

Then

(ii) If P2 is a refinement of PI, then Sgol :S Sgo2' Sgo2 :S Sgol ,

(iii) For any two partitions PI, P2, Sgol :S Sgo2'

Page 110: Surprises and Counterexamples in Real Function Theory

4.6 Continuous functions and integration 101

Letting

rb f = sup Sp la I"

and rb f = inf Sp , la I"

we get

Recall that we say that f is Riemann integrable in [a, bj if f: f = f: f. We have

Theorem 4.6.1. Let f be continuous in [a, b], then f is Riemann integrable in [a, bj.

Proof. (M.J.Norris, [76]). The function f is bounded in [a, bj. Let

cp(x) = l x f(t) dt -lx f(t) dt (a::; x ::; b).

For x ::; b - h, h > 0, we have

t+h rx+h t t" rx+h t+h cp(x + h) - cp(h) = {la f - la f} - {la f - la f} = la f - la f.

Let mh ::; f ::; Mh in [x, x + h], then 0 ::; (cp(x + h) - cp(x)) ::; Mh.h - mh.h, i.e.,

0< cp(x+h)-cp(x) <M -m - h - h h (*)

Now limh--+o Mh and limh--+O mh both exist and equal f(x). Hence, by (*), limh--+o 'P(x+h~-'P(x) exists and is equal to O. Here we are looking at the interval [x, x + h], h > o. A similar argument is available if a - h < x, h < o. It follows that cp' (x) exists and is 0 in (a, b). Further, cp( x) is continuous in [a, bj. Thus,

cp(x) = K, (a constant) in [a, bj and since limx--+a cp(x) = f: f - faa f = 0, we

see that cp(x) == 0 in [a,b], i.e.,

Page 111: Surprises and Counterexamples in Real Function Theory

102 Chapter 4

Let f be a bounded function in [a,b]. Let 4?dx) = J: f, 4?2(X) = J: f, x E [a, b]. The argument, similar to the above, easily yields that 4?~ (x) = f(x) = 4?~(x), at any point x at which f is continuous. Hence, if f is Riemann integrable, then lx U: f) = f(x); and the fundamental theorem of calculus also falls out at almost no extra cost, namely:

Theorem 4.6.2. Let f be a bounded and Riemann integrable funct'ion in [a, b] and let F(x) = J: f, a ~ x ~ b. If f is continuous at a point c E [a, b], then F is differentiable at c and F' ( c) = f ( c) .

Remark 4.6.3. Using the argument similar to the above, it is easy to show that if f is continuous in (a, b) and bounded in [a, b], then f is Riemann integrable in [a,b]. Indeed, with 4?1,4?2 as above, (4?1 - 4?2)'(X) == 0 in (a,b) and 4?1 - 4?2 is continuous in [a,b] and (4?1 - 4?2)(a) = O.

Another interesting result can be proved using the nested interval property.

Theorem 4.6.4. Let f be Riemann integrable in [a, b]. Then the set of points of [a, b], at which f is continuous, is dense in [a, b].

Proof. (K.E.Hirst, [45]). All we need to show is that given any interval [c, dj C [a, b], however small, it has a point of continuity of f, so it is enough to prove that [a, b] has a point of continuity of f (for then, similarly so has [c, dj).

Let E be given, then there exists a 6 such that for any partition p of [a, b], with norm p < 6, we have Sp(f) - sp(f) < E. Choose p with m equal parts so that each part is of length (b - a)/m and that (b - a)/m < 6 and choose E = (b - a)/2. Then

i.e.,

m

2:)Mi - mi) < m/2 ~ m - 2, if m ::::: 4 , (1) i=l

which is the case if we choose m large enough to ensure the following: (i) m ::::: 4 (ii) Norm p = (b - a)/m < 6 (6 corresponding to E = (b - a)/2). Also

(i) =? :3 at least three integers i, 1 ~ i ~ m, for which Mi - mi < 1. (2)

We may thus choose [Xi-l, Xi] = [al, bd say, such that (i) a i- Xi-l, b i- Xi (ii) Mi - mi < 1 (Mi = sup fin [Xi-l.x;], mi = inf fin [Xi-l.x;] ) (iii) Xi - Xi-l = (b - a)/m ~ (b - a)/4 (since m ::::: 4).

Now work with [al' bl ] in exactly the same way as we did for [a, b], the only difference being that we wish to make Mi - mi < 1/2 in (ii) and in general

Page 112: Surprises and Counterexamples in Real Function Theory

4.6 Continuous functions and integration 103

Mi - mi < l/n in the interval [an, bn]. Here the only difference is in the selection of E. We take E = (b - a)/(n + 1) (in [a,b] = [ao,bo], we took it to be (b - a)/2). Then for a suitable 15, for all partitions p of [an-I, bn- 1 ], with Norm p < 15 and length of each subinterval being Xi - Xi-l = (bn - 1 - an -l)/m (note that bn- 1 - an-dim < 15), we have

This implies that

rn

L:)Mi - md < m/(n + 1), if Norm p < 15 . (1)' i=1

It follows that for at least three integers i, 1 ~ i ~ m, we have Mi - mi < l/n, for, if not, then Mi - mi ~ l/n for m - 2 values of i and so

m

:l)Mi - mi) ~ (m - 2)/n. (2)' i=1

Now (1)' and (2)' imply that m-2/n < m/n+1 and hence mn+m-2n-2 < mn =? m < 2(n + 1), which is false if m is chosen large enough to ensure (i) m > 2(n + 1) (for n = 1, i.e., for lao, bo] = [a, b], this condition was m ~ 4), (ii) Norm p = (bn - 1 -an-dim < 15 (15 corresponding to E = (bn- 1 -an-d/(n+ 1)), as required.

We thus get a nested sequence [an, bn] of intervals. Special care has been taken to ensure that at each stage, the subinterval does not share an end point with the "parent" interval. This guarantees that the sequence < an > is strictly increasing and < bn > is strictly decreasing and hence we get a point ~ E n [an, bn] which is actually in n (an, bn). It is easy to see that f is continuous at X =~. For, let E > ° be given. Then by (ii), we have sup f - inf f < l/(n + 1) < E (the sup and inf are taken over [an, bn ] ), if n is large enough. Hence If(x) - f(~)1 < supf - inf f < E, if x,~ E (an,bn); for, x,~ E (an,bn ) is equivalent to ~ E (an,bn ) and Ix-~I < min{bn-~,~-an} = J.

We shall close this section by recording some interesting examples/count­erexamples regarding Riemann integration.

Example 4.6.5. It is well known that the function f on [0,1] defined by f(x) = 0, if x is rational and f(x) = 1, if x is irrational, is not Riemann integrable. It is discontinuous everywhere. Similarly, the function h on [0, 1] defined by h(x) = 1, if x is rational and f(x) = -1, if x is irrational, is not Riemann integrable, whereas the function Ihl is. However, it can be shown that the function g defined on [0,1] by

(x) = {O, when x is irrational or zero

g ~ ,when ° -I x is any rational number ~, in lowest terms and q > °

Page 113: Surprises and Counterexamples in Real Function Theory

104 Chapter 4

is Riemann integrable on [0,1] and the value of the integral is zero. It can also be shown that 9 is discontinuous at each rational and is continuous at each irrational.

Example 4.6.6. Let 1 be defined on [-1,1] by

{1' if x> 0

l(x)= O,ifx=O

-1 , if x < 0

Then 1 is Riemann integrable on [-1,1]' but there is no function F on [-1,1] such that l(x) = F'(x) for all x E [-1,1]' i.e., 1 fails to have a primitive.

Example 4.6.7. The function defined by

1 ( x) = {x2 ~in ~ ,if x =I- 0 O,lfx=O

gives an example of a function having a derivative g(x) at each point x of the closed interval [-1,1]. The function g, therefore, has a primitive, but since 9 is unbounded, it is not Riemann integrable on [-1,1].

Example 4.6.8. Let l(x) == 1, if 0 < x ::; 1 and 1(0) = 0, and if 9 is the function defined on [0, 1] by

(x) = { o, if x is irrational or zero 9 ~ , if x is a rational number ~ in lowest terms and n > 0 ,

then 1 (g( x)) is the function which is 1 for rationals and 0 for irrationals. Thus, 1 0 9 is not Riemann integrable on [0, 1] , whereas both 1 and 9 are.

§4. 7. Continuous functions and their graphs

Let 1 : ]R -+ ]R be a function and denote its graph by G f. Many would tend to think that 1 is continuous if and only if G f (as a subset of ]R2 ) is connected. This two sided equivalence is false, the following being a standard counter example:

Example 4.7.1. Let l(x) = sin 1/x, if x =I- 0, f(O) = O. Then the graph of 1 is connected, as it is easy to see. However, 1 fails to be continuous at x = o.

Page 114: Surprises and Counterexamples in Real Function Theory

4.7 Continuous functions and their graphs 105

Figure 4.22

The converse however is true and is contained in the following:

Theorem 4.7.2. The function f : JR -+ JR is continuous if and only if its graph G j is a closed and connected subset of JR2 .

Remark 4.7.3. In Example 4.7.1 above, the graph Gj of the given function, is not closed. Indeed, all the points (0,0:), 0 < 0: :::; 1 are limit points of G j but do not belong to G j. To see this, let P = (0,0:), with 0 < 0: :::; 1. Then any neighbourhood N of P will have G j passing through it infinitely often (see Figure 4.22), and so P is a limit point of G j .

Proof of Theorem 4.7.2. First, let G j be closed and connected. Since G j is connected, it is easy to see that f will have IVP. So, f : JR -+ JR has the IVP and G j is closed (and connected) and we are to show that f is continuous. Suppose f is not continuous at x = c. Then for some E > 0, there exists a sequence {bn }

of real numbers tending to c, such that If(bn ) - f(c)1 > E for each n, i.e., -E > f(bn ) - f(c) > E. Let us consider the right hand inequality f(bn) > f(c) + Eo

By the IVP, there exists Cn between bn and c such that f(cn) = f(c) + Eo Thus, the sequence of points {(cn' f(cn ))} -+ (c, f(c) +E) ~ Gj, a contradiction, since G j is closed.

Conversely, suppose f is continuous and we have to show that (i) G j is closed and (ii) G j is connected. Proof of (i). If not, then there exists a sequence of points (O:n,f(O:n)) E Gj, O:n -+ 0:, with limit not in G j. However, this limit is (0:, f(o:)) as O:n -+ 0: and I(O:n) -+ 1(0:) (since 1 is continuous), so (0:,1(0:)) ~ G j , which is absurd since for every x E JR, (x,l(x)) E G j, by the definition of G f. Proof of (ii). If not, let Gj C H U K, where Gj = {(0:,1(0:)) I 0: E JR}, H,K f- 0, H n K = 0 and H,K are open. Now put 51 = {o: 1(0:,1(0:)) E H}, 52 = {o: I (0:,1(0:)) E K}. Then 51 ,52 are non-empty, open and disjoint sub­sets of JR such that Sl U 52 = JR, giving a contradiction, since JR is connected.

Definition 4.7.4. We say that a function 1 : JR -+ JR is locally bounded on a set 5 if, for each x E 5, there exists a neighbourhood (x - Ex, X + Ex) of x, in which 1 is bounded.

Page 115: Surprises and Counterexamples in Real Function Theory

106 Chapter 4

Clearly, f bounded implies f locally bounded. The converse, however, is false, for example:

Example 4.7.5. f(x) = x is locally bounded on JR but not bounded. Another example would be f(x) = l/x, if x -j. 0, f(O) = O. This function is locally bounded in (0, (0) but not bounded in (0, (0). The following interesting result holds (parts of which have already been proved):

Theorem 4.7.6. Suppose that f: JR -7 JR has a closed graph Gf (closed as a subset of JR2). Then each of the following conditions implies that f is continu­ous: (i) f is locally bounded, (ii) f has IVP, (iii) G f is connected.

Proof. (i) :::} f is continuous: Suppose f is not continuous at the point x = ~.

Then there exists a sequence {an} -7 ~ such that f(a n) -7 'f) -j. f(~) with'f) E JR, as f is locally bounded. Thus, the sequence of points {(an,f(an))} -7 (~,'f)) in ]R2, contradicting the fact that G f is closed, since the point (~, 'f)) ~ G f and is a limit point of G f. (ii) :::} f is continuous: Already done in the proof of Theorem 4.7.2. (iii) :::} f is continuous: This is the implication {::: of Theorem 4.7.2.

In looking at the converse of Theorem 4.7.6, the hypothesis on G f being closed (in JR2) is redundant, as it follows from the continuity of f (see the con­verse in Theorem 4.7.2). So the converse to Theorem 4.7.6 would read:

Theorem 4.7.7. Suppose that f : JR -7 JR is continuous, then (i) f is locally bounded, (ii) f has the IVP , (iii) G f is connected.

Proof. Here (i) is trivially true; (ii) is a well-known result, while (iii) has already been proved in Theorem 4.7.2 (the converse, (ii)).

However, it would be interesting to see which of the following implications are true or false:

f locally bounded {:} f has the IVP {:} G f is connected,

with no assumption about G f being closed.

Example 4.7.8. Consider the function f(x) = (l/x) sin(l/x), if x -j. 0, 1(0)= o. A rough sketch of the graph of this function may be drawn by the reader. To arrive at it, we note the following properties of this function: (1) f( -x) = f(x), so we need only to look at the region x 2: O. (2) If x is large, l/x is small and positive and so is sin(l/x), hence (l/x) sin(l/x) is small and positive. The last point where the function is zero, is where l/x = 7r, i.e., x = 1/7r ~ .32 ... and when x> 1/7r, 0 < l/x < 7r and so sin(1/x) never vanishes.

Page 116: Surprises and Counterexamples in Real Function Theory

4.8 Convex functions 107

(3) For 0 < .1: < l/,rr, sin(l/x) vanishes infinitely often, viz. at each of the points x = 1/(mr). (4) If need be, we may look at the points at which l' (x) vanishes. We have 1'(x) = -(I/x3 ) cos(l/x) - (l/x2 ) sin(l/x) and this is equal to 0 if tan(l/x) = -(I/x). Solutions of this are awkward. We may draw graphs of y = tan x and of y = -1/ x and check the intersections. All this if need be, but for a rough sketch, it is not necessary.

This function has the IVP but it is not locally bounded at the point x = 0; hence f has IVP does not imply f is locally bounded. Further, for this very function f, its graph is connected, showing G f is connected does not imply f is locally bounded.

Example 4.7.9. Now consider the function

g ( x) = {I , if x > 0 0, if x ~ 0

This immediately shows that f is locally bounded does not imply f has the IVP and also that f is locally bounded does not imply its graph G f is connected.

Remark 4.7.10. It has already been proved (see step 1 in the proof of Theorem 4.7.2) that Gf is connected implies f has the IVP. It finally remains to analyze the implication or otherwise: f has the IVP implies its graph G f is connected, and it turns out that the answer is negative, i.e., f has the IVP does not imply that its graph G f is connected. The counterexample is not easy; it is a slight modification of a function described by F.B.Jones in [51]. This modified function, and other relevant material is given in [19].

§4.8. Convex functions

The function f: [a,b] --+ lR is said to be convex (downwards) if between any two points Xl,X2 E [a,b], the curve y = f(x) lies below or on the chord joining the points Al == (xl,f(xd) and A2 == (X2,J(X2)) of the graph Gf off (see Figure 4.23).

p

Q

A

x X2

Figure 4.23

Page 117: Surprises and Counterexamples in Real Function Theory

108 Chapter 4

The condition simply means that AQ :S AP, i.e., f(x) is less than or equal to the y-coordinate of P, i.e., f(x) :S (f(X2)- f(xd)((x-xd/(X2 -xd)+ f(xd, which on simplification becomes

Since x = AXI + (1- A)x2 , 0 :S A :S 1, this condition (*) becomes the equivalent condition

Another equivalent condition is the symmetrical form (easy to check):

(X3 - x2)·f(xd + (Xl - x3)·f(X2) + (X2 - :x:d.f(X3) > ° (*)z (Xl - X2)(X2 - X3)(X3 - xd - ,

for all distinct Xl, X2, X3 E [a, b], Xl < X2 < X3, which may be written in the form of a determinant:

1 Xl f(xd 1 X2 f(X2) 2': ° . 1 X3 f(X3)

Theorem 4.8.1. Suppose f is convex in [a,b]. Then f is continuous in [a,b].

Proof. Let XI,X2,X3 be any three points of [a,b]. By (*), we have

Take X E ( a, b) and h > 0, sufficiently small, and XI = X - h, X2 = X, X3 = X + h; we get

h h f(x) :S 2h f(x) + 2h f(x + h) ,

which implies that (taking h --+ 0) f(x) :S limHx+ f(t). Also, taking Xl X, X2 = X + h, X3 = X + 2h, we get

1 1 f(x + h) :S 2f (x) + 2f (x + 2h),

so that limHx+ f(t) :S f(x). Thus, f(x) = limHx+ f(t). Similar considera­tions yields that f(x) = limHx- f(t). So that f(:x:) = limHx f(t) and f is continuous in [a, b]

Remark 4.8.2. The function f may fail to be continuous at a and/or at b; for example, if f(x) == ° in (0,1), while at X = 0,1, f(x) = 1.

Example 4.8.3. A convex function, in an open or a half-open interval, may be unbounded, for example, let f(x) = tan x in [0,7r/2).

Remark 4.8.4. If we put X = (Xl + x2)/2 in (*), we obtain

Page 118: Surprises and Counterexamples in Real Function Theory

Exercises 109

which is sometimes taken as the definition of convexity (rather than (*)). We call it the mid-convexity condition and any function f satisfying it, is called mid-convex. It is less restrictive than (*), since it assumes (*) for the mid-point (Xl + x2)/2 of Xl and X2. Indeed, this condition does not imply continuity of the function. To achieve continuity, we have to put further restrictions on f. One such restriction is that f be bounded above. Indeed, the following stronger result is also known to hold, namely: Suppose that f is mid-convex on [a, b], and that f be bounded above in a neighbourhood of a single point ~ of [a, b], then f is continuous at all points of (a, b). The condition f be bounded above, is essential. Indeed, there exists mid-convex functions that are discontinuous everywhere but not bounded in any interval. Here is an example:

Example 4.8.5. Let B = {bihE I be a basis of the vector space lR over Q. Define f on B by putting f(bi) to be equal to an arbitrary ri E Q. Then if x = L, O'.ibi (O'.i E Q,O'.i = 0 for all except finitely many i), we let f(x) = L, ad(bi) = L, airi E Q. It is now trivial to check that if y = L, f3ibi (f3i E Q, f3i = 0 for all but finitely many i), then (**) holds, with equality sign, i.e., f((x + y)/2) :::; (f(x) + f(y))/2; so f is mid-convex. However, f can not be continuous, as a continuous rational valued function would be a constant, which our f is not.

For all this, and much more, see the book by Wayne Roberts and Varberg [116].

Exercises

4.1. Show that the function F(x) = fox sin(l/t)dt is differentiable at x = 0 (note that sin(ljt) is bounded and continuous everywhere, except at x = 0 and so is Riemann integrable over any bounded interval; thus F(x) is defined).

4.2. Let A be the area under the curve y = eX lx, between the x-axis and the two ordinates x = a, x = a + 1. Locate 0'. for which A has the least value.

4.3. Prove the geometric, logarithmic, arithmetic mean inequalities:

IT b-a a+b vab < 1 b 1 < -2- , 0 < a < b, og - oga

by considering the area under the curve y = eX, between the ordinates x = log a, x = log b and certain trapezoia involving the point C = (log atlog b ,0).

4.4. Suppose f is continuous in [0, n] and f(O) = f(n) = O. Then f(x ' ) = f(x") has at least n different solutions with x" - x' E N.

4.5. Let f : lR -t lR be a continuous function and denote the nth iterate of f by r, i.e., P(x) = f(x),P(x) = f(f(x)), ... ,r(x) = f(r-l(x)). A number x is said to be a point of period k for f if fk(x) = x, but fi(x) f. x for 0 < i < k. Prove that if f has points of period k > 1, then f has a fixed point, i.e., a point

Page 119: Surprises and Counterexamples in Real Function Theory

110 Chapter 4

of period l.

4.6. R. Kannan proved (in Bull. Calcutta Math. Soc., 60 (1968),71-76) that if f is a real-valued function such that If(x) - f(y)1 = c(lx - f(x)1 + Iy - f(y)l) for all x, y E ]E., 0 :s c < 1/2, then f has a unique fixed point, i.e., there exists a unique ~ such that f(~) = ~.

Prove that if f is a real-valued function such that If(x) - f(y)1 = c(lx -f(y)1 + Iy - f(x)l), for all X,y E ]E., where 0 :s c < 1/2, then f has a unique fixed point, i.e., there exists a real number E. such that f(O = ~ (B.Fisher, Math. Mag., 48 (1975),223-225).

Page 120: Surprises and Counterexamples in Real Function Theory

111

Chapter 5

The Derivative and Higher Derivatives

§5.1. Continuity of the first derivative

The derivative of a function, even if it exist everywhere, can be discontin­uous. For example, if f(x) = x2 sin~, for x -::j:. 0, f(O) = 0, then for x -::j:. 0, 1'(x) = - cos ~ + 2xsin~, which does not tend to any limit as x tends to 0, although 1'(0) exists and equals 0, as may be easily checked. Thus, f'(x) exists everywhere, but is discontinuous at x = 0.

Recall that a function f is said to have a removable discontinuity at x = c if limx-+c f(x) exists but is not equal to f(c) (which mayor may not exist). The function f is said to have a discontinuity of the first kind (or a jump discontinuity) at a point c, if both limx-+c+ f (x) and limx-+c- f (x) exist but are not equal and f is said to have a discontinuity of the second kind at a point c if either one or both limx-+c+ f(x) and limx-+c- f(x) do not exist. A remarkable result is:

Theorem 5.1.1. The derivative of a function can have a discontinuity only of the second kind.

Proof. We must rule out the two possibilities:

• l' has a removable discontinuity at a point x = a, say;

• l' has a jump discontinuity at x = a.

Now l' ( a) = lim f ( x) - f ( a )

x-+a X - a

and here, since the numerator and the denominator both vanish at x = a, L'Hospital 's rule (see §5.7) may be applied to evaluate this limit and yields

f'(a) = lim !xU(x) - f(a)) = lim f'(x). x-+a lx (x - a) x-+a

Thus, f' can not have a removable discontinuity or singularity. To rule out the second possibility, it is enough to prove IVP for 1', i.e., if

f'(a) < c < 1'(b), then there exists a number ~ E (a, b) such that 1'(~) = c To this end, let ¢(x) = f(x) - cx. Then ¢'(a) < 0, ¢/(b) > ° and we have

to find a number ~ in (a, b) such that ¢' (0 = 0.

Page 121: Surprises and Counterexamples in Real Function Theory

112 Chapter 5

Since ¢' exists in [a, b], ¢ is continuous and so bounded below and attains its infimum in [a, b), call it m, i.e., ¢(O = m = inf ¢(x). Then ¢'(a) < 0 implies that {¢(x) - ¢(a)} /(x - a) < 0 for all x E (a, a + J). Thus, ¢(x) < ¢(a) for all x E (a, a + J), and hence ~ -j a; similarly ~ -j b, i.e., ~ E (a, b).

Now, (¢(x) - ¢(~))/(x -~) 2:: 0 as x -+ ~+, and is less than or equal to 0 as x -+ ~-, i.e., ¢~(~) 2:: 0, ¢'-(~) :::; 0, but ¢~(~) = ¢'-(~) = ¢'(~), since ¢'(~) exists, hence ¢'(O = 0, as required.

Remark 5.1.2. The following argument is offered as a challenge to students by Albert Wilansky (Math. Mag., 38 (1965), 108). (i) Give an example of a function, differentiable everywhere, but with a discon­tinuous derivative at x = 0, (ii) pinpoint precisely where the fallacy in the argument below lies:

Let] be differentiable in an open interval I and let [a, b) c I. Let ~ E [a, b). For k -j 0, let E = (f(~ + k) - ](O)/k - 1'(~). Then

](~ + k) = ](0 + k1'(O + kE , (*)

where E -+ 0 as k -+ O. In (*), taking successively (a) ~ = 0, k = 2h (b) ~ = 0, k = h (c) ~ = k = h, we get

](2h) = ](0) + 2h1'(0) + 2hEl

](h) = ](0) + h1'(O) + hE2

](2h) = ](h) + h1'(h) + hE3

(1)

(2)

(3)

Now (3) implies ]'(h) = (f(2h) - ](h))/h - E3. Substitute for ](2h) and ](h) from (1) and (2) and we get l' (h) = 1'(0) + 2El - E2 - E3 -+ 0 as h -+ 0, showing that l' (x) is continuous at x = O!

The following necessary and sufficient condition for the derivative l' of a function] to be continuous is due to Weinstock [117):

Theorem 5.1.3. Let I be an open interval of which [a, b) is a closed subinterval and suppose f is differentiable in I. Then a necessary and sufficient condition for l' to be continuous in [a, b] is that] be uniformly differentiable in [a, b], i.e., given E > 0, there exists a J = J(E), J independent of x, such that

l(f(x + h) - ](x))/h - 1'(x)1 < E

for all 0 < Ihl < J and for all a:::; x :::; b, x ± hE I.

Proof. By hypothesis, if {hi} is any sequence of non-zero numbers, with x ± hi E I for all x E [a, b], which tends to 0, then the sequence of continuous functions {f(x+hi ) - ](x)/hd is convergent uniformly to ]'(x) in [a, b]. Hence the limit l' (x) of this sequence is continuous in [a, b], as required.

Conversely, by the Mean Value Theorem (see Theorem 5.4.2)

f(x + h~ - f(x) _ 1'(x) = 1'(x + 8h) - 1'(x) , 0 < 8 < 1 , (i)

Page 122: Surprises and Counterexamples in Real Function Theory

5.2 The tangent and the first derivative 113

for each x E [a, b], x + h E I. Since, by hypothesis , l' is continuous and so uniformly continuous in [a, b]'

there exists, for each E > 0, a b = b(E) > 0 (b independent of x) such that 11'(x + Bh) - f'(x)1 < E, whenever 0 < IBhl < b, i.e., whenever 0 < Ihl < o. Therefore, f is uniformly differentiable by (i) and the above observation.

However, the derivative of a function can be unbounded:

Example 5.1.4. The function

has the derivative

f(x) = {X2 s.in ;2 , if x -I- 0 0, If x = 0

f ' () {2X sin --% - £ cos --% , if x -I- 0 x = x x x 0, if x = 0

which is unbounded on the closed interval [-1,1].

After introducing the derivative, the following results are usually discussed in a first course in Calculus: (i) The relationship between the derivative at a point P of y = f(x) and the tangent to the graph of y = f(x) at the point P, (ii) Rolle's Theorem, (iii) The First Mean Value Theorem, (iv) 1'(x) = 0 implies that f is a constant function, (v) Taylor'S Theorem etc.

We shall look at these results and ideas one by one, starting with the first.

§5.2. The tangent and the first derivative

Intuitively, aided by geometry, one may tend to infer that the tangent to the graph G, ofy = f(x) exists at the point (c,f(c» ofG, if and only if f is differentiable at x = c. Intuition is certainly a correct approach to take. However, intuitive results must always be coupled with logical reasoning. We have:

Example 5.2.1. Let f(x) = ~ if Ixl ::; 1, x rational and f(x) = -~ if Ixl ::; 1, x irrational.

The graph of f is a "ghost circle" (see Figure 5.1) and has a tangent at the point (0,1) (see Definition 5.2.3); but f is not differentiable at x = 0, since it is not even continuous (see [Ill])!

The correct result in this direction is the following:

Theorem 5.2.2. (H.Thurston [111]) . If the derivative of a function f exists at a point x = c, then the tangent to the graph G, of y = f (x) exists at the point P == (c, f(c». If the tangent to G, exists at a point P == (c , f(c» and f

Page 123: Surprises and Counterexamples in Real Function Theory

114 Chapter 5

is continuous at x = c, then the derivative of f exists at x = c.

Before we give a proof of this result , we would like to give the precise definition of what we mean by a tangent to a curve X at a point P on the curve (without going into the tricky question of what a curve is).

gap

x rational

gap

Figure 5.1

Definition 5.2.3. Let :F be any set of points in the plane ]R2 and let Po be a nonisolated point of:F. A line L through Po is a tangent to :F at Po if for given f > 0, there exists a is > 0 such that the angle e between PoP and L is less than f for every point P of:F within a distance is of Po (see Figure 5.2).

p

Figure 5.2 Figure 5.3

Thus, according to this definition, the 'figure of 8' (Figure 5.3) has no tangent at Po where it crosses itself.

Proof of Theorem 5.2.2. We first show that if f is differentiable at x = c, the line L through Po with slope f' (c) is a tangent to the graph of f at Po· Po is not an isolated point of the graph because f is continuous at x = c and c is a limit point of the domain of f.

Now, for each f > 0, there exists an 1] > 0 such that the lines with slopes between f' (c) - 1] and f' (c) + rj make angles less than f with L (continuity of

Page 124: Surprises and Counterexamples in Real Function Theory

5.2 The tangent and the first derivative 115

trigonometric functions). For this 7), there exists a 8 > 0 such that

I I(C+hJ,-/(c) I < 7), whenever 0 < Ihl < 6 (c + h in the domain of f), (*)

because f is differentiable at x = c.

tan 81 = f'(c) - Y] ,

tan 8 = f'(c) , tan 82 = f I (c) + Y] •

Figure 5.4

both angles < E

f (c)

c

Thus, whenever 0 < iPoPhl < 8, c + h in the domain of f and 0 < Ihl < 8 (see Figure 5.5), (*) holds, i.e., f' (c) - 7) < (j(c + h) - f(c)) /h < f' (c) + 7), i.e.,

8 Po

f(c+h)

c c+h c c+h

Figure 5.5 Figure 5.6

the slope of POPh lies between l' ( c) ± 7) and so POPh makes an angle less than E with L, i.e., by definition, L is the tangent at the point (c, f(c)) to GI , as required.

We now come to the proof of the second part of the theorem. Suppose that f is continuous at x = c and the graph of f has a nonvertical tangent T at Po. We then show that f is differentiable .at x = c.

Let E > 0 be given and let L be the tangent at Po making an angle a with the x-axis, so that tana = m (m = the slope of L). Let Ph be a point on the graph close to Po (see Figure 5.6) (note that in general, h close to 0 does not

Page 125: Surprises and Counterexamples in Real Function Theory

116 Chapter 5

mean that Ph is close to Po; for example, in our ghost circle, if we choose h small but irrational, Ph is in the lower semi circle and the chord POPh is not small I). Then by the definition of "tangent", the angle e between POPh and L is small and so tane is small, i.e., tan(jJ-a) is small, i.e., Islope of POPh - slope of LI is small, i.e.,

I f(c+h2- f (c) - ml is small, i.e., small enough to be < E , (**)

whenever IPOPhl < 77 (see Figure 5.7).

L

f(c+h)

3 y= f(x) f(c)

Figure 5.7

As f is continuous at x = c, I Po Ph I < 77 if Ihl < 15 (h small implies that If(c+ h) - f(c)1 is small and hence I Po Ph I is small). Hence Ihl < 15 :::} iPoPhl < 77 :::} (** ), i.e., limh-+o (f (c + h) - f ( c)) / h exists and equals m, i.e., f is differentiable at c and f'(c) = m.

§5.3. Implications of first derivative being zero

In this section we analyse the situation when the first derivative of a function is zero. The usual proofs of the result that if f' (x) = 0 for all x E [a, b], for a continuous function defined on [a, b], then f is a constant function, uses the First Mean Value Theorem, which uses properties of continuous functions such as boundedness, attainment of the bounds etc. We now give two direct proofs of this result. We first prove:

Theorem 5.3.1. Let f be a contin1Wus function defined on [a, b]. If f'(x) > 0 for a < x < b, then f is strictly increasing in [a, b]. Also, if f'(x) 2: 0 for a < x < b, then f is nondecreasing in [a, b].

Page 126: Surprises and Counterexamples in Real Function Theory

5.3 Implications of first derivative being zero 117

Proof ([11]). Let p E ( a, b) be any point. We wish to prove that

a < q < p =? f(q) < f(p)

Let S = {x I a < x < p, f(x) 2: f(p)}. Then it is enough to prove that S = 0. Suppose not. Let q = supS. Now f'(p) > 0, so (J(p+h) - f(p))/h > 0 if Ihl is small. Taking h < 0, this shows f(p+h) < f(p) if h is sufficiently small and negative. It follows that q < p, i.e., that a < q < p.

h h

• • • ( • ) • a x q p b

Figure 5.8

Case 1: f(q) 2: f(p)· Since f'(q) > 0, (J(q + h) - f(q))/h > 0, if h is small. Take h small and positive. Then f(q + h) > f(q) 2: f(p), i.e., f(q + h) > f(p), so q + h E S, which is a contradiction since q = sup S. Case 2: f(q) < f(p). Then q rJ. S and by the continuity of f (at q), no point of a small neighbourhood of q is in S (J(q) < f(p) =? f(x) < f(p) if x belongs to a small neighbourhood of q). In particular [q - h, q] n S = 0, which is again a contradiction since q = sup S.

For the second part, let g(x) = f(x) + EX, where E > 0 is chosen suitably small. Then g'(x) = f'(x) +E > 0 for a::; x::; b, so, a fortiori, for a < x < b we can apply the above result to 9 to conclude that 9 is strictly increasing, i.e.,

since E is arbitrary small.

Xl > X2 =? g(xd > g(X2) =? f(xd + EXl > f(X2) + EX2 =? f(xd > f(X2) - E(Xl - X2) =? f(xd 2: f(X2) ,

Theorem 5.3.2. Let f be a continuous function in [a,b] such that f'(x) = 0 for all x E (a, b). Then f is a constant function.

Proof. Observe that f'(x) = 0 =? f'(x) 2: 0 and f'(x) ::; 0, which together imply that f is nondecreasing and that f is nonincreasing. Thus f is a constant function.

Another direct proof of Theorem 5.3.2 ([92]). Let f'(x) == 0 on [a, b]. If f is not a constant, as claimed, then, for some a ::; u < v ::; b, f(u) -::j:. f(v). Hence the chord joining (u, f (u)) and (v, f (v)) has a nonzero slope, i.e.,

f(v) - f(u) = c(v - u) (c -::j:. 0) .

Page 127: Surprises and Counterexamples in Real Function Theory

118 Chapter 5

In this diagram c < 0

feu)

a u v b

Figure 5.9

First, let c > 0. Bisect [u, v] at w. If

f(v) - f(w) < c(v - w) and f(w) - f(u) < c(w - u) (t)

then f( v) - f( u) < c( v -u), which contradicts (*). Hence, for at least one ofthe intervals [u, w], [w, v], (t) fails. Call this interval [Ul' Vl]. By repeated bisection, we obtain a sequence of nested intervals [u,v] = [uo,vo] :J [Ul,Vl] :J ... for each of which f(vj) - f(uj) ;::: c(Vj - Uj). Let ~ = n [Uj, Vj] ~ [u, v]. Hence, in the case c > 0, (f(Vj) - f(uj))/(Vj - Uj) --+ f'(~) ;::: c > 0, which is a contradiction, since f'(x) == ° on [a, b].

If c < 0, the same argument as above, with inequalities reversed, yields a contradiction.

It follows that c = 0, which is again a contradiction to (*). Hence f(u) = f(v).

Remark 5.3.3. The same argument shows that if f'(x) ;::: ° on [a,b], then f(v) ;::: f(u), while if f'(x) :::; ° on [a, b], then f(v) :::; f(u). Indeed, the argument may be generalized to show that if m :::; f'(x) :::; M in [a, b], then m(v - u) :::; f(v) - f(u) :::; M(v - u), namely, it is sufficient to replace c > ° by c > M and c < ° by c < m.

Some more direct proofs of Theorem 5.3.2 are given in [26] and [83]. A rather specialized result is the following:

Theorem 5.3.4. Let f be a continuous function defined in an interval I. If the right derivative of f at each interior point of I exists and is zero, then f is constant in I.

The following lemmas are needed for the proof.

Lemma 5.3.5. Let f be continuous in [a, b], f(a) = f(b) = 0. If the right derivative f~ (x) of f exists at each point x E (a, b), then there exists a point ~ E (a, b) at which f~(~) :::; 0.

Proof. If f is a constant, the lemma is trivially true; so suppose otherwise. Since f is continuous, it assumes either a positive maximum or a negative minimum. We consider these cases separately.

Page 128: Surprises and Counterexamples in Real Function Theory

5.3 Implications of first derivative being zero 119

M

a S b

Figure 5.10 Figure 5.11

Case 1: f has a positive maximum, say at ~ E (a, b), see Figure 5.10 (~ 'I a, bas f(a) = f(b) = 0); so that f(x) ~ f(~) for all x E [a, b]. Then (f(x) - f(~))/(x­~) ~ 0 for all ~ < x ~ b and letting x --+ ~+ gives f~ (~) ~ 0, as required. Case 2: f has a negative minimum, say at d E (a, b), see Figure 5.11. Now look at f~(x) for x E (a,d). If f~(x) ~ 0 for all x E (a, d), then the result is true. If not, choose a point c E (a, d) such that f~(c) > O. Then for all x sufficiently near c, on the right of c, we have (f(x) - f(c))/(x - c) > 0 for x E (c, c + 6). Hence f(x) > f(c) for x E (c, c + 6). Now, let M be the maximum value of f on [c, d]. Since f(x) > f(c) for all x E (c, c + 6) and f(d) is the negative minimum, we have M > f(c) 2': f(d), so f assumes the value M at some point ~ E (c, d). This leads us to case 1, with [c, d] as the initial interval I and M the maximum of fin [c, d], attained at x = ~ E (c, d), i.e., f(~) = M. Hence by case 1, f~ (0 ~ O.

Lelllllla 5.3.6. Let f be continuous in [a, b], f(a) = f(b) = O. If f has a right derivative at each point in (a, b), then there exists a point TJ E (a, b) at which f~(TJ) 2': O.

Proof. Apply Lemma 5.3.5 to - f. Lelllllla 5.3.7. Let f be continuous in [a, b]. If f has a right derivative at each point of ( a, b), then there exist points ~,TJ in ( a, b) such that

f~(~) ~ f(b~ = ~(a) ~ f~(TJ).

Proof. Let g(x) = f(x) - f(a) - [(f(b) - f(a))/(b - a)].(x - a), so that 9 is continuous in the interval [a, b], g(a) = g(b) = 0 and g~(x) exists for all x E (a, b). Hence Lemmas 5.3.5 and 5.3.6 hold for g, giving the required result.

Proof of Theorelll 5.3.4. Choose points a, bEl, a < b, and apply Lemma 5.3.7 to f in [a, b]. Since f~(x) == 0 in (a, b), by hypothesis, it follows from

Page 129: Surprises and Counterexamples in Real Function Theory

120 Chapter 5

above that f(a) = f(b). We have thus proved that for any two points a, bEl, f(a) = f(b) and hence f is a constant function.

Remark 5.3.8. We would like to remind the reader that the first derivative being equal to zero implies that the function is constant, is true only for inter­vals. Otherwise, if f : [1,2] U [3,4] -t lR is defined by f(x) = 1 for x E [1,2] and f(x) = 2 for x E [3,4]' then 1'(x) = 0 for all x E (1,2) U (3,4).

Some relevant counterexamples regarding monotonicity and extreme values are:

Example 5.3.9. The function

has an absolute minimum value at x = O. Its derivative is

1'(x) = {X2(:X(2 + sin~) - cos~) , if x:f. 0 O,lfx=O,

which has both positive and negative values in every neighbourhood of O. So f is not monotone in any interval of the form (a,O) or (0, b). Thus f has an extreme value at a point where the derivative does not make a simple change in sign.

Example 5.3.10. The function

has the derivative

f(x) = {x +.2X2 sin ~ , if x:f. 0 0, If x = 0 ,

1'(x) = {1 +.4xsin ~ - 2cos ~ , if x:f. 0 0, If x = O.

In every neighbourhood of 0, the function 1'(x) has both positive and negative values, however, l' (0) > 0, yet f is not monotone in any neighbourhood of O.

Example 5.3.11 [37]. The function

f(x) = x e~4 sm x3 , IX -r { 4 1 x 2 • 8 'f -I- 0

0, If x = 0

has the derivative

j'(x) = e 4. x - 2 X sIn x3 - cos x3 ,1 x-r { - 1 x 2 [(4 3 1 5)' 8 24 8 1 . f -I- 0

0, If x = O.

Page 130: Surprises and Counterexamples in Real Function Theory

5.4 Rolle's Theorem and the first Mean Value Theorem 121

In every neighbourhood of the origin, this derivative has values arbitrarily near both 24 and -24. Also, for 0 < h::::; 1 and for x = h,

1 2 1 2 1 h 2 3 2 0< e-:;;-x < 1 - -h e-:;;- < 1- -h 4 16 '

and

4x - -x sm - - 24 cos - < 24 + - h . 1 ( 3 1 5) . 8 81 9 3 2 x 3 x 3 - 2

Therefore, 0 < h ::::; 1 implies that

Thus, on the closed interval [-1, 1], the range of f' has supremum equal to 24 and infimum equal to - 24 and neither of these numbers is assumed as a value for f'.

§5.4. Rolle's Theorem and the first Mean Value Theorem

The usual proof of Rolle's Theorem uses the result that the function f is bounded, f attains its maximum at x = ~, say, and then show that f'(~) = O. We give here a new proof using only the intermediate value theorem.

Theorem 5.4.1 (Rolle'8 Theorem). Suppose l' exists in ( a, b), f is contin­uous in [a, b], and f(a) = f(b); then there exists ~ E (a, b) such that f'(~) = O.

Proof (H.Samelson, [99]). Write [a, bl = lao, bolo We first prove that:

there exists an interval [aI, bd C lao, bo], with bl - al = (bo - ao)/2, such that f(al) = f(b l ). (*)

Let g(x) = f(x + (b - a)/2) - f(x). Then

g((a + b)/2) = f((a + b)/2 + (b - a)/2) - f((a + b)/2)

= f(b) - f((a + b)/2)

= - f((a + b)/2) + f(b)

= - f(a + (b - a)/2) + f(a) (since f(a) = f(b))

= -(f(a + (b - a)/2) - f(a))

= -g(a).

Hence, by the intermediate value theorem, g(~) = 0 for some ~ E (a, (a + b)/2), i.e., f(~ + (b - a)/2) = f(~) and letting [aI, bl ] = [~, ~ + (b - a)/2], (*) follows.

Page 131: Surprises and Counterexamples in Real Function Theory

122 Chapter 5

P (% (a+b) , g(% (a+b)))

a % (a+b) b (a, g(a))

Figure 5.12 Figure 5.13

Note that bl - al = ~ + (b - a)/2 - ~ = (b - a)/2, as required and also that since ~ < (a + b)/2, we have ~ + (b - a)/2 < (a + b)/2 + (b - a)/2 = b and so [~, ~ + (b - a)/2] = [aI, b1] C [a, b].

Now apply (*) repeatedly to get a sequence [an, bn] of nested intervals such that

(i) bn -an =(b-a)/2n ,

(ii) [an, bn] C [an-I, bn-d ,n = 1,2, ... ,

Then an increases, bn decreases, and both are bounded and so they both tend to limits, say ~,TJ where ~ = TJ (by (i)) and an::; ~::; bn·

~ ~ ~ ~ a % (3a+b) Y, (a+b) b

Figure 5.14

It follows that (f(bn) - J(an))/(bn - an) ~ J'(~); but by (iii), the quantities (f(bn) - J(an))(bn - an) are all equal to 0, giving f'(~) = O.

Finally, note that this ~ -::p a or b for, suppose for example that ~ = a. Then [aI, bl ] = [~, ~ + (b - a)/2] = [a, (a + b)/2], and [a2, b2 ] = [~, ~ + (b l - al)/2] = [a, (3a + b)/4] (check this last one), so that J(a) = J( (a + b) /2) = J((3a + b) /4) (observe that J(an) = J(bn)). But then we may choose [az, b2] to be equal to [(3a+b)/4, (a+b)/2] rather than equal to [a, (3a+b)/4] and then subsequently there is no danger of the limit ~ of an (or bn) being equal to a. Similarly, ~ -::p b.

Observe that this really is a different proof. The point ~ is not necessarily an absolute maximum or a minimum, nor even necessarily a local maximum or minimum. For yet another proof of Rolle's Theorem see [1].

Following Chorlton [25], we now discuss mean value theorems. The first

Page 132: Surprises and Counterexamples in Real Function Theory

5.4 Rolle's Theorem and the first Mean Value Theorem 123

mean value theorem (MVT) is:

Theorem 5.4.2. Suppose f is continuous in [a, a + hl and differentiable in (a, a + h); then there exists a real number 0, 0 < 0 < 1, such that

f(a + h~ - f(a) = f'(a + Oh).

In general, this number 0 depends on a and h. However, if f is a quadratic polynomial: f(x) = A + Bx + Cx2, then in any interval [a, a + h], it turns out that 0 = 1/2. For, we have

f(a + h) - f(a)

h

as required.

(A + B(a + h) + C(a + h)2) - (A + Ba + Ca2)

h =B+2C(a+h/2)

= f'(a + h/2) ,

Geometrically, this simply means the following: Consider the parabola

P : y = A + Bx + cx2

represented by a general polynomial of degree 2. Let Pl == (Xl, Yl), P2 == (X2, Y2) be any two points on P. Then the abscissa x* of the point P* = (x*, y*) at which the tangent to P is parallel to the chord Pl P2 is the arithmetic mean of the abscissae of the end points of the chord HP2, i.e., x* = (Xl +x2)/2. This may be directly verified using co-ordinate geometry. Analytically it translates into the following:

Theorem 5.4.3. The chord Pl P2 is parallel to the tangent at P*, where, if H == (xl,yd, P2 == (X2,Y2), then P* == ((Xl + x2)/2,y*).

Proof. We are required to prove that the slope of Pl P2 = (dy/dx)p* . Here the left hand side equals (see Figure 5.15)

X2 - Xl

A + BX2 + Cx~ - A - BXl - Cxi

X2 - Xl

= B + C(Xl + X2) ,

while the right hand side equals (B+2C x)p* = B+2C(Xl +x2)/2, as required.

Incidentally, this affords a means of constructing the tangent at any point P == (x, y) of P. For, let Pl , P2 be any points with abscissa x - h, x + h (where X is the abscissa of P). Then the line through P, parallel to Pl P2 is the tangent to Pat P.

Page 133: Surprises and Counterexamples in Real Function Theory

124 Chapter 5

Our object now is to investigate the converse problem, i.e., to determine which real valued functions f have the property that there exists a fixed 0 such that for all a, h, f(a + h) - f(a) = hf'(a + Oh).

Figure 5.15

We have the following remarkable

Theorem 5.4.4. Let f be a real valued differentiable function and suppose that the equation (i.e., the first MVT)

f(a + h) - f(a) = hf'(a + Oh)

holds for all values of a and h, with 0 independent of a and h. Then

f(x) = C + Ex + AX2 .

Proof (T.W. Chaundy, [24]). In (*), first write -h for h and then replace a

by a + h to get

f(a + h) - f(a) = hf'(a + (1 - O)h); (1)

then (*) and (1) imply, that for all a, h

f'(a + Oh) = f'(a + (1 - O)h) . (2)

But x = a + Oh and y = a + (1 - O)h can take all real values x and y, for suitable a and h, provided 0 i= 1/2. Indeed, given x, y, subtracting the second from the first, we get h = (x-y)/(20-1) and then a = x-O((x-y)/(20-1)), so for these values of h and a, any given x and y can be got. So now (2) can be interpreted as saying

f'(x) = f'(y) for all x,y,

i.e., that f' is a constant, say B, which then gives f(x) = C + Bx.

Page 134: Surprises and Counterexamples in Real Function Theory

5.4 Rolle's Theorem and the first Mean Value Theorem 125

Now let 0 = 1/2. Then (*) reads:

f(a + h) - f(a) = hf'(a + h/2) ; (3)

In this, let (i) a ---+ a, h ---+ -h (ii) a ---+ a - h, h ---+ 2h, to get the equations

f(a - h) - f(a) = -hf'(a - h/2) ;

f(a + h) - f(a - h) = 2hf'(a).

(4)

(5)

Now (4)+(5)-(3) implies that f'(a + h/2) + f'(a - h/2) = 2f'(a). In this 'equation substituting (i) a = (x+t)/2, h = x+t and (ii) a = (x+t)/2, h = x-t yields respectively f'(x+t) + 1'(0) = 2f'((x+t)/2), f'(x)+ f'(t) = 2f'((x+t)/2) and these give

f'(x + t) - f'(x) = f'(t) - 1'(0) . (6)

Now let ¢(t) = f'(t) - 1'(0) = f'(x + t) - f'(x) (for all x, by (6)). Then

(f(x + y) - f(x)) - (f(y) - f(O)) =yf'(x + y/2) - yf'(y/2) (by (3))

=y(f'(x) - 1'(0)) (by (6))

=y¢(x) (by definition of ¢)

=x¢(y) ,

by symmetry in x, y, since the extreme left hand side ofthe above is symmetric in x and y.

Hence ¢(x)/x = ¢(y)/y (for all x, y) and so is a constant, say K, i.e., ¢(x) = Kx and so by the definition of ¢, we get f'(x+t) - f'(x) = Kt = K(x+t) -Kx, i.e., f'(x + t) - K(x + t) = f'(x) - Kx (for all t). Now writing y for x + t, this gives f'(y) - J(y = f'(x) - Kx (for all x, y), i.e., that f'(x) - Kx is a constant, say B, or f'(x) = B + J(x, or f(x) = Kx 2 /2 + Bx + C = AX2 + Bx + C.

In this connection, see also [89].

ReIllark 5.4.5. If we assume f to be 3 times differentiable, we can get our result much more easily as follows:

Differentiate the equation (*), i.e., f(a + h) - f(a) = hf'(a + Oh), with respect to h, partially, r times to get (using Leibnitz theorem)

f(r)(a + h) = hf(r+ll(a + Oh).or + G) ddh f(r)(a + Oh).or-l + 0 + 0 + ...

= her f(rH)(a + Oh) + ror-l f(r)(a + Oh).

Now put h = 0 in this to get f(r)(a) = ror-l f(r)(a). Here 0 is fixed, a is any real number and r is any positive integer less than or equal to 3.

r = 1 => f'(a) = f'(a) (nothing new) ,

r = 2 => either f"(a) = 0 for all a or 20 = 1 ,

r = 3 => either f"'(a) = 0 for all a or 302 = 1 .

Page 135: Surprises and Counterexamples in Real Function Theory

126 Chapter 5

If f"' (a) = 0 for all a, well and good, while if 382 = 1, then 28 ~ 1 and so f"(a) = 0 for all a and hence f"'(a) = 0 for all a. Thus f"'(a) = 0 for all a always, which then implies that f(x) = A + Ex + Cx2 , as required.

The number 8 appearing in MVT depends on h, i.e., 8 = 8h . Another interesting property which this 8 has, is the following:

Theorem 5.4.6. Suppose f"(a) exists and is nonzero and f" exists in a neighbourhood of a. Then limh-40 8h exists and equals 1/2.

Proof. We have, by L'Hospital's rule (Theorem 5.7.12),

lim f(a + h) - f(a) - h1'(a) = lim (d(f(a + h) - f(a) - h1'(a)) jdh2 )

h-40 h2 h-40 dh dh

. 1'(a + h) - 1'(a) f"(a) = hm =--

h-40 2h 2

On the other hand, using f(a+ h) - f(a) = h1' (a+8h), the extreme left side of the above equation is (h1'(a+8h)-h1'(a))/h2 = ((f'(a+8hh)- f'(a))/8hh).8h, and letting h -+ 0, this tends to f''(a).limh-408h, since 8hh -+ 0 as h -+ 0, because 0 < 8 < 1. The result now follows from above.

Example 5.4.7. The condition f"(a) ~ 0 is essential, e.g., for the functions f(x) = x3 or f(x) = x4, at the point a = 0, f(a + h) - f(a) = h1'(a + 8h) implies, for f(x) = x3:

1 . f(h) - f(O) = h1'(8h) =} h3 = h.3(8h)2 =} 8 = vI3 ,whIch does not tend to ~

and for f(x) = X4:

1 . f(h) - f(O) = h1'(8h) =} h4 = h.4(8h)3 =} 8 = ij4 ,whIch does not tend to ~.

§5.5. Points of inflection

The derivative l' of a given function f is often used to determine extreme values (the maxima, the minima and the flexes) and the following result is often thought to be true:

1'(c) = 0 implies that c is a local maximum, a local minimum or a point of inflection (for short, a flex) according as f"(c) < 0, f"(c) > 0, or f"(c) = 0 respectively.

This result, as it stands, is incorrect. Let us look at the following examples (A.A.Ball, [7]):

Example 5.5.1. Let (see Figure 5.16)

{3X2 /S-4X+29/2, if2::;x<6,

y = f(x) = -5x2 /16 + 17x/4 - 41/4, if 6::; x ::; 10 .

Page 136: Surprises and Counterexamples in Real Function Theory

5.5 Points of inflection 127

Maximum of the second branch

-- ·····~···,tlti-IJ31~~_~-I •••• , •• L II

, I

(2,8)

1 ...... ; ....... ! ...... !1 ...... ' ...... ; ... I ... , ...... , ........ , ................ ,r,::::::::::::::::::::~:;;;::~ (10 , 1)

1 .. · .. ·i ...... ·! ...... i' ...... : ...... ,·+ .. I ...... r ...... '· .. · .. : ...... t-',,:::::::::::: .. ::: ........................ .

iii I 1 ...... i ...... ·' ...... , ...... ! ...... , .. ·I ...... · .. r .... ·j· .... T· .... r ...... · ...... · ............................ ..

Minimum of the first branch

Figure 5.16

On differentiating we obtain:

d Y { 3x / 4 - 4, if 2 ::; x < 6 , dx = -5x/8+ 17/4, if 6::; x::; 10.

At x = 6, dy/dx = 1/2 and is continuous. Now differentiating again, we get

d2 Y { 3/4 , if 2 ::; x < 6 , d x 2 = -5/8, if 6 ::; x ::; 10 .

In the figure, the point (6,4) belongs to the first branch. The minimum of the first branch is at x = 15/3, while the maximum of the second branch is at x = 34/5. The point (6,4) is a point of transition between the two branches where the respective second derivatives are of opposite signs (check). It is, therefore a flex, but this flex is not defined by the condition d2 y / d x 2 = O. Indeed, d2 y / d x 2 i= 0 for any value of x and at x = 6, d2 Y / d x 2 is actually undefined, as it has a jump discontinuity at the flex.

Figure 5.17 Figure 5.18

Page 137: Surprises and Counterexamples in Real Function Theory

128 Chapter 5

Example 5.5.2.(a). Let y = -x3 , then dy/dx = -3x2 , d2 y/dx2 = -6x. This is defined for all x and y" = 0 holds for x = 0 and gives the flex at x = 0, i.e., at the point (0,0) (see Figure 5.17). (b) Now consider the function y = X1/ 3 . Its graph is shown in Figure 5.18. Turning it clockwise by an angle Jr /2, it becomes exactly the same as the graph of the function y = -x3 (i.e., Figure 5.17).

However, in the case y = x1/ 3 , dy/dx = x-2 / 3 /3 and d2 y/dx2 = -2X-5 / 3 /9, which is defined for all x f. O. Thus y" > 0 when x < 0 and y" < 0 when x > O. Consequently, the point of transition, i.e., the point (0,0), is a flex but y" is not defined at (0,0).

Why this failure? Because the inflectional tangent happens to be parallel to the y-axis. In this circumstance y" can never be defined at the flex, since approaching from one side it needs to be very large and positive (+00) and from the other side, it needs to be very large and negative (- (0).

Thus y" may be undefined at a flex. However, the curvature plots (see § 5.6 for curvature) of the two curves (i) the parabola described in example 5.5.1 above and (ii) y = X 1/ 3 of Example 5.5.2 (b) above are respectively given in Figures 5.19 and 5.20 and it is clear that the two examples are intrinsically different.

Indeed, they illustrate the only two instances in which y" may be undefined at a flex, viz.

1. where there is a discontinuity in 1/ p'. the curvature,

2. where the inflectional tangent is parallel to the y-axis.

K K

s s o 5 10 -2 -1 2

Figure 5.19 Figure 5.20

So then, how does one get all the flexes? Suppose a smooth curve is defined by y = f(x). All the flexes may be found by the following procedure:

Page 138: Surprises and Counterexamples in Real Function Theory

5.5 Points of inflection 129

(a) Identify the points where y" = ° or is undefined.

(b) Determine the signs of y" over the intervals separating the points deter­mined by (a).

(c) Identify a flex if the signs of y" are different over the intervals on either side of the point.

So, things can go wrong if one is not careful enough. Our first expedient, therefore, is to give various definitions of a flex, geometric as well as analytic, that are found III various texts and try to understand their equivalence or otherwise.

Definition 5.5.3 (Geometric). A point of inflection (flex, for short) is a point of a smooth curve C at which the tangent not only touches C but also crosses it.

Inflectional tangent

Figure 5.21

The tangent at a flex is called the inflectional tangent at the flex (see Figure 5.21). We see, from the figure, that a flex occurs when the curve stops bending to the left and starts bending to the right or vice versa.

Let now C be defined by the equation y = f(x). Then the direction of bending can be related to the value of d2 y/dx2 as follows: If d2 y/dx2 > 0, then the gradient dy/dx increases with x and the curve bends to the right, i.e., we have the following definition:

Definition 5.5.4 (Analytic). A flex is a point of transition between two curve segments, one where the second derivative d2 y / d x 2 is positive and one where it is negative.

A function f is said to be concave up on the interval (a, b) if for any- two points x, y E (a, b), and for all a E [0,1], we have (see Figure 5.22):

f(ax + (1 - a)y) :::; af(x) + (1 - a)f(y)

If f is differentiable, (*) is equivalent to saying that l' is increasing in (a, b). Figure 5.23 shows a curve that is convex up and hence the inequality in (*) is reversed.

Page 139: Surprises and Counterexamples in Real Function Theory

130 Chapter 5

PQ = the left side of (*)

f(x) fey)

fey)

Q x ax+(l-a)y y x ax+(l-a)y y

[(x) concave up [(x) convex up

Figure 5.22 Figure 5.23

We may rewrite Definition 5.5.3 in terms of concavity up (or down) as follows:

Definition 5.5.5 (Geometric-Analytic). The point c is a flex of the func­tion 1, if sufficiently close to c, the function is concave up on one side of c and concave down on the other side of c.

Let us now analyse the following four definitions, which are in current use, under the assumption that l' (x) exists (but 1" (x) need not exist) in a neigh­bourhood I of the point c E (a, b) (see [18]).

Definitions 5.5.6. We say that a point c is a flex of a function 1 defined on an interval (a, b) if: (I) There exists an open interval I c (a, b), with c E I such that in I, l' is increasing on one side of c and decreasing on the other side of c. (II) There exists an open interval I c (a, b), with c E I such that in I, l' attains either a maximum or a minimum at c. (III) There exists an open interval I c (a, b), with c E I such that in I, the tangent T at c, to the graph of 1, is greater than or equal to f(x) on one side of c, and is less than or equal to f(x) on the other side of c. (IV) Assuming that 1" (x) exists in a neighbour hood I of c; the function 1" (x) S; 0 on one side of c, but 1" (x) 2: 0 on the other side of c.

We now look at the relationship between these definitions. Since 1" (x) S; 0 implies that l' is decreasing and 1"(x) 2: 0 implies that l' is decreasing, it fol­lows that: (a) c is a flex according to Definition 5.5.6 (IV) then c is a flex of f according to Definition 5.5.6(1). (b) It is clear, from the graphs of functions that are convex up that c is a flex of f according to Definition 5.5.6 (III) if and only if c is a flex of f according to Definition 5.5.5. (c) Let 1 be differentiable on (a, b) and let c E ( a, b). Then c is a flex of 1 according to Definition 5.5.6 (I) implies that c is a flex of 1 according to def­inition 5.5.6 (II) and this implies that c is a flex of 1 according to definition 5.5.6 (III).

The most interesting aspect of these definitions are the following two neg-

Page 140: Surprises and Counterexamples in Real Function Theory

5.5 Points of inflection 131

ative results: (d) There exist functions, differentiable infinitely many times, in a neighbour­hood of a point c, which satisfy Definition 5.5.6 (III) at c but not Definition 5.5.6 (II) at c. (e) There exist functions, differentiable infinitely many times, in a neighbour­hood of a point c, which satisfy Definition 5.5.6 (II) at c but not Definition 5.5.6 (I) at c.

Example 5.5.7. To substantiate (d) above, we take

{(e- X -

2• sin ~)2 , if x> 0,

f ( x) = 0, if x = 0 , _(e- X - 2 sin 1)2 if x < 0 . x' ,

and c = o. A rough sketch of this function is shown in Figure 5.24.

Figure 5.24

The following may now be checked easily: (1) f is differentiable infinitely many times for all x. (2) f(n) (0) = 0, for all n = 1,2, .... (3) Requirements of 5.5.6 (III) are satisfied by f around the origin, since the x-axis is the tangent to the curve at the origin. (4) Requirements of 5.5.6 (II) are not satisfied by f with c = 0, for 1'(x) takes positive and negative values in any interval having the origin as an end point, so that l' (x) does not attain a relative maximum or a minimum at the origin.

Example 5.5.8. To illustrate (e) above, we take f(x) = fox h(t)dt, where

h(x) = {(e-~-2.sin ~)2, if xi- 0, 0, If x = 0 .

A rough sketch of the function h(x) is given in Figure 5.25.

Page 141: Surprises and Counterexamples in Real Function Theory

132 Chapter 5

Figure 5.25

The following may now be easily verified: (1) ] is differentiable infinitely many times in a neighbourhood of the origin. (2) 1'(x) = h(x), by the fundamental theorem of calculus, which is non-negative for all x. (3) l' (0) = O. (4) 1"(x) takes on both positive and negative values in any interval having the origin as an end point.

Results (d) and (e) show that no amount of differentiability makes the three Definitions 5.5.6 (I), (II), (III) equivalent. We have, however, the following result:

Proposition 5.5.9. Let] be analytic on an interval (a, b) (i.e., expandable in Taylor series around any point of the interval) and let c E (a, b). The conditions given in Definitions (I), (II) and (III) are equivalent.

Proof. First note that the hypothesis implies that 1" can not vanish on a set of points having c as an accumulation point (unless] is linear). For, ]" is also analytic and hence determined by its values on such a set.

Thus, for c E (a, b), there exists an open interval I c (a, b), c E I, such that on I, l' is monotone on each side of c. If now the conditions of Definition 5.5.6 (III) are satisfied by ] at x = c, then this monotonicity of l' must be of opposite kinds (i.e., increasing on one side, decreasing on the other), i.e., the conditions of Definition 5.5.6 (I) are satisfied. Thus conditions of Definitions 5.5.6 (I) =}(II) =} (III) =} (I), as required.

For more details, the reader is referred to [113] and [32].

Two more examples of functions differentiable infinitely many times with special properties are:

Example 5.5.10. The function

](x) = e ,IX> { -1/x2 ·f 0

0, if x :S 0

is differentiable infinitely many times, all of its derivatives at x = 0 being equal to O. This function is positive for positive x and vanishes for negative x.

Page 142: Surprises and Counterexamples in Real Function Theory

5.6 Geometric interpretation of the second and the third derivative 133

Exalllple 5.5.11. The function

f(x) = ' {e-1/(X2(1-X)2) if 0 < x < 1

o , otherwise

is differentiable infinitely many times, is positive in (0,1) and vanishes outside.

§5.6. Geometric interpretation of the second and the third derivatives

For a point P on a curve C : y = f (x), the tangent of the angle e, which the tangent line T to C at P makes with the axis of x, is related to the derivative (dyldx)p of f, evaluated at the point P, the relation being:

(1)

The first derivative, therefore, has a geometric significance. A natural question that comes to mind is whether any of the higher order

derivatives, in particular, the second and the third derivative, have some kind of geometric interpretation related to the graph of C.

The tangent T at P to C may be thought of as the limit, as pI -t P, of a secant line joining P and a neighbouring point pI of C. We say T makes a 2-point contact with C at P. We also note that T and C as well as their derivatives, take same values at P. So, instead of saying 2-point contact, we sometimes say that T has a first order contact with C at P.

Our object now is to relate the second derivative (d2 Yldx2 )p at P to the so called curvature K, = (II p) of C at the point P which is defined by the equation

K, = the rate of change of the direction with respect to the arc length, i. e., del d s at P (2)

Intuitively, we may think of the curvature of one curve C 1 being greater than that of another curve C 2 , if C 1 recedes more rapidly from its tangent than C 2 does. To make this idea more precise, we first consider the case of a circle. Its curvature increases as its radius decreases; it is therefore natural to select as a measure of its curvature K" the simplest function f(R) of its radius R, such that f (R) increases as R decreases, viz. K, = f (R) = 1 I R. Let us look at Figure 5.26: Here the arc AB = s = Rw. Hence the measure of the curvature K, of the circle is

K,=IIR=wls

Page 143: Surprises and Counterexamples in Real Function Theory

134 Chapter 5

tangent to e at A

B tangent to e at B

Figure 5.26 Figure 5.27

We extend (*) to any curve. So let AB be an arc of a curve C, which has no flex. Consider the tangents to C at A and B and take the directions of these tangents in the same sense as the arc AB (see Figure 5.27). We call w / Arc AB the average curvature of the arc AB (cf. (*)). As B -+ A, this quotient w/ArcAB tends to a limit I\;c(A), which is called the curvature of C at the point A. The radius of curvature of C at A is defined to be the radius of the circle which would have the same curvature as C has at A, i.e., I\;c(A) = lim(ArcAB/w). To calculate this limit, we proceed as follows (see Figure 5.28).

A(u,13)

Figure 5.28 Figure 5.29

Let y = f(x) be the equation of C and let a be the angle which the tangent to C at A makes with the axis of x. Let B be a point near A, say B = (x+Jx, y+Jy), where A = (x, y), and let Arc AB = Js. Let the tangent at B to C makes an angle a + Ja with the x-axis. Then

. Js ds I\;c(A) = ± hm ""£ = ±- (sign is chosen to makes I\; > 0).

B-+A ua da

However, tana = dy/dx, and so sec2 a (da/dx) = d2 y/dx2 and JS2 = Jx2 +

Page 144: Surprises and Counterexamples in Real Function Theory

5.6 Geometric interpretation of the second and the third derivative 135

r5y2, whence (ds/dx)2 = 1 + (dy/dx)2. Hence,

ds ds dx

and so K, = 1(1 + (y')2)3/2/ y"l. If a length equal to R be marked off from A on the normal to C, on the side

towards which C is concave, the extremity E is called the centre of curvature (see Figure 5.29). The circle with center E and radius R is called the circle of curvature. The coordinates (~, 7]) of E satisfy the two conditions:

(i) (~, 7]) belongs to the normal to C at A,

(ii) EA = R.

Figure 5.30 Figure 5.31

Now, the normal to C at A == (ex,(3) is the line given by (x - ex) + (y -(3).dy/dx = 0, so (i) becomes (~- ex) + (7] - (3).dy/dx = 0, while (ii) becomes (~- ex)2 + (7] - (3)2 = (1 + (dy/dx)2)3 /(d2 y/dX2)2. On eliminating ~ - ex we get

7] - (3 = ±(1 + (y')2)/y" , (3)

which then gives

-ex + ~ = ±y'(l + (y')2)/y" . (4)

We now show that in (3) (and so also in (4)), the + sign holds. To see this note that if y" is positive, then 7] > (3 (see Figure 5.30) and so the + sign holds in (3) and so also in (4). If, however, y" is negative, then 7] < (3 (see Figure 5.31)

Page 145: Surprises and Counterexamples in Real Function Theory

136 Chapter 5

and so rJ - (3 < 0, but so is y", hence again the + sign holds in (3) and so also in (4). Hence, finally the coordinates of the centre of curvature E == (~, rJ) are given by

~ = 0: - y'(1 + (y')2)/y" , rJ = (3 + (1 + (y')2)/y" .

We see how the second derivative y" enters naturally into a geometric config­uration.

We may avoid explaining the motivation behind the intuitive approach and define Ko by the equation Ko = dB / d s, without any further ado. Then

dB dB dx Ko=ds=dx·~·

But ds2 = dx2 + dy2, i.e.,

The equation d y / d x = tan B, on differentiating, gives

Also, (i),(ii) and (iii) immediately give

dB ds (1 + (dy/dx)2)3/2

( i)

( ii)

( iii)

(5)

We define p, the radius of curvature, by the equation p = 1/ Ko. Thus p is the radius of the circle (J (called the circle of curvature at P) through P and two neighbouring points Pl , P2 on C as Pl , P2 ---+ P. This (J is called the osculating circle of C at P. Equivalently, (J has y, y', y" equal to those of C at P and so, rather than talking about a 3-point contact, we sometimes say (J has a 2nd

order contact with C at P (for an interesting problem/solution, see [80]). Let us now go over, briefly, to the third derivative d3 y / d x 3 and see how

we may relate it to a geometrical quantity, the way d y / d x was related to the slope and d2 y / d x 2 to the curvature. Merely saying that d3 y / d x 3 is the rate of change of d2 y / d x 2 with respect to x, is not good enough.

The required geometrical entity is the so called aberrancy of Cat P, viz. the tangent of the angle 6 between the normal to C at P and the limiting position of the line joining P to the mid point M of chords, parallel to the tangent to C at P, as the chord approaches P, assuming that there is a limiting position (see Figure 5.32, where, for convenience, we have taken the tangent to C at P to be the x-axis and P. to be the origin).

Page 146: Surprises and Counterexamples in Real Function Theory

5.6 Geometric intelpretation of the second and the third derivative 137

the limiting position of PM

Normal to (Z at P

A

~M Al ~------~r---~-----------7 M

P tangent to (Z at P

Fignre 5.32

By carrying out this limiting process, it is not difficult to see that the aberrancy is given by:

Theorem 5.6.1.

tanc5 = (dp/d8) 3p

dy dx

(1 + (dy/dx)2) d3 y

3(d2 y/dx2)2 . dx3 .

Aberrancy is thus a quantity which does indeed depend on the third deriva­tive ylll at P. The angle c5 is easily constructed and visualized. The aberrancy is, roughly speaking, the measure of the asymmetry of C with respect to the normal to C at P.

As in the case of the tangent and of the curvature, it is possible to connect this concept with 4-point contact or equivalently the third order contact and osculants to C as follows: In this case, the appropriate osculant is the osculating parabola, i.e., the parabola P, which is the limit of the parabolas through P and three neighbouring points H, P2 , P3 of C to P, as PI, P2 , P3 -t P (hence 4-point contact). This osculating parabola is also the parabola which is such that its value as well as the values of its 1st, 2nd and 3rd derivatives at P are equal to the corresponding values of C and its derivatives at P (hence third-order contact).

\,ye may now define aberrancy in another manner as the tangent of the angle between the axis of the osculating parabola P and the normal to C at P.

Physically, the third derivative ylll (and so also aberrancy) has an instructive interpretation, viz. the jerk. Indeed, the rate of change of distance with respect to time, i.e., d s / d t, is the velocity; the rate of change of velocity with respect to time, i.e., d2 s / d t 2 , is the acceleration; and, finally, the rate of change of acceleration with respect to time, i.e., d3 s / d t3 is the jerk.

In calculus, one defines the centre of curvature 0 of a point P on C to be the intersection of neighbouring normals at P, pI and then the radius of curvature p at P works out to be the line segment PO. Analogously, the limiting position A of the point of intersection of neighbouring axis of aberrancy at P, pI is called

Page 147: Surprises and Counterexamples in Real Function Theory

138 Chapter 5

the center of aberrancy and the length P A is termed the radius of aberrancy of C at P and is denoted by R. Complicated Cartesian expression for Rand the coordinates of A can be derived but let us leave this interesting topic at that. The interested reader may look up the paper [102] by Steven H. Schot.

§5.7. Taylor's theorem and L'Hospital's rule

In its usual form, Taylor's theorem, with Lagrange form of remainder, is stated as:

Theorem 5.7.1. Suppose that f(n-1) (x) is continuous in [a, b] and that f(n) (x) exists in (a, b), then

f(b) = f(a) + b - a f'(a) + (b - a)2 f"(a) + ... + (b - a)n-1 f(n-1) + R I! 2! (n-1)! n,

where Rn = (b_~)n f(n)(~) (Lagrange form of remainder), for some a < ~ < b. n.

Proof. The proof uses Cauchy's mean value theorem. Let

) () b-x '( ) (b_x)n-1 (n 1)( n F(x =f(b)-fx--fx-···- f - x),G(x)=(b-x). I! (n-1)!

Here, F(x) and G(x) are both continuous in [a, b] and F'(x) = -((b-x)n-1 j(n­l)!)!(n)(x), G'(x) = -x(b - x)n-1 i- 0 in (a,b). Hence by the Cauchy mean value theorem,

F(b) - F(a) G(b) - G(a)

F'(~)

G'(~) ,

for some a < ~ < b, which on noting that G(b) = F(b) = 0, gives the theorem.

Remark 5.7.2. If we take G(x) = (b - x)P (p> 0), we get

F(a) F'(7)) (b - 7))n-1 f(n)(TJ) G(a) = G'(TJ) = (n _ l)!p(b _ TJ)p-1 ' (a < TJ < b),

which is Theorem 5.7.1 with the remainder Rn in Schlomilch's form as ((b­a)P j(n - l)!p).(b - TJ)n-p·f(n) (TJ)· Writing TJ = a + B1(b - a), 0 < B1 < 1, we get b - TJ = (b - a)(l - B1 ) and the remainder takes the form:

Rn= (b-a)n (I-B1)n-Pf(n)(a+B1(b-a)) (Schlomilch'sformofremainder). (n - l)!p

Putting p = n in the above, we get the Lagrange's form of the remainder, while putting p = 1, we get Cauchy's form of remainder, viz.

Rn = ~~ -=-~\~ (1- B2)n-1 f(n)(a + B2(b - a)) (Cauchy's form of remainder).

Page 148: Surprises and Counterexamples in Real Function Theory

5.7 Taylor's theorem and L'Hospital's rule 139

Combining these results together, we see that Taylor's theorem shows, on writ­ing b = a + h, that if Ihl < k, then

h h2 hn - 1

f(a + h) = f(a) + I! j'(a) + 2T j"(a) + ... + (n _ I)! + Rn(h) (4)

where

Definition 5.7.3. Suppose that f has derivatives of all orders in (a - k, a + k), for some k > O. Then the series

~ hn f(n)(a) = f(a) + ~ j'(a) + h2 j"(a) + ... ~ n! I! 2! n=O

(5)

is called the Taylor series of the function f(x) at x = a.

Equation (4) immediately gives the following:

Theorem 5.7.4. The series (5) converges to f(a+h) if and only if Rn(h) ---+ 0, as n ---+ 00.

Note that Rn involves f(n)(a + enh), where en is an unknown quantity and depends OIl n.

Example 5.7.5. It may happen that the Taylor series (5), although conver­gent, for a certain a, is convergent to the "wrong" sum, i.e., not to f(a + h). For example, let

f(x) = e . ,1 X r , { _1/x2 ·f -I. 0

0, If x = O.

Then for x =I 0,

j'(x) = (1/x3 ).2e- 1/ X2 ,

j"(x) = (1/x4).2( -3 + 2/x2)e-1/ x2 ,

j"'(x) = (1/x 5 ).2(12 - 18/x2 + 4/x4)e- l / X2 ,

by induction. For x = 0, 1(1(h) - f(O))/hl = le- l / h2 /hl = J(1/h2 )/el / h2 = ,fY/eY (where y = 1/h2), which tends to 0 as y ---+ 00, i.e., as h ---+ o. So 1'(0) exists and equals O.

Page 149: Surprises and Counterexamples in Real Function Theory

140 Chapter 5

Next, 1"(0) = limh--+o((f'(h) - f'(O))/h) = limh--+O((2/h3).e-l/h" - O)/h =

limh--+O(2/h4)/e1/ h2 = limy--+oo2y2/eY = 0, where y = h2. So 1"(0) exists and equals O.

Similarly, py induction, since yO: / eY -+ 0 as y -+ 00, for any 0, we see that f(n) (0) exists and equals O.

Now taking the Taylor expansion at the point a = 0, we find that the Taylor series is f(O) + (h/1!)f'(0) + (h2 /2!)1" (0) + ... = 0+0+0+· .. , which converges to 0, and not to f(O + h) = f(h), as required.

Also, it is easy to check that in this case Rn(h) does not tend to 0 as n -+ 00

(of course, if the contrary were the case, then by Theorem 5.7.4, the Taylor series would converge to f(a + h)). Indeed,

hn Rn(h) = """I f(n)(o + 8h) (in Lagrange 's form)

n. hn 2

= """I x (a polynomial in 1/8h of degree 3n)(1/e1/ h ) n.

1 /2 ..jY ? = (,yn eY ) x (a polynomial in -8 of degree 3n)(where y = l/h-) n.

= (ao + al vIY/8 + a2(vIY)2 /82 + ... + a3n(vIY)3n /83n )/n!(vIY)n.ey

and as n -+ 00 (y = 1/h2 fixed), this does not tend to 0 as required.

As an interesting and useful example, we prove the following:

Theorem 5.7.6. log(l + x) (defined for 1 + x > 0, i.e., for x > -1), with f(O) = log 1 = 0, has the Taylor expansion

x2 x 3 log(l + x) = x - - + - - ...

2 3 , if -1 < x :S 1.

Proof. Let f(x) = log(l + x). Then f'(x) = 1/(1 + x) and indeed, f(n) (x) = (-l)n+1(n -1)!/(1 + x)n. Hence, by Taylor's theorem

h2 hN - 1 log(l + h) = h - - + ... + (_l)N __ + RN(h).

2 N-1

Further, we have already shown that the Taylor series converges to log(l + h) if and only if RN(h) -+ 0 as N -+ 00. We shall establish the convergence for -1 < h :S 1. It is interesting to note that

(i) for 0 < h :S 1, we use Lagrange form of the remainder, viz. RN(h) = (_l)N+lhN /N(l + 8Nh)N ,0 < 8N < 1,

(ii) for -1 < h < 1, we use Cauchy form of the remainder, viz. RN(h) = (_l)N+1hN (1 - 8~ )N-l /(1 + 8~h)N ,0 < 8~ < 1.

So, first let 0 < h :S 1. The Lagrange form gives

Page 150: Surprises and Counterexamples in Real Function Theory

5.7 Taylor's theorem and L'Hospital's rule

Next, let -1 < h < 1. Then Cauchy form gives

IRNI :::; IhlN 1(1 + e~h) , (since 1(1- e~)/(l + e~h)1 < 1 if Ihl < I),

:::; IhlN 1(1 -Ihl) --+ 0 as N --+ 00 if Ihl < 1.

This completes the proof of the theorem.

141

Let us now look at the point ~, sometimes written as ~ = a + e(b - a), o < e < I, appearing in Theorem 5.7.3. Let j be continuous in [0, x] and suppose that j(n-1)(0) exists and j(n)(t) exists in (O,x). By Taylor's theorem, there exists ~(= ~x) E (O,x) such that

j(x) = j(O) + 1'(0) x + 1"(0) x2 + ... + j(n-1)(0) xn- 1 + j(n) (c,x) xn (7) I! 2! (n-1)! n!

j(n) (c,x) n = Pn - 1 (x) + ,x,

n.

say. Then the following result holds:

Theorem 5.7.7. If j(n+1)(t) exists in [O,x], and is continuous from the right at t = 0, and if f(n+1) (0) =I 0, then limx--+o+(c,x/x) = l/(n + 1). (For n = 2, we get Theorem 5.4.6.)

Proof [80]. First of all note that the assumptions of the theorem imply that C, = c'x is uniquely determined if x is small enough.

Apply the mean value theorem to j(n) (C,) in (7). So, there exists T E (O,c,) such that

f(x) = Pn-dx) + (j(n) (c,)ln!)xn

= Pn- 1 (x) + (xn In!)(j(n) (0) + f,j(n+1) (T))

= Pn(x) + f,j(n+1) (T).(Xn In!) . (8)

On the other hand, (7) gives, with n + 1 instead of n,

f(x) = Pn(x) + (j(n+1)(0")/(n+1)!)xn+1 (for some 0" E (O,x)). (8)'

Comparing (8) and (8)', we get f,j(n+1) (T)(Xn In!) = f(n+1) (0") (xn+1 I(n + I)!), i.e.,

(9)

Now, let x --+ 0; since ~,T, 0" E (0, x), we see that c" T, 0" all tend to 0 and since f(n+1)(t) is right continuous and nonzero at t = 0, we may cancellimf(n+1)(t) in (9) to get the required result.

We note that under the usual conditions

f(~) = f(a) + (c, ~ a) f'(a) + (~~!a)2 1"(a) + ... +

+ (~- a)n-1 f(n-1)(a) + (c, - a)n f(n)('y) (0:) (n-1)! n!

Page 151: Surprises and Counterexamples in Real Function Theory

142 Chapter 5

Our aim is to obtain lim€--+a r=~, under certain conditions.

Theorem 5.7.8 ([4]). Suppose a,~ belong to an open interval I, and suppose that

(i) j(n+p)(x) exists jor all x E I and is continuous at a (n::::: 1, p ::::: 1),

(ii) j(n+l)(a) = j(n+2) (a) = ... = j(n+p-l) (a) = 0,

(iii) j(n+p)(a) =I 0;

then

where, I E (a,~) is as in the equation (0:) above.

Proof. Under the assumption of the theorem, we may expand j(n)b) in (0:) up to the pth power, to give

j(n)b) = j(n)(a) + b - a) j(nH)(a) + b - a)2 j(n+2) (a) + ... I! 2!

+ b - a)p-l j(n+p-ll(a) + b - a)P j(n+p)bd, (p-1)! p!

which on using (ii), equals j(n)(a) + (b - a)P /p!)j(n+p)bd. Substituting this in (0:) gives:

Now expand j(O once again, but up to (n + p)th power to give

j(~) =j(a)+ (~- a) f'(a) + ... + (~- a)n j(nl(a) + (~- a)nH j(n+l) (a) + ... I! n! (n+1)!

(~ )n+p-l (~ )n+p + -a j(n+P-l)(a)+ -a j(n+p)b2) , a<'2<~'

(n+p-1)! (n+p)!

and again on using hypothesis (ii), this gives

j(~) =j(a)+ (~- a) f'(a) + ... + (~- a)n j(n)(a)+ (~- a)n+p j(n+p)b2). m I! n! (n+p)!

From (t) and m it follows that

(~- a)n b - a)P j(n+p) ( ) = (~ - a)n+p j(n+p) ( ) , , 11 (+)' 12 , n. p. n p.

Page 152: Surprises and Counterexamples in Real Function Theory

5.7 Taylor's theorem and L'Hospital's rule 143

i.e.,

giving

( '- a)P nlp 1 f(n+ p )(J2) ~-a = (n+p)lf(n+p )(Jl)·

Letting ~ -t a, we see that ",1, 12 all tend to a and the theorem follows.

Remark 5.7.9 (a). Take n = p = 1 and let f"(a) ¥- 0, f"(x) exists and is continuous, then f(~) = f(a) + (~ - a)f' (J). By the theorem, (J - a)/ (~ - a) -t

1/2 as ~ -t a, i.e., as ~ -t a, I approaches the midpoint of a and ~, for (J - a)/(~ - a) -t 1/2 => (J - a)/(~ - a) = 1/2 + E(~), where E(~) -t 0. Hence, I = a + (~ - a) /2 + (~- a)E(~) = (~+ a) /2 + (~- a)E(~) -t (~+ a) /2, as required. (b). Condition (ii) in the theorem is necessary. For take n = p = 1, f(x) = cx + d. Then I can be chosen anywhere in [a,~], for example I can be such (J-a)/(~-a) = 1/3, so that the conclusion ofthe theorem, viz. (J-a)/(~-a) -t 1/2, does not hold. (c). Let f(x) = xN + xN +p , a = O,n = N,p + P; then (J - a)/(~ - a) = (nlp l/(n + p)l)I/P.

The first mean value theorem for integrals states that if f is continuous in [a, x], then there exists a point c, a < c < x, such that

l x f(t)dt = f(c)(x - a).

About this number c, we have the following:

Theorem 5.7.10 ([49]). Suppose f is differentiable at x = a, f'(a) ¥- 0, and that f is continuous in [a,x] (otherwise such a number c would not exist). Then limx--+a(c - a)/(x - a) exists and equals 1/2.

Proof. Consider

L = l~ ((l X

f(t)dt - xf(a) + af(a)) / (x - a)2)

By the mean value theorem for integrals, L is equal to

lim f(c)(x - a) - (x - a)f(a) = lim f(c) - f(a) x--+a (x - a)2 x--+a X - a

= lim f ( c) - f ( a) c - a J;--+a C - a x - a

c-a = f'(a). lim -- .

x--+a X - a

But on applying L'Hospital's rule to evaluate L, we get

L = lim f(x) - f(a) = f'(a) x--+a 2(x - a) 2

Page 153: Surprises and Counterexamples in Real Function Theory

144 Chapter 5

Equating (*) and (**) gives the theorem.

We have looked extensively at the Taylor expansion of a function f, about a point x = a. What is it supposed to tell us about the function f? It tells us that, under certain conditions, f can be represented, in a neighbourhood of the point x = a, as a power series (or a polynomial with a remainder). There are several ways of proving Taylor's theorem, all of which use auxiliary functions, whose choice can not be motivated. Proofs using integration by parts (see, for example, [65]) are better motivated but get the reader lost in technical details.

'\That really matters in Taylor's theorem, is not the form of the remainder but that the function can be approximated by a Taylor polynomial, with an error term (the remainder), which can be estimated. With this in mind, a proof is given below, which depends only on the representation of a function as the integral of its derivative and uses the trivial formula

lIb fl :::: M(b - a), where M = sup f(x). a xE[a,b]

Theorem 5.7.11 ([16]). Suppose that for some n EN U {O}, the functions f, f', f", ... , fen) are continuous in [a, bJ and that f(n+l) exists and is bounded in (a, b), i.e., If(n+l)(x)1 :::: M for x E (a, b); then for x E [a, b],

f'(a) f"(a) 2 fen) n f (x) = f (a) + 1! (x - a) + 2! (x - a) + ... + 7 (x - a) + Rn (x), ( * )

where IRn(x)1 :::: (M/(n + l)!).lx - al n +1 .

Proof. The polynomial

f'(a) f"(a) 2 fen) n Pn(x) = f(a) + -l!-(x - a) + 2!(x - a) + ... + 7(x - a)

(called the Taylor polynomial) is chosen to have the properties:

f(a) = Pn(a) ,j'(a) = P~(a) , .. ' ,f(n)(a) = p~n)(a) (1)

Since Pn is a polynomial of degree n,

p (n+l)(x) = 0 V x E [a bJ n -, , (2)

The remainder Rn is defined by (*), so that Rn(x) = f(x) - Pn(X), and by (1)

R~(a) = R~(a) = ... = R~n)(a) = 0

and by (2),

f(n+l)(t) = R~n+l)(t), a < t < x

Integrating (4) from a to x, and using (3), gives

R~n)(x) = R~n)(a) + IX R~n+l)(t)dt = IX f~n+l)(t)dt

(3)

(4)

Page 154: Surprises and Counterexamples in Real Function Theory

5.7 Taylor's theorem and L'Hospital's rule 145

and the bound If(n+l)(t)1 :::; M gives IR~n)(x)1 :::; J: If(n+l)(t)ldtl :::; M(x­

a). A second integration gives R~n-l)(x) = R~n-l\a) + J: R.~n)(t)dt. So

IR~n-l)(x)1 :::; J: IR~n)(t)ldt:::; J: M(t - a)dt = M(x - a)2/2. Continuing (or using induction), we see that for j = 0,1,2, ... ,n,

IR~n-j)(x)1 :::; (M/(j + l)!).(x - a)j+1 , a:::; x:::; b.

For the desired estimate, we simply take j = n.

Marquis De L' Hospital's rule (to give him, for once, as R.P.Boas Jr. puts it, the spellings that he himself used), was actually discovered by John Bernoulli. Whatever form it appears in and whatever hypothesis are assumed to prove it, it is exclusively used for evaluating indeterminate forms such as §, 0°, :' 0.00 etc. The simplest form it takes (usually to tackle expressions like §) is the following:

Theorem 5.7.12 (L'Hospital's rule). Suppose that f(a) = g(a) = 0, and that A = limx-+a f'(x)/g'(x) exists. Then limx-+a f(x)/g(x) exists and is equal to A.

Proof. Observe that

f(x) g(x)

f(x) - 0

g(x) - 0

f(x) - f(a) g(x) - g(a) (f(x) - f(a))/(x - a) (g(x) - g(a))/(x - a)

(1)

But by the mean value theorem, applied to f and g separately, in the interval between a and x (i.e., [a, x] if a < x, [x, a] if x < a), there exist ~,'rJ between a and x, such that (f(x) - f(a))/(x - a) = f'(~) and (g(x) - g(a))/(x- a) = g' ('rJ). Substituting in (1), we get f(x)/g(x) = f'(~)/g'('rJ) --+ A, by the hypothesis, since ~,'rJ --+ a as x --+ a.

This proof uses the mean value theorem. With a little stronger hypothesis, that suffices for all applications, the rule can be proved without using the mean value theorem. Indeed, we have the following:

Theorem 5.7.13. Suppose that

(i) the derivatives 1', g' of the functions f and g are continuous,

(ii) f, g both tend to 0 or both tend to 00 as x --+ a,

(iii) g'(x) i- 0,

(iv) f'(x)/g'(x) --+ L as x --+ a;

then f ( x ) / g ( x) --+ L as x --+ a.

Proof ([14]). The theorem holds true if we replace, throughout, the limit

Page 155: Surprises and Counterexamples in Real Function Theory

146 Chapter 5

x -+ a by right hand or left hand limits x -+ a+ or x -+ a- ; and so it is enough to prove the theorem for a = 00, since the same proof works for a = -00 or for any finite value of a.

Given E > 0, (iv) implies that -E < f'(t)/g'(t) - L < E, if t is large enough, say t > 6.. Since g' is continuous and never 0, it has a fixed sign, say, without loss of generality, that g'(t) > O. Then the above gives

-Eg'(t) < j'(t) - Lg'(t) < Eg'(t) , if t> 6.. (1)

The right hand inequality implies that f'(t) - (E+L)g'(t) < 0, ift > 6., which implies that f;(f'(t) - (E + L)g'(t))dt < 0, if x> y and hence

f(x) - f(y) - (E + L)(g(x) - g(y)) < 0 . (2)

Case 1 : f, 9 both tend to O. In (2), fix y and let x -+ 00, to get

-f(y) + (E+L)g(y) ~O. (3)

But g'(y) > 0 and g(y) -+ 0, so g(y) < 0 for large y. Hence, by (3), - f(y)/g(y) + L 2: -E if Y > 6.2 , say. Similarly, the left hand inequality of (1) implies - f(y)/g(y) + L ~ E, if Y > 6.2 , say. These imply that f(y)/g(y) -+ L as y -+ 00.

Case 2 : f,g both tend to 00. Then by (2), (noting that g(x) > 0 if x > 6.) f(x)/g(x) - (E + L) < (f(y) - (E + L)g(y))/g(x). In this, fix y. Then, for sufficiently large x, the right hand side is less than E, since g(x) -+ 00; so f(x)/g(x) - L < 2E, for x > ~3, say. Similarly, the left hand inequality of (1) implies that -2E < f(x)/g(x)-L, for x> 6.4 , say. These imply f(x)/g(x) -+ L as x -+ 00.

Remark 5.7.14. The hypothesis that g'(x) is continuous is redundant; all we really use is that if g' oj 0 in an interval, then g' has a fixed sign (because, a derivative can not have a jump discontinuity).

In handling indeterminates of the form 00 , the usual examples that OIle meets are

lim (XX) and lim ( (sin x) tan X) , x-+o x-+o

and both of these limits, and many others, that generally come to mind, equal 1. That this is not always the case, is shown by the following example.

Example 5.7.15. Consider limx-+o+(xa/logX), where a is some constant. This leads to the indeterminate form under consideration. Letting y = x a/ 1og x , we find logy = a, giving logx-+o+(logy) = a. Hence ea = e1imx->o+logy = logx-+o+ e10gy = limx-+o+ y = limx-+o+(xa/logx). Thus, we may vary limx-+o+ (xa/logx) by varying a.

Note that the equation lim e'P(x) = elim 'P(x) is valid, since exponentiation is continuous.

On the same lines, we prove the following useful:

Page 156: Surprises and Counterexamples in Real Function Theory

5.7 Taylor's theorem and L'Hospital's rule 147

TheoreIll 5.7.16 ([115]). Suppose f, g, h are functions, defined in a neigh­bourhood of the origin, that satisfy

(i) 9 is continuous at x = 0 ,

(ii) limx--+o f(x)/x = b (0 < b < (0),

(iii) limx--+oh(x)/logx=c (O<c<oo),

(iv) these functions and their derivatives have the necessary properties for applying L'Hospital's rule near the origin;

then lim f(x)g(x)/h(x) = eg(O)/c . x--+o

Proof. We have that b = limx--+o+ f(x)/x = limx--+o+ f'(x)/I (by (iv)) and similarly c = limx--+o+ h(x)/logx = limx--+o+ h'(x)/(I/x) (by (iv)) (f(x)/x is of type % while h(x)/logx is of type 00/(0).

Now, let y = f(x)g(x)/h(x); then logy = (g(x)/h(x)).logf(x). Hence

lim logy= lim (g(x)/h(x)).log f(x) x--+o+ x--+o+

= lim g(x). lim (log f(x))/h(x) x--+o+ x--+o+

=g(O). lim «I/f(x)).j'(x)/h'(x)) (by (i) and L'Hospital's rule) x--+O+

=g(O) lim 1'(x).«x/f(x))/x.h'(x)) x--+O+

=g(O)b.(I/b)/c (by above)

=g(O)/c.

Hence, as before, elim,-+o+ logy = eg(O)/c, i.e., limx--+o+ e10gy = eg(O)/c, giving

as required.

lim y = eg(O)/c , x--+O+

In attempting to construct specific examples of the indeterminate form 00 , we immediately think of x/ex) where f(O) = 0 and where 1'(x) exists in a neighbourhood (0 - f, 0 + f) of the origin. But under these hypotheses, we have the following surprising result:

TheoreIll 5.7.17. If the limit, as x tends to 0, of x/ex) exists, where f(O) = 0, 1'(x) exists in (-f,f) for sufficiently small f, then it must be equal to l.

Proof ([77]). This limit is equal to limx--+o e1og(x f ('») = e1im,-+olog(xf(·») = eA, say, where, A = limx--+o (f(x) .log(x)) = limx--+o(log x / (1/ f(x))) (leading to the indeterminate form 00/(0). Applying L'Hospital's rule, we find

A = lim «I/x) /( - l' (x) / f2 (x))) = - lim (f(x) /x).(f(x) / l' (x)). x--+o x--+o

Page 157: Surprises and Counterexamples in Real Function Theory

148 Chapter 5

We now claim that under the hypothesis of Theorem 5.7.17, if

lim (f ( x ) / I' ( x ) ) x--+o

exists, it must be O. Assuming this, Theorem 5.7.17 follows, for then

lim f(x)/x = lim ((f(x) - f(O))/(x - 0)) = 1'(0), (since f(O) = 0) x--+o x--+o

(which exists and is finite by the hypothesis of Theorem 5.7.17) and so

lim (f(x)/x).(f(x)/f'(x)) = 1'(0).0 = 0 (by (4)) x--+o

(since 1'(0) exists and is finite). Thus .\ = 0 and so limx--+o xf(x) = eA = I, as required.

It remains to prove the claim. So suppose that limx--+o f (x) / l' (x) exists. Then 0 can not be a limit point of zeros of 1'(x), for otherwise, 0 becomes a limit point of points where f(x)/ f'(x) is either infinite or undefined, so limx--+o f(x)/1'(x) can not exist. Hence we conclude that

f'(x) ::f 0 for 0 < Ixl < b

and so f(x) ::f 0 in (0, b), for otherwise, iffor example, f(xd = 0 (0 < Xl < b), then (f(xd - f(O))/(XI - 0) = 1'(0, for some 0 < ~ < Xl (by the mean value theorem), i.e., 1'W = 0 (since f(xd = 0 = f(O)), which is not possible. So, ' without loss of generality, let

f(x) > 0 in (O,b) (4)

and we must show that lim f(x)/ 1'(x) = L = O. Suppose, to the contrary, that L > 0 (say). Then there exists a number b' such that If(x)/ f'(x) - LI < E if o < x < b'. Take E = L/2 and the inequality reads L/2 < f(x)/ f'(x) < 3L/2 if 0 < x < rS', i.e.,

2/ L > f'(x)/ f(x) > 2/3L if 0 < x < b' (5)

Now, by the mean value theorem (f(x) - f(O))/(x - 0) = l' (~) for some 0 < ~ < x < b', i.e., f(x)/1'(x) = x1'(O/ 1'(x) =x(f(~)/ f(x))(f(x)/ 1'(x))(f'(O/ f(~)) · But by (5), f'(x)/f(x) is bounded by 3L/2, 1'(~)/f(O is bounded by 2/L and f(O/ f(x) is bounded by I, since 1'(x) > 0 (because f(x)/1'(x) -+ L > 0 and f(x) > 0, by (4)) . Therefore f(x)/f'(x) < x.1.(3L/2).(2/L) = 3x in (O,rS"), which implies that the limit of f(x)/1'(x) , as x tends to 0+, is 0, which is a contradiction, since L ::f O. Hence L = O.

Remark 5.7.18. In Example 5.7.15, i.e., limx--+o+ xa/logx , the hypothesis of Theorem 5.7.17 is not satisfied: 1'(x) = -a/x(logx)2 does not exist at 0, so the conclusion fails too.

Under a different kind of hypothesis , this limit , viz., limx--+o+(f9) equals 1. More precisely, we have the following:

Page 158: Surprises and Counterexamples in Real Function Theory

5.7 Taylor's theorem and L'Hospital's rule 149

Theorelll 5.7.19 ([95]). Let f,g be nonzero real analytic functions at x = 0 (i.e., representable, in a neighbourhood of x = 0, by a Taylor series) for which f(x) ;::: 0 for all possible x close to o. If limx--+o+ f(x) = limx--+o+ g(x) = 0, then limx--+o+ f(x)g(x l = l.

Proof. We have

lim f(x)g(x l = lim (exp(log(f(x)g(xl))) x--+o+ x--+o+

= exp( lim (log (f (x ) / (1/ g(x)))) x--+o+

= exp( lim ((1/ f(x)).f' (x) / (_g-2 (x) .g' (x)))) (L'Hospital's rule) x--+o+

=exp( lim (-j'(x)g2(x)/f(x)g'(x))). x--+o+

Now f, 9 are analytic and tend to 0 as x -+ 0, so, by continuity, f(O) = g(O) = O. It follows that in some neighbourhood of 0, f and 9 have the form f = xm.F(x), F(O) f. 0, g(x) = xn.G(x), G(O) f. 0; m, nEW; F, G analytic at x = o. Substituting these in the above, gives

1· f( )g(xl ( 1· xn(xF'(x) + mF(X))G2(x)) 1m x = exp 1m x--+o+ x--+o+ -F(x)(xG'(x) + nG(x))

( . N(X)) = exp hm -(-) , x--+O+ D x

say, where, N(x) and D(x) are analytic at x = O. Since D(O) f. 0, by the continuity at x = 0, limx--+o+ D(x) f. 0 and so limx--+o+ N(x)/D(x) = O.

Relllark 5.7.20. The case lim(f(x)g(xl), as x -+ a, is dealt with by a simple linear transformation of x.

The case when the limit is taken as x -+ 0- is similar to the above theorem. The only requirement needed is f(x) ;::: 0 for x negative, close to O.

Exalllple 5.7.21. If" f, 9 analytic" is replaced by " f, 9 infinitely many times differentiable" , say at the point x = 0, the conclusion of the theorem is false, for example, take

f(x) = {e- 1/

X2 ~f x f. 0, o If x = 0 ;

g(x) = x and

then limx--+o f(x)g(x) = 0 and limx--+o- f(x)g(x) = 00.

In this context, see also [77J and [115J.

The converse of L'Hospital's rule is rarely looked into and it is generally thought that if f and 9 have continuous derivatives and f(a) = g(a) = 0, then

lim f(x)/g(x) = lim j'(x)/g'(x) , x---+a x--+a

in the sense that if either limit exists, so does the other and the two are equal. The following is a counterexample to show that this is not correct.

Page 159: Surprises and Counterexamples in Real Function Theory

150 Chapter 5

ExalTIple 5.7.22. Following [93], let

and

f(x) = {x Si~(1/x4).e-I/x2 , if x -::f 0 0, If x = 0

{e- I/X2 , if x -::f 0

g(x) = 0 ·f = 0 ,IX .

Then f,g are continuous at x = 0, since both f(x) --+ 0 and g(x) --+ 0 as x --+ O. Further, for x -::f 0, f(x)/g(x) = xsin(1/x4) --+ 0 as x --+ 0 and for x -::f 0, it is easy to check that l' (x) / g' (x) = - (2/ x) cos(l/ x4 ) + x sin(l / x4 ) .(1 + x2 /2), and for small x, this is -(2/x). cos(1/x4) + E(X), where E(X) --+ 0 as x --+ 0, and this does not tend to any limit. Indeed, f and g are differentiable infinitely many times, as is easy to check.

There is, however, an occasion when the converse does hold; in fact we have the following:

TheorelTI 5.7.23 (The North-East TheorelTI, [47]). Let f and g be dif­ferentiable near x = a. Suppose that

(i) lim f(x) = lim g(x) = 00 :c--+a x--+a

and (ii) lim j'(x)/g'(x) = >., X-fa

then limx-fa f(x)/g(x) exists and is equal to >..

Proof. Without loss of generality, we may suppose>. > 0, for otherwise, use fr (x) = f(x) + cg(x), instead of f(x), for a large enough c.

Let E > 0 be given. Then there exists "I > 0 such that I log l' (x) / g' (x) -log >'1 < E/2 if 0 < Ix - al < "I. Select Xl such that 0 < IXI - al < 7) and fix it. Then there exists "I' < "I such that if 0 < Ix - al < "I' then f(x) > f(xr) and g(x) > g(xr). This is because f(x) and g(x) tend to 00 as x --+ a, so f(x),g(x) become large (larger than f(xd,g(xd respectively) near a. Then we have

[log (f(X)) -lOg>.[ = [lOg (f(X) - f(X I)) -log>. -log (1 - f(XI)/ f(X)) [ g(x) g(x) - g(xd 1- g(xd/g(x)

[ ( f'(O) [[ (l-f(Xd/!(x))[ ::; log g'(~) -log>. + log l-g(xd/g(x) ,

where ~ lies between x and Xl. Here the first term is less than E /2 by hypothesis (x E (a-ry', a+ry') so ~ E (a-ry, a+ry)). Further, the function logx is continuous and log 1 = 0, so we may choose J < "I' such that If(XI)/ f(x)1 and Ig(XI)/ g(x)1 are small enough to ensure that (1 - f(xd/f(x))/(l- g(xd/g(x)) is near 1 whenever x E (a - J, a + J); in fact so near 1, that the absolute value of the second term is less than E/2 if Ix-al < J (this is possible, since f(x),g(x) --+ 00

as x --+ a). Hence Ilogf(x)/g(x) -log>'1 < E if Ix - al < J, i.e.,

lim (log f(x)/ g(x)) = log >. x-fa

Page 160: Surprises and Counterexamples in Real Function Theory

Exercises 151

and the theorem follows.

Exercises

5.1. Prove that (i) If 1'(x) ---+ 0 as x ---+ 00, then f(x)/x ---+ 0 as x ---+ 00. (ii) If f(x) ---+ 0 as x ---+ 00, and 1'(x) exists, does it follow that 1'(x) ---+ 0 as x ---+ 007 (iii) If g'(x) exists for all x > 1 and if g(x) tends to a limit as x ---+ 00, then there exists a sequence of points Cn , 2n < Cn < 2n+1, such that cn.g'(cn ) ---+ 0 as n ---+ 00. Give an example to show that x.g'(x) need not tend to 0 as x ---+ 00.

5.2. Let f(x) be a real-valued function, defined on (a, b) such that the second derivative f"(x) is continuous in (a,b). Show that for any x E (a,b)

f"(x) = lim [(j(x + h) + f(x - h) - 2f(x))/h2 ]. h---+O

5.3. Let a > 0 and suppose that the differentiable function f : (1,00) ---+ lR is such that l' (t) +af (t) ---+ 0 as t ---+ 00. By considering the function xf(log x/a), prove that if f > 0, then

provided t l , t2 are sufficiently large. Deduce that f(t) ---+ 0 as t ---+ 00.

5.4. Using the above exercise twice, prove that if f : (1,00) ---+ lR is twice dif­ferentiable and f"(t) + 31'(t) + 2f(t) ---+ 2 as t ---+ 00, then f(t) ---+ 1 as t ---+ 00.

5.5. Let F be the class of all functions f: [0,00) ---+ lR that are twice differen­tiable, such that

f(O) = 0, 1'(0) = 1, j'(t) :::; 2, 1f"(t) I :::; 4 (for all t > 0).

Let rn(x) = infJEF If(x)l. Find rn(x), considering separately the cases x:::; 1/2, x > 1/2. Find a function fxo E F such that fxo (xo) = rn(xo), in each of the two cases Xo :::; 1/2, Xo > 1/2.

5.6. Let f have a finite derivative 1'(xo) at Xo. Prove that

(j(xo + h) - f(xo - k))/(h + k) ---+ j'(xo)

as h, k ---+ 0 through positive values. By considering the function x2 sin(n/x), show that the result need not hold if k is allowed to take those negative values for which h + k =j:. O.

5.7. The function f is continuous for x> X and 1'(x) exists and is finite for all x > X. Prove that: (i) 1'(x) ---+ A as x ---+ 00 implies that f(x)/x ---+ A (A finite or infinite). (i) If f(x) tends to a finite limit as x tends to infinity, then 1'(x) can not tend

Page 161: Surprises and Counterexamples in Real Function Theory

152 Chapter 5

to an infinite limit nor to a nonzero finite limit.

5.8. A function f is such that f(O) = 0 and f"(x) exists and is finite for x 2: O. Prove that for some ~, 0 < ~ < x, f'(x) - f(x)/x = (1/2)xf"(~). Hence show that if f(O) = 0, f"(x) > 0 for x > 0, then f(x)/x is an increasing function of x for x > O. Explain the geometric significance of this result.

5.9. Let f be differentiable twice in [0,1] and suppose f(O) = O. Let a > O. Prove that there exists acE (0,1) such that f'(c) = aca - l .f(c)/(1 - ca ).

5.10. Let f, 9 be differentiable functions on [a, b], and suppose g'(.7;) =f. 0 for all x E [a, b]. Prove that there exists c E (a, b) such that

f'(c)/g'(c) = (f(c) - f(a))/(g(b) - g(c)).

5.11. Let f be differentiable on [0,1]' f(O) = 0, f(l) = 1. For each n E N, show that there exist distinct Xl, X2, ... ,Xn such that

n

L l/f'(xi) = n. i=l

5.12. Let f : (0,00) -+ lE. be a differentiable function and suppose that f(x) + f'(x) -+ 0 as x -+ 00. Then f(x) -+ 0 as x -+ 00.

5.13. If f is a function such that (i) f(x) -+ c as x -+ 00 (where c is a fixed real number), and (ii) f"'(x) -+ 0 as x -+ 00. Prove that f'(x) and f"(x) both tend to 0 as x -+ 00.

5.14. Let f be defined for x 2: 1 and suppose that f(l) = f(2) = 4 and that f"(x) exists and is positive in [1,3]. What is the sign of f'(3)?

5.15. Let h, 12,··· ,fn be differentiable functions in [a, b], and suppose h (a) = fn(b) = 0, fi(X) =f. 0 for x E (a, b), i = 1,2, ... ,n. Show that there exists a point c E (a, b), such that

f{( c) / II (c) + fH c) / h (c) + ... + f~ (c) / f n (c) = 0 .

5.16. Functions defined by f(x) = 0 or f(x) = xn satisfy (f(x))m = f(x m ) for all m E Z. Find a real valued function with this property, which is discontinuous at each x E lE..

5.17. Let f, 9 be nonnegative and continuous functions in [1,00). Suppose f is decreasing, g' is continuous in [1,00) and suppose g(x) -+ 00 as x -+ 00. Then

lim (f(I) + f(2) + ... + f(n))/g(n) = lim (f(n)/g'(n)) , n~oo n~oo

provided the right hand side limit exists.

5.18. (Theorem of proportional parts). Prove the following result: Let a < b < c and let f be a real-valued function, defined in [a, c], such that

Page 162: Surprises and Counterexamples in Real Function Theory

Exercises 153

(i) 1" exists in (a, c), (ii) f is continuous in [a, C]i then there exists a ( E ( a, c) such that

f(b) - f(a) = ((b - a)/(c - a)).(j(c) - f(a)) + (b - a)(b - c).1"(()/2

5.19. Let f and 9 be differentiable functions in (0, (0) and for x > 0, suppose that f'(X) = -g(x)/x and g'(x) = - f(x)/x. Characterize all such functions.

5.20. A function f is defined on [a, b], and 1', 1", ... , f(n+l l all exist in [a, b]. Suppose f(il(a) = f(il(b) = ° for i = 0,1, ... ,no Prove that there exists a ~ E (a, b) such that f(n+ll(~) = f(~).

5.21. Let f be continuous in (a, b) and suppose 1'(x) exists in (a,b), except perhaps at c, but the limit of 1'(x), as x tends to c exists and equals A. Prove that 1'(c) exists and equals A.

5.22. Let f be defined and be continuously differentiable in [a, b] (i.e., f' ex­ists in [a, b] and is continuous therein). Suppose there exists a point c E (a, b] such that l' (c) = 0. Then there exists a point ~ E ( a, b) such that l' (~) = (j(~) - f(a))/(b - a).

5.23. Suppose that f is differentiable twice in [0,2] and satisfies (i) If(x)1 :S 1, (ii) 11"(x)1 :S 1. Then 11'(x)1 :S 2 for all x E [0,2].

5.24. Show that there exists a unique c E lR such that for each differentiable function f on [0,1]' with f(O) = 0, f(l) = 1, the equation 1'(x) = cx has a solution in (0,1).

5.25. We know that dy/dx = 1/(dx/dy) always. However this is false for d2 y / d x 2 and d2 x / d y2. Determine all functions y = f (x) for which d2 y / d x 2 = 1/(d2 X/d y2).

5.26. (i) Sketch the graph of the function y = cosx.cos(x + 2) - cos2 (x + 1). (ii) Make a picture of the planar set A = {(x, y) : Ix - 11 + Iyl = I}. Find all points of A at which (x - 1)2 + y2 is maximum or minimum.

5.27. Let f be continuous in (a, b) and suppose 1'(c) exists in a neighbourhood of the point x = c E (a, b). Suppose that there exists a continuous function ()(h), with 0< ()(h) < 1, satisfying the equation f(c+h)- f(c) = h1'(c+h.()(h)). Show that l' (x) is continuous at x = c.

5.28. A continuous function f is such that (j(1 + h) - f(1 - h))/h tends to a finite limit as h --+ 0. Does it follow that 1'(1) exists and is finite? Justify your answer.

5.29. The functions f and 9 are nonconstant, differentiable, real valued func­tions such that for each pair x, y of real numbers

f(x + y) = f(x)f(y) - g(x)g(y) (1)

Page 163: Surprises and Counterexamples in Real Function Theory

154

g(x + y) = f(x)g(y) + g(x)f(y)

If 1'(0) = 0, prove that j2(x) + g2(X) = 1 for all x.

Chapter 5

(2)

5.30. Show that for x E (1, e), there exists a unique y E (e, 00) such that (logy)/y = (logx)/x. Show that for such x,y, we have x+y > xlogy+ylogx.

5.31. Find points, one on each graph, where the functions y = eX and its inverse function y = loge x are closest.

5.32. Find a point a, for which the function y = loga x (a > 1) intersects its inverse y = aX at only one (real) point.

5.33. Prove that if k 2 3, the equation (log x)k = X (x 2 1) has just 2 solutions rk, Sk and rk ---+ e, Sk ---+ 00 as k ---+ 00.

5.34. Prove that for x > 1, the function

f(x) = log((x + 2)/(x + l))/(log((x + l)/x))

is an increasing function of x.

5.35. Prove that the function f(x) = xn.e- x (x 2 0, n 2 2) is increasing in (O,n), decreasing in (n,oo) and has a flex at x = n ± Vii. 5.36. Find

(i) all continuous functionsf : IE. ---+ IE., that satisfy

f(f(x)) = f(x) for all x E IE. ,

(ii) all differentiable functions f : IE. ---+ IE., that satisfy (*).

5.37. Construct functions, f, that satisfy the following properties (one each) or show that no such function can exist:

I. (i) f' exists everywhere, (ii) f(Q) C Q, (iii) 1'(Q) c IE. " Q.

II. (i) f : [0,1] ---+ IE., (ii) f is discontinuous on an everywhere dense set, (iii) f is differentiable on an everywhere dense set.

III. (i) f: [-1,1]---+ IE., (ii) I:~=1 f(l/n) is convergent, (iii) I:~=1 If(l/n)1 is divergent.

Page 164: Surprises and Counterexamples in Real Function Theory

155

Chapter 6

Sequences, Harmonic Series, Alternating Series and Related Topics

§6.1. Sequences and series of real numbers

Recall that a function whose domain is the set N and range a set of real numbers is called a sequence of real numbers. We shall denote a sequence by {sn}. A sequence is said to converge to a real number l, if for each E > 0, there exists a positive number m (depending on E) such that ISn -ll < E, for all n ~ m; and we write Sn -t l; otherwise it is said to be divergent. Every convergent sequence can easily be seen to be bounded (i.e., its range is bounded), however, the converse is not true:

Example 6.1.1. The sequence Sn = (_I)n is bounded, but not convergent. The sequence for which Sn = 1 for n odd and Sn = 2 for n even also has the same property and has positive terms.

A real number ~ is said to be a limit point of a sequence {sn} if every neighbourhood of ~ contains an infinite number of terms of the sequence. For a sequence of real numbers {sn}, limit inferior and limit superior are defined respectively as

lim Sn = lim inf Sn = sup { inf { an, an+l, ... } } n-+oo n--+oo n

and lim Sn = lim sup Sn = inf {sup {an, an+l, ... }}.

n--+oo n--+oo n

It follows that inf Sn :S lim Sn :S lim Sn :S sup Sn and that if {sn} is bounded, then -00 < lim Sn :S lim Sn < 00.

Example 6.1.2. Let {an} and ibn} be sequences given by

{an} : 0,1,2,1,0,1,2,1,0,1,2,1,0,1,2,1, ...

ibn} : 2,1,1,0,2,1,1,0,2,1,1,0,2,1,1,0, ....

Then

Page 165: Surprises and Counterexamples in Real Function Theory

156 Chapter 6

because the five numbers in question can easily be checked to be 0 < 1 < 2 < 3 < 4.

By the completeness property of real numbers, it follows that every bounded sequence with a unique limit point is convergent and conversely. Thus, a bounded sequence {sn} converges if and only if lim Sn = lim Sn = s. We also have (the proof can be found in any text book of Analysis):

Theorem 6.1.3 (Cauchy's general principle of convergence). A neces­sary and sufficient condition for the convergence of a sequence {sn} is that for each £ > 0, there exist a positive integer m such that

for all n 2:: m and for all p 2:: l.

A sequence {sn} is said to be monotonically increasing if Sn+l 2:: Sn for all n and monotonically decreasing if sn+! :S Sn for all n. A necessary and sufficient condition for the convergence of a monotonic sequence is that it is bounded.

A series is the sum of the terms of a sequence. Thus, if Ul, U2, U3, ... is a sequence, then the sum Ul + U2 + U3 + . .. of all terms is called an infinite series and is denoted by I:~=l Un or simply by I: Un· Given the series I: Un,

we form, for each n 2:: 1, Sn = I:Z=l Uk, called the partial sums of the series. If the sequence {sn} of partial sums converges, then the series is regarded as convergent and limn-too Sn is said to be the sum of the series. If {sn} does not converge, we say that the series I: Un does not converge or that it diverges. A necessary condition (easily provable) for the convergence of an infinite series I: Un is that limn-too Un = O. The converse, however, is not true. We have the harmonic series:

Example 6.1.4. The series I:~=l ~ is not convergent, i.e., it is divergent. If I: ~ was convergent to s, say, then the series (1 + 1/2) + (1/3 + 1/4) +

(1/5 + 1/6) + ... would also converge to s. But the bracketed series is strictly greater than (1/2+1/2)+(1/4+1/4)+(1/6+1/6)+··· = 1+1/2+1/3+··· = s, giving s > s, which is a contradiction.

Example 6.1.5. Let an = 1 + 1/2 + ... + lin. Then the sequence {an} is divergent, but for p > 0, an+p-an = 1/(n+l)+·· ·+I/(n+p) :S p/(n+1) ---+ O. This gives an example of a divergent sequence {an} for which limn-too (an+p -an) = O.

As in the case of sequences, for series also, we have:

Theorem 6.1.6 (Cauchy's general principle of convergence for series). A series I: Un converges if and only if for each £ > 0, there exists a positive integer m such that

Page 166: Surprises and Counterexamples in Real Function Theory

6.1 Sequences and series of real numbers 157

for all n 2': m and for all p 2': 1, i. e., if {sn} denotes the partial sums of the series, then ISn+p - snl < E, for all n 2': m and p 2': 1.

Series with nonnegative terms are the simplest and the most well behaved. It can be easily proved that a series with positive terms converges if and only if the sequence of its partial terms is bounded above.

We now recall some common tests of convergence for series of positive terms:

(I) Comparison Test. If 2: Un and 2: Vn are two series of positive terms and k:F 0, a fixed real number (independent of n) and if there exists a posi­tive integer m such that Un :S kvn , for all n 2': m, then

(II)

(III)

(IV)

(V)

(VI)

(a) 2: Un is convergent, if 2: Vn is convergent; and (b) 2: Vn is divergent, if 2: Un is divergent.

Limit Form of Comparison Test. If 2: Un and 2: Vn are two series with positive terms such that limn-too(un/vn ) = l, where l is a nonzero finite real number, then the two series converge or diverge together.

Cauchy's Root Test. If 2: Un is a series with positive terms such that limn-too(un)l/n = l, then the series 2:un

(a) converges, if l < 1, (b) diverges, if l > 1; and (c) the test fails to give any definite information, if l = 1.

D 'Alembert 's Ratio Test. If 2: Un is a series with positive terms, such that limn-too (un+l / un) = l, then the series (a) converges, if l < 1, (b) diverges, if l > 1; and (c) the test fails, if l = 1.

Raabe's Test. If 2: Un is a series with positive term, such that limn-too n( ~ U n +l

-1) = l, then the series (a) converges, if l > 1, (b) diverges, if l < 1, (c) the test fails, if l = 1.

Cauchy's Integral Test. If u(x) is a nonnegative monotonic decreasing integrable function such that u(n) = Un, for all positive integral values of n, then the series 2: Un and Iloo u(x)d x converge or diverge together (the integral Iaoo u(x)dx is said to converge if t(x) = I: u(s)ds tends to a finite limit as x -t 00, otherwise, the integral is said to diverge).

Page 167: Surprises and Counterexamples in Real Function Theory

158 Chapter 6

Example 6.1.7. A series for which the ratio test fails: The series L~=l n\

d ,,",00 1 h th t 1· (n+1)2 - 1 - 1· n+1 b t ,,",00 1 an un=1 n are suc a Imn-+oo ~ - - Imn-+ oo --n-, u Un=1 n d· h ,,",00 1 I d d 1 1 1 1 2 1 Iverges, w ereas un=l :;::t2 converges. n ee , ]2 = '22 + 32 < 22 = 2"' 1 1 1 1 4 _1 _(1)2 h h t""'OO 1 ,,",00 (l)n

42 + 52 + 62 + 72 < 42 - "4 - 2" , .•. , SOWS t a un=l n2 < un=1 2" < 00,

being a convergent geometric series.

Example 6.1.8. The ratio test may also fail by virtue of the nonexistence of the limit l of an +1 • Examples for convergent and divergent series are an ,,",00 2( _1)n -n and ,,",00 2 n -( _l)n . In the first case lim ~ = 2 and L...tn=l un=l n-+oo an

lim an+l = 1. whereas in the second case lim an+l = 8 and lim an+l = l. n-+oo an 8' n--+oo an n--+oo an 2

Example 6.1.9. A series for which the root test fails: The root test fails for . 00 (5+(_1)n)-n_ l 1 1 1 ...L 1 ... the convergent senes Ln=1 2 - 2 + 32 + 23" + ¥ + 25 + 36 + ,

since lim vra;; = ~ and lim vra;; = !-. It also fails for the divergent series n-+oo n--)-(X)

,,",00 (5+(_1)n)n since lim nl'(L - 3 and lim nl'(L - 2 un=l 2' V ~n - - V ~n - . n--+oo 11--+00

Example 6.1.10. A series for which the root test succeeds but the ratio test fails:

The convergent series L~=1 2(-1)n-n of Example 6.1.8 is the one for which (_l)n_n 1

the root test succeeds. Indeed, vra;; = 2--n- --+ 2- 1 = 2" < 1. Similarly,

for the divergent series L~=1 2 n -( _1)n , of Example 6.1.8, vra;; = 2 n_(;;-l)n --+ 21 = 2> 1.

Example 6.1.11. A series for which the ratio test fails, but Raabe's test succeeds.

For example take the series

0: 1+0: (1+0:)(2+0:) j3 + 1 +,8 + (1 + ,8)(2 +,8) + ...

Here (1 + 0:)(2 + 0:) ... (n -1 + 0:) Un = (1 + ,8)(2 +,8) ... (n -1 +,8)

Thus limn -+oo Un+l = limn-+oo n++j3" = 1, and hence the ratio test fails. Also, Un n

1· (Un 1) 1· (n + ,8 1) 1· ,8 - 0: Q 1m n --- = 1m n --- = 1m --,,-=fJ-O:. n-+oo U n +1 n-+oo n + 0: n-+oo 1 + n

Thus, by Raabe's test, the series converges if ,8 - 0: > 1 or ,8 > 0: + 1 and diverges if ,8 < 0: + 1. The test fails for ,8 = 0: + 1.

We now turn to series with arbitrary terms.

Definition 6.1.12. A series whose terms are alternately positive and negative is called an alternating series.

Page 168: Surprises and Counterexamples in Real Function Theory

6.2 The series L l/n and Euler's constant ry

For such series, we have:

Theorelll 6.1.13 (Leihnitz Test). If the alternating series

UI - U2 + U3 - U4 + ... (Un> 0, for all n),

is such that (i) Un+1 ::; Un, for all n, (ii) limn-+oo Un = 0, then the series converges.

159

Definition 6.1.14. A series L Un is said to be absolutely convergent if the series L Iunl is convergent.

Definition 6.1.15. A series which is convergent but not absolutely convergent is called a conditionally convergent series.

Alternating and conditionally convergent series are dealt with in more de­tail in §6.5. We end this introductory section with:

Proposition 6.1.16. If L Un is absolutely convergent, then L Un is conver­gent.

Proof. The series L IUn I convergent implies that IUn+11 + ... + IUm I < E, if m 2': n 2': N and hence IUn+1 + ... + uml < E, if m 2': n 2': N, so that L Un is convergent, by Cauchy's general principle of convergence.

§6.2. The series L ; and Euler's constant I

We have seen in Example 6.1.4 that the series L~=II/n is divergent. It is equally well known that L l/n diverges extremely slowly. While attempting to explore how large the sum 1 + 1/2 + 1/3 + ... + l/n is, a beautiful constant ry emerges from the picture. \Ve turn our attention to this famous Euler's con­stant ry. Letting Sn = 1 + 1/2 + ... + 1/ n to be the nth partial sum of the series, so that Sn ~ 00, we ask: How fast does Sn go to 00 with n? The answer is the following:

Theorelll 6.2.1. Sn/ logn ~ 1 as n ~ 00.

Proof. Let gJ be the partition: 1 < 2 < 3 < ... < n - 1 < n of the interval [1, n]. Then fIn dxX lies between the upper and lower Riemann sums (see Figure 6.1), i.e., sp < fIn dxx < Sp, i.e.,

1/2 + 1/3 + ... + 1/(n - 1) + l/n < logn < 1 + 1/2 + ... + 1/(n - 1) (1)

i.e., Sn < 1+logn and logn < Sn-1/n, i.e., logn+l/n < Sn < logn+l, i.e.,

1 + 1/(nlogn) < Sn/logn < 1 + (1/logn)

and letting n ~ 00, Theorem 6.2.1 follows.

Page 169: Surprises and Counterexamples in Real Function Theory

160 Chapter 6

y

y= lIx

--~--~----~-----+--------L-----------+X

o 2 3 ..... . ... n

Figure 6.1

However, a much stronger result holds, viz.

Theorem 6.2.2. Let Tn = Sn -logn = 1 + 1/2 + ... + lin -logn, then Tn converges to a limit '/ (called Euler constant ,/) as n -+ 00, where 0 ::; '/ ::; l.

Proof. Rewrite (1) above as

lin < 1 + 1/2 + ... + 1/(n - 1) + lin -logn < 1 (2)

If we can show that Sn -logn tends to a limit, say, ,/, as n -+ 00, it will follow from (2) that 0 ::; '/ ::; 1, as required. But by (1)

0< {logn - (1/2 + .. . + lin)} < 1 - lin < 1 (3)

and the quantity Tn in curly brackets, being the sum of the shaded regions in Figure 6.2, is monotone increasing and bounded above by 1 (by (3)) and hence tends to a limit.

y

y = [Ix

o 2 3 ............ n-l n ......... ..... .

Figure 6.2

Page 170: Surprises and Counterexamples in Real Function Theory

6.2 The series I: l/n and Euler's constant 'Y 161

It follows that Tn = 1 - Tn also tends to a limit (call it 'Y) as n tends to infinity, completing the proof of the theorem.

Using a slightly more refined argument, we can easily prove the following

Theorelll 6.2.3. 1/2 ::::; 'Y ::::; 1.

Proof. In the proof of Theorem 6.2.2, we considered rectangles and the area under the curve y = l/x. The refinement is to consider trapezia as follows: \Ve have

-+-+ .. +- < - < - 1+-)+-(-+-)+ .. ·+-(--+-1 1 1 In d x 1 ( 1 1 1 1 1 1 1 ) 23 nIx 2 2 22 3 2n-1 n

(4)

Here the first inequality is got by using rectangles (as in (1) above), while the second one comes from using trapezia, for, the area of the (r - l)th trapezium is equal to (see Figure 6.3)

1 1 1 1 2,h(f(r - 1) + f(r)) = 2, ·1.(r _ 1 +~) ,

so starting with r = 2 and going up to r = n, (4) follows. The extreme right side in (4) equals (1/2)(-1 + 2Sn - l/n), while the

extreme left side is Sn - 1. Hence

1/2 + 1/2n < Sn -logn < 1

E:Jer- 1) f( r)

h

r-l r

Figure 6.3

(5)

and letting n ---+ 00, Theorem 6.2.3 follows, provided that the limit in question exists; but this has already been established in the proof of Theorem 6.2.2.

y = I /x

n-J n n+ 1

Figure 6.4

Page 171: Surprises and Counterexamples in Real Function Theory

162 Chapter 6

Remark 6.2.4. Actually, if we let Xn = {Sn-l -log(n - I)} - {Sn -logn}, it is easy to see, using a diagram, that Xn > 0 for all n.

Indeed Xn = {logn - log(n - I)} - (Sn - Sn-d .

Here the quantities log n - log( n - 1) and Sn - Sn-l are just the shaded areas shown in Figures 6.4 and 6.5 respectively. The difference is the shaded area shown in Figure 6.6, which is clearly greater than O. Thus Sn -logn decreases as n increases and so, by (5), 'Y exists, as required.

Using the Mercator expansion (see Theorem 6.2.5 below)

x 2 x 3 10g(1+x)=x- 2 + 3 -··· (-l<x::;l),

so that Xn = (Sn-l - Sn) -log(l-l/n) = (-l/n) + (1/2).1/n2 + (1/3).1/n3 + ... > 0, as required. However, the proof using a diagram is far more instructive.

n-\ n 0+ \

Figure 6.S

We now take a digression and give elementary proofs of the following two well known expansions. Proof of the second of these has already been given using Taylor series in Chapter 5.

0-1 n 0+ \

Figure 6.6

Page 172: Surprises and Counterexamples in Real Function Theory

6.2 The series I: l/n and Euler's constant; 163

Theorem 6.2.5. I. (James Gregory, 1668).

x 3 x 5 x 7

tan- 1 x = x - :3 +"5 - 7"" + ... , if Ixl :::; 1 .

II. (Nicolaus Mercator, 1669).

x2 x3 X4 log(l + x) = x - 2 + :3 - ""4 + ... , if -1 < x :::; 1.

Proof. Sum the geometric progression S = 1 - x + x 2 - ... + (-X),,-l to get S(1 + x) = 1 - (-x)", or 1/(1 + x) = (1 - x + x2 - x3 + ... + (_X),,-l) + (-x)" /(1 + x). Now replace x by t 2 and integrate from 0 to x to get

i.e.,

where

l x dt x 3 x 5 X 2n- 1 l x t2n dt -- = x - - + - - ... + (_l)n __ + (_l)n __ o 1 + t 2 3 5 2n - 1 0 1 + t 2 '

x 3 X 2n - 1 tan- 1 x = x - - + ... + (_l)n __ +R (x) . 3 2n - 1 n ,

(1)

In the range [0, x] (or [x, 0] if x < 0), 1 :::; 1 + t2 , i.e., 1/(1 + e) :::; 1 and so

(2)

Hence

which tends to 0 as n --+ 00, if Ixl :::; 1. However, if Ixl > 1, say Ixl = 1 + h (h > 0), then Ixnl/n = Ixln /n = (1 + nh + ... + hn)/n > nh/n which does not tend to 0 as n --+ 00, i.e., (1) is not convergent. That completes the proof of the first result.

Page 173: Surprises and Counterexamples in Real Function Theory

164 Chapter 6

Also, 1/(1 + t) = Sn + (_t)n /(1 + t), where Sn = 1 - t + t 2 - .• . (_t)n-1. Integrating from 0 to x, we get

log(l + x) = fox {(Sn + (( _t)n)/(l + t))}dt

=x-x2 /2+x 3 /3-···+(-1)n-1 xn/n + fox((-t)n/(l+t))dt.

Let En(x) = foX ((-t)n/(l + t))dt. We prove that En(x) -+ 0 as n -+ 00 , if -1 < x :::; l.

First, let 0 :::; x :::; 1. If t > 0, then 1 + t > 1 and so 1/(1 + t) < 1 or tn /(1 +t) < tn. Hence IEn(x)1 = f;(t n /(1 +t)) dt < fox tndt = x n+1 /(n+ 1) :::; l/(n+ 1) -+ 0 as n -+ 00.

• • • -1 x o -x

Figure 6.7

Next, let -1 < x < O. Put s = -t, then En(x) = (-l)n fo- X (-s)nds/(l- s), and since l-s > l+x, sn/(l-s) < sn/(l+x), we get IEn(x)1 < fox snds/(1+ x) = (_x)n+1 /(1 + x)(l + n) < 1/(1 + x)(n + 1) (since 0 < -x < 1), which tends to 0 as n -+ 00 , i.e., En(x) -+ 0 as n -+ 00.

Observe that if x > 1, the series does not converge as xn /n does not tend to 0, while if x:::; -1, log(l + x) is not defined, so the question of convergence to log(l + x) does not arise.

Putting x = 1 in the above expansion, we get

Corollary 6.2.6.

and

7r 111 -=1--+---+··· 435 7

1 1 1 log 2 = 1 - "2 + "3 - 4 + ....

The second result of this corollary may also be derived in a neat way as follows (see [35]):

We know that

r2 dx = log2. J1 x

Subdivide the interval [1,2] into n equal parts and consider lower approximating rectangles of height 1/(1 + l/n), 1/(1 + 2/n), .... As n -+ 00, the sum of the areas of these n rectangles tend to (see Figure 6.8) the integral f12 dx/x, i.e.,

[ 1 1 1 1 1 1] lim - + + ... + = log 2 11-700 1 + l/n n 1 + 2/n n 1 + n/n n

Page 174: Surprises and Counterexamples in Real Function Theory

6.2 The series I: l/n and Euler's constant,

i.e.,

( 1 1 1 ) lim -- + -- + ... + -- = log 2 n~oo n + 1 n + 2 n + n

Now, let

S2n = 1 - 1/2 + 1/3 - 1/4 + ... + 1/(2n - 1) - 1/2n

= 1 + 1/2 + 1/3 + ... + 1/2n - 2(1/2 + 1/4 + ... + 1/2n)

= 1 + 1/2 + .. ·1/2n - (1 + 1/2 + ... + l/n)

= l/(n + 1) + l/(n + 2) + ... + l/(n + n) ,

165

which tends to log 2 (by (* )). Also S2n+l = S2n + 1/(2n + 1) tends to log 2 too.

y y= l/x

Yzt-------t--+---+-...;;:::."!ooo....-

x o 1 l+lIn 1+2/n 2

Figure 6.8

Before we go on to some more properties of the Euler constant " we give another simple derivation of Leibnitz-Gregory series for 7r / 4 and Mercator series for log 2 (see [55]) i.e.,

00

7r/4= 1 -1/3+1/5 - 1/7+··· +( _l)k /(2k + 1)+··· = 2) _l)k /(2k + 1) (i) k=l

00

log2 = 1-1/2+1/3-1/4+·· +( _l)k /k+··· = 2:) _l)k-l /k (ii) k=l

Consider the integral In = J01T/

4 tann X dx (n 2: 2). Since, for 0 :=:; x :S 7r /4, we have O:S tanx :S 1, we see that O:S tann +1 x :S tann x, for all x E [0,7r/4] and for all n = 0,1,2, .... So, being the area under tann x,

In decreases as n increases. (1)

Page 175: Surprises and Counterexamples in Real Function Theory

166 Chapter 6

Next, it is easy to verify the formulae

Using (1), these give

In + In- 2 = l/(n - 1)

In + In+2 = l/(n+ 1)

1/(2(n + 1)) < In < 1/(2(n - 1))

The reduction formula (2) gives

hn = 1/(2n -1) -1/(2n - 3) + ... ± 1- (±7f/4)

I 2n+I = 1/(2n)-1/(2n-2)+·· ·±1/2-(±log2)/2

Let us analyse (4) for signs. Using (2) , we get the following:

h=1-Io =1-7f/4,

14 = 1/3 - h = 1/3 - 1 + 7f /4 = (-1)( -1/3 + 1 - 7f /4) ,

I6 = 1/5 -1/3 + 1- 7f/4 ,

18 = 1/7 -1/5 + 1/3 -1 + 7f/4 = (-1)(-1/7 + 1/5 -1/3 + 1-7f/4) ,

Taking absolute values, we get , for all even suffixes

hn = I~ ~;~: -~I , n = 1,2, ....

Now using (3), this gives

lin (-l)k 7f1 1 2(2n + 1) < (; 2k + 1 -"4 < 2(2n - 1)

, n = 1,2, ...

Letting n -t 00, we obtain (i). Next, analyzing (5) for signs, we get, using (2):

r/4 II = 10 tanxdx = [-logcosxl~/4 = (log 2)/2 ,

h = 1/2 - II = 1/2 - (log 2)/2 ,

h = 1/4 - h = 1/4 - 1/2 + (log 2)/2 = (-1)( -1/4 + 1/2 - (log 2)/2) ,

h = 1/6 - h = 1/6 - 1/4 + 1/2 - (log 2)/2 ,

Taking absolute values, we get , for all odd suffixes

n (_l)k- I log2 2: 2k - -2- , n = 1,2, .... k=I

(2)

(2)'

(3)

(4)

(5)

Page 176: Surprises and Counterexamples in Real Function Theory

6.2 The series L lin and Euler's constant '/ 167

Now using (3), this gives

1 I n (_l)k-l IOg21 1 4(n+ 1) < L 2k - -2- < 4n ' n = 1,2, .....

k=1

Letting n -+ 00, we obtain (ii).

For geometrical ways of looking at the expansion of log 2, see [59] and [34].

Now, some more properties of '/. As already said before (in Chapter I), it is not known if '/ is rational, irrational or transcendental. It has been calculated to many significant figures; the value is approximately given by '/ = .57721566490 .... The following mnemonic for remembering the value of '/ has been composed by Morgan Ward [114]:

These numbers proceed to a limit Euler's subtle mind discerned.

Here the number of letters in each word agrees with the decimal digits of the value of 'f. Since the next significant figure in the decimal expansion of '/ is 0, the sentence comes to a natural stopping place!

Our first aim is to calculate '/ to say, 5 significant figures. We write

3n = log n + '/ + En , where En -+ 0 as n -+ 00 . (6)

Since log(n + a) -log n -+ 0 as n -+ 00, it follows that 3n -log(n + a) -+ '/ as n -+ 00 and so we may write (6) as

3n = loge n + a) + 'Y + En , (7)

where En -+ 0 as n -+ 00 and the number a is arbitrary (but fixed), to be chosen suitably. Equation (7), with n - 1 in place of n, gives

3n - 1 = log(n - 1 + a) + '/ + En-l (8)

Subtracting (8) from (7) gives lin = log((n + a)/(n - 1 + a)) + En - En-I.

Now take a = 112, as then the terms lin and 1/n2 will not occur. Indeed, we have lin = log((n + 1/2)/(n -1/2)) + En - En-l = 10g(1 + 1/2n) -log(l-1/2n) + En - En-I. Now use the expansions 10g(1 + x) = x - x 2/2 + x 3 /3 - ... and 10g(1 - x) = -x - x 2 /2 - x 3 /3 - "', so the above becomes l/n = l/n + 2((1/2n)3/3 + (1/2n)5/5 + ... ) + En-l - En ,giving En - En-l = 2((1/2n)3/3 + (l/2n)5/5 + ... ), which is greater than the first tern 1/(12n3), but

En-l - En < (1 + 3(1/2n)2/5 + 3(1/2n)4/7 + ... )/(12n3)

< (1 + 1/n2 + 1/n4 + ... )/(12n3 ) (to estimate crudely)

= 1/(12n3 (1 - 1/n2))

= 1/(12n(n2 - 1)) .

Page 177: Surprises and Counterexamples in Real Function Theory

168 Chapter 6

Thus, 1/12n3 < En-l - En < 1/(12n(n -l)(n + 1)). Now sum these inequalities from n to m (n < m) to obtain

m m

L 1/(12r3 ) < En-l - Em < L 1/(12(r - l)r(1' + 1)). r=n r=n

Here the last sum above, using partial fractions, works out to be equal to 1/(12.2n(n - 1)) and letting m -+ 00, we get, since Em -+ 0,

00

L 1/(12r3 ) < En-l < 1/(24n(n - 1)) (9) r=n

Now, when a < x < f3, we say that" x is less than f3 by less than f3-a" meaning x falls short of f3 by a quantity less than f3 - a. Thus (9) can be read as: En-1

is less than 1/24n(n - 1) by less than A = (1/24n(n - 1) - I:::n 1/12r3 ).

We now wish to estimate A. We have A = (1/12) I:::n l/(r - 1)1'(1' + 1) - (1/12) I:::n 1/r3 (going back to the summation, so that earlier, we need not have used partial fractions and summed !), i.e., A = (1/12) I:::n 1/r3 (r­l)(r + 1) < (1/12) I:::n l/(r - 2)(r - l)r(r + 1)(1' + 2) = X/12, say, since 1/r3 (r -l)(r + 1) < 1/(r2 - 4)r(r - l)(r + 1). But now using partial fractions to sum this series, we get X = 1/4(n - 2)(n - l)n(n + 1). Indeed,

00

24X = L(l/(r - 2) - 4/(r - 1) + 6/1' - 4/(1' + 1) + 1/(1' + 2)) r=n

= l/(n - 2) - 3/(n - 1) + 3/n - l/(n + 1)

= 6/((n - 2)(n - l)n(n + 1)) ,

as required. So now finally we have:

En-1 is less than 1/24n(n - 1) by less than 1/48(n - 2)(n - l)n(n + 1). (10)

Choose a numerical value of n, say n = 12. Then S12 = log 12~ + i + E12 (by (7) rather than (6)). But S12 = 3.103210678 ... and log 12~ = log 100/8 = 2 log 10 - 310g 2 = 2.525728654 .... Hence

i + E12 = .577482024... , (11)

where E12 is less than .0002671 ... by less than .0000009 ... (a check: E12 is less than 1/24.13.12 by less than 1/48.11.12.13.14, i.e., the given figures), i.e., .0002662 < E12 < .0002671. This together with (11) gives

.5772149 < i < .5772158,

the actual value of i being .57721566 .... So we have calculated i to within an error of 5 in the sixth decimal place and have found that:

Sn = 10g(n+1/2)+i+ 1/ 24(n(n+1))-<5, (12)

Page 178: Surprises and Counterexamples in Real Function Theory

6.2 The series L lin and Euler's constant, 169

where 8 < 1/(48(n - l)n(n + l)(n + 2)) (by (7) and (10)). Our aim now is to prove the following striking result

Theorem 6.2.7. limn -+oo n.((l + 1/2 + 1/3 + ... + lin -logn) -I) exists and equals 1/2.

To prove this result, we shall find an interesting integral representation of ,. Indeed, we have the following:

Theorem 6.2.8. limn -+ oo (1 + 1/2 + 1/3 + ... + lin -logn) exists and equals

Proof (Lakshman Rao [60]). Verify that

1/1' = 100 e-rxdx , l' > 0

Now integrate (1) from 1 to n to get

logn = in dr/r = in (100 e-rxdx) dr = 100

(in e-rxdr) dx

(1)

= 100 (e- X - e-nx)dxlx (2)

(it is easy to check that interchange of the two integrals is valid-see Lemma 6.2.9; (2) is called the Frullani integral). Hence

1 1 an = 1 + - + ... + - - log n

2 n

= (oo(e- X + e-2x + ... + e-nx ) dx _ roo e-x - e-nx dx (using (1) and (2)) Jo Jo x

= - ~ l °°( e-x - e-(n+l)x e-x - e-nx )

o 1- e- X x (on summing the G.P.)

= e _ ~ dx + e-nx - - __ dx (on regrouping) (3) l°°( -x -X) 100 ( 1 1)

o 1 - e-X x 0 x eX - 1

We now verify the inequalities:

1/2-x18 < 1/x-1/(eX -1) < 1/2 , (for x> 0) (4)

First note that eX - 1 > 0 if x > O. and so we may multiply throughout by x(e X - 1) and clear denominators. Thus, it is enough to prove

(i) (x2-4x+8).e x >x2+4x+8,

Page 179: Surprises and Counterexamples in Real Function Theory

170 Chapter 6

(ii) (x-2).ex +x+2>0.

Checking (ii). Expanding eX = 1+x+x2 /2!+x3 /3!+···, terms upto x 2 cancel out. For n 2: 3, it is enough to prove that l/(n - 1)! > l/n!, i.e., that n > 2, which is true.

Checking (i). Here too, terms up to x 2 cancel. For n 2: 3, coefficient of xn works out to be n 2 - 5n + 8 and so it is enough to prove that this coefficient is greater than 0, which is clear since it is equal to (n - 5/2)2 + 7/4.

The second integral in (3) can be estimated as Jooo e~nx(l/x - l/(eX -

l))dx < Jooo e~nxdx/2 = 1/2n (using the second inequality of (4)). Hence, (3) gives

0:::; Ian - roo ( e~x~ _ e~X) dxl = I roo e~nx (.!. ___ 1 ) dxl < J:.-. 10 1 - e X x 10 x eX - 1 2n

Letting n -* 00, we get Theorem 6.2.8. The result used in the proof is the following:

Lemma 6.2.9.

Proof. Write cp(r) = Jooo e~rxdx and let E > 0 be given. Then there exists

a ~ such that I Jox e~Txdx - Jooo e~rxdxl < E whenever X 2: ~,for the left side is equal to Ir;e~TXdxl = [e~rx/-r]~ = l/reTx . Here, rerx > 1.ex

(since r E [1, n] (see the second integral)), which tends to 00 as X -* 00, so l/reTx < E if X 2: ~, as required.

Now choose ~ such that I J; e~rxdxl < E/n, whenever X 2: ~. Then

and so J;Uooo e~TXdx)dr = JlnUOx e~rxdx)dr + JlnU; e~rxdx)dr. In the first integral on the right, the order can be interchanged, since e~TX is a con­tinuous function of rand x. Here, bringing that first integral to the left side, we get

by (*) above, if X > ~, and the lemma follows.

Page 180: Surprises and Counterexamples in Real Function Theory

6.2 The series L l/n and Euler's constant 'Y 171

Proof of TheoreIll 6.2.7. Write equation (3) as

and so by (4) we get

or 1/2n - 1/8n2 < an - 'Y < 1/2n. Multiplying by n and letting n -+ 00,

Theorem 6.2.7 follows.

ExaIllple 6.2.10. Writing 5n = 1 + 1/2 + ... + l/n, it follows that "f < 51' + 5 s - 51's:::; 1, for all integers r, s ~ 1. Indeed, we have 51' + 5 s - 51's = (51' + 5s - 51') - (521' - 51') - (531' - 521') - ... - (5sr - 5(s-1)r) :::; 5s - r(1/2r)­r(1/3r) _ ... - r(l/sr) = 1, as required. Further, we have already seen that

5n - log n decreases and tends to 'Y as n tends to 00. (**)

So, 51's -logrs :::; 51' -logr (by (**)), i.e.,

0:::; 51' - 8rs -logr + logrs and 'Y < 5 s -log s (by(**)).

Adding these two inequalities gives that 'Y < 51' + 5 s - 51's' Another way of proving the above is as follows:

Put f(r, s) = 51' + 5 s - 51's' Then

l(r, s) - l(r - 1, s) = (51' + 5s - 51's) - (51'-1 + 5 s - 5(1'-1)8) = l/r - (l/(rs - s + 1) + l/(rs - s + 2) + ... + l/rs)

< l/r - sirs = 0

Hence, l(r, s) is a decreasing function of r and so also of s, by symmetry.

(i) Let r = s = 1. Then, 1 being decreasing, 1(1,1) is maximum, and so f(r, s) :::; 1(1,1) = 1, as required.

(ii) Let r, S -+ 00. Then

f(r, s) = 5r + 5s - 51'S

= (logr + 'Y + cr) + (log S + 'Y + cs) - (logrs + "f + crs)

= 'Y + Cr + Cs - Crs

-+ 'Y ,

as r,s -+ 00, since all the c's tend to O. Further, since l(r,s) decreases, it is greater than 'Y.

Page 181: Surprises and Counterexamples in Real Function Theory

172 Chapter 6

Example 6.2.11. We can use the above notions to find the sum of the series

00

S = (log 1)/1 - (log 2)/2 + (log 3)/3 - ... = 2:) -If+l(logr)/r . r=l

The series is convergent since the terms decrease and tend to O. Let f(n) = L~=l(log r) /r- (log2 n)/2. By Cauchy's integral test, L~=1 (logr) /r- (log2 n)/2 is convergent if and only if the integral It log xd x/x - (log n) /2 is convergent, which is the case (since the expression works out to be 0). Thus, f(n) converges to a limit A, say, as n -+ 00 and so f(2n) - f(n) -+ A - A = 0, as n -+ 00. But now

2n 1 1 2 n 1 1 2 f(2n)-f(n)=L ogr _ og 2n _ L ogr + og n

r 2 r 2 r=l r=l

= {~ (-l)r+llogr +2 ~ 10g2r}_ 10g22n _ t logr + 10g2 n ~ r ~ 2r 2 r 2 r=l r=l r=l

= f (-1)r+llogr +{2 t 10g2r _ t 10gr}_ (log2+logn)2 + r 2r r 2

r=l r=l r=l

10g2 n +-2-

2n (It+1l { n } 1 22 = ~ - r ogr + 10g2. ~ l/r - o~ -10g2.logn

= '"' - ogr +10g2 '"' --logn -~ (on shuffling), 2n ( 1t+ll { n 1 } 1 22 ~ r ~r 2 r=l r=l

and letting n -+ 00, we get that 0 = S + ')'(log 2) - ~ . log 2.log 2, which gives

S = log 2 (log J2 - ')') .

§6.3. Some number-theoretic aspects of the harmonic series

Since L l/n is divergent, the partial sums SN = L:=l l/n are larger than every positive integer as N increases. It is a remarkable fact that SN is never actually equal to an integer if N > 1, i.e., we have the following:

Theorem 6.3.1 The partial sums SN = 1 + 1/2 + 1/3 + ... + l/N of the harmonic series are never an integer, for any value of N 2:: 2.

Proof. Let 2t be the highest power of 2 which is less than or equal to N, so

Page 182: Surprises and Counterexamples in Real Function Theory

6.3 Some number theoretic aspects of the harmonic series

that 1 < 2t ::; N, but 2t > N. Then

1 1 1 1 1 1 SN = - + - + - + ... + - + -- + ... +-

1 2 3 2t 2t + 1 N 2tu + 2t- 1u + 2t u/3 + ... + u + ... + 2tu/N

2t u

173

where u is odd. Here each term in the numerator, except u, is even; so the numerator is odd and hence the 2t in the denominator can not cancel out, showing that S N is not an integer.

Remark 6.3.2. Since I: l/n diverges very slowly, it will take quite a number of terms to add up to even a small number like say 10. The problem is to look for the integer nA such that SnA 2': A but SnA -1 < A. It may amuse the reader to note, for example, that the sum of a quarter of a million terms of the series is still less than 20, whereas in order to surpass 100, it is necessary to add more than 15 million trillion trillion trillion terms! In [15], R.P.Boas 'almost' solves the problem. Indeed, he says that" it is 'almost' true that nA = [e A-, + ~] ". Heuristically, since we know that Sn = log n + "/- En, where En ---+ 0 as n ---+ 00,

so Sn > A if and only if logn + "/- En > A, i.e., if and only if n > eA-'.eEn •

Since eEn is close to 1, we see that nA is close to [eA-,] , as required (more or less!) .

Boas refines the equation Sn = logn + "/ - En, using the Euler-Maclaurin formula, to the equation

Sn = logn + "/ + 1/2n + [00 (t - [t]- 1/2).C2dt .

Consequently, Sn is larger than A, if logn > A - "/- 1/2n + 1/8n2 and is less than A if logn < A - "/- 1/2n. This leads to Boas' result. For details see the paper of Boas cited above.

Example 6.3.3. LetA = 3. We have, approximately, 1 = 1.00000 ... , 1/2 = 0.50000 ... , 1/3 = 0.33333 ... , 1/4 = 0.25000 ... , 1/5 = 0.20000 ... , 1/6 = 0.16666 ... , 1/7 = 0.142857 ... , 1/8 = 0.12500 ... , 1/9 = 0.11111 ... , 1/10 = 0.10000 ... ; adding, we get SlO = 2.9288 ... , which is less than 3, but Sl1 = SlO + 1/11 = 2.9288 ... + 0.0909 ... = 3.0197, which is greater than 3. Thus SlO < 3, Sl1 > 3. It follows that n3 = 11.

Let now A = 10 and let us try to determine nlO. Clearly, the crude method used above for A = 3, will be most impractical. By equation (12) (immediately above Theorem 6.2.7), we have Sn = log(n + 1/2) + "/ + (1/24n(n + 1) - 8), where 8 < 1/48(n - l)n(n + l)(n + 2). We try to solve 10 = log(v + 1/2) + ,,/, i.e., log(v + 1/2) = 10 - .57722 = 9.42278, i.e., v + 1/2 = e9.42278 and using tables this gives v + 1/2 as just under 12367 and so v lies between 12366 and 12367 (it is very near 12366 + 1/2, to the left of it (see Figure 6.9)).

Page 183: Surprises and Counterexamples in Real Function Theory

174 Chapter 6

v

• r • 12366

Figure 6.9

The value of S12367 is thus equal to log(12367 + 1/2) + '/ + 15', which is just larger than log(v + 1/2) + '/ + 15' = 10 + 15' > 10 (just because 15' is very small) and further S12366 = S12367 - 1/12367 = 10 + 15" - 1/12367 < 10, since 6" is very very smalL Thus nlO = 12367 (for a more detailed account of this, see RG.Lyness, Note on ,/, Math. Mag., 24 (1940), 206-209).

Let us now consider the following similar question. We know that as n increases, the quantity S(m, n) = l/(m + 1) + l/(m+ 2) + ... + l/(m +n) tends to 00 for any fixed value of m. We have the following:

Theorem 6.3.4. The quantity S(m, n) crosses 1 for some value of n between the two numbers [me] + 1 and [(m + l)e], i.e.,

S(m, [me]) < 1 < S(m, [(m + l)e]) .

Proof. From Figure 6.10, we see that

m m+l m+2 ... n-l n m m+! m+2 ... n n+!

Figure 6.10 Figure 6.11

in dx S(m,n) < -,

m X

while from the Figure 6.11, we see that

in+1 dx - < S(m,n).

m+1 X

These two give loge((n + l)/(m + 1)) < S(m,n) < loge(n/m). Since we want 1 < S(m,n), we must ensure that 1 < loge((n + l)/(m + 1)), i.e., that e < (n + l)/(m + 1), or (m + l)e - 1 < n, while S(m, n) will definitely be less than

Page 184: Surprises and Counterexamples in Real Function Theory

6.3 Some number theoretic aspects of the harmonic series 175

1 if loge n/m < 1, i.e., if n < m.e. So, as soon as n exceeds m.e, i.e., as soon as n 2: [m.e] + 1, SCm, n) may cross 1 and it will definitely cross 1 as soon as n exceeds m.e + e - 1, and so being an integer, for some value of n between [m.e] + 1 and [em + l)e] (both inclusive), SCm, n) will cross 1.

We now prove the following striking result

Theorem 6.3.5 (See [81]). Every positive rational number is the sum of a finite number of distinct terms of the harmonic series (without the condition "distinct ", the result is trivial: any positive rational A/ B equals

,1/ B + 1/ B + ... + 1/ E, ) v

(A terms)

Proof. Let alb be any given positive rational and first let A/ B > 1. Since L l/n is divergent, there exists a unique positive integer no such that

no no+l

Sno = L l/i < A/ B ~ L l/i = Sno + 1/(no + 1). (I) i=l ;=1

If the second inequality in (I) is an equality, we are through; so suppose that strict inequality holds in (I), at the second place too, i.e.,

A/B-Sno (=AdBl, say) < 1/(no+l). (II) 1

Let nl be the unique positive integer such that

(lIIh

i.e., nl is determined by nl < Bd Al ~ nl + 1. If equality holds here, we are through, for A/ B = Sno + Ad Bl (by (II)d = Sno + 1/(nl + 1) (by the supposed equality in (lII)d. So suppose that strict inequality holds in (IlIh. Then

0< (A1(nl + 1) - Bd/Bdnl + 1) = A2/B2 (say) with (A2,B2 ) = 1, (lIh

i.e., 0 < (AI (nl + 1) - B 1)/ BI (nl + 1) = A2/ B2 (A2' B2 in their lowest terms), so A2 ~ Al (nl + 1) - B 1, while the right hand inequality of (lIIh implies that AI(nl + 1) - Bl < Al and these imply

(th Now we check that

(IV)

i.e., (Al(nl + 1) - Bd/B1(nl + 1) < l/nl(nl + 1), or AdBl -1/(nl + 1) < l/nl(nl + 1) or AdBl < l/nl, which is true by (lIIh. Let n2 be the unique integer such that

(lIIh

Page 185: Surprises and Counterexamples in Real Function Theory

176 Chapter 6

i.e., n2 < E2/A2 :::; n2 + 1, while (IV) implies that N 1 (nl + 1) < BzlA2; hence nl (nl + 1) < n2 + 1, i.e., nl < nl (nl + 1), so

(th

If equality holds in the left inequality of (II I)z, we are again through, for then

A/B = Sno + AdB I

= Sno + l/(nl + 1) + A2/ B2

= Sno + l/(nl + 1) + 1/(n2 + 1)

(by (II)d

(by (IIh)

(by the assumed equality of (IIIh).

So suppose strict inequality holds in (III)z. Now proceed as from (II)z to get

0< AzlB2 -1/(n2 + 1) = A3/B3 (say) ,with (A3,B3) = 1, (IIh

i.e., 0 < (A2(n2 + 1) - B 2)/ B 2(n2 + 1) = A3/ B3, ((A3, B3) = 1), so A3 :::; A2(n2+1)-B2' while the right hand side of (III) 2 implies A2(n2+1)-B2 < A2. These imply

(t)z

N ow check that

(IV)z

Let n3 be the unique integer such that

(IIIh

Prove, as above (see after (IIIh), that

and so on. The process must terminate after finitely many steps, since at each step, the numerator strictly decreases (see (th, (th, ... ) and so a stage comes when Am/Em becomes 1/ B m , which is a term of the series L l/n, giving

A/B = Sno + l/(nl + 1) + 1/(n2 + 1) + ... + l/(nm -l + 1) + l/Bm .

Further, by (+)1, (+)2, ... , the ni are all distinct, so the condition that the terms be distinct is satisfied too. This completes the case A/ B > 1.

So, now let A/ B < 1. Let no be uniquely determined by l/no < A/ B < 1/ (no - 1), where clearly we may suppose strict inequality is valid on the right above, otherwise we are through. Let 0 < A/ B - l/no = Ad B 1 . Let nl be determined (uniquely) by 1/ (nl + 1) :::; Ad B 1 < 1/ nl. This is exactly the equation (I I I) 1 and so we may proceed as in the case A / B > 1. This completes the proof of the theorem.

Remark 6.3.6. For actual computations, we summarize the procedure below:

Page 186: Surprises and Counterexamples in Real Function Theory

6.3 Some number theoretic aspects of the harmonic series 177

• Let A/B > 1.

• Let no be determined by

Sno = 1 + 1/2 + ... + l/no < A/ B < 1 + 1/2 + ... + l/no + 1/(no + 1) .

• Let ArlBl = A/B - Sno.

• Let nl be determined by nl < BriAl ~ nl + 1. Let A2/B2 = ArlBl -1/(nl + 1).

• Let n2 be determined by n2 < B z / A2 ~ n2 + 1. Let A3 / B3 = A2 / B2 -1/(n2 + 1)

and so on. Since ... < A3 < A2 < AI, in finitely many steps, we get Am = 1, i.e., I/Bm = Am/Bm = Am~rlBm~l -1/(nm~l + 1), and then

A/B = Sno + 1/(nl + 1) + 1/(n2 + 1) + ... + 1/(nm~l + 1) + I/Bm .

Let us take some actual examples to illustrate this.

Example 6.3.7. 2 = 1 + 1/2 + 1/3 + 1/6.

The number 3 requires hard work. We proceed as follows: \Ve first exhibit the representation, then explain the working.

11111111111 1 3 = 1 + "2 + 3" + 4" + 5" + 6" + -;:; + 8 + 9 + 10 + 29 + 1239 + 4311720 '

i.e., 3 = 510 + 1/29 + 1/1239 + 1/4311720. Indeed, SlO = 2 + 2431/2520 < 3, but Sl1 = 2 + 2431/2520 + 1/11 = 2 + 29261/27720 > 3. Thus

0< Arl Bl = 3 - SlO = 89/2520 < 1/11. (1)

Define nl by nl < BriAl ~ nl + 1, i.e., 28 < 2520/89 = 28 + 28/89 < 29, giving 1/29 < 89/2520 < 1/28. So,

0< ArlBl -l/(nl +1) = 89/2520-1/29 = 59/73080 = A2/B2 . (2)

Define nz by n2 < 73080/59 ~ n2 + 1, i.e., 1238 < 73080/59 ~ 1239. So

A3 Az 1 59 1 59 1 1 (3) ---- ----- ---

4311720· B3 Bz nz + 1 73080 1239 8.5.7.9.29 3.7.59

Hence

3 = SlO + 89/2520 (by (1))

= SlO + 1/29 + 59/73080 (by (2))

= SlO + 1/29 + 1/1239 + 1/4311720 (by (3))

Page 187: Surprises and Counterexamples in Real Function Theory

178 Chapter 6

Proposition 6.3.8. For any positive integer r,

7'(1 - l/(n + l)l/r) < 1 + 1/2 + ... + l/n ::; r(nl/r - 1) + 1.

Proof. Consider the curve y = l/(x + 1)1+1/r and the area A between its graph, the x-axis and the ordinates x = 0, x = n (see Figure 6.12). We have

A < 1 + 1/2 + ... + l/n .

However, A = Jon (x + 1)-l-l/rdx = [-r/(x + l)l/r]~ = r(l -l/(n + l)l/r), which proves the first inequality.

o 1 2 n-l n

Figure 6.12

For the second inequality, consider the curve y = l/xl-~ and the area B between its graph, the x-axis and the ordinates x = 1, x = n (see Figure 6.13). We have B > 1/2+ 1/3+··· + l/n. However, B = Jon x-1+~dx = r [xl/r]~ =

r(n1/ r - 1). Hence 1 + 1/2 + 1/3 + ... + l/n ::; r(n1/ r - 1) + 1, as required. Observe that in Figure 6.13, the rectangles are respectively of area greater than 1/2,1/3, .. . 1/n.

For an alternative method of proving this result, see [87].

o 2 n-1 n

Figure 6.13

Page 188: Surprises and Counterexamples in Real Function Theory

6.4 The restricted harmonic series 179

§6.4. The restricted harmonic series

Let 59 denote the series obtained from the harmonic series upon the deletion of all terms which contain the digit 9 (it could be any other digit; for example, if it is 8, the series would be called 58) i.e.,

1 11 111 11 59 = L -;;: = 1 + "2 + :3 + ... + "8 + 10 + 11 + ... + 18 + 20 + ... +

9<tn

Since such a reduction seems to leave a richer series than that with prime denominators, which is known to be divergent (see [33]), one would expect this series to be very much divergent. However, a big surprise is that this series is convergent. This was first proved in 1914 by A.J.Kempner (see [56], and also [48]). It is easy to prove that all these series 5 i , with sum Ui, say, are convergent (see [46]).

Let us first consider 50 = 2::0 <t n l/n, i.e., the sum is taken over all those n's which do not contain the digit O. Let Uo denote its sum. We have the following:

Theorem 6.4.1. 20.18 < Uo < 28.29.

Proof.(i) We leave as it is, the sum from all the one digit terms: 1 + 1/2 + ... + 1/9, since no zero appears in any of the terms. (ii) Next, the sum from the two digit terms without a zero is (1/11 + 1/12 + ... + 1/19) + (1/21 + 1/22 + ... + 1/29) + ... + (1/91 + 1/92 + ... + 1/99) < 9· 1/10 + 9 . 1/20 + ... + 9 . 1/90. (iii) The sum from the three digit terms without a zero equals (1/111 + 1/112 + ... + 1/119) + (1/121 + 1/122 + ... + 1/129) + ... + (1/191 + ... + 1/199) + (1/211 + ... + 1/219) + ... + (1/291 + ... + 1/299) + ... + ... < 92 . 1/100 + 92 .1/200 + ... + 92 .1/900; and so on.

Hence

50 < (1 + 1/2 + ... + 1/9) + (9/10)(1 + 1/2 + ... + 1/9) + + (9/10)2(1 + 1/2 + ... + 1/9) + ...

= (1 + 1/2 + ... + 1/9)(1 + 9/10 + (9/10)2 + ... ) = (1 + 1/2 + ... + 1/9)10 ~ 28.29.

The convergence of 50 now follows. To get a lower bound for 50, we proceed exactly as above:

((i)') Leave the one-digit terms as they are, since they have no zero in them, to give the sum 1 + 1/2 + ... + 1/9. (( ii)') The sum of the two-digit terms without a zero is greater than 9/20 + 9/30 + ... + 9/100. (( iii)') The sum of the three-digit terms without a zero is greater than 9 .

Page 189: Surprises and Counterexamples in Real Function Theory

180 Chapter 6

1/200 + 92 . 1/300 + ... + 92 . 1/1000; and so on, hence

0"0 > (1 + ~ + ... + ~) + ~ (~ + ... + ~) + (~) 2 (~ + ... ~) + ... 2 9 10 2 10 10 2 10

( 1 1) 9 (1 1 ) ( 9 9 2 ) = 1 + 2" + ... + 9" + 10 2" + ... + 10 1 + 10 + (10) + ...

( 1 1) 9 (1 1) = 1 + 2" + ... + 9" + 10 10 1 + 2" + ... + 10 - 1

( 1 1) 9·9 = 1 + - + ... + - (1 + 9) - -2 9 10

~ 20.18

The approximate calculations of the other O"'s is of about the same order of difficulty. For example, we prove, with equal ease, the following:

Theorem 6.4.2 (See [38]). Let Sl = 2:1 f:- n lin. Then Sl is convergent with sum 0"1, where 14.68 < 0"1 < 18.29.

Proof. Exactly as before, the one-digit numbers , without a 1 contribute 1/2 + 1/3 + ... + 1/9; the two-digit numbers, without a 1 contribute (1/20 + 1/22 + ... + 1/29) + (1/30 + 1/32 + ... + 1/39) + ... + (1/90 + 1/92 + ... + 1/99) < 9.1/20 + 9.1/30 + ... + 9.1/90 = (9/10)(1/2 + 1/3 + .. . + 1/9). The three-digit numbers, without a 1, similarly contribute (9/10)2(1/2 + 1/3 + ... + 1/9) ; and so on. Hence, we get

0"1 < (1/2 + 1/3 + ... + 1/9)(1 + (9/10) + (9/10)2 + ... )

= (1/2 + 1/3 + ... + 1/9).10

~ 18.29.

A lower estimate can be worked out as follows: The contribution from the one­digit numbers, without a 1, equals (1/2+ 1/3+···+ 1/9); the contribution from the two-digit numbers, without a 1, by above, is more than 9.1/30 + 9.1/40 + ... + 9.1/100 = (9/10)(1/3 + 1/4 + ... + 1/10); the contribution from the three­digit numbers, without a 1, is similarly more than (9/10)2(1/3+1/4+·· +1/10); and so on. Hence, we get

0"1 > (~+ ~ + ... + ~) + (~ + ~ + ... +~) (~+ (~) 2 + ... ) 2 3 9 3 4 10 10 10

= (~ + ~ + ... + ~) + (~ + ~ + .. . + ~) 9 (summing the G.P.) 2 3 9 2 3 10

= (~+ ~ + ... + ~) 10 - 9 (~ - ~) 2 3 9 2 10

= (~ + ~ + ... + ~) 10 - 3.6

~ 14.68

Page 190: Surprises and Counterexamples in Real Function Theory

6.5 Rearrangements of the alternating series 2:( _l)n /n 181

Renmrk 6.4.3. This argument can be refined and better bounds can be obtained. In [5], Robert Baillie gave a method of finding the value of ai,

i = 1,2, ... ,9, to a high degree of accuracy and also gave a table giving these values to 20 decimal places. Baillie remarks that " it seems plausible that the ai are all irrational".

Remark 6.4.4. We would like to give an explanation as to why the conver­gence of these series Si is to be expected. Let us fix an i, say i = 1. When we look at the number of removed terms, we see that, in the beginning at least, they are very few and so we tend to think that the left over series would di­verge. However, as we look at large n, we see that if for examplc, n contains 50 digits, in all probability it would contain i = 1 as a digit and so would get removed. Thus, as n increases, most n get removed, which is what leads to the convergence.

More precisely, it is easy to chcck that (i) the proportion removed from the one-digit numbers is 1/10 (1 from 1,1/2, ... , 1/10); (ii) the proportion removed from the two-digit numbers is 18/90 (1/10,1/11, ... ,1/19,1/21,1/31,1/41, ... ,1/91 from 1/10,1/11, ... ,1/99); (iii) the proportion removed from the three-digit numbers is 252/900 (more cumbersome to calculate); and so on and we see that 1/10 < 1/5 < 252/900 < ....

§6.5. Rearrangements of the alternating series 2:( -l)n In We now look at the alternating series 1 - 1/2 + 1/3 - ... (convergent to

10g2 = S, say) which is so very closely related to the harmonic series. This is a typical example of a conditionally convergent series. Unexpected things can happen with such a series, for example, if we rearrange its terms as

1 - 1/2 - 1/4 + 1/3 - 1/6 - 1/8 + 1/5 - ... ,

where each positive term is followed by two negative ones, it is easy to see that this rearranged series converges to ~S. Indeed,

S3n = (1 - 1/2 - 1/4) + (1/3 - 1/6 - 1/8) + ... + + (1/(2n - 1) - 1/2(2n - 1) - 1/4n)

= (1/2 - 1/4) + (1/6 - 1/8) + ... + (1/2(2n - 1) - 1/2(2n)) 1

= 2(1 - 1/2 + 1/3 - 1/4 + ... + 1/(2n - 1) - 1/2n)) ,

converges to ~S, as n -t 00. Further, S3n+l = S3n + 1/(2n + 1) -t ~S and S3n+2 = S3n + 1/(2n + 1) - 1/2(2n + 1) -t ~S as n -t 00. It follows that Sn -t ~S, as required.

Such a phenomena is not possible with, for example, the alternating series

1 - 1/2 + 1/22 - 1/23 + 1/24 - ••• ,

Page 191: Surprises and Counterexamples in Real Function Theory

182 Chapter 6

a geometric progression with sum 2/3. By making a similar rearrangement, we get the series (1-1/2 -1/23) + (1/22 -1/25 -1/27) + ... , which also converges to the same sum, viz. 2/3, for

83n = (1 - ~ - 2~3 ) + C12 - 215 - 2\ ) + ... + (22~-2 - 24~-3 - 24~-1 ) = (1 - 5/23 ) + (1/22 - 5/27 ) + ... + (1/22n- 2 _ 5/24n- 1 )

= ( 1 + ;2 + 214 + ... + 22~-3 ) - (253 )

-+ 4/3 - 2/3 = 2/3 ,

as required.

The reason for why this happens for 1 - 1/2 + 1/3 - . .. but not for 1 -1/2 + 1/22 - 1/23 + ... ? is due to the fact that this latter series is not only convergent but also absolutely convergent.

We now develop the theory of unconditionally convergent series and prove the beautiful theorem of Riemann in the sequel. This will enable us to under­stand the reason behind the two interesting phenomena described above.

Definition 6.5.1. The series :z= a;l is called a rearrangement of the series :z= an if each an is an a~ and each a;l is an an, once and once only.

Definition 6.5.2. We say that :z= an is unconditionally convergent if every rearrangement of :z= an is convergent.

Our object now is to prove the following three results:

TheoreIll 6.5.3. If all rearrangements of:Z= an converge (i.e., if L an is un­conditionally convergent, according to Definition 6.5.2), then they all converge to the same sum.

In view of our opening remarks about the alternating harmonic series, The­orem 6.5.3 shows that all rearrangements of 1 - 1/2 + 1/3 - 1/4 + ... do not converge.

TheoreIll 6.5.4. :z= an is unconditionally convergent if and only if :z= an is absolutely convergent.

TheoreIll 6.5.5 (RieIllann). Let a :::; (3 be any two real numbers and let:z= an be a series of real numbers, with partial sums 8 n , which is convergent but not absolutely convergent. Then there exists a rearrangement :z= a~ of :z= an with partial sums 8~, such that

lim sup 8;1 = (3 , lim inf 8~ = a . n--+oo n--+oo

Page 192: Surprises and Counterexamples in Real Function Theory

6.5 Rearrangements of the alternating series L( _1)n In 183

and

{o ,if an 2: 0 qn = .

-an, If an ::; 0 .

The series LPn and L qn both diverge to +00, for if both were convergent, then L lanl = L(Pn + qn) = LPn + L qn would be convergent, which is not, while if LPn were convergent and L qn divergent or vice versa, then Lan = L(Pn - qn) = LPn - L qn would be divergent, which it is not.

Now let PI, P2 , ... be the nonnegative terms of L an, in the order in which they occur and let Ql, Q2, . .. be the absolute values of the negative terms of L an, also in the order in which they occur. Thus, LPn and L Qn are same as LPn and L qn and are therefore divergent, i.e. :

LPn and L Qn are divergent.

Now start adding terms of LPn one by one till PI + P2 + ... + Pn, - 1 < f3 and PI + P2 + ... + Pn, - 1 + Pnl 2: f3. This is possible since LPn is divergent (if f3 < 0, take nl = 1).

Next, subtract sufficiently many Q's from PI + P2 + ... + Pnl , one by one till PI +P2 + .. '+Pnl -Ql -Q2 - ... Qm,-l > a and PI +P2 + .. '+Pnl -Ql­Q2 - ... Qm,-l - Qml ::; a. Again this is possible since L Qn is divergent.

Now again add the P's from nl + 1 onwards, one by one, to the last sum till PI +P2 + .. '+Pnl - Ql - Q2 - ... Qm,-l - Qml +Pnl +1 + ... +Pn2 - 1 < f3 and PI +P2+·· '+Pnl -QI-Q2-" ·Qm,-I-Qm, +Pn1 +1 + .. ,+Pn2-1+Pn2 2: f3. Then again subtract from this sum the Q's from ml + 1 onwards, one by on till PI + P2 + ... + Pnl - Ql - Q2 - ... Qml -1 - Qml + Pnl +1 + ... + Pn2 - 1 + Pn2 - Qml +1 - ... Qm2-1 > a and PI + P2 + ... + Pnl - Q1 - Q2 - ... Qm,-1 -Qm1 + Pn, +1 + ... + Pn2 - 1 + Pn2 - Qm,+l _ ... Qm2-1 - Qm2 ::; a, and so on, alternate "just" crossing f3 to the right and then "just" crossing a to the left.

This gives us a rearrangement of L an, say L a~, viz. n1 positive terms of L an followed by ml negative terms, then n2 positive terms of L an followed by m2 negative terms; and so on.

Let S~ be the partial sums of L a~. The subsequence {SU of {S~}, whose last terms are Pnl , Pn2 , . .. is convergent to f3, as for such a S~,

IS~ - f31 = I(P1 + ... + Pnl - Ql - ... - Qm,) + ... + + (Pnk_l+l + ... + Pnk - 1 + Pnk ) - f31

::; Pnk

-+ 0,

as k -+ 00 (see Figure 6.14).

Page 193: Surprises and Counterexamples in Real Function Theory

184 Chapter 6

Figure 6.14

Similarly, the subsequence, whose last terms are Qml' Qm2' ... is convergent to 0:.

Remark 6.5.6. If (3, say, is 00, we may choose an increasing sequence of real numbers, for example 1,2,3, ... and then construct our rearrangement as follows:

PI + ... Pn, - 1 < 1 ,

PI + ... Pnl -1 + Pnl 2: 1 ;

PI + ... Pnl - Q1 - ... Qml + Pnl +1 + ... + Pn2 - 1 < 2 ,

PI + ... Pnl - Q1 - ... Qml + Pnl +1 + ... + Pn2 - 1 + Pn2 2: 2 ;

and so on. Then as above, the subsequence whose last terms are P"" P"2' ... is divergent to 00.

Similarly, if 0: = -00, (3 finite, or if 0: = -00 and (3 = 00, we may suitably modify the proof.

Proofs of Theorems 6.5.3 and 6.5.4. First let I: an be absolutely conver­gent. Then by the general principle of convergence, given f > 0, there exists a N = N(E) such that

(t)

Let I: a~ be an arbitrary rearrangement of I: an and let the partial sums of I: a~ and I: an be respectively S~ and Sn. Since I: an is convergent, Sn -+ S, say. Being a rearrangement of I: an, we see that

The numbers aI, a2, ... ,aN appear in the sum ai + a~ + ... + a~, if M is large enough. m

Then, since aN, aN -1, ... ,a2, a1 cancel out (perhaps more of them cancel out), we have

IS~ - SMI ::; laN+1 + aN+2 +···1 ::; laN+11 + laN+21 + ... < E,

if M is sufficiently large (to ensure (t) and (+)). Since S M -+ S, we see that S~ -+ S, showing that the arbitrary rearrangement I: a;1 is convergent and converges to the same sum as I: an is convergent to. This proves Theorem 6.5.3 and one implication of Theorem 6.5.4.

Page 194: Surprises and Counterexamples in Real Function Theory

6.5 Rearrangements of the alternating series L( _l)n /n 185

Conversely, let L an be not absolutely convergent (but of course L an is convergent). Then by Theorem 6.5.5, there is a rearrangement of L an which is not convergent (chose f3 i= a in Theorem 6.5.5), i.e., L an is not unconditionally convergent, which proves the other implication of Theorem 6.5.4.

As a very instructive example to illustrate all this, we look at the alternating harmonic series, which is convergent but not absolutely convergent. We have the following beautiful:

Theorem 6.5.7. The rearrangement of L( -1)n /n given by

{(I + 1/3 + 1/5 + ... + 1/(2p - 1)) - (1/2 + 1/4 + ... + 1/2q)} + + {(1/(2p + 1) + ... + 1/(4p - 1)) - (1/2(q + 1) + ... + 1/4q)} + ...

in which, at each stage, p positive terms are followed by q negative ones, has sum equal to ~ log( 4p / q) .

Proof. Let TN be the sum of the first N terms of the new series, each term being a sum of terms inside the (curly) brackets above. These N brackets contain (p + q)N terms ofthe original series. We add and subtract 1/2 + 1/4 + ... + 1/2p to the first bracket and 1/2(p + 1) + 1/2(p + 2) + ... + 1/4p to the second bracket, and so on, to get

TN = S(p+q)N

= { (1 + ~ + ... + _1_) _ (~ + ~ + ... + ~) + 3 2p - 1 2 4 2q

+ (~ + ~ + ... + ~) - (~ + ~ + ... + ~) } + 2 4 2p 2 4 2p

+ { (_1_ + ... + _1_) _ ( 1 + ... + ~) + 2p + 1 4p - 1 2(q + 1) 4q

+(2(P~ 1) + ... + 4~) - (2(P~ 1) + ... + 4~)} +up to N brackets

= { (1 + ~ + ... + ~) - ~ (1 + ~ + ... + ~) - ~ (1 + ~ + ... + ~) } 2 2p 2 2 q 2 2 P

+{(_1_+ ... +~) _~ (_1 + ... +~)_ 2p + 1 4p 2 q + 1 2q

- ~ (p: 1 + ... + 2~) } + ... +

+ {(2(N _11)p+ 1 + ... + 2~P) - ~ CN ~ l)q + ... + ~q) -~ CN ~ l)p + ... + ~p) }.

But by (6) in §6.2, 1 + 1/2+···+ l/n = IOgn+,+En, where En -+ 0 as n -+ 00.

Page 195: Surprises and Counterexamples in Real Function Theory

186 Chapter 6

It follows that

1 1 TN = log2Np + 'J + E2Np - "2 (log Nq + 'J + ENq) - "2(logNp + 'J + ENp)

1 2 2 1 1 = "2log4N P /Nq.Np+E2Np - "2ENq - "2ENp,

which converges to ~ log4p/q, as n -+ 00.

We may now drop the (curly) brackets and then the partial sums of the rearranged series will not just be groups of p+ q terms but will have T(p+q)N+l, T(p+q)N+2, ... , T(p+q)N+p+q = T(p+q+l)N as additional partial sums, but they all converge to the same limit clearly, for example

T(p+q)N+2 = T(p+q)N + (±l/((N - l)p + 1)) + (±l/((N - l)p + 2))

and the second and the third terms tend to 0 as N -+ 00, giving what is re­quired. This completes the proof of the theorem.

With conditionally convergent series, other peculiar things can happen, for example with the Cauchy product. We know that:

Theorem 6.5.8. If 2: an and 2: bn are absolutely convergent, then their Cauchy product 2: en, where en = aobn + a1bn- 1 + ... + anbo, is absolutely convergent and 2: en = (2: an)(2: bn)

This fails with conditionally convergent series, for example:

Example 6.5.9. Let

S = 1/v'i - 1/V2 + l/h - 1/V4 + ...

which is conditionally convergent. The Cauchy product of S with itself is the series

1/v'i.v'i - (1/v'iV2 + 1/V2v'i) + (l/v'i.h + 1/V2.V2 + l/h.v'i) - ...

Hence the absolute value of the nth term is equal to

1/v'i.,;n + 1/V2 . ...;n=1 + ... + 1/v'ri.v'i ,

and since ,jr . ..jn - r + 1 ::; (n + 1)/2 (as the geometric mean is less than or equal to the arithmetic mean), the above term is larger than or equal to (2/(n+ l)).n, which does not tend to 0 as n -+ 00, so the series is not convergent.

Example 6.5.10. The Cauchy product of the two series

2 + 2 + 22 + 23 + ... + 2n + ...

-l+l+l+l+···+ln + ...

is - 2 + 0 + 0 + 0 + ... + On + . .. .

Page 196: Surprises and Counterexamples in Real Function Theory

6.5 Rearrangements of the alternating series 2::( _1)n /n 187

This gives the example of two divergent series whose Cauchy product converges absolutely.

We have only considered what are called simple rearrangements of the alternating series and we shall continue doing so, i.e., we do not consider, for example, the rearrangement 1 + 1/7 - 1/4 + 1/3 - 1/2 + ... of the alternating harmonic series. We may put this down as the following:

Definition 6.5.11. We say that a rearrangement of 2::( _1)n-l /n is a simple rearrangement if the subsequences of positive terms and negative terms occur in their original order.

In any convergent series, we know that the convergence is unaffected if we tamper with any number of initial terms (however, the convergence may be to a different limit (sum)). In Theorem 6.5.5, in particular, all we require is that eventually P positive terms should be followed by q negative ones. With this in mind, for a rearrangement 2:: an of the alternating harmonic series, we let a = limn--+ oo Pn/n (if this limit exists), where Pn = the number of positive terms in {aI, a2, ... ,an}. We have the following (essentially a restatement of theorem 6.5.5).

Theorem 6.5.12 (see [28]). A simple rearrangement 2:: an of2::(-I)n-l/n converges (in lE. U {±oo}) if and only if a, as defined above, exists. Moreover, the sum of the rearranged series is then equal to log 2 + pog(a/(1 - a)), if a -=I- 1 and log 2, if a = 1.

Proof. Letting qn = the number of negative terms in {aI, a2, ... ,an}, we see that Pn + qn = n. Then

n ~ h

L ak = L 1/(2i - 1) - L 1/2i . k=l i=l i=l

Let r n = 2::~=1 l/k -logn, so that r n is positive and decreasing and tends to " the Euler constant, as n tends to infinity. Now we have

and

Pn 2pn Pn

L 1/(2i - 1) = L l/i - L 1/2i i=l i=l i=l

1 = (log 2Pn + r 2Pn) - "2 (log Pn + r Pn) .

Page 197: Surprises and Counterexamples in Real Function Theory

188 Chapter 6

Hence

lim ~ak= lim ((IOg2pn+r2pn)-~(logPn+rpn)-~(IOgqn+rq)) n--+<Xl ~ n--+<Xl 2 2 n

k=l

= nl~~ (lOg 2 + ~IOg(Pn/qn) +r2Pn - ~rpn - ~rqn) = log 2 + ~ lim (log(Pn/ qn)) + "I - ~"I - ~"I (if the limit exists)

2 n--+<Xl 2 2 1 .

= log 2 + -2 hm (log(Pn/ qn)) , (if the limit exists) (*) n--+<Xl

i.e., 2: an converges if and only if limpn/qn exists. Further, (*) is exactly Theorem 6.5.12. To this end, we have I/o: = limn--+<Xl n/Pn = limn--+<Xl(Pn + qn)/Pn = 1 + limn--+<Xl qn/Pn, and so limn--+<Xl qn/Pn = (I/o:) - 1 = (1 - 0:)/0:, as required. This completes the proof.

For integers p, q, define

11 111 1 1 S =-+-+ ... +------ ... ---+--+

P, q 2 4 2p 3 5 2q + 1 2p + 2 1 1 1 1 1 + -- + ... + - - -- - -- - ... - -- + ...

2p + 4 4p 2q + 3 2q + 5 4q + 1 )

as the series with blocks of P positive terms followed by blocks of q negative terms alternatively, (a rearrangement of the alternating harmonic series), with sum log 2 + ~ log(p/q). Also put

P, = (1 + ~) (1 + ~) ... (1 + ~) (1 _ ~) ... (1 __ 1 ) (1 + _1 ). P, q 2 4 2p 3 2q + 1 2p + 2

( 1 + 2p ~ 4)- .. ( 1 + 4~)( 1 - 2q ~ 3) ( 1 - 2q ~ 5)- .. ( 1 - 4q ~ 1)- .. , as the product obtained by rearranging terms of the product

correspondingly. We have the following striking

Theorem 6.5.13 (See [88]). Pp,q = JPlq.

Page 198: Surprises and Counterexamples in Real Function Theory

6.5 Rearrangements of the alternating series 2:( _l)n /n

Proof. Let

357 2n + 1 == --- ... --

= (2n). 2n + 1 . n 22n

But by Wallis' product (see §1.3), we have

as n ---+ 00, i.e.,

2244 2n 2n 7r

i 3 3"5 ... 2n - 1 2n + 1 ... ---+ "2'

( 24 2n) 2 3"5'''2n+1 (2n+1)

7r

---+ "2' which, on taking reciprocals, gives that

189

<(In = (3/2)(5/4)··· ((2n + 1)/(2n)) c::: 2vn.J(1 + 1/2n)/.;7r c::: 2vn/.;7r

and we have

(1 - 1/3)(1- 1/5)··· (1- 1/(2n + 1)) = (2/3)(4/5)··· (2n/(2n + 1)) = l/«Jn .

Hence «Jpn/«Jqn = (1 + 1/2)(1 + 1/4) ... (1 + 1/(2pn))(1-1/3)(1-1/5)··· (1-1/(2qn+1)) is equal to the partial product Ppn, qn (i.e., product with Pn positive terms followed by qn negative ones). Letting n ---+ 00, we get

. «Jpn . 2ffn/..fo r::t:./ Pp , q = hm - = hm / /:;; = y p;q .

n--+oo «Jqn n--+oo 2ffn v 7r

Corollary 6.5.14. Sp,q - SI,1 = logJ(p/q).

Proof. We have

lOgPl ,1 - SI,1 = log((l + 1/2)(1- 1/3)(1 + 1/4)(1- 1/5)···) -

- (1/2 - 1/3 + 1/4 - 1/5 + ... ) = (log(l + 1/2) - 1/2) + (log(l - 1/3) + 1/3) + + (log(l + 1/4) - 1/4) + ...

and this series is absolutely convergent since Ilog(l - x) + xl = Ix2/2 + x3 /3 + ···1 ::; (lxI 2/2)(1 + 21xl/3 + 21x1 2/4 + ... ) < ~lxI2(1 + Ixl + Ixl 2 + ... ) =

Page 199: Surprises and Counterexamples in Real Function Theory

190 Chapter 6

IxI 2 /(2(1 - Ix!)) :::; Ix1 2 , if 1 :::; 2(1 - Ix!), i.e., if Ixl :::; ~, which is the case for the above series. Hence the value of the series is unaltered by rearranging it and so is equal to log Pp, q - Sp, q. Thus log P1,1 - Sl,l = log Pp, q - Sp, q or Sp, q - Sl,l = log Pp, q -log P1,1 = log J(p/q) -log Jl7l (by Theorem 6.5.13) = log J (p / q), as required.

Example 6.5.15. What convergent rearrangements of the alternating har­monic series will have sum which is a rational number?

Indeed, let Pn be equal to the number of positive terms in the first n terms and let qn be equal to the number of negative terms in the first n terms and let Pn/ qn -t k as n -t 00. Then by Theorem 6.5.12, the sum of this rearranged series is log 2 + ~ log k. If S (= log 2) is the sum of the alternating harmonic series, then the change A in the sum is 1 log k. Write k = c2 /4 so that the sum of the rearranged series equals log 2 + l (2 log c - 210g 2) = log c and we want this to be a rational number min, say. Thus c = em/ n giving k = ie2m/ n .

So now take any sequence {an/bn } of rationals having limit ie2m/n (for example convergents of its continued fraction expansion) and take Pn = an and qn = bn .

For example, take k = 1/4. Then the rearrangement is

1111111111 1----- -- -+- - - - -- - - -+- - ...

2 4 6 8 3 10 12 14 16 5

and the sum is log 2 + ~ log k = 0, since k = 1/4.

As an example of the conditional convergence of series, we show:

Proposition 6.5.16. The series

is convergent.

Proof. Let Un be the sum of blocks of consecutive terms of the same sign. Thus

U1 = 1,

U2 = 1/2 + 1/3 ,

U3 = 1/4 + 1/5 + 1/6 ,

U4 = 1/7 + 1/8 + 1/9 + 1/10 ,

... ,

To get a formula for the nth term Un, we note that 1 + 2 + ... + (n - 1) terms of the harmonic series have preceded it, so it begins with 1/ (~(n -l)n + 1) and goes up to l/(1(n - l)n + n) (= 2/(n2 + n)), i.e.,

Un = 2/ (n 2 - n + 2) + 2/ (n 2 - n + 3) + ... + 2/ (n 2 + n)

Page 200: Surprises and Counterexamples in Real Function Theory

6.5 Rearrangements of the alternating series 2: ( -1) n / n 191

and putting n + 1 for n we get

U n +l = 2/(n2 + n + 2) + ... + 2/(n2 + 3n) + 2/(n2 + 3n + 2) .

So 0 < Un < n(2/(n2 - n + 2)) (number of terms times the largest term = n times the first term). Letting n -+ 00, we see that Un -+ O.

Next, we estimate Un - U n+l. We have Un > n(2/(n2 + n)) = 2/(n + 1) (number of terms times the smallest term = n times the last term) and U n +l < 2(n + 1)/(n2 + n + 2) (by the above inequality, with n + 1 for n), so -Un +l > -2(n + 1)/(n2 + n + 2). It follows that Un - U n+l > 2/(n + 1) - 2(n + 1)/(n2 + n + 2) = 2( -n + 1) which is not greater than 0, so we need a better estimate. The inequality Un > 2/(n + 1) is, as it is, good enough. As for Un +1, we have un +1 < (smallest of the first n terms times the number of terms + the last term) =n(2/n(n+3))+2/(n2+3n+2). Henceu n -un +l > 2/(n+l)-2/(n+ 3)-2/(n+l)(n+2) = 2(n+l)/(n+l)(n+2)(n+3) = 2/(n+3)(n+2) > O. It follows that Un is a decreasing sequence and we have seen above that Un -+ O. Hence the series 2:( _1)n+l.un is convergent. But then 5 is also convergent, for let 5 N be the partial sum of the first N terms of 5, and Un the partial sums of Ul - Uz +U3 - U4 + .... Then 5N = Un + rn for a suitable nand rn

is the sum of the first m terms of U n +l (sign ± included), for a suitable m (for example if N = 18, then 518 = U5 - 1/16 - 1/17 - 1/18 = U5 + r5, where r5 is the sum of the first 3 terms of U6). As N -+ 00, so does n and Un tends to a limit U, say. Also Irml < U m +l -+ 0, hence 5N -+ U.

Following [36], we shall now give estimates for the sum 5 of the alternating series 1 - 1/2 + 1/3 - 1/4 + ....

Grouping two terms at a time and collecting them into a fraction we get

5 = (1 - 1/2) + (1/3 - 1/4) + ... = 1/1.2 + 1/3.4 + 1/5.6 + 1/7.8 + .. . > 1/1.2 + 1/3.5 + 1/5.7 + 1/7.9 + .. . 111

= 1/1.2 + "2(1/3 - 1/5) + "2(1/5 - 1/7) + "2(1/7 - 1/9) + ...

1 = -(1 + 1/3 - 1/5 + 1/5 - 1/7 + 1/7 - 1/9 + ... )

2 1

= "2(1 + 1/3) = 2/3.

Moreover,S = 1- (1/2 - 1/3 + 1/4 -1/5 + 1/6 - 1/7 + ... ) and again making

Page 201: Surprises and Counterexamples in Real Function Theory

192

a common denominator of two terms at a time, we find that

Hence

s = 1 - (1/2.3 + 1/4.5 + 1/6.7 + ... ) < 1 - (1/2.4 + 1/4.6 + 1/6.8 + ... )

1 = 1 - -(1/1.2 + 1/2.3 + 1/3.4 + ... ) 4 1

= 1 - -(1 - 1/2 + 1/2 - 1/3 + 1/3 - 1/4 + ... ) 4

= 1 - 1/4 = 3/4.

2/3 < S < 3/4.

Chapter 6

Other estimates for the partial sums of this alternating harmonic series are also available.

Let Cln = 1 - 1/2 + 1/3 - 1/4 + ... + (_1)n+1 In. Since Cln ~ Cl = 10g2, as n ~ 00, we have

I Cl - Cl n I < E if n 2: N .

Now, given E = l/k (k a positive integer), we can calculate at what stage (*) begins to hold, i.e., the least value on N such that (*) holds. Indeed, we have

iCl - Clni = l/(n + 1) - l/(n + 2) + l/(n + 3) - .. .

= l/(n + 1) - (l/(n + 2) - l/(n + 3)) - .. .

<1/(n+1)

and this is less than l/k as soon as n + 1 > k, so

ICl - Clnl < l/k if n 2: k. (i.e., N = k will do) (~)

But of course we have used very crude estimates here and by no means will k be the smallest value of N. The following result improves this value.

Theorem 6.5.17. ICl - Clnl < 1/2k , if n 2: k.

Proof. Let Rn = Cl - Cl no Then

and

IRn- 1 1 = l/n - l/(n + 1) + l/(n + 2) - l/(n + 3) + ... = l/(n(n + 1)) + l/(n + 2)(n + 3) + ... ,

IRnl = l/(n + 1) - l/(n + 2) + l/(n + 3) - l/(n + 4) + ... = l/((n + l)(n + 2)) + l/((n + 3)(n + 4)) + ....

Page 202: Surprises and Counterexamples in Real Function Theory

Exercises 193

Now compare these with the series

1 1/2n = 2"(l/n - l/(n + 2) + l/(n + 2) - l/(n + 4) + l/(n + 4) - ... )

= l/n(n + 2) + l/(n + 2)(n + 4) + ... ,

and we at once see that IRn-11 > 1/2n > IRnl (since l/n(n+ 1) > 1/n(n+2) > l/(n + l)(n + 2) and so on for each term) and this is what is required.

There is another very simple, even closer, approximation to IJ. Let

1 IJ~ = 1-1/2+ 1/3 -1/4+··· + (_1)n+ 1 /n + (-1)n+22"(1/(n + 1))

1 = IJn + (-l)n2"(l/(n + 1)) .

We then have:

Theorem 6.5.18. IIJ - IJ~I < 1/2n(n + 1).

Proof. The preceding inequality gives

1/2n> IRnl > 1/2(n + 1)

and on subtracting 1/2(n + 1) throughout, we get

1/2n(n + 1) > IRnl - 1/2(n + 1) > 0 .

Then

IJ - IJ~ = IJ - IJn + IJn - IJ~

1 = Rn - (-l)n2"(l/(n + 1))

= (-l)n(IRnl- 1/2(n + 1)) .

Hence IIJ - IJ~I < 1/2n(n + 1).

Thus, for example, if 2k = 1000, then by (~), 11J-lJnl < 1/1000, if n 2: 1000, whereas, according to Theorem 6.5.17, IIJ - IJn I < 1/1000, if n 2: 500. Also, IIJ - IJ~I < 1/1000, when n 2: 22, by Theorem 6.5.18.

Exercises

6.1. True or false? Justify

(i) L a~ convergent =? L a~ convergent.

(ii) an 2: 0, L an convergent =? L a~ convergent.

Page 203: Surprises and Counterexamples in Real Function Theory

194 Chapter 6

(iii) :z= an convergent =? :z= lanl/n convergent.

(iv) The sequence {an} converges if for each r 2: 2, {arn} converges.

(v) The sequence {an} converges if there exists a k, 0 ::; k < 1, such that lan+danl ::; k for all sufficiently large n.

(vi) The sequence {an} converges if an's are bounded and an+1 - an -+ 0 as n -+ 00.

(vii) The series 2: an is convergent, an i=- 0,1, then 2: a; is convergent.

(viii) The series :z= an is convergent, an -::f- 0,1, then 2: an/1- an is convergent.

(ix) The series :z= an is convergent, an -::f- 0, 1. If an > 0 for all n, then :z= a;t is convergent.

(x) The series 2: an is convergent, an -::f- 0,1. If an > 0 for all n, then 2: an/(l - an) is convergent.

6.2. Let {an}, {bn} be sequences such that an -::f- 0, :z= an is convergent and bn/an -+ 1 as n -+ 00. Prove that if an > 0 for all n, then 2: bn is convergent. Give an example to show that 2: bn need not converge if the restriction an > 0 is removed.

6.3. Let {an} be a decreasing sequence of positive numbers. Using the general principle of convergence, or otherwise, prove that if there is a positive number k such that an 2: kin for infinitely many n, then 2: an diverges.

6.4. :Z=(cn + idn) is convergent and all the cn's , dn's are greater than or equal to O. Let 5 m be the point in the complex plane representing :Z=(cn + idn ).

Show that the length of the polygonal path 5 1 5 2 ... 5 N remains bounded as N -+ 00. Does the result remain true if the condition Cn, dn 2: 0 is replaced by Cn + dn 2: O? Justify your answer.

6.5. If dn 2: 1 for all n and the sequence {dn + d-;;l } converges, prove that the sequence {dn } converges.

Produce an example to show that this result is no longer true if the condition dn 2: 1 is replaced by dn 2: k, where k < 1.

6.6. 2: an is convergent and an > O. The numbers Pn are defined by

By using the inequality 1 + x < e (x > 0), or otherwise, prove that Pn tends to a limit as n -+ 00. Hence or otherwise prove that if Rn = 2::n+1 ai, then 2:~=1 (an/ Rn) is divergent.

Page 204: Surprises and Counterexamples in Real Function Theory

Chapter 7

X· The infinite exponential XX

and Related Topics

195

The object of this chapter is to answer the following attractive and pertinent questions:

Q1. When is xY = yx (x, y positive real numbers)?

Q2. More generally, when is xY ::; yx (x, y positive real numbers)?

Q3. Is there an explicit formula for y as a function of x, that gives the solutions of the equation xY = yX (this is where the title of the chapter comes in)?

§7.1. The equation x Y = yX and the parameterization

The first question is most interesting and has attracted attention for some 250 years; from the time of Bernoulli (1728), who was looking for integral solutions of x Y = yX, to the work of Maher and Breuch (1963) and of Sato (1972), who characterized the algebraic solutions of x Y = yX. For an exhaustive set of references and an excellent account of a part of the material of this chapter, see [58].

Let us begin by looking at the graph of the function

f(x , y) = xY - yX = 0 ,

so that the solutions of f = 0 are precisely the ones to satisfy xY = yX. More­over, by plotting f = 0, we get the various regions in which the sign of f is maintained, thus answering the second question above. Taking xyth roots, we need only consider the equation

To get the solutions of this equation, we first plot the graph of the auxiliary function¢(x) =xl/x. One easily checks the following features of this function:

1. ¢ has a maximum equal to e1/e at x = e,

2. ¢(x) ~ 0 as x ~ 0, ¢(x) ~ 1 as x ~ 00,

Page 205: Surprises and Counterexamples in Real Function Theory

196 Chapter 7

3. ¢(x) increases in (O,e] and decreases in [e,oo) (d¢/dx > 0 in (O,e) but is less than 0 in (e,oo)).

We may thus plot the graph of ¢(x) = xlix (see Figure 7.1).

y

max of Xlix

e lie ,.,..-- ~

/ A B l"-I

) ~I/~ n 1/ll

x o ~ 2 e 3 4 11 5

Figure 7.1

Now to determine a pair (x,y), x f- y such that xY = yX, i.e., xlix = yl/y, all we need to do is to take a line A parallel to the x-axis to meet the graph of ¢ in points A,B, say, with A = (~,e/~), B = (rJ,17l /7)). But e/~ = T]1/7), since A is parallel to the x-axis. So we have determined ~,T] such that e/~ = T]1/7) , as required. From the graph of ¢, it is now clear that if

(i) A is above or at el/e (the maximum of ¢), no such pair (~, T]), ~ f- T] exists.

(ii) A is below or at 1, again no such pair (~, T]), ~ f- T] exists.

(iii) A is strictly between 1 and e 1/ e, then for each ~ E (1, e), there exists a unique T] E (e,oo) such that e/~ = T]1/7) (i.e. ~7) = T]~).

(iv) As ~ --t 1, T] --t 00 and vice versa and as ~ --t e, T] --t e.

It follows that the graph of xY = yX, i.e., the set {(x, y) I xY = yX ,x, Y E JE.} consists of the following:

(1) The line x = y, the branch of equal points called the trivial branch. (2) The points (~, T]) determined above.

Page 206: Surprises and Counterexamples in Real Function Theory

7.1 The equation xY = yX and the parameterization

3

e

2

y I

IV

III

II

--------~----~-----.--,-r------------------x

o 2 e 3

Figure 7.2

197

By (iv) above, this graph is as shown in Figure 7.2. These two branches divide the first quadrant into 4 parts denoted by I, II, III, IV, shown in the figure. In IV and II, f(x, y) = xY - yX < 0, while in I and III, f(x, y) > O. To see this we merely have to check on anyone (suitable) point in each part, for example in part IV, we may take the point (1,2) and get f(1,2) = 12 - 21 = -1 < 0 and so f(x, y) < 0 for all points of IV. Similarly in I, we take the point (3,4) and get f(3,4) = 17> 0, and so f(x,y) > 0 for all points ofI, etc.

Example 7.1.1. Which of 7re and e7r is bigger? Indeed, the line y = e manages to always remain in parts IV and II, only

crossing the point (e, e) on the boundary. It follows that for any point (x, e) on this line (x i- e), x Y - yX < 0, i.e., x e - eX < 0 for all x i- e, 0 < x < 00,

which itself is an interesting inequality. Taking x = 7r, we get 7re < e7r •

Although, technically all the required pairs (x, y) for which xY = yX (and even those for which x Y < yX or x Y > yX) are now available, taking any value of x (1 < x) the unique corresponding y for which xY = yX is still not explicitly available; all we have is that given x = ~, the corresponding y = 77 is given by ~Y = y~. So how does one get y as a function of e

The nontrivial branch (which is what matters) can be parameterized as

x = Sl/(8-1) , Y = S8/(8-1) , where S E lR+ .

Observe that s = y/x and x 8 = y. So yX = x 8X = xY. Thus for each unequal pair (~, 77), there exists a unique (Y E lR+, such that ~ = (y1/(a-1), 77 = (ya/(a-1). In fact we have the following :

Theorem 7.1.2. Let~, 77, (Y be as above, i.e., ~ = (y1/(a-1), 77 = (ya/(a-1),. then

(i) ~1) = 77~ ,

Page 207: Surprises and Counterexamples in Real Function Theory

198 Chapter 7

(ii) a- = T)/~ ,

(iii) (Carlini-1889) ~.a- = ~(]".

Proof. (i) We have to prove that

(a-l/((]"-l))(]"~/(~-l) = (a-(]"/((]"-1))(]"1/(~-1)

or, taking logarithms, that (a-(]" /((]"-l)) (1/ (a--l)) log a- = a-1/((]"-1) (a- / (a--l)) log a-. Cancelling (log a-) / (a--l), it remains to show that a-(]" /((]"-l) = a-.a-1/((]"-1) , which is easily checked. (ii) T)/~ = a-(]"/((]"-l) /a-1/((]"-1) = a-((]"/((]"-l)-l/((]"-l)) = a-k-1)/((]"-1) = a-.

(iii) We must show that (a-1/((]"-1)).a- = (a-1/((]"-1)t. Here, the left hand side is equal to a-(1/((]"-l)+l) = a-(]"/((]"-l), while the right hand side is equal to a-(1/((]"-1)).(]" = a-(]"/((]"-l), too.

Observe that (iii) gives us the solution of when the product and power functions are equal.

With the above parameterization, we can produce lots of simple solutions (C T)) of ~1) = T)t,: Take s = 1/2,2/3,3/2,2,3, ... , which give respectively

~ = 4,27/8,9/4,2,)3, ... and T) = 2,9/4,27/8,4,3)3, ....

Thus, for example (y'3)3'/3 = (3v'3)v'3. Note that if s gives the pair (~,T)), then l/s gives the pair (T),~).

For s = I, there is an apparent singularity, but it is only apparent, for if we take the limit as s --+ 1 and substitute s = t + 1, we get the usual expression for e, viz.

lim(1 + t)l/t = X = e = y = lim(1 + t)(l+l/t) . t~O t~O

This parameterization enables us to characterize some other solutions of significance, for example we shall prove the following results:

1. The only integer solutions of x Y = yX are (~, T)) = (2,4) and (4,2).

2. Solutions (~, T)) of x Y = yX, with ~,T) E Q are given by s = m/(m + 1) or (n + 1)/n (m, n EN).

3. Solutions (~, T)) of x Y = yX, with ~,T) algebraic numbers are given by s = m/n E Qt.

4. Solutions (~, 1]) of x Y = yX, with ~, 1] algebraic integers (a real number' Ct is called an algebraic integer if a satisfies a monic polynomial with rational integral coefficients) are given by s = m or l/n (m, n E N).

Theorem 7.1.3. The only pair (m, n) that satisfies xY = yX, with m, n E N, m > n, is (4,2).

First Proof. Write m = n + a, a E N. Substituting in the equation gives

(n + a)" = n(n+a) or (1 + a/n)n = n a .

Page 208: Surprises and Counterexamples in Real Function Theory

7.1 The equation xY = yX and the parameterization 199

Here, the left hand side is equal to

n.a n(n - 1) a2 n(n - 1) ... (n - r + 1) aT 1+-+ + ... + -+ .. . +

n 2! n 2 r! n T n(n - 1) ... (n - (n - 1)) an

+ -n! nn

=l+a+- 1-- +- 1-- 1-- + . .. + a2 ( 1) a3

( 1) ( 2) 2! n 3! n n

aT( 1)( 2) (r-1) an

( 1) ( n-1) + -:;:r 1 - ;-;: 1 -;-;: ... 1 - -n- + ... + n! 1 -;-;: ... 1 - -n-

a2 aT an <l+a+-+···+-+· ··-

2! r! n!

Now (*) gives n a < ea or n < e. It follows that n = 1,2. Here n = 1 gives m 1 = 1m , i.e., m = 1, giving the trivial solution (1,1); while n = 2 gives m 2 = 2m , i.e., m = 4 (since for m > 4 we get 2m > m 2 ).

Second Proof. Write (x, y) for a solution, so that x> y > O. Write x = y + a (a E N). Then xY = yX implies that (y + a)Y = y(y+a).

Factor y = pr 1 p~2 . .. P~' as a product of prime powers. Then

(y + a)Y = ya.(prlp~2 .. . p~. )Y

and so p~iY i(y +a)Y, i.e., pfi iy+a and so yiy+a, giving yia, say a = kyo Then x = y + a = y + ky (k ::::: 1) = y(l + k) = yK, say, K ::::: 2, and so xY = yX implies (Ky)Y = yKy implies that (Ky)Y = (yK)Y and hence Ky = yK, so that

K = yK- l (K::::: 2) .

K = 2 gives 2 = Y and so x = 2y = 4. Now, let K > 2. Write y = 1 + t, where t ::::: 1 (as t = 0 implies y = 1,

giving the trivial solution) . Then by (U),

K = (1 + t)K-l = 1 + (K - l)t + ((K - l)(K - 2)/2!).t2 + . . . + tK- 1

> 1 + (K -l)t

::::: 1 + (K - 1) (since t ::::: 1)

=K,

i.e. , K > K, which is a contradiction. So K > 2 does not give any solution.

We now consider the rational solutions of the equation xY = yX, which we write as (see [104])

(logx)/x = (logy)/y .

If y = log x, dy/dx = l/x ---+ 0 as x ---+ 00.

Page 209: Surprises and Counterexamples in Real Function Theory

200

tan e = m = lie a=e

y

Chapter 7

----------~~~~~--~~--------------- X 2 e 3

Figure 7.3

So the graph of y = log x becomes parallel to the x-axis as x -+ 00. First we determine m for which the line y = mx is tangent to the curve y = logx. Suppose the tangent at P == (a,loga) to y = logx goes through (0,0). Then m = (dYldx)p = 11a. So the tangent is y = xla and (see Figure 7.3) since P lies on it, we have loga = (lla).a = 1, giving a = e, i.e., m = lie. Hence, we have essentially proved the following:

Lemma 7.1.4. The line y = mx meets the curve y = log x in no real pO'int, one real point, two real points, according as m satisfies

1 Ie < m < 00 ; m = 1 lear m < 0 , 0 < m < 1 Ie.

Now take 0 < m < lie and let {y = logx} n {y = mx} = {Xl,X2}. Then

m = (logxdlxl = (logx2)lx2

and we simply have to determine all the rational solutions of (* )". Let xl, X2 be rational solutions and let x21xl = r E Ql. We let Xl < X2 as

in Figure 7.4, so that r > 1 and we write r = s + 1 (s rational and positive). Substituting in (*)" gives (logxdlxl = (logrxdlrxl' which on simplification, gives xir-l) /r = rl/r, i.e., Xl = rl/(r-l) and so X2 = rr/(r-l), i.e. , Xl = (1 + 8)1/8, X2 = (1 + 8)(1+8)/8. Write 8 = alb (a, bEN, (a,b)=l). Then

Xl = ((a + b)lb)b/a , X2 = ((a + b)lb)(aH)/a

and it is easy to see, since (a , b) = 1, that both (a + b)l/a and bl /a must be integers: say (a + b)l/a = nl, bl /a = n2, so that nl > n2, say nl = n2 + t (t positive integer).

Page 210: Surprises and Counterexamples in Real Function Theory

7.1 The equation xY = yX and tile parameterization 201

y

o x

Figure 7.4

Then

so that (n2+t)a = nf = a+b and n~ = b. Subtracting, we get (n2+t)a-n~ = a, i.e.,

a.n~-lt + (a (a - 1)/2!).n~-2e + ... + ta = a .

Here, n2, t are positive integers and so, unless a = 1 (and then nl = 1 + b, n2 = b, t = 1), the left hand side is clearly greater than a, i.e., a > a, which is impossible. It follows that a = 1 and hence s = l/b (b = 1,2, ... ). Taking b = 1,2, ... successively, we get respectively

( l+b)b (3)2 (4)3 Xl = -b- = 2, "2 ' 3" ' ... which is an increasing sequence,

and

( l+b)l+b (3)3 (4)4 x2 = -b- =4'"2 ' 3" , ... which is a decreasing sequence, (**)'

and as m increases to 1/ e, Xl increases to e, while X2 decreases to e. However, if m < (log 2) /2, the corresponding Xl is less than 2. But we have seen that 2 is the smallest value of Xl for which Xl, X2 are both rational. It follows that if 0 < m < (log 2) /2, at least one of the corresponding Xl, X2 is not rational. This gives the following surprise (see Figure 7.5):

Page 211: Surprises and Counterexamples in Real Function Theory

202 Chapter 7

y

y= logx

(e , log e) = (e , 1)

tan a = m = % (log 2)

o X2 =4 x

Figure 7.5

Theorem 7.1.5 If (log 2)/2 ::; m < lie, the line y = mx meets the graph of y = log x in Xl, X2 (Xl < X2) and all possible such rational pairs (Xl, X2) are given by (**) and (**)' above. If, however, 0 < m < (log 2) 12, at least one of the corresponding Xl, X2 is not rational.

Remark 7.1.6. Observe that (**) and (**)' give the result (2) of the four results stated above Theorem 7.1.3, as may be easily checked. Remember that result (2) gives the full set (~, '1]) and ('I],~) of solutions, whereas (**) and (**)' gives only one pair.

Theorem 7.1. 7. Solutions of x Y = yX (0 < X < y) with x, y real algebraic numbers are parameterized by

X = S8/(8-1) , Y = 8 1/(8-1)

where 8 E Q n (0,1). The solutions with x, y real algebraic integers are param­eterized by

X = (k + l)l/k , Y = (k + l)l+l/k , k = 1,2, ....

Proof. It is easy to check that the x, y given in (*)' are algebraic numbers where as those in (*)" are algebraic integers and that either set satisfies the equation x Y = yX. In fact this has already been done for the set (* )'. For the set (*)", we have

and

Page 212: Surprises and Counterexamples in Real Function Theory

7.1 The equation x Y = yX and the parameterization 203

which is as required. Note also that the condition 0 < x < y entails 8 E (0,1), for x/y = 88/(8-1)-1/(8-1) = 8 and so 8 < 1 if and only if x < y; and 0 < x implies 0 < 8. Conversely, let 0 < x < y, where x, yare real algebraic numbers and satisfy xY = yX. Write 8 = x/y and let U = 8/(8 -1) = x/(x - y) (check), v = 1/(8-; 1) = y/(x-y) (check), so that 8,U,V are all real algebraic numbers. We are to show that 8 is rational (note that with this value of 8 (viz. x/y), 88/(8-1) = x and 81/(8-1) = y, as before).

Suppose 8 is irrational. Then u, v would be irrational too (if, for example, U is rational, i.e., 8/(8 - 1) = alb, then 8b = 8a - a, i.e., 8 = a/(a - b) E Q). But 8u = X,8 v = y (check: 8v = (x/y)y/(x-y) = y since (x/y)Y = yX-Y as xY = yX) and so by the Gelfond-Schneider Theorem (Chapter 1), x, y would be transcendental, which is a contradiction as we are supposing (x, y) to be an algebraic solution. So 8 is rational, say 8 = min, 0 < m < n (since 0 < 8 < 1), as required.

Next, let x be an algebraic integer; then so is xn-m. But xn- m = (8 s~l )n-m = (m/n)(m/n).(n-rn)/«m-n)/n) = (m/n)-m = nm/mm , with (nm,mm) = 1. It follows that m = 1 and since 0 < m < n, we see that 2 :S n. We then get x = 88/(8-1) = (I/n)(1/n)/(I/n-l) = n(I/(n-l)) (since 8 = min = I/n), and similarly y = nn/(n-1) (n = 2,3, ... ). Letting k = n - 1 (so that k = 1,2, ... ) this gives

x = (k + I)I/k , Y = (k + I)(k+l)/k, k = 1,2,3, ....

That completes the proof of statements (3) and (4) stated above Theorem 7.1.3. The above proof is due to D.Sato [100].

Remark 7.1.8. Questions (3) and (4) were parts of a problem posed by K.Mahler in 1963 (see [64]) and solved by many (see [60]). The result may also be written as follows:

Suppose m, n are algebraic numbers satisfying mn = nm with mn(n -I)(m - I)(m - n) -I- 0 (i.e., m, n -I- 0,1, m -I- n). Then there exist integers h, k with h, k -I- 0, h -I- k, (h, k) = 1, such that m = (h/k)k/(h-k) , n = (k/h)h/(k-h).

If further, m, n are algebraic integers, then h or k equals 1.

Example 7.1.9. We show that m(n=) = n(mn) has no solution in positive

integers m, n with m -I- n. Indeed, without loss of generality, let n > m 2: 1.

Case 1: m = 1. Then our equation becomes 1 = nn, so n = 1 = m, as required. Ca8e 2: m = 2, n = 3 or 4. Our equation becomes 2(n2

) = n(2n).

For n = 3, the left hand side equals 29 while the right hand side equals 38 ,

so the two can not be equal. For n = 4, the left hand side equals 216 while the right hand side equals 416 , and again the two sides are different. Case 3: m = 2, n 2: 5. Now the function f(x) = xlix is decreasing for x > e and so 41/ 4 > n 1/n (since n 2: 5 > e), i.e., 4n > n4, or taking square roots, 2n > n 2 and so, a fortiori, 2n(1ogn) > n2(1og2), or n(2n) > 2(n2) and since m = 2, this gives n(mn) > m(n=).

Page 213: Surprises and Counterexamples in Real Function Theory

204 Chapter 7

Case 4: n > m 2: 3. Again, since x 1/ x decreases, m1/m > n1/n or mn > nm or mn(logn) > nm(logm) (a fortiori) or n(mn

) > m(n=).

§7.2. The infinite exponential xX x·

We have already mentioned that given ~, the corresponding TJ satisfying the equation ~'1 = TJ"-, is not readily available, even after the parameterization x = Sl/(8-1), Y = S8/(8-1), where s E lR+. For example, if say ~ = y'2 (so that 1 < ~ < 2), trying to get s from the equation y'2 = SI/(8-1) is not possible and without this s, the corresponding TJ = S8/(8-1) is not available. Thus we really have to have something even better than the above parameterization. This is exactly where the beautiful "function"

x· y = h( x) = XX ,i.e., y equal to the limit of the sequence x, xx, .-rxx , . .. , (*)

comes into the picture. Before we consider the connection of this function with that of f(x,y) = x Y - yX, we must determine the values of x for which h(x) is meaningful. Thus, for example, if x = 2, the sequence 2,22 ,222 , ...

is obviously divergent and it would seem that for x > 1, the sequence would diverge. However, it surprises most to learn that there are values of x greater than 1, for which h(x) converges. As it turns out:

TheoreIll 7.2.1. The infinite exponential, as defined above, converges for all values of x in the range [e-e, e1/ e], i. e., for all x satisfying

.06598803585 ... = e-e ::::; x ::::; e1/e = 1.444667861 ...

We first give a graphical motivation of this result (see D.F.Barrow [8] or M.C.Mitchelmore [68]).

Graphical proof (Illotivation) of TheoreIll 7.2.1. Let (1 = a, (2 = aa, (3 = aa" = a(2, ... ,(n = a(n-l and ask for what values of a, the limit, as n tends to infinity, of (n exists. The trick is to consider the graph r of the function y = aX, which, in spite of having a more or less uniform shape, varies a great deal, for different values of a, in its orientation and its relative position to the graph G ofthe line y = x, and it is this fact that will be seen to account for convergence or divergence of the sequence {(n} for various values of a. We need to split different cases: Case 1: 1 < a < e1/e. To fix our ideas, let us take a = y'2 = 1.4142 .... The graph rand G of y = (y'2)X and y = x respectively are shown in Figure 7.6.

We see that the set r n G = {(2,2), (4,4)} and the points Q1 = (y'2,y'2v2),

Q2 = (y'2v2, y'2v2v'2) , ... can be successively got by chasing the zigzag shown in Figure 7.6, starting with a = y'2 on the x-axis and going up to Q1 on r,

Page 214: Surprises and Counterexamples in Real Function Theory

x· 7.2 The infinite exponential XX 205

then to PIon G, onto Q2 on r, and so on. It is clear from the graph that

y'2' Qi --+ (2,2), showing that v'2 --+ 2.

y

r: y= (,J2y ---l---

---'~------~'---~~'--------------T-+X -12\12 2 o 4

Figure 7.6

However, there is a second point of intersection of rand G, viz. (4,4), but clearly the process stops at the point with smaller coordinates (we shall prove the existence of this limit analytically as well; however, the graphical "proof" is far more revealing, illuminating and instructive).

Case 2: a = e l /€. This zigzag process described above fails if rand G do not intersect in real points. As a increases, a time will come when G is tangent to r. The critical value, when this happens, is a = e l /€. So let us draw the graph r of y = (eI/e)x (see Figure 7.7).

The point (e, e) E r. Let us determine the tangent to r at (e, e). We have y = ex/e and so logy = xle, giving dyldx = yle = ex/e Ie. At the point (e, e), the value of d y I d X is equal to e I e = 1. The equation of the required tangent is therefore y = X + c. Since this goes through (e, e), c = 1 and so the tangent is the line y = x, as required.

Page 215: Surprises and Counterexamples in Real Function Theory

206 Chapter 7

y

A'-, where A = e1/e

2 e 3

Figure 7.7

Now, taking x = el/e on the x-axis, we see that the zigzag of points Ql, Q2, . .. --+ (e, e), in particular,

--+ e .

Case 3: a > el/e. The curve r : y = aX has graph shown in Figure 7.8, and a·

it is obvious that for such values of a, the infinite exponential a a diverges. Case 4: 1 < a < el/e. Here, exactly as for the case a = vI2 in case 1, the sequence

converges. There will, of course, be 2 points of intersection of rand G, since aX --+ 00 faster than x (since a > 1) and at x = 0, y = 1, so r must cross G

twice. The graph of various a, 1 :::; a :::; el/e are shown in Figure 7.9 and a a a·

converges to the smaller of the x-coordinates of the two points of intersection.

Page 216: Surprises and Counterexamples in Real Function Theory

x· 7.2 The infinite exponential XX

y

2

Q1 = (a,aa)

y = aX

P1 = (aa , aa)

----~--~~-.~~,,-----------------.x

Figure 7.8

y

Figure 7.9

y=ax, where a is general

x

207

For various a < 1, the graphs of y = aX behave differently, since, as X -+ 00,

aX -+ 0 and as X -+ -00, aX -+ 00 and so the graphs r have shape shown in Figure 7.10.

As it turns out, we need to consider the graph

r* : x = aY

as well, which is the mirror image of r : y = aX in the line G : y = x and we further need to determine the points r n r * in this case (i.e., when 0 < a < 1),

Page 217: Surprises and Counterexamples in Real Function Theory

208 Chapter 7

i.e., all the real solutions of the pair of equations

(i) y = aX and (ii)x=aY

y

x Figure 7.10

Now one solution of (*) is x = y = (, say. Let P be the point ((, (). If the absolute value of the gradient of (i) at P is greater than 1, then clearly the graph of (i) will drop below the graph of (ii) as x moves to the right of P and further since it stays above the x-axis, it follows that the two graphs must cross each other once more, at Q, say. By symmetry about G, there is then a third point R of intersection of rand r * (see Figure 7.11).

Y

r* : x = aY

r: y = aX

G :x=y

r: y = aX

.................. """ .................. r ...• Jx~L --~-----------------------------=~~~--+x

Figure 7.11 (1,0)

Page 218: Surprises and Counterexamples in Real Function Theory

x· 7.2 The infinite exponential XX 209

The problem now is to find when (i.e., for what values of a) the graph f has absolute value of the gradient greater than 1 at P. We shall see that the critical value is at a = e-e. For then, our f is y = (e-e)x = e-ex and so dyldx = e-ex.log(e-e) = -e.e-ex. To get P, we solve y = e-ex with y = x, giving e-ex = x. Since there is only one real solution (because f is decreasing) and X = lie is a solution, it is the solution, i.e., P = (lie, lie). Hence (dYldx)p = (_e)e-e(l/e) = -1.

So now the gradient of f, at the point P E f n G changes from -1 to 1 as a varies from e-e to e1/e. For a > e1/e, (dYldx)p > 1 while for a < e-e, (dYldx)p < -1 and as a --+ 0, (dYldx)p --+ -00 and so in the range 0 < a < e-e, l(dYldx)pl > 1 and f n f* consists of 3 points, as shown in Figure 7.11. In case this graphical consideration does not satisfy the reader, we may prove the above analytically as follows: Since y = aX, d y I d X = aX log a. Then (dYldx)p = xloga = log aX = logx (since aX = y = x at P) and this is less than -1 if and only if

logx < -1 ¢} x < e- 1

¢} aX < e- 1

¢} xloga <-1

¢} log a < -1 I x

¢} loga < -e

¢} a < e-e ,

(since -l/x < -e by (*))

as required. Now let us collect all the information that we have observed so far that is relevant to the behaviour of f : y = aX.

Let a > O. Mark off, on the real line, certain points as shown below:

• • • • o e-e e lie

e -e = .06598803585 ..... . el/e = 1.444667861. .....

Figure 7.12

Let

(i) f : y = aX (ii)f*:x=aY (iii)G:y=x,

so that f, f * are mirror images of each other in G. Then the following results hold:

1. If e-e :::; a:::; 1, f n f* consists of only one point, say P = (a,a) E G. The gradient of (i) at P is greater than or equal to -1 (it is equal to -1 when a = e-e and is 0 when a = 1).

Page 219: Surprises and Counterexamples in Real Function Theory

210 Chapter 7

2. If 0 < a < e-e, r n r* consists of 3 points, say, R = (a,;3), P = (r,')'), Q = (/3, a), a < /3, and the gradient of (i) at P is less than or equal to -1 and as a -+ 0, this gradient tends to -00.

3. If 1 < a < eI/e, r n r* consists of 2 points, say, P = (a,a), Q = (/3,;3), a < /3 and the gradient of (i) at P lies between 0 and 1 (it is 0 if a = 1 and is 1 if a = el/e; for a = 1, Q does not exist, while for a = el/e, Q and P coincide).

4. If a = el/e, r n r* consists of one point, say, P = (a,a) E G, in fact G is tangent to both of rand r *, and the gradient of (i) at P is 1.

5. If el/e < a, r n r* = 0.

For the convergence of (n = a(n-l, (1 = a, for 0 < a < 1, two cases arise:

(i) 0 < a < e-e ,

(ii) e-e ~ a < 1 .

It will be found that in case (ii), (n is convergent to the unique point P = r n r * while in case (i), (n has no limit but two limit points (accumulation points) R and Q, as described above and indeed (2n -+ R, (2n-l -+ Q, as n -+ 00. In fact we have R -+ (0,1), Q -+ (1,0), as a -+ O. Case 5: e-e ~ a < 1. Let us take a typical value of a, say a = 1/2 (e-e ~ 1/2 < 1). The graph of y = (1/2V is shown in Figure 7.13.

y

Y=(~r

x a = ~ (Yzyl. A

Figure 7.13

The sequence (n, n = 1,2, ... , is the x-coordinate of Qn and these Qn's are given by a spiral instead of a zigzag as was the case for 1 < a ~ e l /". It is now

Page 220: Surprises and Counterexamples in Real Function Theory

x· 7.2 The infinite exponential XX 211

clear, but perhaps not as clear as in the zigzag case, that the spiral of points Qi, Pi tends to the unique point of intersection P of rand G. Case 6: 0 < a < e-e .The relevant graph for a typical a with 0 < a < e-e, now looks like Figure 7.14. Here, the spirals alternatively tend to the points Rand Q, as required. Analytic proof of Theorem 7.2.1. We now give an analytic proof of all these cases discussed above. More specifically, we prove: Let a > 0 be a real number and let (1 = a, (2 = a(l = aa, ... ,(n = a(n-l.

• The sequence {(n} is convergent to a limit A if and only if e-e :S a :S eI/e .

• If 1 < a < eIle , the equation X = aX has two roots 0,(3 (0 < (3, say) and A equals the smaller root o.

• If a = eIle , the equation X = aX has a unique root 0 and A = 0, while if a = 1, trivially A = 1.

• If eIle < a, (n diverges to 00.

• If 0 < a < 1, the (i satisfy

o < (1 < (3 < (5 < ... < (6 < (4 < (2 < 1.

Writing A = lim (2n-I, B = lim (2n, we have that if e-e :S a:S 1, A = B = A. If 0 < a < e-e, then A < Band (n oscillates with A, B as two limit points.

y f* : x = aY

G :x=y

f: y = aX

f* : x = aY

f: y = aX

--~---------------------------+~~-=----~ X aa

Figure 7.14 (1, 0)

Page 221: Surprises and Counterexamples in Real Function Theory

212 Chapter 7

First, let 1 < a:S: e1/e. We have (n/(n-1 = a(n-1/a(n-2 = a(n-1-(n-2).

Hence (n > (n-l if and only if (n-1 > (n-2. But (2 = aa > a = (1, so by induction (n > (n-1 for all n, i.e. the sequence {(n} is increasing.

Next we show that the sequence {(n} is bounded above bye. First notice that, since a :s: e1/e < e, so aa :s: (e1/e)a < (e 1/e)e = e (since a < e). Now use induction. If (n-l < e, then (n = a(n-1 :s: (e 1/e)(n-1 < (e 1/e)e = e (by induction hypothesis). It follows that (n tends to a limit, A say, which is less than or equal to e. Letting n -+ (Xl in the defining equation (n = a(n-1, we get A = aA , where A :s: e, i.e.,

A is a root of the equation x = aX and A :s: e.

To get this root, we draw the graphs r : y = aX and G : y = x. In the range 1 < a :s: e1/e, the graphs rand G, for a = 1, a = e1/e and a typical a are shown in Figure 7.15.

y

e 1----------;;(

o a. 2 e 3 4 ~

Figure 7.15

Page 222: Surprises and Counterexamples in Real Function Theory

x· 7.2 The infinite exponential XX 213

For a i- el/e , the equation X = aX has two roots 0:,(3 (0: < (3, say) and indeed 0: < e < (3. Also, since A ::; e, it follows that A = 0:, the smaller of the two roots.

Next, let el/e < a. We have seen that in this case, the two curves rand G have no real intersection (see case 3 and Figure 7.8 of the graphical motivation). Suppose lim (n exists and equals A. Then

i.e., that A is a root of the equation X = aX, showing that r n G i- 0. Let now 0 < a < 1. We show, by induction, that

We need one pertinent inequality throughout, viz., if 0 < a < 1 (as is the case) and if 0 < p < q, then

The proof of this is immediate from the graph r of y = aX (0 < a < 1), which has the shape shown in Figure 7.16.

y

G: y=x

(1 , a)

r: y = aX

x o p a q

Figure 7.16

Since, as x increases, r decreases (Le., comes down onto the x-axis), it follows that aq < aP and both these numbers are less than 1 (even if q is to the right of 1).

Page 223: Surprises and Counterexamples in Real Function Theory

214 Chapter 7

Now using this inequality repeatedly, we have

o < (1 = a 1 < aa = (2 < 1 ,

(3 = a(aa) < aa = (2 ,

(1 = a 1 < a(aa) = (3 ,

( aU)) (2 = aa > a a = (4 ,

(since a < aa)

(since aa < 1)

since a < a(aU) and since (3 < (2, so a(2 < a(3, i.e., (3 < (4. These inequalities

imply (1 < (3 < (4 < (2, which starts off our induction. Now the induction step. Suppose (induction hypothesis)

Then

i.e.,

(2r+1 = a(2r < a(2r-l = (2r ,

(2r-1 = a(2r-2 < a(2r = (2r+1 ,

(2r+2 = a(2r+l < a(2r-l = (2r ,

(2r+l = a(2r < a(2r+l = (Zr+2 ,

which gives the induction step.

(since (2r-l < (2 r)

(since (2r < (2r-2)

(since (2r-1 < (2r+d

(since (2r+1 < (2r)

Finally, to see that 0 < (n < 1 for all n, we proceed as follows: Since 0 < a < I , so 0 < aa < I, i.e., 0 < (2 < 1. If 0 < (n-1 < I, then

o < a(n-l < I, i.e., 0 < (n < I, so by induction, the result follows . Thus (2n - 1 increases while (2n decreases and both are bounded by 0 and

1. Hence they tend to limits, say (2n-1 tends to A, (2n tends to B, where

Now, in the relation (n = a(n-l, let n tend to 00, first through odd values and then through even values and we get A = aB , B = aA , i.e., (A, B) is a point common to the two curves r : y = aX and r * : x = aY • But we have seen that if e-e :S a :S I, then r n r * is a single point P, which belongs to G : y = x. It follows that A = B = lim (n exists (in case e-e :S a :S 1) and is the unique solution of the equation x = aX (i.e., the unique intersection of r, r * and G), as required.

Let 0 < a < e-e. In this case r n r * consists of three distinct points R = (0:,,8), P = (" 'Y), Q = ((3,0:), (0: < (3). Since (A , B) Ern r *, we see that if A:j:. B, then A = 0:, B = (3 and although {en} has two limit points, it has no limit. It remains therefore to show that A :j:. B.

Page 224: Surprises and Counterexamples in Real Function Theory

x· 7.2 The infinite exponential XX 215

The equations A = aB , B = aA (established above) imply A = aB = a(a A)

and B = aA = a(a B), i.e., that A, B are roots of the equation X = a(a'). Let

(1)

If A is a root of ¢, i.e., A = a(a A), then log A=aA log a or log(l/ A) = aA log(l/a)

and so loglog(l/ A) = Aloga + loglog(l/a). Let

<f;(x) = loglog(l/a)+x log a-log log(l/x) . (2)

Then it is easy to check that ¢(x) = 0 if and only if <f;(x) = 0, and indeed

> > ¢(x) is o ¢} <f;(x) is o (3)

< <

(for example, ¢(xo) > 0 ¢} Xo > a(a'O) ¢} logxo > aXO loga ¢} log(l/xo) < _aXO • log a ¢} log log(l/ xo) < Xo log a+log( -log a) ¢} log log(l/ xo) < Xo log a+ loglog(l/a) ¢} <f;(xo) > 0). Also

<f;'(.r) = loga - (1/ log(l/x)).(l/(l/x)).( -1/x2 ) = loga -l/(xlogx). (4)

So 'lj/(x) = 0 when log XX = ljloga, i.e., when XX = el/loga.

\Ve now consider the equation XX = C (0 < c < 1) and record:

Lemma 7.2.2. The equation XX = C (0 < c < 1) has no real root if 0 < c < l/el/e; one real root if c = l/el/e; two real roots if l/el/e < c < 1.

Proof. Consider the graph T of y = XX (sketch shown in Figure 7.17). Taking logarithms and differentiating gives (l/y).dy/dx = 1 + logx; so there is an extreme at X = l/e and indeed it is a minimum, equal to e-l/e. As x increases from 0 to 1, y = XX decreases from 1 to the value l/el/e at x = l/e and then increases from l/el/e to 1.

To continue with the analytic proof, as <f;' (x) = 0 when XX = e1/ log a (= c, say), we get that <f;' (x) = 0 has no root, one root and two roots according as 0< el/loga < e-1/e, el/loga = e-1/e and e-1/e < el/loga < 1 respectively, i.e., according as a > e-e, a = e-e and a < e-e, respectively (check that l/loga is less than, equal to or greater than -1/ e ¢} log a is less than, equal to or greater than -e ¢} a is less than, equal to or greater than e-e respectively).

Page 225: Surprises and Counterexamples in Real Function Theory

216 Chapter 7

y

--------~~--~--~--~L---~-L------_.x

o ~ 1/e 1

Figure 7.17

Now consider the graph of y = 'l/J(x) and note that as x -+ 0, 'l/J(x) -+ -00

and as x -+ 1, 'l/J(x) -+ 00. It follows that 'l/J(x) crosses the x-axis an odd number of times (see Figures 7.18-7.21), i.e., that 'l/J(x) = 0 has an odd number of roots. We now split two cases. Case (i): e-e ::; a < 1. Then by above 'l/J'(x) = 0 has either no root or one root and so only the two possibilities (i), (i)' in the diagrams (Figures 7.18-7.21) hold, since in the other possibilities, the equation 'l/J' (x) = 0 has 2,4, . .. roots respectively.

y y

x -r--~--~---+~ X

Figure 7.18 Figure 7.19

Page 226: Surprises and Counterexamples in Real Function Theory

X· 7.2 The infinite exponential xX

y

(ii)

Figure 7.20

217

y

(iii)

Figure 7.21

Thus 'lj!(x) = 0 has only one root and hence ¢(x) = 0 also has only one root (see (3)). Since A, B are roots of ¢(x) = 0, it follows that A = B; so in this case the sequence (n converge to A = B. Case (ii): 0 < a < e-e. We have seen in this case that the equation XX

el/loga (= c) has two roots Xl,X2, say, in the interval (0,1), i.e.,

(5)

(for we must check that c = el/loga > l/e l / e , i.e., that l/loga > -l/e, i.e., that log a < -e, i.e., that a < e-e, which is true in this case).

We have also seen in Lemma 7.2.2 that these two roots lie on different sides of l/e, i.e.,

Xl < l/e < X2 . (6)

Then X < Xl implies that XX > X~l = ell log a => 'lj!' (x) > 0 (see Figure 7.22).

Y Y \jI '(x) > 0

\ /J r-....

x 0 x Xl lie X 2 1

Figure 7.22 Figure 7.23

Page 227: Surprises and Counterexamples in Real Function Theory

218 Chapter 7

To check the above implications; the first one follows by looking at the graph of the function y = XX in [0, 1], since x < Xl. To see the second implication, we have 'ljJ'(x) > 0 {:} log a > l/log(xX) (see (4)) {:} l/loga < 10g(xX) {:} xx> ell log a, as required.

Similarly, X > Xl :::} XX < el/loga :::} 'ljJ'(X) < O. So (see Figure 7.23)

X = Xl is a maximum of'ljJ(x) €3 similarly X = X2 is a minimum of'ljJ(x). (7)

In particular,

(8)

So now, by (2), we have 'ljJ(xd = 10glog(1/a)+xl loga-loglog(1/xd. However, by (8), 'ljJ'(xd = 0 and so by (4), loga = (l/xd log Xl, so xlloga = 1/ 10gxI = -l/log(l/xd. Therefore, we have

So

10glog(1/a) = 10gl-log(-Xllogxd

= 10g(1/( -xllogxd)

= 10g(1/xllog(1/xd)

= 10g((1/xd/ 10g(1/xd)

= 10g(1/xd -loglog(l/xd.

'ljJ(xd = (log(l/xd -loglog(l/xd) - l/(log(l/xd) -loglog(l/xd

=r-1/r-2logr=A(r) , say,

where

r = 10g(1/xd > loge = 1 (since Xl < l/e).

(9)

(10)

Then A'(r) = 1 + 1/r2 - 2/r = (r - 1)2 /r2 ~ 0 and in fact it is greater than 0, since r > 1, by (10), i.e., A'(r) > 0, showing that A(r) is increasing and so A(r) > A(l) = 0, or by (9) we have that

'ljJ(xd > O. (11)

Further, by (2)

'ljJ(l/e) = 10glog(1/a) + (l/e) log a = 10glog(1/a) - (l/e) 10g(1/a). (12)

But

(log z) / z < 1/ e , if z > e (13)

(check: We have to show that h(z) = z - elogz > O. We have dh/dz = 1- e/z > 0 if z > e, and so hex) is increasing and h(e) = 0, so h(z) > 0).

Page 228: Surprises and Counterexamples in Real Function Theory

X· 7.2 The infinite exponential xX

y

Y = \!I(x)

maXImum

-----r--+-~--r_-.--.-----~+__+-------.x o lie X2

minimum

Figure 7.24

219

Taking z = 10g(1/a), this shows that (loglog(l/a))/log(l/a) < lie (for, z = 10g(1/a) is greater than e since 1/a > ee as a < e-e), i.e., 10glog(1/a) < (lie) 10g(1/a) (since 10g(1/a) > 0 as a < 1), and so, by (12),

1j;(l/e) < 0 . (14)

It follows that the graph of 1j;(x) = 0 is as shown in Figure 7.24 (see (6), (7) and (11)), showing that 1j; (x) = 0 has three roots in (0, 1); call them (31, (32, /3s, where

0<(31 < Xl < (32 < lie < X2 < (33 < l.

Since, by (3), ¢(x) is larger than, equal to or less than 0 if and only if 1j;(x) is larger than, equal to or less than 0, respectively, it follows that ¢(x) has the same three roots (31, (32, (33. Remembering again that 0 < a < e-e, we now show that B = (33, A = (31. Indeed, we shall show, by induction, that (2n > lie for

all n. We have (0 = 1 > lie. If (2n-2 > lie, then (2n = aaC2n- 2 > aa1

/e > lie

(this last inequality, since 1j;(l/e) < 0 by (14) and so ¢(1/e) < 0). Thus, (2n > lie and so lim(2n = B 2': lie. But B is a zero of ¢ and the only zero of ¢ larger than lie is (33· So B = (33.

Next we show, again by induction, that (2n+l < Xl for all n. The induction

step is simple. Suppose (2n-l < Xl, then (2n+l = aaC2n - 1 < aa~l < Xl (the last inequality comes from (11) because 1j;(xt) > 0 and so ¢(xt) > 0), completing the induction step.

To start the induction, it remains to show that a < Xl. Now z 2': elogz for all z > 0 (check: It is enough to prove that 9 (z) = z - e log z :s: o. Now

Page 229: Surprises and Counterexamples in Real Function Theory

220 Chapter 7

g'(z) = I - e/z = 0 if z = e and g"(z) = e/z2 > 0 at z = e, and so z = e is a minimum and g(e) = 0, so g(z):S 0). Hence l/yiXl ~ elogl/yiXl > 2logl/yiXl = logl/Xl, i.e., l/Xl > (logl/xd 2 , i.e., l/xllog(l/xd > logl/xl or 1/(-Xllogl/xr) > -lOgXl. But loga = l/(xdogxr) < lOgXl (taking logarithms in (5)), by above, we get that a < Xl, as required.

So now, (2n+l < Xl for all nand (2n+l is increasing and tends to A, so A :S xl. But A is a zero of ¢ and the only zero of ¢ less than or equal to Xl is (31. Hence A = (31. Thus A -I- B and so (n oscillates with A, B as limits.

This at last completes the analytic proof of Theorem 7.2.1.

§7.3. Applications and examples

In the last section we have given graphical and analytic proofs of the con­vergence or otherwise of the infinite exponential of the title of this chapter. Our object now is to establish the connection of this infinite exponential with the function f(x, y) = x y - yX in the range of convergence of the exponential. We have the following striking result:

Theorem 7.3.1. In its interval of convergence, i.e., for z E [e-e,e l/e ] , the functions

and z = g(x) = xlix

are partial inverses of each other. More specifically

(i) g(h(z)) = z, for all z E [ee, el/e ] (= the full range of convergence of h(z)),

(ii) h(g(x))=x, for all X E [l/e, e].

Proof. Since exponentiation is continuous (and increasing), it follows that

X = h(z) =} ZX = zh(z) =} ZX = z(Zz· ) = h(z) = X =} z = xlix = g(x).

Hence, we have

z· (i) g(h(z)) = g(ZZ ) = g(x) = z; and

(ii) h(g(x)) = h(Xl/x) = h(z) = x.

Page 230: Surprises and Counterexamples in Real Function Theory

7.3 Applications and examples

x

e

I1e

~~------------------~----~---+ z . e

e Figure 7.25

e lie

221

We now determine the range where these functions are meaningful. We take them up by turn.

In (i), h(z) is meaningful only in the range of its convergence, i.e., when Z E [e-e, el/e ], as required. Note that 9 is defined for all x > o.

For (ii) to hold, g(x) must lie in the range of convergence of h, i.e.,

But this is the same thing as saying that lie::; x ::; e, since at x = lie, we have xlix = e-e and xlix increases from x = lie right up to x = e (where el/e is the maximum of xlix), so llee ::; Xl/x. Similarly, at x = e, we have xl/x = el/e . For a clear understanding of this, we sketch, in Figure 7.25, the graph of the function

x = h(z) = ZZ z·

Page 231: Surprises and Counterexamples in Real Function Theory

222 Chapter 7

ExaIllple 7.3.2. Evaluate

Indeed, for x > 0, let

fn(x)= (1+XV(1+(X+1)Vh+ ... +(X+n-1h/(1+(X+n))))). (1)

Clearly, fn(x) increases as n increases, but

x + 1 = J(l + x(x + 2))

= V(1 + xJ(l + (x + l)(x + 3)))

== ... ... . ..

= (1 + xV((l + (x + l)J(l + ... + (x:- n - l)(x + n + 1)))))

> fn(x) ,

since J(l + (x + n)) < x + n + 1, i.e., fn(x) is bounded above by x + 1 and so tends to a limit, say f(x). Thus f(x) :s x + 1. Also

f(X»J(l+XV(1+XV(l+X~ =R ,say.

For this, compare fn(x) with the right hand side; i.e., fn(x) > R, for all nand so f(x) ~ fn(x) > R. Then

R2 = 1 + XJ(l + XV(l + xV(l + x~ = 1 + xR,

giving R = (x ± J(x2 + 4))/2 and since R > 0, the + sign holds, so R = (x + J(x2 + 4))/2> x. Thus

x < f(x) :s x + 1

Let x + 1 - f(x) = cp(x), so that (2) becomes

° :s cp(x) < 1 .

Now squaring (1), we get

(2)

(2)'

f~(x) = 1 + XV(l + (x + l)V(1 + ... + (x + n - l)J(l + (x + n))))

= 1 + Xfn-l (x + 1)

Page 232: Surprises and Counterexamples in Real Function Theory

7.3 Applications and examples

and letting n ---+ 00, this gives P(x) = 1 + xf(x + 1), i.e.,

Hence,

(1 + f(x))(l - f(x)) + xf(x + 1) = 0

or (1 + f(x))(<p(x) - x) + x(x + 2 - <p(x + 1)) = 0

or (1 + f(x))<p(x) + x(x + 2) - x(l + f(x)) = x<p(x + 1)

or (1 + f(x))<p(x) + x((x + 1 - f(x))) = x<p(x + 1)

or (1 + x + f(x))<p(x) = x<p(x + 1)

or <p(x)jx = <p(x + l)j(x + 1 + f(x)) :S <p(x + l)j(x + 1).

O:S <p(x)jx :S <p(x + l)j(x + 1) :S ... :S <p(x + n)j(x + n) < 1j(x + n)

223

(this last inequality by (2)') and this tends to 0 as n ---+ 00, i.e., <p(x) = 0, giving f(x) = x + 1.

So now, the given expression of the example is equal to f(2) = 2 + 1 = 3.

Example 7.3.3. For the equation yX = x Y ,

(i) determine those positive values of x, for which the equation has only trivial solution y = x,

(ii) for those x > 0 for which nontrivial positive solutions y exist, determine all such solutions,

(iii) if x is selected from the range (0, e), determine the probability that a nontrivial solution for y, satisfying yX = xY , lies in the same interval.

Indeed, since xY = exp(ylogx), it is enough to compare ylogx and xlogy. Further, the function exp is increasing, so

> > x Y is yX {:} (logx)jx is (logy)jy;

< <

thus it is enough to consider the function g(x) = (logx)jx, defined for x > O. Now gl(X) = (1 - logx)jx2 , gl/(x) = (-3 + 2logx)jx3 . Therefore g is

negative for 0 < x < 1, increasing for 0 < x :S e, attains a maximum of 1 j e at x = e and is decreasing for x 2: e. Further, g(l) = 0 and g(x) tends to the x-axis asymptotically as x ---+ 00. Hence

(i) the equation yX = xY has only the trivial solution y = x, only when either x = e or 0 < x :S 1.

(ii) If 1 < x, x i- e, there exists exactly one nontrivial solution of yX = xY .

Let y be this solution. Then, if 1 < x < e, we have y > e, while if x > e, then 1 < y < e.

Page 233: Surprises and Counterexamples in Real Function Theory

224 Chapter 7

(iii) The probability is 0, because nontrivial solutions of yX = x Y exist, in this case, only if 1 < x < e and then the corresponding y is greater than e.

We shall now discuss the convergence or otherwise of the infinite continued radical

V(a+ {j(a+ ~ where r stands for the rth root and r E N, a E IE.+ , the set of positive real numbers . Writing UI = {la, U2 = {/(a + ud, ... , Un = {/(a + un-d, ... , we have the following:

Lemma 7.3.4. The sequence {un} is convergent.

Proof. u~+l = a + Un, U~ = a + Un-I =? u~+l - U~ = Un - Un-I. Hence, if Un > Un - I then u~+l > u~ or un+l > Un· But U2 > UI (for it is enough to prove that U2 > ul, i.e., that a + UI > a, which is true since UI = (jIi > 0 as a> 0). So, by induction, Un is an increasing sequence.

y y ~ f(x)

---=o+-:-) .ra-----+ x

Figure 7.26

Now we prove that the equation f(X) = xr - X - a = ° has a unique positive root; call it a, for, its derivative is r xr-l - 1 which is greater than ° (i.e., X r- I > l/r) if X ;::: 1, so xr - X - a increases after X ;::: 1. At X = 1, xr - X - a = IT - 1 - a = -a < 0, so xr - X - a vanishes just once after X = 1, say at X = a. Further, in [0,1), i.e., if X E [0,1), xr < X, i.e., xr - X < O. So X r - X - a < O.

Thus, for all X E [0,1), f(X) < O. It follows that f(X) = 0, only at X = a in the range [0, (0). Now u~ < u~'+l = a + Un (since {un} is increasing) , i.e., u~ - Un - a < 0 so Un < a, i.e., {un} is bounded above by a. Hence, {un} is a convergent sequence, which converges to u, say. Letting n -+ 00 in u~ = a+un, we get ur = a + u.

From the above it follows that U is the positive real root a of xr -X -a = 0, i.e. , we have proved:

Theorem 7.3.5. Let r ;::: 2 be an integer and let a E IE.+. Then the expression

is meaningful, i. e., convergent, with limit a, which is the unique positive real root of the equation xr - X - a = O.

Page 234: Surprises and Counterexamples in Real Function Theory

7.3 Applications and examples 225

Example 7.3.6. Taking r = 2 in the above, we get

where 0:2 - 0: - a = 0, 0: > 0, i.e., 0: = (1 + )(1 + 4a))/2. Taking a = 6, for example, we get

(1 + )(1 + 24))/2 = 3 .

In general, if a = k(k - 1),

Taking r = 2, a = 1, we get

the golden ratio, which incidentally, is also equal to the continued fraction

1 1+ 1

1+ 1 1+~

Finally, taking r = a = 2, we get

There is a similar infinite expression involving logarithms and fits in the above pattern ideally. The interested reader may look up [40].

Page 235: Surprises and Counterexamples in Real Function Theory

226

Appendix I

§A.1. Stirling's formula and the trapezoidal rule

The object of Stirling's formula is to find an asymptotic value for n!. We have:

Theorem A.I.1 (Stirling's formula). n! is asymptotically equal to

Vn.n n .J27r/en .

Proof. Consider the curve y = logx (see Figure A.I.1). The gradient dy/dx = l/x, which is greater than 0 and decreasing as x increases; so the curve is convex downwards.

G

y = log x c

B

N

o A(a,O) O(a+l.O) A=(a,O) O=(a+l,O)

Figure A.l.l Figure A.1.2

Hence, area of the trapezium ABCD is less than J:+1logtdt, i.e., & (log a +

log(a + 1)) < J:+1logtdt. Summing from a = 1 to a = N - 1 gives

~((logl +log2) + (log 2 +log3) + ... + (log(N -1) +logN)) < IN logtdt. 2 1

Here the left hand side is equal to log N! - ~ log N, while the right hand side is equal to N log N - N + 1. Hence

CN = (N +~) logN-N+1-logN! > O. (1)

Page 236: Surprises and Counterexamples in Real Function Theory

A.l Stirling's formula and the trapezoidal rule 227

However, CN is equal to the difference of area under y = log x from 1 to Nand the trapezoidal approximation to this area. Hence

C N increases as N increases . (2)

We now show that CN is bounded above; indeed that

CN < 1/4 . (3)

For this we look at the Figure A.1.2. Let the tangents to y = log x at B, C meet DC, AB in G, E respectively. The coordinates of G are (0: + 1, (log 0:) + 1/0:) and the coordinates of E are (0:, log (0: + 1) - 1/(0: + 1)). This is because B = (0:, log 0:), the equation of the tangent at B is y = x / 0: + (log 0: - 1), the slope of this tangent is (d y / d x) B = 1/0: and this tangent passes through B. This gives G.

Similarly, since C = (0: + 1, log(o: + 1)), the equation of the tangent at Cis y = x/(o: + 1) + log(o: + 1) - 1, which then gives E. So now

. 1 Area of trapezmm ABGD = "2 (log 0: + (1/0: + log 0:))

and

. 1 Area of trapezmm AECD = "2 (log(o: + 1) + log(o: + 1) - 1/(0: + 1)).

Since the area of each trapezium is greater than the area under y = log x, it follows that the average area of these trapezia is greater than the area under the curve, i.e.

1 (1 ) 1 ( ) 1"'+1 "2 "2(2 logo: + 1/0: +"2 2log(0: + 1) - 1/(0: + 1) > '" logtdt.

Again summing this from 0: = 1 to 0: = N - 1, we get (see (3))

i N 1 1 1 logtdt < logN! - -logN + - - -

1 2 4 4N'

which gives CN < 1/4 - 1/4N < 1/4. Also (2) and (3) imply that CN -t C as N -t 00, where 0 < C :S 1/4. But now CN = 10g(NN VN) - log eN + 10ge-logN! = 10g(NN VN.e-N.e/N!). Since CN -t C, we get eCN -t eC, i.e., NNVN/NLeN -t eC- 1 , or

lim NLeN /NN VN exists and equals e1- C . N--+oo

(4)

The existence of this limit is the essential part of Stirling's formula. We have, however, still to find the value of the limit. This is done by using the expression for 7r in Corollary 1.3.6 as follows: Let the left side of (4) be denoted by f n

Page 237: Surprises and Counterexamples in Real Function Theory

228 Appendix - I

(replacing N by n), so that n! = ynnne-n fn. Then the left side of Corollary 1.3.6 equals

f~ 2J]n«2n + l)/n) ,

(on cancellation) and letting n ~ 00, we see that this tends to e4(1-C) /4e2(1-C)

= e2 (1 - C) /4. This , coupled with Corollary 1.3.6, gives e2(1-C) /4 = 1f /2, giving e1- C = V(21f), which completes the proof of Stirling's formula.

The above proof is due to A.J.Coleman [27].

Theorem A.1.2 (The trapezoidal rule). Let f, f' be continuous in [a, b] and suppose f" exists and is continuous in (a , b) i then, there exists a ~ E (a, b) , such that

E = lb f(x)dx - (b - a)(f(a) + f(b))/2 = - f"(~)(b - a)3/12 (*)

Proof. Let y = ¢(x) be the equation of the line joining the point A with B , so that (see Figure A.1.3):

B

A

a Figure A.I.3 b

E = lx f(t) dt -lx ¢(t) dt .

Put g(x) = J: f(t) dt- J: ¢(t) dt+ (2«x-a)/(b-a))3 -3«x-a)/(b-a))2)E. Then g(a) = g(b) = 0 and so by Rolle's theorem, there exists u, a < U < b, with g'(u) = O.

Now, g'(x) = f(x) -¢(x) + (6.«x -a)/(b-a))2 .(l/(b- a)) -6.«x - a)/(b­a)).(l/(b - a))).E, and so g'(a) = g'(b) = 0 and so again by Rolle's theorem, there exist points c, d such that a < c < u < d < b, and with gil (c) = g"(d) = 0, and we conclude, once more, by Rolle's theorem, that there exists a point C c < ~ < d, with g"'(~) = 0, i.e.,

0= g"'(O = f"(~) + 12.E/(b - a)3 ,

Page 238: Surprises and Counterexamples in Real Function Theory

A.l Stirling's formula and the trapezoidal rule 229

since ¢"(x) == 0, ¢ being linear; and that proves the theorem. As a corollary to the trapezoidal rule we prove the existence part of the

limit in Stirling's formula:

Corollary A.1.3. limn-Hx) Kn = limn--+oo nnvn/n!en exists (and equals K, say).

Proof ([50]). Let f(t) = logt, a = k - 1, b = k in the trapezoidal rule, to obtain (see Figure A.1.4)

rk logtdt-(log(k-1)+logk)/2 = (-1/12).f"(~k) = 1/12~~ k-1 < ~k < k.

lk-l Summing this from k = 2 to k = n gives

/,n 1

1 log t dt - 2 ((log 1 + log 2) + (log 2 + log 3) + ... + (log(n - 1) + logn))

1 = (nlogn - n + 1) - (log 2 + log3 + ... + logn) + 210gn

= (lognn -logen + 1) -logn! + log v'ri

= (lognnv'ri/n!en) + 1

= 10gKn + 1.

o 2 ....... k-l k ....

Figure A.IA

It follows that 10gKn = L~=2 1/12~~ -1. But now, since k-1 < ~k < k, we have l/~k < 1/(k-1), and hence L~=2 1/~~ < L~=2 1/(k-1)2 = L%':11/k2 = 1[2/6, i.e., log Kn is bounded above. Moreover, log Kn increases as n increases, because positive terms get added to it, when n increases. It follows that log Kn tends to a limit and therefore so does K n , as required.

The value of this limit can not be determined by this method; however, we

Page 239: Surprises and Counterexamples in Real Function Theory

230 Appendix - I

can give an estimate for it as follows: We have

f 1/(12~D < (1/12) f 1/(k - 1)2 < (1/12) IX) (l/x2) dx= 1/(12(n - 1)) k=n+1 k=n+1 12-1

(here the inequality between summation and integral follows from Figure A.l.5, where the shaded area is less than the area under the curve y = l/x2 from n-l to 00).

Similarly, since 1/~k > l/k, so 1/~~ > l/k2, we have

f 1/(12~~) > (1/12) f l/k2 > (1/12)100

(l/x2)dx=I/(12(n+l)) k=n+ 1 k=n+1 n + L

(again the inequality between the summation and integral follows from Figure A.l.6 where the shaded area is greater than the area under the curve y = 1/ x 2

from n + 1 to 00).

n-J n n + J n+l n+2 n +3

Figure A.l.5 Figure A.1.6

Thus 00

1/12(n + 1) < L 1/12~~ < 1/12(n - 1) , k=n+1

as required. Taking n = 2, this gives

00

1/36 < L 1/12~~ < 1/12 . 1.,=3

Adding 1/12~~ - 1 throughout, gives

1/12~~ + 1/36 - 1 < log K < 1/12~~ + 1/12 - l.

Putting in the estimate 1 < 6 < 2, i.e., 1/4 < 1/~~ < 1, this gives 1/48 + 1/36 - 1 < 10gK < 1/12 + 1/12 - 1, i.e., -137/144 < 10gK < -5/6, which

Page 240: Surprises and Counterexamples in Real Function Theory

A.l Stirling's formula and the trapezoidal rule 231

gives the following

Corollary A.1.4. e-137/144 < K < e-5 / 6 , where K = limn-+oo nn"fii/n!en

(which we know, from Stirling's formula, to be equal to 1/ J (27r)).

We now estimate the error in trapezoidal rule for a concave curve.

(4, log 4)

2 3 4

Figure A. 1.7

For example, consider the area between y = log x and the polygonal ap­proximation by vertices (1, log 1), (2, log 2), ... , (see the shaded area in Figure A.1. 7). This region is unbounded, yet the area is finite. This finiteness of the shaded area is a corollary of the following (see [98])

Theorem A.1.5. Let f be defined for x:::: 1 and suppose f(x) :::: 0, f(x) :::: 0, 1" (x) ::; 0, for all x :::: 1 (which is the case for the function y = log x, which has been drawn in Figure A.1.7). For n E N, let

n n-l

Tn = r f(t) dt - 2:)the area of the relevant trapezia) , il 1

then Tn::; (f(2) - f(1))/2.

Proof. Let i-I, i, i + 1 be 3 consecutive integers and consider Figure A.1.8, which is the key to the proof. The line t is tangent to the graph of f at the point B = (i, f(i)). The area of the shaded region represents the errors of adjacent summands in the trapezoidal approximation.

Now translate the shaded region R between Band C, by one unit to the left, to become the region R' and note that no point of R' lies below t. Thus R' and the shaded region S between A and B are disjoint, except for B. The two shaded regions between i-I and i, (viz. Sand R') can now be similarly

Page 241: Surprises and Counterexamples in Real Function Theory

232 Appendix - I

shifted to the left by one unit (between i - 2 and i-I) and so on till everything between 1 and n is fitted between 1 and 2.

i-2

R' C1,==' m::~~~~R~= C

i-I

A, B, C, C' are points, R, R', S are regions (shaded), t is the tangent at B.

Figure A.1.8

i+1

Since the curve is increasing (because l' :::: 0), all the shaded regions be­tween 1 and n are fitted in the triangle 6, with vertices (1, f(l)), (1, f(2)), (2, f(2)) (see the figure for n = 4, i.e., Figure A.1.9). It follows that Tn::; Area of 6 = (1(2) - !(1))/2, which completes the proof.

Letting n -+ 00, we get

Corollary A.1.6. L:~=1 U;:+l f(x) dx - (1(n) + f(n + 1))/2) ::; (1(2) -f(1))/2.

Proof. J;:+1 f(x) dx - (1(n) + f(n + 1))/2 is the trapezoidal error between In, n + 1] and the corollary follows.

o 2 3 4

Figure A. 1.9

Page 242: Surprises and Counterexamples in Real Function Theory

A.l Stirling's formula and the trapezoidal rule 233

This shows that the infinite series L Ri (Ri are the shaded regions in the diagram) is convergent, with sum less than or equal to (f(2) - J(1))/2.

Using the proof of Theorem A.I.1 and the trapezoidal rule (Theorem A.I.2), we prove the following interesting

Corollary A.1.7. The shaded region for the Junction J(x) = logx, between 1 and 00 (see Figure A.l.4) has area equal to 1 -log J(27r).

Proof. This shaded region has area precisely equal to (see proof of Corollary A.I.3)

jn 1 lim ( logtdt--((logl+log2)+(log2+log3) + .. '+(log(n -l)+logn)))

n-7CXJ 1 2

= lim (log Kn + 1) (as proved in Corollary A.I.3) n--+oo

as required.

= logK + 1

= log(1/J(27r)) + 1 (by Stirling's formula)

= 1 -log J(27r) ,

To provide yet another example of the trapezoidal rule, we derive the existence of Euler's constant ,,(, in the following:

Corollary A.1.S. "( = limn--+oo (1 + 1/2 + ... + l/n -logn) exists and satisfies

2 + ((3) < 6"( < 3 + ((3) ,

where, ((3) = 1 + 1/23 + 1/33 + ....

Proof. Take J(t) = l/t in Theorem A.I.2, so that f'(t) = -1/t2 , f"(t) = 2/t3 .

The trapezoidal rule now gives

rk dt _ (l/(k _ 1) + 1/k)/2 = -(1/12)'!"(~k) , lk-l t

where k - 1 < ~k < k. We sum this equation from k = 2 to k = n, to get

/,n dt 1 n

- - -((1 + 1/2) + (1/2 + 1/3) + ... + (l/(n - 1) + l/n)) = L -1/6~~ , 1 t 2 k=2

which gives, on simplification,

1 n

1 + 1/2 + ... + l/n -logn = 2(1 + l/n) + (1/6). L~~ . k=2

Letting n ---t 00, this yields

00

1 + 1/2 + ... + l/n -logn -+ 1/2 + (1/6) L~~ . k=2

Page 243: Surprises and Counterexamples in Real Function Theory

234 Appendix - I

Now use the estimate k - 1 < t,k < k to get 1/k3 < l/t,Z < l/(k - 1)3; hence the above gives

00

((3) - 1 < L t,~ < ((3) . k=2

Thus, we have proved that "I = limn-+oo(l + 1/2 + ... + l/n -logn) exists and satisfies 1/2 + ((3)/6 - 1/6 < "I < 1/2 + ((3)/6, i.e., 2 + ((3) < 6"1 < 3 + ((3), where, ((3) = 1 + 1/23 + 1/33 + ... , as required.

§A.2. Schwarz differentiability

This is a rather specialized topic, one that offers excellent problems for non-routine examinations. Throughout our discussion, we suppose f to be a real-valued function defined on an open interval I of which [a, b] is a closed subinterval (this is to avoid left hand limits at b and right hand limits at a).

Schwarz differentiability, also known as symmetric differentiability (s-diffe­rentiability; similarly s-differentiable, s-derivative for Schwarz derivative, etc) is defined as follows:

Definition A.2.1. For each x E [a, b] C I, if limh-+o{ (f(x+h) - f(x- h))/2h} exists and is finite, we say f is s-differentiable in [a, b] and the limit is called the s-derivative (i.e., the symmetric derivative or the Schwarz derivative) of f at x and is denoted by f(s)(x).

The following are some easy consequences of this definition: (1) If f'(x) (the usual derivative) exists at x, so does f(s)(x) and f'(x) = f(s)(x). Proof. (f(x + h) - f(x - h))/2h = (f(x + h) - f(x))/2h + (f(x) - f(x - h))/2h and taking limits as h -+ 0, the result follows. (2). The converse to (1) is false; for example let f(x) = Ixl; then at the point x = 0, we have f(s)(O) = lim(f(O +h) - f(O - h))/2h = lim(f(O + h) - f(O­h))/2h = lim(h - h)/2h = 0 exists, but 1'(0) does not exist. (3) s-differentiability does not even imply continuity. Take, for example, the function

f(x) = {o ~f xi 0, 11fx=0

Then at the point x = 0, limh-+o(f(O + h) - f(O - h))/2h exists and equals 0, i.e., f(s) (0) = 0, but f is not continuous at x = O.

We see that the existence of f(s)(x) at the point x, is a much weaker require­ment than the existence of f'(x) at x. In view of this, all the basic elementary theorems regarding differentiabilty, such as Rolle's theorem, the first mean value theorem, etc. will have to be reformulated in terms of s-differentiability.

Thus, for example, we have (4) It is not true that f(x) is a constant when f(s) (x) = 0 in [a, b], for example,

Page 244: Surprises and Counterexamples in Real Function Theory

A.2 Schwarz differentiabilit.Y 235

let

f(x) = {o ~f x 1= 0 , 11fx=0.

Then f(sl(x) = 0 for all x but f is not a constant.

This failure is expected, since, we have (5) Rolle's theorem too, is false for s-derivatives. For example, the function f(x) = Ixl in the interval [-1,1].

We shall deal with this problem after proving some easier results and the reformulated Rolle's theorem and the mean value theorem. (6) It may happen that f( S l (a) exists and is finite but f (a) does not exist (is not defined); for example, let f(x) = 1/x2. Then f(sl(x) = 0 for all x, including x = 0, but f(O) is not defined.

Example A.2.2. For Dirichlet's function

f(X)={l ifxEQ, o otherwise,

f(S)(x) exists for all x E Q but not for x ~ Q. (7) Even if f is continuous at x = a and f(s)(a) exists, it does not follow that f'(x) exists; for example take f(x) = Ixl and the point x = O. (8) Even if f(s) (x) is continuous in a neighbourhood of a and f(x) is continuous at x = a, it does not follow that f'(a) exists. For example, take f(x) = l/n if x = ±l/n, (n EN), f(x) = 0 otherwise. It is easily verified that f(s)(x) = 0 for all x, so that f(s)(x) is continuous for all x. Further, f(x) is continuous at x = O. However, 1'(0) does not exist, because for h = ±l/n, (f(O+h) - f(O)/h) equals ±1, while for other values of h, this equals O/h = 0, so the limit of the quotient does not exist as h -+ O.

Theorem A.2.3. If f(s) is continuous at x = a and f is continuous in a neighbourhood of a, it does follow that 1'(a) exists.

The proof of this is rather more involved than one would imagine at first glance and uses results analogous to Rolle's theorem and the mean value the­orem, which we now develop.

Theorem A.2.4. Let f be continuous in [a, b], let f(s) exist in (a, b) and let f(b) > J(a). Then there exists a number C E (a,b) such that J(S) (c) 2 o. Proof. Let k be such that f(a) < k < J(b). The set E = {x I f(x) > k , a < x < b} is bounded below by a. Let c = inf E, then c 1= a since J (a) is not greater than or equal to k.

Page 245: Surprises and Counterexamples in Real Function Theory

236 Appendix - I

8 8 • • a b=c E

Figure A.I.IO

Also c #- b, for otherwise, since j(x) is continuous at x = c, and j(b) > k, it would follow that there exists a J > 0 such that j(x) > k for all x E (b-J, b+J) and so numbers less than c (e.g. c - J/2) would belong to E, contradicting the fact that c is the infimum of E. Furthermore, j(s)(x) 2:: 0, for, there exist points x > c in every neighbourhood of c such that j(x) > k while j(x) ::; k for x E [a, c], so for h > 0, we have

(f(c+ h) - j(c - h))/2h > (k -1)/2h > 0 where l::; k ,

giving limh-to[(f(c + h) - j(c - h))/2h] 2:: O. But since j(s)(c) exists, so j(s)(c) 2:: O.

A similar result holds with the inequality reversed, i.e.,

Theorem A.2.5. Let j be continuous in [a,b], let j(s)(x) exist in (a,b), and let j(b) < j(a). Then there exists a number c E (a,b) such that j(S) (c) ::; o. Theorem A.2.6 (Analogue of Rolle's theorem). Let j be continuo1ts in [a, b], let j(s) exists in (a, b) and let j(a) = j(b) = o. Then there exists an Xl E (a, b) such that j(s)(XI) 2:: 0 and an X2 E (a, b) such that j(.') (X2) ::; o. Proof. If j(x) == 0, the result is trivial. So let j be not identically zero; then there exists either a number c such that j(c) > 0 or a number d such that j(d) < 0 or both. By the above theorems, there exist Xl, X2, with either a < Xl < c < X2 < b or a < X2 < d < Xl < b such that j(s) (xd 2:: 0 and j(s)(X2) ::; O.

Theorem A.2.7 (Analogue of mean value theorem). Let j be continu­ous in [a,b] and let j(s) exist in (a,b). Then there exists points Xl,X2, with a < XI,X2 < b such that j(s)(X2) ::; (f(b) - j(a))/(b - a) ::; j(s)(XI).

Proof. This follows from Theorem A.2.6 in exactly the same way the mean value theorem for ordinary derivatives follows from Rolle's theorem (for ordi­nary derivative): just rotate the axes!

Theorem A.2.8. Let j and j(s) be continuous in (a,b). Then f'(x) exists and f'(x) = j(s)(x).

Proof. Take h sufficiently small to ensure a < x + h < b. Then by Theo­rem A.2.7, there exist Xl, X2 with X < Xl, Xz < X + h such that j(s) (X2) ::; (f(x + h) - j(x))/h ::; j(s)(XI). If strict inequality holds at either end, we apply the intermediate value theorem for continuous functions. So there exists X3 such that j(s)(X3) = (f(x + h) - j(x))/h, X < X3 < X + h. Letting h --t 0

Page 246: Surprises and Counterexamples in Real Function Theory

A.2 Schwarz differentiability 237

and noting that the left hand limit exists, we get f(s)(x) = f'(x).

We now give:

Proof of Theorem A.2.3. Given E > 0, there exist a neighbourhood Na of a such that f(x) is continuous in Na and, since f(s)(x) is continuous at a, so

(1)

for all x E N a. By A. 2.7, there exist Xl, X2, strictly between a and a + h, such that

here h -j. 0 is such that a + h E N a . Now (1) and (2) imply that

f(S) (a) - E < (f(a + h) - f(a))/h < f(S) (a) + E.

Letting h ---+ 0, it follows that f'(a) exists and equals f(s)(a).

(2)

(3)

\Ve are now ready to prove a result analogous to the result: if f'(x) = 0, then f(x) is a constant, in the Schwarz derivative set up. Indeed, we have the following:

Theorem A.2.9. Suppose f is defined and is continuous in an open interval I of which [a, b] is a closed subinterval and suppose f(s) exists in I. Then

f(s)(x) = 0 in I => f(x) is a constant in [a,b].

Proof. Since f(s)(x) = 0 in I, it is a continuous function and so by Theorem A.2.S, f'(x) exists in [a, b] and equals f(s)(x), i.e., equals O. Thus f'(x) = 0 in [a, b] giving f(x) = k.

Before we go on to uniform differentiability, we quickly introduce the second Schwarz derivative of f(x), written as f(ss)(x). It is defined to be the Schwarz derivative of f(s) (x). Thus

f(ss)(x) = lim [(f(S) (x + h) - f(s)(x - h))/2h] (provided the limit exists) h-+O

lim f(x+h+k)-f(x+h-k) _ lim f(x-h+k)-f(x-h-k) . k-+O 2k k-+O 2k

= hm 2h h-+O

But if the limit exists, as we suppose it does, then we may choose k = h; so that

f(SS)(x) = lim [(f(x + 2h) - f(x))/2h]- [(f(x) - f(x - 2h))/2h] h-+O 2h

= lim f(x + h) - 2f(x) + f(x - h) (writing h/2 for h) h/2-+0 h2

= lim f(x + h) - 2f(x) + f(x - h) (as h/2 ---+ 0, so does h) h-+O h2

Page 247: Surprises and Counterexamples in Real Function Theory

238 Appendix - I

Although, it looks difficult, the following result is easily proved:

Theorem A.2.10 (Schwarz, see [72]). Suppose f is continuous in I and suppose f(SS) (x) = 0 for all x E I; then f(x) is a linear function of x on any interval [a, b] C I.

Proof. Let E > 0 and let

[ ( f(b) - f(a)) ] ¢(x) = f(x) - f(a) + b _ a .(x - a) + E(X - a)(x - b) ,

so that ¢(x) is continuous in [a, b] and ¢(a) = ¢(b) = O. Furthermore, using that f(SS) (x) = 0 and the fact that if a function 9 is differentiable then g'(:£) = g(s)(x), it may be easily checked that

We shall show first that

¢(x) ~ 0 't/x E [a,b]. (1)

Suppose not; then ¢(x) assumes its maximum value ¢(xo) at an interior point Xo E (a, b) (¢ being continuous and so bounded and since ¢ attains its max­imum). Then (¢(xo + h) - 2¢(xo) + ¢(xo - h))/2h ~ 0 (since ¢(xo) is the maximum value). Letting h ---+ 0 gives ¢(SS) (xo) ~ 0, which contradicts (*).

Similarly the function

1j;(x) = - [f(X) - (f(a) + (f(b~ = ~(a)) .(x - a))] + E(X - a)(x - b) ,

satisfies

1j;(x) ~ 0 't/ x E [a, b] . (2)

Combining (1) and (2) gives

if(x) - (f(a) + (f(b~ = ~(a)) .(x - a)) i ~ EI(x - a)(x - b)1

and since E is arbitrary, it follows that the left hand side is 0, which gives

f(x) = f(a) + ((f(b) - f(a))/(b - a)).(x - a) ,

a linear function of x, as required.

Remark A.2.11. We know that l' exists implies that f(s) exists and f(s) = 1'. It may be shown that f" exists implies that f( S8) exists and f( Ss) = f", for f" exists => (f')' exists => (f(s))(s) exists and equals (f(s))' and hence f(ss) exists and equals (f')' = 1".

Page 248: Surprises and Counterexamples in Real Function Theory

A.2 Schwarz differentiability 239

Alternatively, let g(h) = f(x + h) + f(x - h), so that (by the Cauchy mean value theorem)

f(x + h) - 2f(x) + f(x - h) g(h) - g(O)

as required.

h2 h2

= g'(Bh)/2Bh (0 < B < 1)

= (f'(x + Bh) + f'(x - Bh))/2Bh

-+ 1"(x) as h -+ 0 ,

The converse is false, for example, let

f(x) = fox tsin(l/t) dt;

then f(ss) exists at the point x = 0 but 1" does not.

We would now like to introduce the concept of uniform Schwarz di.fferen­tiability in [a , bJ.

Definition A.2.12. Let f be Schwarz differentiable at each point x E I. We say that f is uniformly Schwarz differentiable in [a, bJ if given E > 0, there exists a number 6 = 6(E), 6 independent of x, such that l(f(x + h) - f(x - h))/2h­f(s)(x) I < E if Ihl < 6 for all x E [a , b], x ± h E I.

According to Reich [85], the concept of uniform Schwarz differentiability is due to S.N. Mukhopadhyay.

Our aim is to show that l' (x) exists, given the existence of f( S) (x) and certain other suitable conditions (d. 7, 8 and Theorem A.2.3). We prove the following two simple results:

Theorem A.2.13 (Simeon Reich [91]). Suppose f(s) is continuous at the point a and suppose that f is uniformly s- differentiable in a neighbourhood of a. Then 1'(a) exists and equals f(s)(a).

Proof. Let E > 0 be given. By the uniform s-differentiability of f, there exist 61 ,62 > 0, with l(f(a + t + h) - f(a + t - h))/2h - f(s)(a + t)1 < E/2 for all It I < 61, and all 0 < Ihl < 62. By the continuity of f(s)(x) at a, If(s)(a + t) - f(s)(a)1 < E/2 for all It I < 63 (63 > 0). It follows that

I f(a + t + h) ~f(a + t - h) _ f(S)(a)1

~ I f(a + t + h) ~f(a + t - h) _ f(s)(a + t)1 + If(S)(a + t) - f(S)(a)1

< E/2 + E/2 = E ,

if 0 < Itl, Ihl < 6/2, where 6 = min{61,62,63}. Putting t = h, we obtain l(f(a + 2h) - f(a))/2h - f(s)(a)1 < E if 0 < Ihl < 6/2, or with 2h = hI,

Page 249: Surprises and Counterexamples in Real Function Theory

240 Appendix - I

IU(a + hd - f(a))/h l - f(s)(a)1 < t if 0 < hI < 6, i.e., f'(a) exists and equals f(s)(a).

Theorem A.2.14. Suppose f(s) is continuous in I and let

F(x) = l xf(S)(t)dt, xE[a,b].

Then F(S) (x) exists and equals f(s)(x) (cf. the fundamental theorem of Calcu­lus).

Proof.

t+ h t- h F(x + h) - F(x - h) = la f(s)(t) dt - la f(s)(t) dt

Now

= t-h f(s)(t) dt + l x+h f(s)(t) dt _ rX-

h f(s)(t) dt la x-h la

l x+h

= x-h f(s)(t) dt .

If(S)(t)1 = If(s)(t) - f(s)(x) + f(s)(x)1 :::; If(s)(t) - f(s)(x)1 + If(s)(x)1

< t + IjCs)(x)1 ,

if It - xl:::; h, since f(s)(t) is continuous at x. So

l x +h

I(F(x + h) - F(x - h))/2hl :::; (1/12hl) x-h If(s)(t)1 dt

:::; (1/12hl) l~:\t + If(s)(x)l) dt

= (1/12hl)(t + If(s)(x)1 ).12hl = If(s)(x)1 + E.

Letting t -+ 0 and noting that then h -+ 0, this gives the required result.

Theorem A.2.15. Suppose f and f(s) are continuous in I, then f is uniformly s-differentiable in [a, b].

Proof. Let

¢(x) = f(x) -lx f(s)(t) dt (x E I) .

Then ¢(x) is continuous and s-differentiable in I and by Theorem A.2.14, ¢(s)(x) = f(s)(x) - fCs)(x) = 0 (for x E [a,b]). It follows, by Theorem A.2.9, that ¢(x) = k, a constant, i.e., that f(x) = k + fax fCs)(t)dt (for x E [a,b]).

Page 250: Surprises and Counterexamples in Real Function Theory

A.2 Schwarz differentiability 241

Hence

I f(x + h) ~f(X - h) _ f(S)(x)1

= 12~ (k + lx+

h f(S) (t) dt - k _lX

-h

f(S) (t)dt) - jcs) (x) I

= 1(1/2h) (l~:h f(s)(t) dt) - f(S)(X)1

= IjCS) (x + Bh) - f(s)(x)1 , BE [-1,1],

by the mean value theorem for integrals. Now f(S) (x) is continuous in [a, b] and hence uniformly continuous. So given £ > 0, there exists a 6 > ° (independent of x) such that If(s)(x + h) - f(s)(x)1 < £ if Ihl < 6, x E [a,b]. It follows that l(f(x + h) - f(x - h))/2h - f(s)(x)1 < £ if Ihl < 6, for all x E [a, b], i.e., f(x) is uniformly s-differentiable in [a, b].

Remark A.2.16. The continuity of f is necessary for the conclusion of The­orem A.2.15 to hold; for example, let f(x) = ° if x =j:. 0, and 1 if x = 0. Then f(s)(x) = ° for all x and so, is continuous everywhere, but f is not uniformly s-differentiable in any [a, b] which contains the point 0.

The converse of the above theorem also holds, viz.

Theorem A.2.17 (Mukhopadhyay [69]). If f is continuous and uniformly s-differentiable in I, then f(s) is continuous in [a, b].

Proof. Let ~ E [a, b]. We have to show that If(s)(~+h) - f(s)(~)1 < £ if Ihl < 6. Since f(s) is uniformly s-differentiable, so -£/3 < (f(x+k) - f(x-k))/2k­

f(s)(x) < £/3 if Ikl < 61 (for all x). Take x = ~ + h, to get

f(x+h+k)-f(x+h-k) ~ f(s)(1: I) f(x+h+k)-f(x+h-k) ~ ( ) 2k 3 < <., + ~ < 2k + 3 1

Take x = ~ and change sign throughout and rewrite to get

f(~ + k) - f(~ - k) ~ < _ f(s)(~) < _f(~ + k) - f(~ - k) +~ (2) 2k 3 2k 3

Adding (1) and (2) gives

(f(~+h+k)- f(~+h-k) - f(~+k)+ f(~-k))/2k-2£/3 < f(s)(~ + h)- f(s)(~)

< (f(~ + h + k) - f(~ + h - k) - f(~ + k) + f(~ - k))/2k + 2£/3,

if Ikl < 61, i.e., say for A = f(~ + h + k) - f(~ + h - k) - f(~ + k) + f(~ - k),

A/2k - 2£/3 < If(s)(~ + h) - f(s)(OI < A/2k + 2£/3, if Ikl < 61 (3)

Page 251: Surprises and Counterexamples in Real Function Theory

242 Appendix - I

But now, since f is continuous at x = ~ ± k, we have f(~ + k + h) - f(~ + k) < 2k.(E/6) if Ihl < 62 and f(~ - k + h) - f(~ - k) < 2k.(E/6) if Ihl < 63. Then

IAI = If(~ + h + k) - f(~ + h - k) - f(~ + k) + f(~ - k)1 < 2k.(E/6) + 2k.(E/6)

if Ihl < min{62,63} = 6, say, i.e.,

-E/3 < A/2k < E/3 , if Ihl < 6 . (4)

Substituting in (3) gives

-E/3 - 2E/3 < If(s)(~ + h) - f(s)(~)1 < E/3 + 2E/3,

if Ihl < 6, Ikl < 61. Now making sure that Ikl < 61, this completes the proof.

Theorem A.2.18 (Mukhopadhyay [71]). If f is uniformly s-difJerentiable in I, then

(i) f is continuous in [a, b] if f(s) is bounded in [a, b],

(ii) f(S) is bounded in [a, b] if f is continuous in I.

Proof. Suppose that f is discontinuous at the point c E [a, b]. Then there exists 1] > 0, such that for each 6 > 0, there are points x* E [a, b] n (c - 6, c + 6) such that

If(x*) - f(c)1 > 1] (1)

Further, since f(s) is bounded in [a, b], there exists M > 0, such that

If(S)(x)l::; M for all x E [a,b]. (2)

Hence, taking x = (x* + c)/2, h = (x* - c)/2 in the definition of uniform s-differentiability, we get

I f(x + h) ~f(x - h) _ jCS)(x)1

= If(¥ +~) ;f(¥ - ~) _ f(s) (x* 2+ c) I = IU(x*) - f(c))/(x* - c) - f(s)((x* + c)/2)1

~ 1]/lx* - cl - M (by (1) and (2)).

Since x* may be chosen as near to c as we want, this quantity above is greater than or equal to N, however large N be, and this contradicts the definition of uniform differentiability of f in I.

Conversely, by Theorem A.2.17, it follows that f(s) is continuous in [a, b] and so bounded.

Page 252: Surprises and Counterexamples in Real Function Theory

A.2 Schwarz differentiability 243

Theorem A.2.19. If f is continuous at a and uniformly s-differentiable in a neighbourhood N of a, then f'(x) exists at x = a.

Proof. Since f is continuous at a, it is bounded, i.e., 1 f (x) 1 ::; M, in some neighbourhood Na of a, which we may assume, is contained in N. Let [c, d] c Na eN (c < a < d), so that f is uniformly s-differentiable in [c, dj.

J, N, N, are intervals

~~ d (( ( [

a c

• ) )

FigureA.l.ll

Then there exists ho > 0 such that l(f(x + ho) - f(x - ho))/2ho - f(s)(x)1 < EO

for x E [c,d], for some EO. Hence If(s)(a)1 < Eo+I(f(x+ho) - f(x-ho))/2hol ::; EO + 2M /2ho, for all x E [c, d], i.e., f(s) is bounded in [c, dj and so by Theorem A.2.18 (i), f is continuous in [c,dj and hence further, by Theorem A.2.17, f(S) is continuous in some subinterval [Cl' d1 ], with Cl < a < d1 , of [c, d]. It follows, by Theorem A.2.8, that f'(a) exists.

Theorem A.2.20. If f is continuous and uniformly s-differentiable in I, then f is uniformly differentiable in [a, b].

Proof. For E > 0, there exists a 15 > 0 such that

l(f(x+h)- f(x-h))/2h- f(s)(x)1 < E/2, if Ihl < 151 , for all x E I. (1)

Further, by Theorem A.2.17, jcs) is continuous in [a, b] and so uniformly con­tinuous; hence there exists a 152 , such that

Let 15 = min{J1 ,J2 } and choose h such that Ihl < J. Now replace x by x + h/2 and h by h/2 in (1) to get (since Ih/21 < Ihl < Jd

l(f(x+h)- f(x))/h- i s)(x+h/2)1 < E/2. (3)

Hence

If(X + ~- f(x) f(S)(x)i=I f(x + ~)-f(X) _ f(s)(x + ~)-f(S)(x + ~)-iS)(x)1

< E/2 + E/2 = E ,

for all x E [a,b]. This also shows that f'(x) = f(s)(x) in [a,b].

Combining this with Theorem A.2.14, we obtain:

Theorem A.2.21. If f is continuous and uniformly s-differentiable in I, then

Page 253: Surprises and Counterexamples in Real Function Theory

244 Appendix - I

l' is continuous in [a, bJ.

For most of the results given in this section see [3], [84], [72], [91], [71 J and [69J; see also [70J.

§A.3. Cauchy's functional equation:f(x + y) = f(x)+ f(y)

Theorem A.3.l. Let f : JR ~ JR be a function that satisfies the functional equation

f(x + y) = f(x) + f(y)

for all x, y E JR. Then f(rx) = r.f(x) for all r E Q, and indeed f(x) = ex for all x E Q, for some e E lR.

Proof. Step 1. We first show that f((m/n).t) = (m/n).f(t) for all t and all positive rationals min. Now (*) gives, by induction, f(Xl + X2 + ... + x n ) = f(xd + f(X2) + ... + f(x n ) and putting Xk = x for each k, it follows that f(nx) = nf(x) (x E JR, n E 1'\1).

Write x = (m/n).(nx/m) = (m/n).t (t = nx/m E JR, m,n E 1'\1) . Then nx = mt and so f(nx) = f(mt), i.e. nf(x) = mf(t), i.e. , nf((m/n)t) = mf(t), i.e. f((m/n)t) = (m/n)f(t).

Putting t = 1 and f(l) = e, this gives f(m/n) = (m/n).c, i.e., f(x) = ex for all positive rationals x. Step 2. 'iVe now show that f(x) = ex for all rational numbers x. For x = 0, this is trivial, as on putting x = y = 0, f(O) + f(O) = f(O), i.e., f(O) = o. For x < 0, we proceed as follows:

First put y = -x in (*) to get f(x - x) = f(x) + f(-x), i.e., f(O) = f(x) + f ( -x) , i.e. ,

f(-x)=-f(x) "Ix.

Then for x < 0, sat x = -y, y > 0, we have f(y) = ey (since y > 0), i.e., f( -x) = c( -x) , i.e., - f(x) = -ex (by (t)), i.e., f(x) = ex, as required.

Theorem A.3.2. Suppose f is continuous, then(*) implies that f(x) = ex for all x E JR.

Proof. Let x E JR and let Xn be a sequence of rational numbers that tends to x. Then f( xn ) = exn , for all n. Letting n ~ 00, we get limn~oo f(x n ) = ex. But since f is continuous, limn~oo f(x n ) = f(x); hence f(x) = ex for all x E JR.

We also have:

Theorem A.3.3. Suppose f satisfies (*) and is continuous at a single point x = a; then f(x) = ex for all x E lR.

Proof. It is enough to prove that f is continuous everywhere. To see this, let ~ be any point. Then we must show that limx~t; f(x) = f(~). We have

Page 254: Surprises and Counterexamples in Real Function Theory

A.3 Cauchy's functional equation f(x + y) = f(x) + fey) 245

limx~~ f(x) = limx_~+a~ a f(x - ~ + a + ~ - a) and writing x - ~ + a = t, this gives

lim f(x) = lim f(t + ~ - a) X~~ t~a

= lim (f(t) + f(~ - a)) t~a

= lim f(t) + f(~ - a) t~a

= f(a) + f(~ - a) (since f is continuous at t = a)

= f(a + ~ - a) = f(~).

The above was observed by G.Darboux in 1875.

Theorem A.3.4. Suppose that f satisfies (*) and f(x) is nonnegative for all sufficiently small positive x i then f (x) = cx for all x E JR.

Proof. Let y > 0 be sufficiently small, so that fey) ::::: 0 by hypothesis. Then

f(x + y) = f(x) + fey) ::::: f(x) ,

i.e., f is monotone increasing. Further fer) = cr for all r E Q. Now let ~ be any real number and choose an increasing sequence rn of

rational numbers that tend to ~ and a decreasing sequence Rn of rational numbers that also tend to~. Then rn < ~ < Rn for all n = 1,2, .... Further, crn = fern) ::; f(~) ::; feRn) = eRn, since f is increasing. Here Rn and rn are rational numbers. Letting n -+ 00 we get c~ ::; f(~) ::; eC as required.

Remark A.3.5. Suppose that f satisfies (*) and is less than or equal to 0 for all sufficiently small positive numbers x, then in this case too f(x) = ex for all x E JR.

Theorem A.3.6. Suppose that f satisfies (*) and is bounded on an arbitrarily small interval (a, b) i then f (x) = cx for all x E R

Proof. Let g(x) = f(x) - x.f(l). Then g(x + y) = g(x) + g(y) (check) and 9 is bounded on (a, b). But if r E Q, then

g(r) = fer) - r·f(l)

= r.f(l) - r.f(l) = 0 (by Theorem A.3.1).

Hence g(x + r) = g(x) + g(r) = g(x) (for all r E Q). If now ~ is any real number, there exists a rational number r~ (in fact infinitely many such rational numbers) such that ~ + r~ E (a, b). Hence 9 is bounded everywhere. We claim that g(x) == 0; for, if at some point p, g(p) = A ::j:. 0, then for n E N, g(np) = n.g(p) = nA and so for sufficiently large n, 9 would assume arbitrarily large values, contradicting its boundedness. Thus 9 == 0 and so f(x) = x.f(l) = cx, say.

These considerations lead to the following

Page 255: Surprises and Counterexamples in Real Function Theory

246 Appendix - I

Theorem A.3.7. If Cauchy's functional equation

f(x + y) = f(x) + f(y)

is satisfied for all x, y E IR and if f satisfies anyone of the following conditions:

(i) f is continuous at a point x = a,

(ii) f(x) 2: 0 for 0 < x < t, for some t > 0,

(ii)' f(x) ::; 0 for 0 < x < t, for some t > 0,

(iii) f (x) is bounded in (a, b), where (a, b) is some open interval,

then f(x) = cx, c E IR, for all x E IR.

A natural question that comes to mind is, "Are there any functions f : IR -+ IR, other than f(x) = cx, that satisfy Cauchy's functional equation? If such a function could exist, then, by the above theorem, it would

(i) be totally discontinuous (i.e., discontinuous at each point),

(ii) neither be greater than or equal to 0 in (0, t) nor be less than or equal to o in (0, t), for any t > 0,

(iii) be unbounded in each interval (a, b).

Making use of the vector space basis of the (infinite dimensional) vector space IR over Q, it is easy to construct all functions that satisfy Cauchy's func­tional equation. Indeed, we have the following:

Theorem A.3.B. The most general solution f, which satisfies Cauchy's func­tional equation, can be constructed as follows: Choose a basis lffi of IR over Q, so that if x E IR is any real number, then we can write uniquely

Now define

where f(bd, f(b2 ) , .. · ,f(bn ) are arbitrarily chosen real numbers. Then f sat­isfies the required condition. Moreover, such a function is contin'uous if and only if there is a constant c such that f(bd = c.bk , for all k = 1,2, ... ,n, and then f(x) = cx for all x.

Proof. Let x = rlbl + ... + r"b", Y = Slbl + ... + s"bn , (rk, Sk can be zero) . Then (**) gives

f(x + y) = f(x) + f(y) ,

Page 256: Surprises and Counterexamples in Real Function Theory

A.3 Cauchy's functional equation f(x + y) = f(x) + f(y) 247

for, x + y = (rl + sdb1 + ... + (rn + sn)bn and therefore by (**), f(x + y) = (rl + sdf(bd + ... + (rn + sn)f(bn), while

f(x) + f(y) = rd(bd +r2f(b2) + .. ·+rnf(bn) +sd(b1 ) +s2!(b2) + .. ,+snf(bn)

= (rl + sdf(bd + ... + (rn + sn)f(bn) ,

i.e., Cauchy's equation is satisfied. Further, for x = rlbl + ... + rnbn, by Theorem A.3.1, we have

f(x) = f(rl bl + ... + rnbn)

= f(r1bd + ... + f(rnbn) (since f satisfies Cauchy's equation),

= rd(b1 ) + ... + rnf(bn) (again by Theorem A.3.1).

We have thus proved that f (x) has to be defined by (**), i.e., that there is no other f. Such an f, if continuous, and only then, is given by f(x) = ex, by Theorem A.3.2. This completes the proof of the theorem.

Example A.3.9. Suppose f : ~ ~ ~, satisfies Cauchy's functional equation. If three vertices of a parallelogram lie on the graph r of the function f, then the fourth vertex does also lie on r.

----------~// --- " , ,

------

Figure A.1.12

With the notation of Figure A.1.12, it is enough to prove that A = f(al). We have

(i) a3 - al = a4 - a2 ,

These conditions give

by (i) above.

A = f(a2) + f(a3) - f(a4)

= f(a2 + a3 - (4)

= f(ad ,

Page 257: Surprises and Counterexamples in Real Function Theory

248

Appendix - II

Hints and solutions to exercises

Exercise 1.1. Let I be the set of intervals [1'1,1'2] with rational end points; let I be the set of isolated points of E and let S be the set of sparse points of E. First note that I is countable, for fix 1'1, then {[1'1,1'2] 11'2 E QI} is countable and there are only count ably many 1'1 E QI.

If x E I, find 8 such that E(x,8) = {x} and so for any 80 :s 8, E(x,80 ) = { x }. Hence there exists a rational 80 such that E (x, 80 ) = {x}. N ow map x -+ [x - 80 ,x + 80 ], This map is one to one from I into T Hence I is countable.

Proceed likewise for S: Find a rational number 80 such that E(x,80 ) is countable and the map x -+ [x - 80 , x + 80 ] , is a one-to-one map of S into I.

Exercise 1.2. (a) '* (b) : Let sup A = 0:. Construct an increasing sequence {an} with an E A n [0: - l/n,o:]. This is possible since 0: = supA. Then clearly an -+ 0:. But by (a), lim an E E, i.e., 0: E E. Similarly, inf A E E. (b) '* (c): We have sup IXn - xml < I: if m, n 2 N, i.e.,

Xn < XN + I: , if n 2 N ,

i.e., {xn} is bounded. Let 0: be a limit point of {xn}. As in the proof of (a) '* (b), choose a monotone subsequence Xn" Xn2 , ... ,Xnk , ... of {xn} which tends to 0:. Then 0: is the supremum or the infimum of {xnk }. Hence, by (b), 0: E E (with A = {xnk } C E). Now IXn - xnkol < 1:/2, ifn,nko are large (by (*)) and IXnko - 0:1 < 1:/2 if nko is large, because xnko tends to 0:. Therefore

IXn - 0:1 = IXn - x nko + xnko - 0:1

:s IXn - xnko I + IXnko - 0:1

< 1:/2 + 1:/2 = I: ,

if n is large. Therefore the full sequence {xn} -+ 0: E E, as required. (c) '* (a): Let {xn} be a bounded monotone sequence in E, which tends to 0:.

We have to prove that 0: E E. Now for this sequence {xn }, we have IXn -xml :s IXn - 0:1 + IXm - 0:1 < 1:/2, if m, n 2 N, say. Hence sup {Ixn - xml} < 1:, if m,n > N. Therefore, this supremum tends to 0 as N -+ 00. Therefore by (c), {xn} tends to a limit which belongs to E, i.e., 0: E E.

Page 258: Surprises and Counterexamples in Real Function Theory

Hints and solutions to exercises 249

Exercise 1.3. Let a1 = a2 = 0, where, for n ~ 3, an is equal to the length of a side of a regular polygon of n sides, inscribed in a unit circle, so that clearly an ~ 0 as n ~ 00 , but nan is equal to the length of the complete polygon, which tends to the circumference of the circle, which is 27r, a transcendental number.

Exercise 2.1. 6(s,t) = 0 implies that there exists xn,Yn, n = 1,2, ... , with IXn - Ynl < lin, such that f(xn) = 8, f(Yn) = t. But IXn - Ynl < lin < 6 (if n ~ N) implies that If(xn) - f(Yn)1 < f (since f is continuous in [a, b] and so uniformly continuous), i.e., 18 - tl < f, which is false since 8 :j:. t.

Next, choose Xn < Yn with f(xn) = r, f(Yn) = t, such that IXn - Ynl = 6(r, t) + lin. Since r < 8 < t, f takes the value 8 between Xn, Yn, say at Wn, i.e., Xn < Wn < Yn , such that f(wn) = 8. Then IXn-wnl:S IXn-Ynl = 6(r, t)+1/n (for all n) and so is less than or equal to 6(r, t), i.e., 6(r,8) :S 6(r, t).

Finally, if 6(r, 8) = 6(r, t) for some r < 8 < t, then there exist Xn < Wn < Yn, with f(xn) = r, f(wn) = 8, f(Yn) = t and then (wn - xn) - (Yn - xn) ~ 0 as n ~ 00, i.e., Yn - Wn ~ 0 as n ~ 00, and so f(Yn) - f(wn) ~ 0, by the continuity of f, i.e., It - 81 < f, which is false since t:j:. 8.

Exercise 2.2. (a). Let 61 > 62. Then ¢(x, 6d ~ ¢(X,62), for all x, since the left side is the oscillation of f in (x - 61, X + 6d while the right side is the oscillation of f in (x - 62, X + 62). Therefore, sup x ¢(x, 6d ~ sUPx ¢(x, 62), i.e. 'ljJ(6]) ~ 'ljJ(62). (b) First, let f be bounded: m :S f(x) :S M. Then for all X,Y E lR, If(x) -f(Y)1 :S M -m, so ¢(x, 6) :S M -m for all x, 6 (> 0) and hence 'ljJ(6) :S M -m for all 6 > 0, i.e., 'ljJ is bounded. Conversely, let 'ljJ be bounded, say sup'ljJ(6) = M (inf is greater than or equal to 0, of course). Then for all x and all 6 > 0, ¢(x, 5) :::; 'ljJ (5):::; M. In this, take x = 0 (say) to get ¢(0,5):::; M for all 5 > 0 and so If(O) - f(Y)1 :::; M, if -6 < Y < 6 for all 6 > 0, i.e., M - f(O) :S f(y) :S M + f(O) for all Y (since 6 can be any positive real number), i.e., f is bounded, as required. (c). First, let f be uniformly continuous on lR. Then given f > 0, there exists an T) > 0, such that If(x) - f(Y)1 < f if Ix - yl < T) and so ¢(x, 6) :S f if 6 < T)

(for all x E lR), and so 'ljJ (6) :S f if 6 < T) , i.e. , 'ljJ(6) ~ ° as 6 ~ O. Conversely, let 'ljJ(6) ~ 0 as 6 ~ 0, and we just work backwards to get f is uniformly continuous on lR.

Exercise 2.3. I:::} I I: Suppose I I is false. Let f > 0 be given. Then there exists x as close to a as we want, such that

If(x) - AI ~ f . ( i)

Choose a sequence {xn} with Ix-al < lin (so that for large n, Xn is sufficiently close to a) such that If(xn) - AI ~ f (see (i)) , i.e.,

f(xn) does not tend to A . ( ii)

Page 259: Surprises and Counterexamples in Real Function Theory

250 Appendix - II

However, since IXn - al < l/n so {xn} -+ a and so by (I), f(xn) -+ A, a contradiction to (ii). II:::} III: By II, given 10 > 0, there exists a el > 0 such that If(x) -AI < 10/2 if Ix-al < el and similarly If(y) -AI < 10/2 if Iy -al < el; and then If(x) - f(y)1 :::; If(x) - AI + If(y) - AI < 10/2 + 10/2 = 10 if 0 < Ix - al < el, 0 <; Iy - al < el, which is II I. III :::} I: Let {xn} -+ a and we want to prove that {f(xn)} is convergent. By III, given 10 > 0, there exists a el > 0 such that If(xn) - f(xm)1 < 10 if IXn - al < el, IXm - al < el, but since Xn -+ a so IXn - al < el if n 2: N, i.e., IXn - al < el, IXN - al < el, if n 2: N and so by III, we have If(xn) - f(XN)1 < 10, i.e., -10 < f(xn) - f(XN) < 10, i.e., f(xn) < f(XN) + 10 for all n 2: N, i.e., f(xn) is a bounded sequence and so contains a convergent subsequence, f(xn, ),f(xn2 ), ... ,f(xnk ), ... , say, which tends to a limit Las k -+ 00. We claim that the entire sequence {f(xn)} -+ L. To see this, select n large and nk

(i.e., k) large so that Ix-al < el, IXnk -al < el. Then by III, If(xn) - f(xnk)1 < 10, but now If(xn) - LI = If(xn) - f(x nk ) + f(x nk ) - LI :::; If(xn) - f(xnk)1 + If(xnk ) - LI < 10 + 10; hence f(xn) -+ L, as required.

Exercise 2.4. (i). Let Xl < X2 and we have to prove that f(xr) < f(X2). We have

00

f(X2) - f(xr) = '~:)¢X2 (7"n) - ¢Xl (7"n))/2 n ; n=l

the rearrangement is allowed since the series is absolutely convergent. Here

{

1 - 1 if 7"n :::; Xl ,

(¢X2 (7"n) - ¢Xl (7"n)) = 1 - 0 if Xl < 7"n :::; X2 ,

0-0 if X2 < 7"n,

= {~ if 7"n tJ. (Xl,X2] , if 7" n E (Xl, X2] ,

so the above sum is equal to I:Tn E(Xl,X2]1/2n > 0, since such 7"n exist. (ii) First let Xo be irrational and let Xo < X; then

00

f(x) - f(xo) = 2:)¢x(7"n) - ¢xo(7"n))/2n = L 1/2n n=l

and we can ensure that this is less than any 10, by making sure that the first few terms, say, 1/2,1/22 , ... ,1/2j do not appear in this sum (note that j = j(f)), i.e., that 7"1,7"2, ... ,7"j tJ. (Xl, X2], by choosing X close enough to Xo. Similarly for X < Xo. That proves continuity for the irrational x's.

Finally, if Xo is rational, say equal to 7" no, then for X < Xo we have If (x) -f(xo)1 = I:Tn E(x,xo]1/2n and this sum contains the term 1/2n0 since 7"no E

(x, xo] and is not less than 10 if 10 < 1/2n0. That completes the proof.

Exercise 2.5. f is continuous and so bounded and attains its bounds, so S f. 0.

Page 260: Surprises and Counterexamples in Real Function Theory

Hints and solutions to exercises 251

Let A = sup 5 and we have to prove that A E 5. Suppose, to the contrary, that A tf. 5, then f(A) is not the supremum of f(x) and so by the continuity of f, f(xo) is not the supremum of f(x) for each Xo E [a - E,A + E]; but the interval [A - E, A + E] contains points of 5, i.e., points Xo where f(xo) is the supremum of f(x), which is a contradiction.

Suppose, to the contrary, that g(x) is not identically 0 in [0,1]. Let B = sup g(x), and without loss of generality, assume that B > O. Let 5 be the set of those points of [0,1], where B is attained. By the first part 5 i- 0 and 5 contains sup 5 = A, say. Then Ai-I (nor is A equal to 0) (since g(A) = supg(x) = B > 0, while g(l) = 0), so 0 < A < 1. Therefore, there exists a k, 0:::; A - k < A < a + k :::; 1, with g(A) = Hg(A + k) + g(A - k)}. Here g(A + k) :::; B (since B = sup g) and g(A - k) :::; B, g(A) = B. Hence g(A + k) = B, i.e., A + k E 5, a contradiction since A = sup5.

Exercise 2.6. Define fo(x) = x, JI(x) = f(x), iz(x) = f(J(x)), ... , fn+1(x) = f(Jn(x)). Note that f is continuous (by the given condition). Suppose f(x) > x for all x. This means fn(x) is a strictly increasing sequence for all x, i.e., in particular, {fn(O)} is a strictly increasing sequence. Now for each n,

Ifn+1 (0)- fn(O) I:::; klfn(O)- fn-l (0) I:::; k2lfn_l (0)- fn-2(0)1:::;· .. :::; knlf(O) - 01·

Hence for m > n,

Ifm(O) - fn(O)1 = Ifm(O) - fm-l(O)+ fm-l(O) - fm-2(0)+···+ fn+1(O) - fn(O)1

:::; Ifm(O) - fm-l(O)1 + Ifm-l(O) - fm-2(0)1 + ... + + Ifn+1(O) - fn(O)1

:::; f(0)(km- 1 + km- 2 + ... + kn)

:::; f(O).kn(lj(l- k))

--+ 0 as n --+ 00.

Therefore, by the general principle of convergence, {fn(O)} is convergent, say to the limit Lt. But now {f(Jn(O))} is the same sequence (check), and so tends to Lt, i.e., fn+l (0) --+ Lt, but fn+l (0) also tends to Lt, which contradicts our supposition that f(x) > x for all x. Similarly, " f(x) < x for all x " is false. Hence either f(xo) = Xo for some Xo as required and we are through, or

3 Xl,X2 such that f(xd<Xl,f(X2»X2. (*)

Also, f is continuous, so g(x) = f(x) -x is continuous and g(xJ) < 0, g(X2) > 0 and so by the intermediate value property, there exists Xo such that g(xo) = 0, i.e., f(xo) = Xo·

Now the uniqueness of Xo· If also f(yo) = Yo, then If(yo) - f(xo)1 :::; klyo - xol, but f(yo) = Yo, f(xo) = Xo and so Iyo - xol :::; klyo - xol < Iyo - xol, which is a contradiction unless Yo = Xo. This completes the proof.

Finally, the last part. Consider the function f(x) = J(l + x2). We shall show that If(x) - f(y)1 < Ix - yl· We have

IJ(l + x2)-J(1 + y2)IIJ(1 + x2)+J(1 + y2)1 = 11+x2-1-y21 = Ix+yllx-yl,

Page 261: Surprises and Counterexamples in Real Function Theory

252 Appendix - II

and therefore

I[J(1 + X2) - J(1 + y2)]j(X - y)1 = Ix + yl/IJ(1 + X2) + J(1 + y2)1

~ (Ixl + Iyl)/( J(l + :Z;2) + J(1 + y2))

<1

(since Ixl < J(l + x2), Iyl < J(l + y2)). Hence IJ(1 + x2) - J(l + y2)1 < Ix - yl, as required.

But now J(l + x2) is never equal to x; indeed J(l + x 2 ) > x, always.

Exercise 2.7. Suppose, to the contrary, that for some a < b, we have f(a) i f(b) and without loss of generality, let f(a) < f(b). Choose an irrational p with f(a) < p < f(b) and let g(x) = f(x) - p. Then 9 is continuous in [a, bJ and g(a) < 0, g(b) > 0, and so by the intermediate value property, there exists c E (a, b) such that g(c) = 0, i.e., f(c) = p. But f is rational valued and p is irrational- a contradiction.

Exercise 2.8. Suppose f(x) -7 A as x -7 00. Let E > 0 be given; then If(x) - AI < E/2 if x 2: ~. Now in the closed interval [O,~], since f is continuous, it is uniformly continuous. In the interval [~, (0), we have

If(xI) - f(X2)1 = If(xI) - A + A - f(X2)1

~ If(Xl) - AI + I!(X2) - AI

<E/2+E/2=E, if Xl,X22:~,

so f is uniformly continuous in [~, (0). The converse is false, for example, take f(x) = x, which is uniformly con­

tinuous in [0,00], but f does not tend to a finite limit as x tends to 00.

Exercise 2.9. Easy.

Exercise 2.10. The trick is to use simultaneous induction on n, i.e., our induc­tion hypothesis is

(i) P2n-l(X) = 0 has only one real root, and

(ii) P2n (x) = 0 has no real root;

and we have to prove that

(i)' P2n+I(x) = 0 has only one real root, and

(ii)' P2n+2(X) = 0 has no real root.

Now P~n+l (x) = P2n (x), and by (ii), it has no real zero and P2n (0) = 1, so being a continuous function, it stays above the x-axis for all x, i.e., retains positive sign for all x, i.e., P~n+l (x) > 0 for all x. It follows that

P2n+1(X) is an increasing function of x, for all x. (a)

Page 262: Surprises and Counterexamples in Real Function Theory

Hints and solutions to exercises 253

Further, for x large and negative, P2n+I(X) < 0, while for x large and positive, P2n+ I (x) > 0; so P2n+ I (x) is equal to 0 exactly once, proving (i)' (note that (a) alone does not prove (i)', for example eX satisfies (a) but it is never 0).

Let a2n+ I be the unique real root of P2n+1 (x) = O. Since P2n+1 (x) = 1 + x + x2/2! + ... , we see that P2n+1 takes positive values for any positive x and so

(b)

Now P~n+2(X) = P2n+1 (x), which is >, =, < 0 according as x is >, =, < a2n+1. Thus the minimum of P2n+2(X) is at a2n+1. We shall show that

(c)

i.e., the minimum of P2n+2 is positive and so P2n+2(X) > 0 for all x, proving (ii)'. To prove (c), note that

P2n+2(a2n+d = P2n+l (a2n+1) + a 2n+2 /(2n + 2)!

= 0 + a positive quantity

> 0,

as required. The last part of the question is almost trivial. By the uniform convergence of the series, the function y = exp(x) = I: x T /r! is a differentiable function of x for all x E lE. and term by term differentiability is legal, i.e., y' = (exp(x))' = exp(x), i.e., dy/dx = y. To prove that exp(x) is non­decreasing, it is enough to prove that y' > 0 for all x, i.e., that exp(x) > 0 for all x. This is easily checked from the series for exp(x).

Exercise 2.11. First, suppose limx-ta+ f(x) = a and limx-tb- f(x) = (3 both exist. Define f(a) = a, f(b) = (3; then f is now a continuous function, defined on the closed interval [a, b]. Hence f is uniformly continuous on [a, b] and so, on (a, b).

Conversely, suppose f is uniformly continuous on (a, b) and we have to prove that the two limits in question exist. Now f(x) --+ A as x --+ a+ if and only if given E, there exists a 15 such that If(xd - f(X2) I < E for all Xl, x2 lying in a right neighbourhood of a. But by uniform continuity such a 15 exists; indeed If(xd - f(x2)1 < E for all XI,X2 E (a,b) with IXI - x21 < 15. That completes the proof.

Exercise 2.12. (i) f(O) is determined and equals O. Indeed, being continuous at x = 0, we have f(O) = limx-to+ f(x) = limx-to- f(x), so f(O) is determined. If f(O) were positive, then, being continuous at x = 0, f(x) > 0 in a sufficiently small neighbourhood of 0, a contradiction to the hypothesis. Thus f(x) is not greater than o. Similarly, f(O) is not less than O. So f(O) = O. (ii) f(O) is not determined by this condition; for example, let f(x) = x. sin(l/x), for x -I O. Then If(x)1 = Ixll sin(l/x)1 ::; Ixl < E, if 0 < Ixl < 15; so the condition

Page 263: Surprises and Counterexamples in Real Function Theory

254 Appendix - II

is satisfied but f(O) is not defined. (iii) Write f(h) = a + Eh, where Eh ---+ 0 as h ---+ O. Then

(f(h) + f( -h) - 2f(O))/h = (a + Eh + a + Ch - 2f(0))/h

= (2a + Eh + Ch - 2f(0))/h

---+ 1 , as h ---+ 0 (given).

It follows that 2a = 2f(0), giving f(O) = a.

Exercise 2.13. For x = ±1, ±1/2, ±1/3, ... and only these values of x, we have sin(1f/x) = O. Further, it is enough to prove that f is integrable in [0,1], since the prooffor [-1,0] is identical. So let n > O. For x E (l/(n+ 1), l/n), sin(1f Ix) is positive if n is even and is negative if n is odd. It follows that

. (. ( / ) ) {1 if n is even , sIgn sm 1f x = -1 if n is odd.

Hence the graph of f is as shown in Figure A.2.1. This has count ably many points of discontinuity; hence it is integrable.

-1 II

Figure A.2.1

Exercise 2.14. (i). The condition ensures that f(x) ---+ 0 as x ---+ 00.

Indeed, If(x) - g(x)1 < E if x 2 6., i.e., g(x) - E < f(x) < g(x) + E if x 2 6.. Here, the right side is less than E, since g(x) < 0 and -E < f(x), since f(x) > O. Thus -E < f(x) < E if x 2 6., i.e., If(x)1 < E if x 2 6., which means f(x) ---+ 0 as x ---+ 00.

(ii) If A :j;; 0, this does not ensure that f(x) tends to a finite limit, for example, let

f(X)={_l, ~fxE[2n,2n+1),n=O,~,2, ... 1, IfxE[2n+1,2n+2),n-O,l,2, ...

Page 264: Surprises and Counterexamples in Real Function Theory

Hints and solutions to exercises 255

Then j2 = 1, which tends to 1, but f does not tend to any limit as x --+ 00.

However, if A = 0, then f2 --+ 0 :::} f --+ 0 as x --+ 00.

Exercise 2.15. (i) Take y = x to get f(2x) = (f(X))2 2: 0 for all x. Suppose f(O) = 0 for some a. Then f(a + x) = f(a).f(x) = 0 for all x, i.e., f(x) == 0, a contradiction. (ii) Let ~ be equal to supf in [-X, X], attained at~, i.e., f(~) =~. Suppose, to the contrary, that inf f = O. Then, given E > 0, there exists 7] such that o < f(7]) < E. SO f(7] + x) = f(7])·f(x), i.e., f(x) = (1/ f(7])).f(7] + x) > (I/E)·f(7] + x). Now take x = ~ - 7]; so f(~ - 7]) > (I/E).f(~) = (I/E).~ > ~, if E < 1, a contradiction.

Exercise 2.16. The open intervals (x - 6x , x + 6x ) form an open cover of [0,1]. By the Heine- Borel theorem, there exists a finite subcover, say, {Oi = (Xi -6xi , xi + 6x J}, i = 1,2, ... ,n, where, without loss of generality, assume that Oi n Oi+1 i- 0 for each i = 1,2, ... ,n - l.

Choose Pi E Oi n 0i+l; then

(1) f (0) - f (pd = PI E Q ,

(2) f(X2) - f(pd = P2 E Q ,

(3) f(X2) - f(P2) = P3 E Q ,

(4) f(X3) - f(P2) = P4 E Q ,

Now (1) :::} f(pd E Q, since f(O) = 0, and so (2) :::} f(X2) = f(pd + P2 E Q. Therefore, by (3), f(P2) E Q, and so by (4), f(X3) E Q, and so on. Thus f(Xi) E Q for all i and so f(x) E Q, by the equation f(x) - f(x') E Q.

•• • • • • • • • • • • • P2

Figure A.2.2

The condition f(O) = 0 is essential, for, take f(x) = V2 for all x E [0,1]. This function satisfies the conditions of the theorem (except that f(O) = 0) but fails to satisfy the conclusion.

Exercise 2.17. Hint: Define f(O) = 0, if x is algebraic, and f(x) = 1 if x is not algebraic, and note that a and am are either both algebraic or both transcendental.

Exercise 2.18. No: For otherwise there would exist Xl i- X2 such that f(xd i­f (X2). Suppose, without loss of generality, that Xl < X2, say Xl + P = X2. Then f(XI + p) = f(X2), i.e., f(xd = f(X2), which is a contradiction.

Page 265: Surprises and Counterexamples in Real Function Theory

256 Appendix - II

Exercise 2.19. Let E be given. vVe have to show that there exists a closed interval [e, d] C [a, b] such that f(x) < E for all x E [e, d]. Consider the upper Riemann sum Sp for the partition

p : a < a + (b - a)/n < a + 2(b - a)/n < ... < a + (n - l)(b - a)/n < b .

If the result is false, then

Sp > E/n + E/n + ... + E/n = E .

By Darboux's theorem Sp tends to the Riemann integral of f over [a, b], which is given to be equal to O. So Sp ought to be less than E if norm p is small enough, which contradicts (*) if n is large enough.

~ow take E = 1 and find [a1' b1] (the above [e, d]) contained in [a, b] such that

f(x) < 1 for all x E [a1, b1]. Then f:: f = 0 (for 0 = f: f = faa, f+ f:,' f+ f:, f· Here each term is greater than or equal to 0, since f 2: 0, and so each term is equal to 0).

Now repeat the process with E = 1/2 to get [a2' b2] C [a1' b1] such that f(x) < 1/2 for all x E [a2' b2 ], and so on.

Thus f(x) < l/r in far, br]. By the nested interval property, there exists a ~ in [a, b] which is in each far, brL so 0 ~ f(O < l/r for all r = 1,2, ... ; hence o ~ f(~) ~ 0, giving f(O = O.

Exercise 2.20. The following graphs show examples of functions for which (i) (*) is satisfied, (**) fails; (ii) (*) fails, (**) is satisfied.

x=J

x=J-JI10

Figure A.2.3

x-(1-ro)

if 1 - /0 ~ x ~ 1 + 110 ,

if 1 + 110 < x,

if x < 1 - 1~.

Figure A.2.4

{I if 1 <

(ii) f(x) = 0 - x, if x < 1.

Page 266: Surprises and Counterexamples in Real Function Theory

Hints and solutions to exercises 257

Exercise 2.21. First of all, since f is continuous, it is integrable and so, since dg(x)ldx = f(x), we see that g(x) has to be of the form g(x) = I: f(t) dt + g(a). Further, g(x + 'if) = 1:+11: f(t) dt + g(a) = I: f(t) dt + 1:+11: f(t) dt + g(a) = g(x) + 1:+11: f(t) dt. However, since g(x + 'if) = g(x), this implies

1:+11: f(t) dt = 0, i.e., 1011: f(t) dt = 0, which is the required necessary and sufficient condition.

Exercise 2.22. We modify the function

f(x) =

(-l)I.X if1/2 :Sx<1,

(-1)2. x if 1/3:S x < 1/2,

(-1)3. x if 1/4:S x < 1/3,

... , (-l)n.x if 1/(n + 1) :S x < lin,

... ,

whose graph is shown in Figure A.2.5, to make it continuous, without changing the value of the function at the points x = 1,1/2,1/3, .. .. The graph of the modified function is also drawn in Figure A.2.6.

Figure A.2.S Figure A.2.6

Exercise 2.23. (i) The function fn is given by

fn(x) = {(n -l)x if 0:S x:S lin, _x_ + n-2 if lin < x < 1. n-I n-I --

Page 267: Surprises and Counterexamples in Real Function Theory

258 Appendix - II

It follows that

-1 {x/(n - 1) if 0 ~ x ~ (n - l)/n, gn(X) = fn (x) = (n _ l)x _ (n - 2) if (n - l)/n ~ x ~ 1.

{x/(n - 1)

= in - l)x - (n - 2) if 0 ~ x ~ (n - l)/n,

if (n - l)/n ~ x < 1

if x = 1.

Hence, as n --t 00, we see that

{o if 0 ~ x < 1, f;:l(x) ---+ 1

if x = 1 .

This, in any case, is clear from the graph of fn(x). (ii) Let x E (0,1] be any point. Find m such that 2m.x > 1/3, but 2m- I x ~ 1/3. Then g(x) = 2x, g2(X) = g(g(x)) = g(2x) = 4x, ... , gm(x) = g(gm-l(x)) = g(2m- 1 .x) = 2mx, since 2m- I x ~ 1/3, and gm+l(x) = g(gm(x)) = g(2m.x) = (2m X + 1)/2 (noted above), since 2m.x > 1/3. Here again, (2m.x + 1)/2 = 2m- 1 .x+1/2> 1/2> 1/3, and thereforegm+2(x) = g((2m.x+1)/2) = ((2m.x+ 1)/2 + 1)/2 = (2m.x + 3)/2, and so on, up to gm+n(x) = (2m.x + 2" - 1)/2n, and letting n --t 00, we see that this tends to 1. Thus

lim gn(x) = 1 for all x E (0,1] . n-+oo

Finally, if x = 0, then g(O) = 0 and so g(g(O)) = 0, and so on, i.e.,

gn(O) --t 0 as n --t (X) •

Exercise 2.24. f(x) =psinx+qxcosx+x2,andsof(-x) = -psinx-qxcosx+ x2 . Adding we get f(x) + f(-x) = 2x2 . Putting x = 2 gives f(2) + f(-2) = 8 or f(-2) = 8 - f(2) = 5.

Exercise 2.25. Since x > sinx for x > 0, it follows that f(x) > O. Further, f' (x) = (sin2 x - x2 cos x) /x 2 sin2 x and so it is enough to prove that f' (x) > 0, i.e., that sin2 x > x 2 cosx. Let g(x) = x - sinx(cosx)-1/2. Then g"(X) = (1/4) sinx(cosx)-5/2(cos2 x - 3). Here obviously g"(X) < 0 if x E (0, r./2) and hence g'(x) is decreasing in (0,r./2), or g'(x) < g'(O) (if 0 < x < r./2) = 0 (check). Since g'(x) < 0, we see that g(x) is decreasing, i.e. , g(x) < g(O) (if 0< x < r./2) = O. Thus g(x) < 0 in (0,r./2), i.e. , x < sinx/Jcosx and since all the quantities involved are positive, we get x2 cos x < sin2 x, as required.

Exercise 2.26. The graph of the function y = log x shows that it is concave in [a, b]. where 0 < a < b. This implies (see Figure A.2.7) that the ycoordinate of A is larger than the y-coordinate of B, i.e. , that

log(>.a + (1- >.)b) > >'loga + (1- >.) 10gb.

Page 268: Surprises and Counterexamples in Real Function Theory

Hints and solutions to exercises 259

Q

a h+(l-A)b b

Figure A.2.7

Now take a = sinx, b = sinx + cosx, ,\ = tanx, so that for 0 < x < 71:/4, ,\ E (0,1). Then (*) gives log(tanx.sinx + (1 - tanx)(sinx + cosx)) > tanx.log(sinx)+(l-tanx).log(sinx+cosx), i.e., log (cos x) > tanx.log(sinx)+ (1- tanx).log(sinx + cos x) > tanx.log(sinx), (since in the second term, 1 - tan x > 0 and sinx + cosx = v'2.cos(7I:/4 - x) > 1), i.e. log(cosx) > log((sinx)tanx), i.e., cosx > (sinx)sinx/cosx, i.e. , (cosx)COS X > (sinx)sinx.

Exercise 2.27. (i) Let a = cosx. Then sina = sin(cosx). Since sina < a, we have

sin( cos x) < cos(sin x) . (1)

But sin x < x and so

cos (sin x) < cosx. (2)

Now (1) and (2) give sin (cos x) < cos (sin x). Another m ethod of doing (i): Without loss of generality assume that -71: < x::; 71:. We first show that cos(sinx) =j:. sin(cosx). For, otherwise, cos(sinx) = sin(cosx) = cos(7I:/2 - cosx) and so sinx = ±(7I:/2 - cos x), since both sinx and 71:/2 - cosx E (-71:,71:]. Thus

either sinx + cosx = 71:/2 or sinx - cos x = -71:/2.

But sin x ± cos x = v'2. sin(x ± 71:/4) and we get that

- v'2 ::; sin x ± cos x ::; v'2 , or, since -71:/2 < -v'2 and v'2 < 71:/2,

-71:/2 < sinx ± cosx < 71:/2,

contradicting (*). Now, by continuity and the fact that cos (sin 0) > sin (cos 0), it follows that cos (sin x) > sin( cos x). (ii) Let sina = cos x; then a = sin-1(cosx). But sina < a, i.e. ,

cos x < sin -1 ( cos x) (1)

Page 269: Surprises and Counterexamples in Real Function Theory

260 Appendix - II

Also x> sinx, and so sin- I x> sin- l (sin x) = x and therefore

cos(sin-l x) < cos x .

Now (1) and (2) imply cos(sin-l(x)) < sin- 1 (cosx). (iii)

I sin(rx)1 = I sin((r - l)x + x)1

= I sin((r - l)x) cos x + cos((r - l)x) sin xl

::; I sin((r - l)x)1 + I sinxl ,

and the result follows by induction.

(2)

Exercise 2.28 (Mathematics Magazine, 53 (1980), 245). The following function disproves the statement:

f(x) = {7r - 217r - .xl if 17r - xl < 7r /2, o otherwIse .

The graph of this function is easily drawn. The function is zero outside of (7r /2, 37r/2). In (7r /2, 7r), we have 0 < 7r - X < 7r/2 and so f(x) = 7r- 2(7r -x) = 2x - 7r, while in (7r,37r/2), we have 17r - xl = x - 7r, since 7r - x < 0; so f(x) = 7r - 2(x - 7r) = -2x + 37r. We shall show that for each x E lR, except x = 7r, fn(x) = O.

First of all, for x = 7r, Ix - 7r1 = 0 < 7r/2, so fen) = 7r , hence f(f(7r)) = f(7r) = 7r, and so on, i.e., fn(7r) = 7r =I- 0 for any n .

For other values of x: If x lies outside the interval (7r /2, 37r /2), f(x) = 0 and so f(f(x)) = f(O) = 0, and so on. If x lies in the above interval, we proceed as follows: If x is such that Ix - 7r1 :::: 7r /2 n for some n E N, then

f(x) = 0 .

Now for all x inside [7r /2, 37r /2]' Ix -7r1 is greater than or equal to 7r /2 n for some n (starting with n = 1), so for all these x too, fn(x) = 0 for some n. Therefore now it remains to prove (*) . We use induction: For n = 1, Ix -7r1 :::: 7r /2 implies f (x) = 0 (definition of f). Suppose (*) holds for n = k and let

Then Ij(x) -nl = 17r-2In-xl-7rI:::: 7r/2k (by the definition of j, noting that Ix - nl < 7r/2 and by (m. So by the induction hypothesis, jk(f(x)) = 0, i.e., jk+1 (x) = 0, as required.

Exercise 4.1. First note that IF(x) I ::; 3x2 for all x E lR; for since F(x) = F( -x) for all x and F(O) = 0, it is enough to prove this for x > O. Now if 0 < y < x, then

j 'XSin (l/t)dt =lx(t2 sin(l/t) /t2)dt = x 2 cos(l/ x) _y2 cos(l/y) -lX2t cos(l/t)dt. y y y

Page 270: Surprises and Counterexamples in Real Function Theory

Hints and solutions to exercises 261

Therefore

Hence

as required. So now we have

lim (F(x) - F(O))/(x - 0) = lim (F(x)lx) = X, x~o x~o

say, Then limx~o -3x2 Ix :'S: X :'S: limx~o 3x2 Ix and it follows that X = O.

+1 [ ] 00+1 Exercise 4.2. The required area A = J: y dx, and so dAldx = y a

(e OO+1 I(a + 1)) - (e OO I a), and for the extrema, this is equal to 0, i.e., e OO .e/(a + 1) = eOO la, i.e. a = 1/(e - 1).

Remark. Among all areas bounded by y = f(x), the x-axis and two ordinates n units apart, the extreme one is that whose left ordinate is the solution for x of f(x + n) = f(x). Note that no matter how irregular y = f(x) may be between x and x + n the two bounding ordinates are of equal length.

l/(e-I) e/(e-l)

Figure A.2.8

Exercise 4.3. A routine exercise, try!

Exercise 4.4. Use induction on n. For n = I, x' = 0, x" = 1 is a solution, by (ii) . The induction step: By the horizontal chord theorem, there exists a, a + 1 E [0, n] such that f(a) = f(a + 1). Define a function 9 on [0, n - 1] by

() { f ( x ) if 0 :'S: x :'S: a , 9 x = f (x + 1) if a :'S: x :'S: n - 1 .

Page 271: Surprises and Counterexamples in Real Function Theory

262 Appendix - II

Then 9 is continuous in [0, n - 1] and g(O) = g(n - 1).

g = f in [0, a]

a a+l n-l n

Figure A.2.9

Hence by the induction hypothesis, there exist n - 1 solutions of g(xl) = g(X"), ·th.1I I E "T (I ") (I ") ( I ") B t Wl X -x n,say, Xl'Xl' X2,X2 , ... , Xn-l,Xn-l' U now

{f(Xn - f(x;) if X;I < a,

o = g(xn - g(xD = f(X;' + 1) - f(x;) if x; ::::: a ::::: X~I , f (X;I + 1) - f (x; + 1) if a < x; .

So we have n - 1 distinct solutions of f(x ' ) = f(x") with x" - Xl E N. These n-1 solutions, together with the solution (a, a+ 1) gives us n distinct solutions, as required.

Exercise 4.5 (See Amer. Math. Monthly, 82 (1975),985-992). Case 1: f(a) > a. Consider the sequence of points a, f(a), p(a), ... ,fk- 1(a), fk(a) = a. Then there exists a point b in the sequence, say b = fi(a), such that f(b) < b (i.e., f (P (a)) < fi (a)), for otherwise the sequence would monotonically increase and could not return to a. Now let g(x) = f(x) - x. Then g(b) = f(b) - b < 0, but g(a) = f(a) - a> ° (case 1). So by the intermediate value theorem, there exists a point c, between a and b, such that g(c) = 0, i.e., such that f(c) = c. Case 2: f(a) < a may be similarly dealt with.

Exercise 4.6. Let x be any real number. We have

Jr(x) - r+1 (x)J = Jf(r- 1 (x)) - f(r(x))J

::::: cUr-1 (x) - r+1 (x))J + Jr(x) - r(x)J)

= cUr-1(x) - r+1 (x)J)

= c(Jr-1) - r(x) + r(x) - r+1(x)J

::::: c(Jr-1(x) - r(x)J + Jr(x) - r+1(x)J)

Page 272: Surprises and Counterexamples in Real Function Theory

Hints and solutions to exercises 263

Thus, Ir(x) - r+l (x)l(l - c) :::; clr-l(x) - r(x)l, or Ir(x) - r+l(x)1 :::; (c/(l- c)).lr-l(x) - rCx)l, and repeated application of this shows that

Ir(x) - r+l(x)1 :::; (c/(l- c))2Ir-2(x) - r-l(x)1

< ........ . :::; (c/(l - c))nlx - f(x)1 .

Hence, using (*),

Ir(x) - r+r(x)1 :::; Ir(x) - r+1(x) + r+1(x) - r+2(x) + ... + + r+r-l(x) - r+r(x)1

:::; Ir(x) - r+1(x)1 + Ir+1(x) - r+2(x)1 + ... + + Ir+r-l(x) - r+r(x)1

= (c/(l- c))nlx - f(x)1 + (c/(l- c))n+1lx - f(x)1 + ... + + (c/(l - c))n+r-llx - f(x)1

= Ix - f(x)l(c/(l- c))n(1 + c/(l- c) + ... + (c/(l ~ C)r-l)

= Ix - f(x)l(c/(l - c))n((1- c/(l- cY)/(l- c/(l- c)))

= Ix - f(x)l(c/(l - c))n(l - c).(l - c/(l - c))" /(1- 2c)

:::; Ix - f(x)l(c/(l - c))n.((l - c)/(l - 2c)) ,

since 1 - c/(l - c) :::; 1. But c/(l - c) < 1, and so the right hand side can be made smaller than any E if n :::: N. Thus {fn(x)} is a Cauchy sequence and so convergent to a point (, say. Then we have

I( - f(O :::; I( - rex) + rex) - f(OI :::; I( - r(x)1 + Ir(x) - f(OI

= I( - r(x)1 + If(r-l(x)) - f(()1 ,

which, by the hypothesis is less than or equal to I ( - r (x) I + c( I r-l (x) -f(OI + I( - r(x)I)· Now let x -+ 00, then rex) -+ ( and we get I( - f(()1 :::; 0+ c(l( - f(OI + I( - (I), i.e., I( - f(OI :::; cl( - f(OI, and since c -I- 1, we get f(O = (, as required.

Exercise 5.1. (i) By the mean value theorem, (f(x)f(a))/(x - a) = 1'(c), for some c E (a, x). Choose a so large that If' (c) I < E (c is also large, being larger than a). Then (f(x) - f(a))/(x - a) -+ 0 as x -+ 00, i.e., f(x)/(x - a) -f(a)/(x - a) -+ 0 and so f(x)/(x - a) -+ 0, i.e., (f(x)/x)(x/(x - a)) -+ 0, but x/ex - a) -+ 1, so f(x)/x -+ O. (ii) No, for example, let f(x) = (sinx2)/x. (iii) Again by the mean value theorem, (g(2n+1) - g(2n))/(2n+1 - 2n) = g'(cn) (2n < Cn < 2n+1), i.e., 2ng'(cn) = g(2n+l) - g(2n). But now Icng'(cn)I < 12n +lg'(cn )1 = 2(g(2n+1) - g(2n)) -+ 2(l - l), where g(x) -+ l as x -+ 00, as required.

Page 273: Surprises and Counterexamples in Real Function Theory

264 Appendix - II

The example. Take g(x) = sinxjx, then x.g'(x) = cosx - (sinx)jx does not tend to any limit.

Exercise 5.2. The right hand side is equal to limh-+o{(J(x + h) - f(x))jh­(J(x) - f(x - h))jh}/h = limh-+o{1'(x + ()h) - 1'(x - h + ()'h)}/h, (as by the mean value theorem (J(b) - f(a))/(b - a) = 1'(a + (b - a)()), with a = x - h, b = x for the second term, where (), ()' -+ 0 as h -+ 0), which on rewriting is equal to limh-+o((J'(x + ()h) - 1'(x + ()h + ()'h - ()h - h))/h(l + () + ()')).(1 + () - ()') = limh-+o 1"(x + ()h - h(l + () - ()') + h(l + () - ()')()").(1 + () - ()'), where (), ()', ()" -+ 0 as h -+ 0 (by the mean value theorem, applied to l' with a = x + ()h - h(l + () - ()'), b = x + ()h), which then tends to 1"(x) by the continuity of 1" (x), as required.

Exercise 5.3. Let ¢(x) = x.f((logx)/a), then d¢jdx = (l/a)(J'((logx)/a) + af((logx)/a)) -+ 0, as (logx)/a -+ 00, by hypothesis, i.e., as x -+ 00.

Further, since ¢' (x) -+ 0 as x -+ 00, we see by Exercise 5.1, that ¢( x) / x -+ 0 as x -+ 00, i.e., f((logx)/a) -+ 0 as x -+ 00, i.e., f(x) -+ 0 as x -+ 00, as required.

Now by the mean value theorem, (¢(td - ¢(t2))/(t1 - t2) = ¢'(~) -+ 0 (t2 < ~ < td as t2 and therefore t1, as well as ~ -+ 00 (proved above).

Thus if t1, t2 > .6., I(¢(td - ¢(t2))f(t1 - t2)1 < E. In this, replace t1 byeah

and t2 by eat2 . This then gives the required result.

Exercise 5.4. Let ¢(t) = f'(t) + 2f(t) - 2, then ¢/(t) = 1"(t) + 21'(t), and so ¢'(t) + ¢(t) = 1"(t) + 3f'(t) + 2f(t) - 2 -+ 2 - 2 = O. Hence by Exercise 5.3, ¢(t) -+ 0 as t -+ 00, i.e.,

1'(t) + 2f(t) - 2 -+ 0 , as t -+ 00 .

Now let 'ljJ(t) = f(t) - 1, then 'ljJ'(t) = f'(t). Therefore 'ljJ'(t) + 2'ljJ(t) = 1'(t) + 2f(t) -2 -+ 0 as t -+ 00 (by above). Therefore, again by Exercise 5.3, 'ljJ(t) -+ 0 as t -+ 00, i.e., f(t) -+ 1 as t -+ 00.

Exercise 5.5. Hint: Use the first and the second mean value theorems, viz.

(i) f(a + h) = f(a) + h1'(a + ()h) (for some 0 < () < 1),

(ii) f(a + h) = f(a) + hf'(a) + (h2 /2)1" (a + ()'h) (for some 0 < ()' < 1).

Exercise 5.6. Since 1'(xo) exists, we have (J(xo + h) - f(xo))fh = 1'(xo) + Eh, i.e., f(xo + h) - f(xo) = h1'(xo) + hEh and, by putting -k for h, f(xo - k) -f(xo) = -kf'(xo) - kE-k· Subtracting, we get f(xo + h) - f(xo) + f(xo) -f(xo - k) = (h + k)1'(xo) + hEh + kE-k. Therefore the quotient (j(xo + h) -f(xo - k))/(h + k) = 1'(xo) + (hEh + kE-k)/(h + k). Here, since both hEh and kE-k tend to zero faster than h or k or h + k, we see that the latter term above tends to zero as h, k tend to 0, which proves the result.

For the function f(x) = x2 sin (71-jx) , take Xo = O. Then

(J ( h) - f ( - k) ) / (h + k) = (h 2 sin 7r / h + k2 siwlr / k ) / (h + k) .

Page 274: Surprises and Counterexamples in Real Function Theory

Hints and solutions to exercises 265

Now choose k = -h + h2 , then the above quotient equals

which does not tend to 0 as h tends to o.

Exercise 5.7. (i) We have

f(x)/x = ((f(x) - f(x'))/(x - x')).((x - x')/x) + f(x')/x ,where X < x' ::; x.

= f'(c).(x - x')/x + f(x')/x , (by mean value theorem).

Now let x ~ 00 and see that f(x)/x ~ f'(c), x' < c < 00. Letting x' ~ 00,

it follows that c ~ 00, so the right hand side tends to A (finite or infinite), as required. (ii) Let f(x) ~ A as x ~ 00, say f(x) = A + g(x), where g(x) ~ 0 as x ~ 00.

Then (f(x + h) - f(x))/h = (A + g(x + h) - A - g(x))/h = (g(x + h) - g(x))/h. Here keep h fixed and let x ~ 00 and we get (f(x+h) - f(x))/h ~ 0 as x ~ 00,

i.e., by the mean value theorem, f'(x+eh) ~ 0 as x ~ 00 (0 < e < 1). Letting h ~ 0, this gives f'(x) ~ 0 as x ~ 00.

Exercise 5.S. By the second mean value theorem, f(a + h) = f(a) + h.f'(a) + (h2 /2).f"(a + eh), (0 < e < 1). In this put a = x, h = -x and we get

f(O) = f(x) + (-x)f'(x) + (( _X)2 /2).f"(x - ex) ,

i.e., f'(x) - f(x)/x = (x/2).f"(x(1 - e)), as required, with ~ = x(l - e). Here o < ~ < x, for we have to show that 0 < x(l - e) < x, i.e., since 0 ::; x, that o < 1 - e < 1, which is true, since 0 < e < 1. If f"(x) > 0 for x > 0, then l' (x) is an increasing function of x. It follows that the graph of f is as shown in Figure A.2.10, which then clearly shows that f(x)/x too is an increasing function of x.

Figure A.2.l0

Exercise 5.9. Hint: Let g(x) = (x a -l).f(x), then g(O) = g(l) = O. Now apply Rolle's theorem to g(x) in [0,1].

Exercise 5.10. Hint: Let h(x) = (f(x) - f(a))/(g(x) - g(a))/(g(b) - g(a)) -f(x) + f(a). Since g'(x) -::J 0 in [a, b], it follows that g(b) - g(a) -::J O. Check further that h(a) = h(b) = O. Hence, by Rolle's theorem, there exists c E (a, b) such that h'(c) = 0, proving the result.

Page 275: Surprises and Counterexamples in Real Function Theory

266 Appendix - II

Exercise 5.11 (See American Math. Monthly, 87 (1980), 817). Put ao = 0, an = 1. Since f is differentiable on [0, 1] and so continuous, there exist ak (ak-l < ak ::; 1) such that f(ad = k/n. This is done successively for each k as follows:

First we want al with ao = 0 < al < 1 = an, such that f(ad = l/n. This is possible because f(ao) = f(O) = 0, f(an) = f(l) = 1. So there exist al (0 = ao < al < an = 1), and f(ad = l/n.

Next, f(ad = l/n, f(an) = 1 and l/n < 2/n < 1, so there exists a2 (al < a2 < 1) such that f(a2) = 2/n; and so on.

Hence by the mean value theorem, there exist Xk, ak-l < Xk < ak, such that

So now we have

n n

L 1/ f'(Xk) = L n(ak - ak-d = n(an - ao) = n(l- 0) = n. k=l k=l

Since the ak are strictly increasing and ak-l < Xk < ak, the points Xk are distinct, as required. Another solution: The intermediate value theorem (IVT) is:

f continuous in [a, b], f(a) < c < f(b) => ::J ~ E (a, b) such that f(~) = c.

(i) Let ao = O. Then ao = f(O) < f(l) = 1 = an. Hence by the IVT, there exists al E (0,1) such that f(ad = l/n. (ii) Since l/n < 2/n < I, i.e., f(ad < 2/n < f(l), so by the IVT, there exists a2 E (aI, 1) such that f(a2) = 2/n. (iii) Since 2/n < 3/n < 1, i.e., f(a2) < 3/n < f(l), so by the IVP, there exists a3 E (a2' 1) such that f(a3) = 3/n; and so on; thus we have (iv) 0 = ao < al < ... < an-l < an = 1, with f(ak) = k/n, and so by IVT, there exists ~ E (ak-l,ak) such that

f'(~k) = f(ak) - f(ak-d = k/n - (k - l)/n = 1 ak - ak-l ak - ak-l n(ak - ak-d

Therefore L~=l 1/ f'(~k) = L~=l n(ak - ak-J) = n(an - ao) = n(l - 0) = n, where, since the ak are strictly increasing, the ~k are distinct.

Exercise 5.12. Let E > 0 and choose .6. > 0 such that

If(x) + f'(x)1 < E for all x 2 .6.

By Cauchy's mean value theorem, with f(x).e x for f and eX for g, we have, for X >.6., a number b E (.6.,x) such that

Page 276: Surprises and Counterexamples in Real Function Theory

Hints and solutions to exercises 267

It follows that 1(f(x)eX - f(6.)e~)/(eX - eX~)1 = If(b) + 1'(b)1 < E (by (*)), if x 2: 6.. Here the left hand side is equal to I(ex(f(x)- f(6.)e~-X))/ex(1-e~-X)I. So If(x)-f(6.)e~-XI < EI1-e~-xl or If(x)1 < If(6.)I.e~-x+EI1-e~-xl < 2E, if x is large enough (do not forget this, for given E, 6. is fixed). This completes the proof.

Exercise 5.13. For a > 0, the mean value theorem implies

f(a + 1) = f(a) + 1'(a) + (1/2)1"(a) + (1/6)1"'(6) , where a < 6 < a + 1,

and

f(a -1) = f(a) - 1'(a) + (1/2)1"(a) - (1/6)1"'(6) , where a -1 < 6 < a.

Subtract and add these to get

f(a + 1) - f(a - 1) = 21'(a) + (1/6)1'''(6) + (1/6)1'''(6) ,

f( a + 1) + f( a -1) = 2f( a) + 1" (a) + (1/6)1'" (6) + (1/6)1'" (6) - (1/6)1''' (6) .

Let now a -t 00. Then these become c - c = lima--+oo 21'(a) + 0 + 0 and c + c = 2c + lima--+oo f" (a) + 0 - 0, proving the result.

Exercise 5.14. The second mean value theorem implies

f(a + h) = f(a) + (h/1!)1'(a) + (h2 /2!)1"(a + (}h) , 0 < () < 1 .

Take (i) a = 2,h = -1, (ii) a = 3,h = -1, (iii) a = 2,h+ 1, to get (i) f(l) = f(2) - 1'(2) + (1/2)1"(2 - (}d, i.e., since f(l) = f(2) = 4,

1'(2) = (1/2)1"(2 - (}1) > 0

(ii) f(2) = f(3) - 1'(3) + (1/2)1'(3 - (}2) ,

(iii) f(3) = f(2) + 1'(2) + (1/2)1"(2 + (}3).

By (ii), 1'(3) = f(3) + (1/2)1"(3 - (}2) - f(2), and substituting for f(3) from (iii), this is equal to f(2) + f'(2) + (1/2)1"(2 + (}3) + (1/2)1"(3 - (}2) - f(2), which is greater than 0 by (*) and the hypothesis.

Exercise 5.15. Let f(x) = JI(x).h(x) ... fn(x), so that f is differentiable in [a, b] and f(a) = f(b) = 0, f(x) #: 0 for x E (a, b). So, by Rolle's theorem, there exists c E ( a, b), such that

1'(c) = O.

Now, for x E (a, b), we have

n n

1'(x) = L JI(x) ... fi-1 (x)·fl(x)·fi+1 (x) ... fn(x) = f(x). L fl(x)/ Ji(x). i=l i=l

Page 277: Surprises and Counterexamples in Real Function Theory

268 Appendix - II

Putting x = e, we get 0 = f'(e) = fee). :L~=I fICe)/ fi(e), (by (*))i but e E (a, b) and so fee) -f. o. It follows that :L~=I fICe)/ li(e) = O.

Exercise 5.16. Let

f(x) = {o if x is algebraic, 1 if x is transcendental.

This function is discontinuous everywhere and since x, xm are either both al­gebraic or both transcendental, f has the desired property.

Exercise 5.17. Put Sen) = f(l) + f(2) + ... + fen). Then Sen) - f(l) ::; Iln f(x)dx::; Sen). Dividing by g(x) and noting that f(I)/g(n) tends to 0 as n tends to infinity, we get limn--+oo S(n)/g(n) = limn--+oout f(x) dx/g(n)). Here It f(x) d x = g( 00) = 00, so by L'Hospial's rule, this equals limn--+oo fen) / g' (n).

As an application, we prove:

1 + 1/2 + 1/3 + ... + l/n ~ logn as n -t 00.

Indeed, take f(x) = l/x, g(x) = logx. These satisfy the hypothesis of the above exercise and hence also the conclusion, i.e., the required limit equals

lim (l/n)/(d(logx)/dx) = lim ((1/n)/(I/n)) = l. n~oo n~oo

Exercise 5.18 (Proof by N.R.C. Dockeray, Math. Mag., 15 (1931),435). Let

f(x) x2 x 1 f'(x) 2x 1 0 f(a) a2 1 f(a) ? 1

<I>(x) = a so that <I>'(x) = a- a

feb) b2 b 1 feb) b2 b 1 fee) e2 1 fee) ? 1 e c- e

(expand <I> by the first row and differentiate). Now <I> (a) = <I> (b) = 0, and so by Rolle's theorem, there exists a ~ E (a,b) such that <I>'(l;") = O. Similarly, there exists an 7) E (b,e) such that <I>'(7)) = 0, i.e., <I>'(x) vanishes at x = ~,7) and so there exists a (E (~,7)) C (a,e) such that <I>I/(e) = O. But again we have

<I>I/(x) =

f"(x) f(a) feb) fee)

2 0 0

a2 a 1 '" b2 b 1 = -(b-e)(e-a)(a-b).f"(x) -2 ~(b-e).f(a),

e2 e 1

i.e., there exists a ( E (a, c) such that

f(a)/(a - b)(a - c) + f(b)/(b - e)(b - a) + f(e)/(e - a)(e - b) = f"(()/2 ,

which is what we have to prove (check) but, if anything, this is a neater form!

Page 278: Surprises and Counterexamples in Real Function Theory

Hints and solutions to exercises

Exercise 5.19. We have

d(x(f(x) + g(x))) = xf'(x) + f(x) + xg'(x) + g(x) dx

= -g(x) + f(x) - f(x) + g(x)

=0

Hence x(f(x) + g(x)) is a constant, say, 2A. Similarly

d((f(x) - g(x))/x) = (x(f'(x) _ g'(x)) - (f(x) _ g(x)))/x2

dx = (-g(x) + f(x) - f(x) + g(x))/x2

= O.

269

Hence, (f(x) - g(x))/x is a constant, say, 2E. These equations imply f(x) = A/x + Ex, g(x) = A/x - Ex (A, E arbitrary constants).

Exercise 5.20. Use induction on n. For n = 0, let g(x) = e-x.f(x). Then g(a) = g(b) = O. By Rolle's theorem there exists c E (a, b) such that g'(c) = 0, i.e., e-cf'(c) - f(c)e- C = 0, i.e., f'(c) = f(c), as required.

For n > 0, let h(x) = 2::7=0 fi(x). Then h(a) = h(b) = 0 and h(x) - h'(x) = f(x) - f(n+l)(x). Apply the result for n = 0 to the function h(x). So there exists c E (a, b) such that 0 = h(c) - h'(c) = f(c) - f(n+l) (c), as required.

Exercise 5.21. By the mean value theorem, (f(c + h) - f(c))/h = f'(~), for some ~ between c and c + h (h may be negative). Letting h to go to 0, we get

f'(c) = lim f'(~) = A . h--+O

Exercise 5.22.' For x E [a, b], define g(x) = f'(x) - (f(x) - f(a))/(b - a). We now consider cases: Case 1: f(c) > f(a). Choose d in [a, c], such that f(d) is a maximum for f(x) in [a,c] (so f'(d) = 0) and let e E (a, d) be such that

f'(e) = (f(d) - f(a))/(d-a) (first mean value theorem in [a, dj) (*)

Then g(e) = f'(e) - (f(e) - f(a))/(b - a) > 0 (by (*)) and

g(d) = f'(d) - (f(d) - f(a))/(b - a) = -(f(d) - f(a))/(b - a) < 0 ,

since f'(d) = 0 and f(d) is a maximum. Thus g(e) > 0, g(d) < 0 ans so, since g(x) is continuous, by the intermediate value theorem, there exists a ~ E (e,d) such that g(O = 0, as required. Case 2: f(c) < f(a). Proof is similar to that in case 1. Case 3: f(c) = f(a). Proof is again similar to that in case 1.

Exercise 5.23. We have f(2) = f(x + (2 - x)) = f(x) + (2 - x)f'(x) + {((2 -

Page 279: Surprises and Counterexamples in Real Function Theory

270 Appendix - II

x)2)/2}f"(x + 81(2 - x)), and f(O) f(x + (-x)) = f(x) + (-x)f'(:r) + {x2/2} f"(x+82( -x)), where 0 < 81,82 < 1. Subtracting, we get f(2) - f(O) = 2f'(x) + ((2 - x)2/2)f"(t) - (x2/2)f"(t), say, where, since 0 < 81,82 < 1 and o ::; x ::; 2, we get 0 ::; h, t2 ::; 2 (check). Hence

21f'(x)1 ::; If(2)1 + If(O)1 + 1((2 - x)2/2)f"(tdl + l(x2/2)1"(t2)1 1 .

::; 1 + 1 + 2"((2 - X)2 + x2 ) (smce If I ::; 1, 11"1::; 1) ,

= x2 - 2x +4

::; 4 (4 occurs at x = 0,2 in [0,2]) ,

or 1f'(x)1 ::; 2 in [0,2], as required.

Exercise 5.24. Let g(x) = x2. Then g(O) = 0, g(l) = 1, so the equation g'(x) = ex has the unique solution e = 2 (2x = ex and so e = 2). If f is any differentiable function on [0,1], with f(O) = 0, f(l) = 1, then the function h(x) = f(x) - g(x) is differentiable on [0,1] and h(O) = h(l) = O. Hence, by Rolle's theorem, there exists a point ~ E (0,1) such that h' (0 = 0, i.e., f'(~) - g'(~) = 0, i.e., f'(~) = 2~, i.e., e = 2 works and no other e works, as required.

Exercise 5.25. Differentiate the relation dx/dy = 1/(dy/dx) with respect to y, to give

d2x/dy2 = d(I/(dy/dx))/dy = (d(1/(dy/dx))/dx).(dx/dy)

= (( -1/ (dy / dx )2).( d2y / dx2 )) .(1/ (dy / dx))

= -d2y/dx2 /(dy/dx)3 .

On cross multiplying, this gives d2y/dx2 = -(dy/dx)3.(d2x/dy2) = -(dy/dx)3. (1/(d2y/dx2 )), (since we are assuming that d2x/dy2 = 1/(d2y/dx2)), which gives (d(dy/dx)/dx)2 = -(dy/dx)3.

Let now dy /dx = ¢>(x). Then this becomes d¢>/dx = (_¢»3/2, i.e., (_¢»-3/2. d(-¢» = -dx, which gives, on integration, (_¢»-1/2/(_1/2) = -x - a, say, or 2/A = x+a, i.e., dy/dx = -4(x+a)-2, giving y = 4/(x+a) - b, say, i.e., the family of translates of the rectangular hyperbola xy = 4, i. e., (x + a) (y + b) = 4.

Exercise 5.26. (i) Since y = cosx. cos(x + 2) - cos2(x + 1),

dy /dx = - cos x. sin(x + 2) - cos(x + 2). sin x + 2 cos (x + 1). sin (x + 1)

= -(sin(x + x + 2)) + sin(2(x + 1))

=0.

It follows that y = K, a constant. Taking x = 11"/2, we find y = - cos2(11" /2 + 1) = - sin2 1 = K, i.e., the graph is a straight line, parallel to the x-axis, through the point (11"/2, - sin2 1).

Page 280: Surprises and Counterexamples in Real Function Theory

Hints and solutions to exercises 271

(ii) To sketch the graph of Ix - 11 + Iyl = 1, first let x ;:::: 1, i.e., x-I;:::: 0 and our function becomes

{2 - x if y > 0

y= x - 2 if Y < 0 .

If now x-I::::; 0, i.e., x ::::; 1, our function becomes

{X if Y > 0

y= -x if y < 0 .

Thus the graph is shown in Figure A.2.11.

Figure A.2.ll

(1) (2)

(3) (4)

Exercise 5.27. 1'(c) = limh--+o(f(c + h) - f(c))/h = limh--+o hf'(c + he(h))/h. Now B(h) is continuous, so B(h) is bounded in a neighbourhood of h = 0 and hence h.B(h) -+ 0 as h -+ O. So the above equals the limit of 1'(c + q) as q tends to O. It follows that 1'(x) is continuous at x = c.

Exercise 5.28. It does not follow that 1'(1) exists, for example take f(x) = Ix - 11-Exercise 5.29. Differentiate (1) and (2) with respect to y:

1'(x + y) = f(x)1'(Y) - g(x)g'(y) ,

g'(x + y) = f(x)g'(y) + g(x)f'(y) .

In this put y = 0 and we get 1'(x) = -g(x)g'(O) and g'(x) = f(x)g'(O). Eliminating g'(O), we get f(x)1'(x)+g(x)g'(x) = O. Integrating, we get P(x)+ g2(x) = K. Here, if K = 0, then j2 = 0 and g2 = 0, i.e., f == 0, and g == 0, which is not the case since we are given that f and g are not constants.

Now j2(x + y) + g2(X + y) = (j2(x) + g2(X))(j2(y) + g2(y)), so K = K.K. Since K =I- 0, it follows that K = 1, as required.

Exercise 5.30. Consider the curve r: y = logx. We have dy/dx = l/x. At the point P(a, {3), let the tangent go through (0,0). It is easy to check that then (a,{3) = (e,I). Thus for Xo < e, the line joining (0,0) and (xo,logxo) cuts r in a further point (Yo, log yo), as required.

Page 281: Surprises and Counterexamples in Real Function Theory

272 Appendix - II

Next, logy - logx = J; dtlt. Since the graph of f(t) = lit is concave upwards, this integral is less than its trapezoidal approximation (see §A.l), i.e., logy -logx < (y - x)(l/x + l/y)/2, i.e.,

2(10gy -logx)/(y - x) < l/x + l/Y

N ow the line through the three points (0, 0), (x, log x), (y, log y) (collinear, since x log y = y log x) has slope (log y -log x) I (y - x) = log x I x = log y I y and so

log x I x + log y I y = 2 ((log y - log x) I (y - x)) ,

which, by (*) above, is less than l/x + l/Y.

Figure A.2.12

x y

Figure A.2.13

Exercise 5.31. By symmetry, we need to find the point P = (a, b) on y = eX that is closest to the line y = x. This point has its tangent parallel to y = x (clearly). Hence the point satisfies (d(eX)ldx)p = 1, i.e., (eX)(a,b) = 1, i.e., ea = 1, i.e., a = 0, which gives b = ea = eO = 1, i.e., P = (0,1). It follows that the two required "closest" points are

(0,1) on y = eX and (1,0) on y = log x .

Exercise 5.32. Let P = (p,p) be an intersection of y = log a x and y = aX (by symmetry, it lies on the line y = x). For the function y = loga x, we have dyldx = l/x.loge a and so at P, since we want (dYldx)p = 1, we have 1 = I/(p.loge a), which gives

p = II loge a . (1)

Similarly, for the function y = aX, we have dy I dx = aX. loge a and so at P, 1 = aP.loge a, giving

(2)

Now (1) and (2) imply aP = p and taking logarithms to base a, this implies

p = 10gaP (3)

Page 282: Surprises and Counterexamples in Real Function Theory

Hints and solutions to exercises 273

Hence 1jlogea = p (by (1)) = 10gaP (by (3)) = loge pj loge a, which implies loge P = 1, i.e., p = e.

So now, since P = (p, p) lies on the curve y = aX, we get, p = aP , i.e., e = ae, i.e., e.loge a = 1, i.e., loge a = 1je, i.e., a = el/e . thus the two curves y = (el/e)x and y = loge!/e x meet at just one (real) point (see Figure A.2.14).

/ /

/

/ /

/

Figure A.2.14

y=x

Exercise 5.33. Write t = log x, then (*) holds if and only if

k.logt=t, i.e. ,1jk=logtjt=F(t),

say, so that F(t) -t 0 as t -t 00.

Now FI(t) = (1-logt)jt2 , which is greater than 0 if t E (O,e) and is less than 0 if t E (e, (0); so F(t) is increasing in (e, (0) and F(t) has a maximum at t = e. But F(l) = 0, F(e) = 1je > 1j3; so there exist two solutions of the equation (*).

F(t)

t

Figure A.2.1S

Page 283: Surprises and Counterexamples in Real Function Theory

274 Appendix - II

Further, as k ---+ 00, 1/ k ---+ 00, so the two roots tend to 1 and 00. These are the limits of the roots of the equation log tit = 1/ k as k ---+ 00; an equation got from (*) by taking logarithms, so the roots of (*) tend to e and 00 as k ---+ 00.

Exercise 5.34. We have

10 (x+1)( 1 1) 10 (x+2)( 1 1) l' ( x) = g ----:r X+2 - x:tl - g x:tl x:tl - X (log (x + 1) -logxF '

so it is enough to prove that 1'(x) > 0, i.e., that

log((x + l)/x).(l/(x + 2) - l/(x + 1)) > log((x + 2)/(x + l)).(l/(x + 1) -l/x),

or, simplifying, it is enough to prove that

log((x + l)/x).(l/(x + 2)) < log((x + 2)/(x + l)).(l/x) (*)

Now l x+2

log(x + 2) -log(x + 1) = dt/t> L(1/(x + 2)) , x+1

since t :::; x + 2, so l/t 2: l/(x + 2), i.e.,

log((x + 2)/(x + l)).(l/x) > (l/(x + 2)).(1/x) (1)

Similarly

. rX +1 (10g(x+1)/x)(1/(x+2)) = (1/(x+2))}x dt/t < 1/(x+2) = (1/(x+2)).L(1/x),

since x:::; t :::; x + 1, so l/x 2: l/t 2: l/(x + 1), i.e., l/t :::; l/x; hence,

(log (x + l)/x)(l/(x + 2)) < (l/(x + 2))(1/x) . (2)

Now (1) and (2) give (*).

Exercise 5.35. We prove that fen + h) > fen - h) (0 < h < n). For, this is the same thing as showing that ((n+h)ne-n-h)/((n-h)ne-n+h) > 1 (on taking log­arithms), which is the same thing as showing that log((n + h)/(n - h)) > 2h/n. Here the left side is the area H below the curve y = l/x, above the x-axis, between the two ordinates x = n - h and x = n = h, while the right hand side is the area T of the trapezium formed by the tangent to y = f(x) at x = n, the x-axis, and the above two ordinates.

Exercise 5.36. (i) Since IE. is connected and f is continuous, f(IE.) (<;;; IE.) is connected and so f(IE.) is an interval, I, say, i.e., I = (a, b), [a, b], [a, b), (a,b], (oo,a), (-oo,a], (a,oo), [a,oo), (00,-00). We shall show that I = [a,b], [a, 00), (-00, a], (-00,00), i.e., I is closed. Let a be, say, the left end point of I. First suppose a is finite (a mayor may not belong to I). The case

Page 284: Surprises and Counterexamples in Real Function Theory

Hints and solutions to exercises 275

I = {a} is trivial, as then f(x) = a, for all x. So suppose I has positive length, i.e., points to the right of a, close enough to a, are in I. Noting that f(a) = a for all a E I (by (*)), we have f(a) = f(lim x --+ a+ x) = limx --+a+ f(x) (since f is continuous) = limx --+ a+ x (by above) = a, i.e., f(a) = a and so a E I, as required. If a is infinite, say -00, then the right neighbourhood of a is in I trivially. Thus in all cases I is closed (possibly unbounded), as claimed.

This now gives us all the required continuous functions that satisfy (*) as follows: First let I = [a, b] = f(lR) be bounded. Then the graph of y = f(x) lies between the two horizontal lines y = a and y = b (see Figure A.2.16).

f (c)

a b c

Figure A.2.16

Further, f restricted to [a, b] ([a, b] on the x-axis now) is the identity function. Thus all the required functions are any continuous continuations of the bold line (shown in Figure A.2.16) on either side, that keep within the limits y = a and y = b. Thus for example, if c is any point on the x-axis, f(c) E [a, b], and so f(f(c)) = c, as required.

If I is a ray, say [a, 00), Figure A.2.16 will help in completing the proof. (ii) This will follow if we show that I, the range of f, is either equal to IR, or to a singleton point {K}. Then in the former case, since f (x) = x for all x E I, we see that f(x) = x for all x, while in the latter case, f(x) E 1= {K} for all x, i.e., f(x) = K.

If 1= IR, we are through, so let I be one of [a,b], [a,oo), (-oo,b]. It is enough to deal with one case, say I = [a, 00) (other cases being similar).

Take x near a, to the right of a; then f(x) = x and so

lim (f(x) - f(a))/(x - a) = lim (x - a)/(x - a) = 1 . x--+a+ x--+a+

However, for x to the left of a,

x - a < 0, but f(x) 2: a (since f(x) E I for all x) (**)

Page 285: Surprises and Counterexamples in Real Function Theory

276 Appendix - II

and so

lim (f(x) - f(a))/(x - a) = lim (f(x) - a)/(x - a) :=:; 0 (by (**)) x--+a- x--+a-

(the limit exists, since f is differentiable). This shows that the left and the right derivatives of f(x) at x = a can not be equal, so f(x) is not differentiable at x = a. It follows that I = {a}, as required.

Exercise 5.37. I (F.D. Hammer, American Math. Monthly, 82 (1975), 415-416). Let f(x) = L~=l g(n! x)/(n!)2, where g(y) is the periodic function of period 1, defined in [-1/2,1/2]' by g(y) = y(l - 4y2), so that g(n) = 0 for all n E Z and g(n + 1/2) = 0 for all nEZ, and g(x) has a continuous derivative for all x and this derivative is equal to 1 at all integers, so (i) is satisfied. A rough graph of the function g(x) is drawn in Figure A.2.17.

Figure A.2.17

Further, for any x E iQ, f(x) has at most finitely many nonzero terms and they are rational, i.e. , (ii) is satisfied.

Finally, the formal derivative of f(x) converges uniformly and absolutely and therefore converges to the derivative of f(x). For x E iQ, the derivative of f(x) is the same as the series for e = L l/n!, except for at most finitely many terms, which are rational. Thus, for x E iQ, f'(x) = e + 0:, 0: E iQ, i.e., (iii) holds.

II. Let

f(x) = {1/22n- 1 ~f x = k/2n , where k is an odd integer less than 2n ,

o otherwIse.

Since the number of odd k that are less than 2n is 2n - 1 , we see that f(x) equals 1/22n - 1 for 2n - 1 values of x, i.e., 2n - 1 times. Hence the variation of f(x) in [0 , 1] is L~=l 2n - 1 /22n - 1 = L~=l 1/2n = 1, i.e., f(x) is of bounded variation and therefore can be written as a difference of two monotone increasing func­tions, i.e ., f = h - iz· It follows that f is differentiable almost everywhere, i.e., on a dense set. However, f is discontinuous for x = k /2 n , which forms a dense subset of [0,1].

A nother example: Let

f(x) = {q-q. if.x:= pi?, (p, q) = 1, o If x IS IrratIOnal.

Page 286: Surprises and Counterexamples in Real Function Theory

Hints and solutions to exercises 277

Then f(x) is discontinuous at each rational, but differentiable at each algebraic irrational. III. If such a function exists, then by the second mean value theorem, for each n = 1,2, ... , there exists a ~n E (0, l/n), such that

f(l/n) = f(O) + f'(O)/n + f"(~n)/2n2 .

Now L f(l/n) is convergent, so f(l/n) -+ 0 as n -+ 00 and since f(x) is continuous at x = 0, f(O) = O. Further, f"(X) is continuous, and so bounded, say 1f"(x)1 :::; M. Then L 1f"(~n)/2n21 :::; L M/2n2 and so f"(~n)/2n2 is absolutely convergent. Now by (*), noting that f(O) = 0, we have

N N N L f(l/n) = L f'(O)/n + L f"(~n)/2n2. n=l n=l n=l

Here, the left hand side is convergent and the second term on the right hand side is convergent, but the first term on the right hand side is divergent, which is a contradiction, unless l' (0) = 0 and (*) gives

Here the right side is convergent while the left side is divergent-a contradiction. So such a function can not exist.

Exercise 6.1. (i) is true, for the hypothesis implies that a;' -+ 0 and so a;' < 1 if n > N, therefore a~ < a;' if n > N (even if an < 0), and so I: a~ is convergent. (ii) is false, for example take

an = {1/n2 if n is not a square, l/n if n is a square.

(iii) is false, for example take an = (_l)n / log n. (iv) does not ensure convergence, for example take

{n if n is a prime,

an = o otherwise.

(v) ensures convergence, for suppose lan+1/anl :::; k for n > N. Then for M > N, we have

i.e., laml :::; km-nlaNI -+ 0 as M -+ 00, since k < 1, i.e., an -+ O. (vi) does not ensure convergence, for example take the sequence

1 0 123 3 2 1 0 1 2 3 4 5 543 2 1 0 . . . . . - - - - - - - - - - - - - - -2'2' 3' 3' 3' 4'4'4'4' 5' 5'5' 5' 5' 6'6'6'6'6'6'

Page 287: Surprises and Counterexamples in Real Function Theory

278 Appendix - II

Then an-1 - an --t 0 as n --t 00, an are bounded but an oscillate between 0 and 1. (vii) L an convergent implies that an < 1 for all n > N and hence a;, < an if n > N so that L a; is convergent. (viii) ani (I-an) < 2an +2a; if n > N (check), so L anl(l-an) is convergent, since L an and L a; are convergent. (ix) La;' is not necessarily convergent, for example, take an = (-I)nlfo. (x) L anl(l - an) not necessarily convergent, for example, let

v (22-1 pairs 1/2-1/2) (3 2 -1 pairs 1/3-1/3)

111 1 1 1 +---+---+ ... +---+ ... n n n n n n " " v

((n2-1) pairs 1/n-1/n)

Here, L an is convergent with sum 0 clearly, since the series is alternating and monotone decreasing and an --t 0; but

(22-1 pairs 1-1/3) (32 -1 pairs 1/2-1/4)

1 1 1 1 1 1 + -- - --+-- ---+ ... +--- -- + ... n-I n+1 n-I n+I n-I n+1 " v '

((n2-1) pairs 1/(n-1)-1/(n+1))

Here 1 I (n - 1) - 1 I (n + 1) = 2 I (n 2 - 1), and so each block of n 2 - 1 pairs sums to 2 and so our series under consideration is divergent.

Exercise 6.2. We have bnlan = 1 + En, where En --t 0, and so

bn = an + Enan , since an > 0

< an + an, if n > N, since En < 1

= 2an ·

So L bn is convergent.

Exercise 6.3. Let E > 0 (and k > 0) be given. For N arbitrarily large, select an n (n > m > N) such that an ~ kin and then

lam+! + ... + ani> (n - m)an

~ (n - m)kln

=k-mkln

> E,

(since an is decreasing)

(since an ~ kin)

Page 288: Surprises and Counterexamples in Real Function Theory

Hints and solutions to exercises 279

if n is large enough, i.e., we can not make lam+1 + ... + ani arbitrarily small and so 2: an is divergent, by the general principle of convergence.

Exercise 6.4. Observe that 2:(cn + idn) convergent means 2: Cn and 2: dn are convergent. The length AN ofthe segment SN- 1 SN is the distance between the points 2:~:11 (cn + idn) and 2:~=1 (cn + idn ), which is equal to J(c'Jv + d'Jv) :s: CN +dN, since CN and dN are both greater than or equal to O. Hence the length

N=l N=l N=l

which stays bounded as .!vI ---+ 00, since 2: CN and 2: dN are convergent.

The result is false if the condition Cn, dn 2: 0 is replaced by Cn + dn > O. For example, let Cn = (-I)11+1/n, dn = (-I)n/n. Then A = J(c'Jv+d'Jv) =

J(I/N2 + I/N2) = V2/N; therefore 2:~=1 AN = V2. 2:~=1 (I/N), which is unbounded, since 2: 1/ N is divergent.

Exercise 6.5. Let A = limsupdn , A = lim inf dn. Then A,A both exist and are finite, for if say A were infinite, i.e., dn unbounded, then dn + d;;I 2: dn would be unbounded, which can not be the case since {dn + d;;,l} is convergent.

Suppose that A > A (2: 1, since dn 2: 1). Choose subsequences

Then l/dn"l/dn21 ... ---+ I/A and l/dm"l/dm2 , ... ---+ I/A. Therefore d111 + l/dn" dn2 + 1/ dn2 , ... ---+ A + 1/ A and dm, + l/dm" dm2 + l/dm2 , ... ---+ A+ 1/ A. But the sequence {dn + l/dn} is convergent to S, say; so any subsequence is also convergent to S. It follows that S = A + 1/ A = A + 1/ A. This gives (A - A)(AA - 1)/ AA = 0, i.e., either A = A or AA = 1 and both are impossible since A > A ~ l.

The example: Let 0 < k < 1 and let dn = k, l/k, k, l/k, ... (then all terms are greater than or equal to k), so l/dn = l/k, k, l/k, k, ... and hence dn + l/dn = k + l/k ---+ k + l/k, but dn does not tend to any limit.

Exercise 6.6. We have Pn+I/ Pn = 1 + an+1 > 1, so Pn+1 > Pn, i.e., Pn increases, but

where A = 2::1 ai remains bounded since 2: ai is convergent.

If now 2: an/ Rn were convergent, then Qn = (l+aI/ Rr)(I+a2/ R2) ... (1+

Page 289: Surprises and Counterexamples in Real Function Theory

280 A.ppendix - II

ani Rn) would tend to a limit, by the first part,

i.e., (a1: ~ .~.' .. ) . (:: : : : :) ... ( an ::~: .~.' .. ) i.e., (al + a2 + ... )/(an+l + ... ) ~ f-t ,

i.e., )./(an+l + ... ) ~ a finite limit,

which can not be the case, since an+l + ... ~ 0, by the general principle of convergence.

Page 290: Surprises and Counterexamples in Real Function Theory

Bibliography

[1] Abeian, A., An ultimate proof of Rolle's theorem, Amer. Math. Monthly, 86 (1979), 484 - 485.

[2] Allendoerfer, C.B., The proof of Euler's equation, Amer. Math. Monthly, 55 (1948), 94 - 95.

[3] Aull, C.E., The first symmeric derivative, Amer. Math. Monthly, 74 (1967),708 - 711.

[4] Azpeitia, A.G., On the Lagrange remainder of the Taylor formula, Amer. Math. Monthly, 89 (1982), 300 - 301.

[5] Baillie, R., Sums of reciprocals of integers missing a given digit, Amer. Math. Monthly, 86 (1979),372 - 374.

[6] Baker, A., Transcendental Number Theory, Cambridge University Press, London (1975).

[7] Ball, A.A., Identifying points of inflection, Math. Gaz., 63 (1979), 225-229.

[8] Barrow, D.F., Infinite exponentials, Amer. Math. Monthly, 43 (1936), 150 - 160.

[9] Beagle, E.G. and Ayres, W.L., On Hildebrandt's example of a function without a finite derivative, Amer. Math. Monthly, 43 (1936), 294 - 296.

[10] Beigel, R, Fundamental period, Math. Mag., 54 (1981), 142 - 143.

[11] Bers, L., On avoiding the mean value theorem, Amer. Math. Monthly, 74 (1967), 583.

[12] Billingsley, P., Van der \Vaerden's continuous, nowhere differentiable function, Amer. Math. Monthly, 89 (1982), 691.

[13] Blank, A.A., A simple example of a Weierstrass' function, Amer. Math. Monthly, 73 (1966),515 - 519.

281

Page 291: Surprises and Counterexamples in Real Function Theory

282

[14] Boas, R.P., Jr., L'Hospital's rule without mean value theorems, Amer. Math. Monthly, 76 (1969), 1051 - 1053.

[15] Boas, R.P., Jr., Partial sums of the harmonic series, Amer. Math. Monthly, 78 (1971), 864 - 870.

[16] Brauer F., A simplification of Taylor's theorem, Amer. Math. Monthly, 94 (1987), 453 - 455.

[17] Brubaker, D.A., A proof that not both 7re and 7r + e are algebraic, Math. Mag., 44 (1971), 267.

[18] Bruckner, A.M., Some nonequivalent definitions of inflection points, Amer. Math. Monthly, 69 (1962), 787 - 789.

[19] Burgess, C.E., Continuous functions and connected graphs, Amer. Math. Monthly, 97 (1990), 337 - 339.

[20] Bush, K.A., Locally recurrent functions, Amer. Math. Monthly, 69 (1962), 199-206.

[21] Carter, F.S., On Van der Waerden's nowhere differentiable function, Amer. Math. Monthly, 91 (1984), 307 - 308.

[22] Castellanos, D., The ubiquious 7r, Part I and Part II, Math. Mag., 61 (1988), 67 - 98 and 148 - 163.

[23] Caveny, R. and Caveny, J., Periodic sums of functions with incommen­surable periods, Math. Mag., 55 (1982), 280 - 285.

[24] Chaundy, T.W., On the formula ofthe mean, Math. Gaz., 34 (1950), 219 - 220.

[25] Chorlton, F., A fixed feature of the mean value theorem, Math. Gaz., 67 (1983), 49 - 50.

[26] Cohen, L.W., On being mean to the mean value theorem, Amer. Math. Monthly, 74 (1967), 581 - 582.

[27] Coleman, A.J., A simple proof of Stirling's formula, Amer. Math. Monthly, 58 (1951),334 - 336.

[28] Cowen, C.C., Davidson, K.R. and Kaufman, R.P., Rearranging the alter­nating harmonic series, Amer. Math. Monthly, 87 (1980), 817 - 819.

[29] Cox, R.H. and Kurtz, L.C., Real periodic functions, Amer. Math. Monthly, 73 (1966), 761 - 762.

[30] Darst, R.B., Some Cantor sets and Cantor functions, Math. Mag., 45 (1972), 2 - 7.

Page 292: Surprises and Counterexamples in Real Function Theory

283

[31] Dunham, W., A historical gem from Vito Volterra, Math. Mag., 63 (1990),234 - 237.

[32] Ewing, G.M., On the definition of inflection point, Amer. Math. Monthly, 45 (1938),681 - 683.

[33] Eynden, C.V., Proof that 2: lip diverges, Amer. Math. Monthly, 87 (1980),394 - 397.

[34] Feingold, A., New geometric representation of an old infinite series, Amer. Math. Monthly, 73 (1966), 528 - 531.

[35] Forder, H.G., Direct derivation of the series for log 2, Math. Gaz., 12 (1925), 390.

[36] Freilich, G., Increasing continuous singular functions, Amer. Math. Monthly, 80 (1973),918 - 919.

[37] Gelbaum, B.R. and Olmsted, John M.H., Counterexamples in Analysis, Holden - Day, Inc., San Francisco (1964).

[38] Gibbins, N.M., To find the upper and lower limits to the sums of the series formed by omitting from the series 2: lin, the fractions whose denominators contain (a) the digit 0, (b) the digit 1, Math. Gaz., 13 (1926), 31 - 33.

[39] Gibbins, N.M., Infinite series for fifth formers, Math. Gaz., 28 (1944), 170-172.

[40] Goodstein, P., Limits of iterated logarithmic functions, Math. Gaz., 42 (1958), 295 - 296.

[41] Halberstam, H., Transcendental numbers, Math. Gaz., 58 (1974), 276 -284.

[42] Halperin, 1., Fundamental theorem of the Calculus, Amer. Math. Monthly, 61 (1954), 122 - 123.

[43] Hilderbrandt, T.H., A simple continuous function with a finite derivative at no point, Amer. Math. Monthly, 40 (1933), 547 - 548.

[44] Hille, E., Gelfond's solution of Hilbert's 7th problem, Amer. Math. Monthly, 49 (1942), 654 - 661.

[45] Hirst, K.E., A property of Riemann integrable functions, Amer. Math. Monthly, 75 (1968), 168 - 169.

[46] Housberger, R., Mathematical Gems II, Chapter 10, 98 - 103, Math Assoc. America.

Page 293: Surprises and Counterexamples in Real Function Theory

284

[47] Hu, M.C., On the L'Hospital's rule for indeterminate forms 00/00, Math. Mag ., 44 (1971), 217 - 218.

[48] Irwin, F., A curious convergent series, Amer. Math. Monthly, 23 (1916), 149 - 152.

[49] Jacobson, B., On the mean value theorem for integrals, Amer. Math. Monthly, 89 (1982), 300 - 301.

[50] Johnsonbaugh, R., The trapezoid rule, Stirling's formula and Euler's con­stant, Amer. Math. Monthly, 88 (1981),696 - 698.

[51] Jones, F.B., Connected and disconnected plane sets and the functional equation f(x + y) = f(x) + fey), Bull. Amer. Math. Soc., 48 (1942) , 115 - 120.

[52] Jones, J.P. and Toporowski, S.T., Irrational numbers, Amer. Math. Monthly, 80 (1973), 423 - 424.

[53] Jungck, G. , The extreme value theorem, Amer. Math. Monthly , 70 (1963), 864 - 865.

[54] Katznelson, Y., and Stromberg, K., Everywhere differentiable, nowhere monotone functions, Amer. Math. Monthly, 81 (1974), 349 - 354.

[55] Kazarinoff, D.K. , A simple derivation of the Leibnitz-Gregory series for 7r/4 , Amer. Math. Monthly, 62 (1955),716 - 727.

[56] Kempner, A.J., A curious convergent series, Amer. Math. Monthly, 21 (1914), 48 - 50.

[57] Knight, W.J. , Functions with zero right derivatives are constant, Amer. Math. Monthly, 87 (1980), 657.

[58] Knoebel, R.A., Exponentials reiterated, Amer. Math. Monthly, 88 (1981),235 - 252.

[59] Kost, F., A geometric proof of the formula for log 2, Math. Mag., 44 (1971),37 - 38.

[60] Lakshmana Rao, S.K., On the sequence for Euler's constant , Amer. Math. Monthly, 63 (1956) , 572 - 573.

[61] Lang, S., Algebra, Addison-Wesley Publishing Company, Inc. (1994).

[62] Levy, P., Sur une generalization du the ore me de Rolle, C.R. Acad. Sci., Paris , 198 (1934) , 424 - 425.

[63] Libin, C.l. and Macintyre, A.J., On e and its approximation, Math. Mag. , 43 (1970),98 - 101.

Page 294: Surprises and Counterexamples in Real Function Theory

285

[64J Mahler, K., Problem 5101, Amer. Math. Monthly, 70 (1963), 571; solu­tion in Amer. Math. Monthly, 71 (1964), 564.

[65J Marsden, J. and Weinstein, A., Calculus, Benjamin-Cummings (1980), 574 - 576.

[66J Meyer, B., On the Cauchy convergence criterion, Amer. Math. Monthly, 62 (1955), 488.

[67J Minassian, D.P. and Gaisser, J.W., A simple Weierstrass function, Amer. Math. Monthly, 91 (1984),254 - 256.

[68J Mitchelmore, M.L., A matter of definition, Amer. Math. Monthly, 81 (1974),643 - 647.

[69J Mukhopadhyay, S.N., On s-differentiability I, Pmc. Nat. Acad. Sci., In­dia, 36 A (1966), 525 - 533.

[70J Mukhopadhyay, S.N., On s-differentiability IV, Pmc. Nat. Acad. Sci., Hungary, 17 (1966), 129 - 136.

[71J Mukhopadhyay, S.N., On s-differentiability V, Amer. Math. Monthly, 74 (1967), 542 - 544.

[72J Natanson, 1.P., The theory of functions of a real variable, Vol. II, Fred­erick Ungar, N.Y. (1964).

[73] Neuser, D.A. and Wayment, S.G., A note on the intermediate value prop­erty, Amer. Math. Monthly, 81 (1974),995 - 997.

[74] Niven, 1., Transcendence of 1f, Amer. Math. Monthly, 46 (1939),469 -471.

[75] Niven, 1., A simple proof that 1f is irrational, Amer. Math. Monthly, 53 (1947), 509.

[76] Norris, M.J., Integrability of continuous functions, Amer. Math. Monthly, 59 (1952),244 - 245.

[77] Paige, L.J., A note on indeterminate forms, Amer. Math. Monthly, 61 (1954), 189 - 190.

[78] Papadimitriou, 1., A simple proof of the formula I: k-2 = 1f2/6, Amer. Math. Monthly, 80 (1973), 424 - 425.

[79] Pennington, W.B., Existence of a maximum of a continuous function, Amer. Math. Monthly, 67 (1960),892 - 893.

[80] Poffald, E.1., The remainder in Taylor's formula, Amer. Math. Monthly, 97 (1990), 205 - 213.

Page 295: Surprises and Counterexamples in Real Function Theory

286

[81] Posey, E.E., Proteus forms of wild and tame arcs, Duke Math J., 31 (1964), 63 - 72.

[82] Posey, E.E. and Vaughan, J.E., Functions with a proper local maximum in each interval, Amer. Math. Monthly, 90 (1983), 281 - 282.

[83] Powderly, M., A simple proof of a basic theorem of Calculus, Amer. Math. Monthly, 70 (1963), 544.

[84] Problem E 1852, Amer. Math. Monthly, 74 (1967), 72l.

[85] Problem 4151, Amer. Math. Monthly, 53 (1946), 342.

[86] Problem 6 of Putnam Math. Competition, Amer. Math. Monthly, 61 (1954),546.

[87] Problem E 746: Bounds for finite harmonic series, Amer. Math. Monthly, 54 (1947), 340 - 342.

[88] Problem 4138 (posed and solved by G. Polya), Amer. Math. Monthly, 53 (1946), 279 - 282.

[89] Problem 5880: Condition for a quadratic; solution by Price, D.B., Amer. Math. Monthly, 81 (1974),177 - 178.

[90] Randolf, J.F., Distances between points of the Cantor set, Amer. Math. Monthly, 73 (1966), 187 - 188.

[91] Reich, S., s-differentiability and differentiability, Math. Mag., 44 (1971), 214 - 216.

[92] Richmond, D.E., An elementary proof of a theorem in Calculus, Amer. Math. Monthly, 92 (1985), 589 - 590.

[93] Rickert, N.W., A Calculus counter-example, Amer. Math. Monthly, 75 (1968), 166.

[94] Rosenbaum, J.T., Some consequences of the universal chord theorem, Amer. Math. Monthly, 78 (1971),509 - 513.

[95] Rotando L.M. and Korn, H., The indeterminate form 00 , Math. Mag., 50 (1977), 41 - 42.

[96] Rouse, H.R., The natural logarithm is transcendental, Amer. Math. Monthly, 73 (1966), 187 - 188.

[97] Royden, H.L., Real analysis, Printice-Hall ofIndia, New Delhi (2000).

[98] Rudio, F, Archimedes, Huygens, Lambert, Legenclre, Vier abhandlungen Uber die Kreisnessung, Leipzig, 1892.

Page 296: Surprises and Counterexamples in Real Function Theory

287

[99] Samelson, H., On Rolle's theorem, Amer. Math. Monthly, 86 (1979),486.

[100] Sato, D., Algebraic solutions of x Y = yX (0 < x < y), Proc. Amer. Math. Soc., 31 (1972),316.

[101] Schenkman, E., The independence of some exponential values, Amer. Math. Monthly, 81 (1974),46 - 49.

[102] Schot, S.H., Aberrancy: Geometry of the third derivative, Math. Mag., 51 (1978),259 - 275.

[103] Simmons, G.F., Introduction to Topology and Modern Analysis, McGraw - Hill Book Company, Auckland (1963).

[104] Slobin, H.L., The solutions of xY = yX, X > a, y > a, x =I- y, and their graphical representation, Amer. Math. Monthly, 38 (1931),444 - 447.

[105] Staib, J.H., Sequences vs. neighbourhoods, Math. Mag., 44 (1971), 145-146.

[106] Stein, S.K., Error of the Trapezoidal method for a concave curve, Amer. Math. Monthly, 83 (1976),643 - 645.

[107] Swift, W.C., Simple constructions of non-differentiable functions and space-filling curves, Amer. Math. Monthly, 68 (1961), 653 - 655.

[108] Takacs, 1., An increasing continuous singular function, Amer. Math. Monthly, 85 (1978),35 - 37.

[109J Tall, D., The blancmange function, continuous everywhere but differen­tiable nowhere, Math. Gaz., 66 (1982), 11 - 22.

[110] Thielman, H.P., Types of functions, Amer. Math. Monthly, 60 (1953), 156 - 161.

[111] Thurston, H., Tangents to graphs, Math. Mag., 61 (1988),292 - 294.

[112] Utz, W.R., The distance set for the Cantor discontinuum, Amer. Math. Monthly, 58 (1951),407 - 408.

[113] Walker, A.W., What is a point of inflection?, Amer. Math. Monthly, 63 (1956), 182 - 183.

[114] Ward, M., A Mnemonic for ,,(, Amer. Math. Monthly, 38 (1931),522.

[115] Watson, G.C., A note on indeterminate forms, Amer. Math. Monthly, 68 (1961),490 - 492.

[116] Wayne Roberts, A and Varberg, D.E., Convex junctions, Academic Press (1973) .

Page 297: Surprises and Counterexamples in Real Function Theory

288

[117] Weinstock, R., On continuous differentiability, Amer. Math. Monthly, 64 (1957), 492.

[118] Wilker, J.B., Transcendental galore, Math. Gaz., 66 (1982), 258 - 261.

Page 298: Surprises and Counterexamples in Real Function Theory

Index

algebraic integer, 198

Cantor ternary set, 20 Cauchy's integral test, 157 Cauchy's root test, 157 centre of curvature, 135 circle of curvature, 136 common multiple, 90

least, 90 comparison test , 157

limit form, 157 curvature, 133, 134

average, 134 curve

length of, 33

D'Alembert 's ratio test, 157 discontinuity

jump, 111 of the first kind, 111 of the second kind, 111 removable, 27, 111

Euler constant "t, 160

flex , 129, 130 function

absolutely continuous , 48 algebraic, 50 Cantor, 30 concave up, 129 convex, 107 convex up, 129 locally connected, 105 locally recurrent, 85 mid-convex, 109 periodic, 86

289

rational, 51 Riemann integrable, 101 uniformly differentiable, 112 van der Waerden's, 56

horizontal chord, 96

infimum, 2 intermediate value property, 78 isolated point, 19

L'Hospital 's rule, 145 limit inferior, 155 limit superior, 155 local maximum

proper, 48

Mean Value Theorem, 123

nowhere dense set, 24 number

algebraic complex, 17 real , 3

irrational, 3 Liouville, 6 transcendental, 3

numbers commensurable , 90 incommensurable, 90

ordered field , 1 complete, 2

osculating circle, 136

perfect set, 19 period, 86

Page 299: Surprises and Counterexamples in Real Function Theory

290

fundamental, 86 point of inflection, 129

Raabe's test, 157 radius of curvature, 134, 136 rearrangement, 182

simple, 187

Schwarz derivative, 234 second, 237

Schwarz differentiability, 234 uniform, 239

sequence, 155 convergent, 155 divergent, 155

series, 156 alternating, 158 convergent, 156

absolutely, 159 conditionally, 159 unconditionally, 182

divergent, 156 sparse point, 24 Stirling's formula, 226 supremum, 1 symmetric differentiability, 234

tangent , 114 inflectional, 129

ternary expansion, 20

Wallis' product, 13

Page 300: Surprises and Counterexamples in Real Function Theory

Texts and Readings in Mathematics

1. R. B. Bapat: Linear Algebra and Linear Models (Second Edition) 2. Rajendra Bhatia: Fourier Series (Second Edition) 3. C. Musili: Representations of Finite Groups 4. H. Helson: Linear Algebra (Second Edition) 5. D. Sarason: Complex Function Theory (Second Edition) 6. M. G. Nadkarni: Basic Ergodic Theory (Second Edition) 7. H. Helson: Harmonic Analysis (Second Edition) 8. K. Chandrasekharan: A Course on Integration Theory 9. K. Chandrasekharan: A Course on Topological Groups 10. R. Bhatia (ed.): Analysis, Geometry and Probability 11. K. R. Davidson: C* - Algebras by Example 12. M. Bhattacharjee et al.: Notes on Infinite Permutation Groups 13. V. S. Sunder: Functional Analysis - Spectral Theory 14. V. S. Varadarajan: Algebra in Ancient and Modern Times 15. M. G. Nadkarni: Spectral Theory of Dynamical Systems 16. A. Borel: Semisimple Groups and Riemannian Symmetric Spaces 17. M. Marcolli: Seiberg - Witten Gauge Theory 18. A. Bottcher and S. M. Grudsky: Toeplitz Matrices, Asymptotic

Linear Algebra and Functional Analysis 19. A. R. Rao and P. Bhimasankaram: Linear Algebra (Second Edition) 20. C. Musili: Algebraic Geometry for Beginners 21. A. R. Rajwade: Convex Polyhedra with Regularity Conditions

and Hilbert's Third Problem 22. S. Kumaresan: A Course in Differential Geometry and Lie Groups 23. Stef Tijs: Introduction to Game Theory 24. B. Sury: The Congruence Subgroup Problem 25. R. Bhatia (ed.): Connected at Infinity 26. K. Mukherjea: Differential Calculus in Normed Linear Spaces

(Second Edition) 27. Satya Deo: Algebraic Topology: A Primer (Corrected Reprint) 28. S. Kesavan: Nonlinear Functional Analysis: A First Course 29. S. Szabo: Topics in Factorization of Abelian Groups 30. S. Kumaresan and G. Santhanam: An Expedition to Geometry 31. D. Mumford: Lectures on Curves on an Algebraic Surface (Reprint) 32. J. W. Milnor and J. D. Stasheff: Characteristic Classes (Reprint) 33. K. R. Parthasarathy: Introduction to Probability and Measure

(Corrected Reprint) 34. A. Mukherjee: Topics in Differential Topology 35. K. R. Parthasarathy: Mathematical Foundations of Quantum

Mechanics 36. K. B. Athreya and S. N. Lahiri: Measure Theory 37. Terence Tao: Analysis I (Second Edition) 38. Terence Tao: Analysis II (Second Edition)

Page 301: Surprises and Counterexamples in Real Function Theory

39. W. Decker and C. Lossen: Computing in Algebraic Geometry 40. A. Goswami and B. V. Rao: A Course in Applied Stochastic

Processes 41. K. B. Athreya and S. N. Lahiri: Probability Theory 42. A. R. Rajwade and A. K. Bhandari: Surprises and Counterexamples in Real Function Theory 43. G. H. Golub and C. F. Van Loan: Matrix Computations (Reprint of the

Third Edition) 44. Rajendra Bhatia: Positive Definite Matrices 45. K. R. Parthasarathy: Coding Theorems of Classical and Quantum Information Theory 46. C. S. Seshadri: Introduction to the Theory of Standard Monomials 47. Alain Connes and Matilde Marcolli: Noncommutative Geometry,

Quantum Fields and Motives 48. Vivek S. Borkar: Stochastic Approximation: A Dynamical Systems

Viewpoint 49. B. J. Venkatachala: Inequalities: An Approach Through Problems 50. Rajendra Bhatia: Notes on Functional Analysis 51. A. Clebsch (ed.): Jacobi's Lectures on Dynamics

(Second Revised Edition) 52. S. Kesavan: Functional Analysis 53. V. Lakshmibai and Justin Brown: Flag Varieties: An Interplay of

Geometry, Combinatorics, and Representation Theory 54. S. Ramasubramanian: Lectures on Insurance Models 55. Sebastian M. Cioaba and M. Ram Murty: A First Course in Graph

Theory and Combinatorics 56. Bamdad R. Yahaghi: Iranian Mathematics Competitions, 1973-2007 57. Aloke Dey: Incomplete Block Designs 58. R.B.Bapat: Graphs and Matrices